0% found this document useful (0 votes)
2K views

Blueprints Surgery PDF

Uploaded by

Zay
Copyright
© © All Rights Reserved
We take content rights seriously. If you suspect this is your content, claim it here.
Available Formats
Download as PDF, TXT or read online on Scribd
0% found this document useful (0 votes)
2K views

Blueprints Surgery PDF

Uploaded by

Zay
Copyright
© © All Rights Reserved
We take content rights seriously. If you suspect this is your content, claim it here.
Available Formats
Download as PDF, TXT or read online on Scribd
You are on page 1/ 491

Authors: Karp, Seth J.; Morris, James P.G.

; Zaslau, Stanley
Title: Blueprints Surgery, 5th Edition
Copyright ©2008 Lippincott Williams & Wilkins

2008

Lippincott Williams & Wilkins


Philadelphia
Baltimore, MD 21201

978-0-7817-8868-7

0-7817-8868-4

Acquisitions EditorCharles W. Mitchell

Senior Managing EditorStacey Sebring

Editorial AssistantCatherine Noonan

Marketing ManagerJennifer Kuklinski

Creative DirectorDoug Smock

Associate Production ManagerKevin P. Johnson

CompositorInternational Typesetting and Composition

Fifth Edition

Copyright © 2008 Lippincott Williams & Wilkins, a Wolters Kluwer business.

351 West Camden Street 530 Walnut Street

Baltimore, MD 21201 Philadelphia, PA 19106

Printed in China

All rights reserved. This book is protected by copyright. No part of this book may be
reproduced or transmitted in any form or by any means, including as photocopies or
scanned-in or other electronic copies, or utilized by any information storage and retrieval
system without written permission from the copyright owner, except for brief quotations
embodied in critical articles and reviews. Materials appearing in this book prepared by
individuals as part of their official duties as U.S. government employees are not covered
by the above-mentioned copyright. To request permission, please contact Lippincott
Williams & Wilkins at 530 Walnut Street, Philadelphia, PA 19106, via email at
[email protected], or via website at lww.com (products and services).

987654321

Library of Congress Cataloging-in-Publication Data

Karp, Seth J.

Surgery / Seth J. Karp, James P.G. Morris ; questions and answers provided

by Stanley Zaslau.—5th ed.

p. ; cm.—(Blueprints)

Rev. ed. of: Blueprints surgery / Seth J. Karp, James P.G. Morris. 4th ed.

c2006.

Includes bibliographical references and index.

ISBN-13: 978-0-7817-8868-7

ISBN-10: 0-7817-8868-4

1. Surgery—Outlines, syllabi, etc. I. Morris, James, 1964- II. Zaslau,

Stanley. III. Karp, Seth J. Blueprints surgery. IV. Title. V. Series.

[DNLM: 1. Surgical Procedures, Operative—Examination Questions.

WO 18.2 K18s 2009]

RD37.3.K37 2009

617'.910076—dc22

2008035981

DISCLAIMER

Care has been taken to confirm the accuracy of the information present and to describe
generally accepted practices. However, the authors, editors, and publisher are not
responsible for errors or omissions or for any consequences from application of the
information in this book and make no warranty, expressed or implied, with respect to the
currency, completeness, or accuracy of the contents of the publication. Application of
this information in a particular situation remains the professional responsibility of the
practitioner; the clinical treatments described and recommended may not be considered
absolute and universal recommendations.
The authors, editors, and publisher have exerted every effort to ensure that drug
selection and dosage set forth in this text are in accordance with the current
recommendations and practice at the time of publication. However, in view of ongoing
research, changes in government regulations, and the constant flow of information
relating to drug therapy and drug reactions, the reader is urged to check the package
insert for each drug for any change in indications and dosage and for added warnings and
precautions. This is particularly important when the recommended agent is a new or
infrequently employed drug.

Some drugs and medical devices presented in this publication have Food and Drug
Administration (FDA) clearance for limited use in restricted research settings. It is the
responsibility of the health care provider to ascertain the FDA status of each drug or
device planned for use in their clinical practice.

To purchase additional copies of this book, call our customer service department at (800)
638-3030 or fax orders to (301) 223-2320. International customers should call (301)
223-2300.

Visit Lippincott Williams & Wilkins on the Internet: http://www.lww.com. Lippincott


Williams & Wilkins customer service representatives are available from 8:30 am to 6:00
pm, EST.
Authors: Karp, Seth J.; Morris, James P.G.; Zaslau, Stanley
Title: Blueprints Surgery, 5th Edition

Copyright ©2008 Lippincott Williams & Wilkins

> Front of Book > Authors

Authors P

Seth J. Karp MD
Attending Surgeon
Beth Israel Deaconess Medical Center

Assistant Professor of Surgery

Harvard Medical School

Boston, Massachusetts

James P.G. Morris MD, FACS


Thoracic and General Surgeon
The Permanente Medical Group

Chief of Surgery

South San Francisco Kaiser Hospital

South San Francisco, California

Questions and answers provided by

Stanley Zaslau MD, MBA, FACS


Associate Professor
Division of Urology

West Virginia University

School of Medicine

Morgantown, West Virginia

Contributors
Ramzi Alami MD
Bariatric & General Surgeon
Department of Surgery
The Permanente Medical Group
South San Francisco Kaiser Hospital
South San Francisco, CA

Chapter 10

Rona L.T. Chen MD, FACS


General Surgeon
Department of Surgery
The Permanente Medical Group
South San Francisco Kaiser Hospital
South San Francisco, CA

Chapter 14

Grant Cooper MD
Fellow
Spine, Sports and Musculoskeletal Medicine
Orthopedics and Rehabilitation Medicine
Beth Israel Medical Center
New York, NY

Chapter 25

Jason Cooper MD
Plastic Surgery Resident
Department of Surgery
Brigham and Women's Hospital
Harvard Plastic Surgery
Boston, MA

Chapter 24

David Le MD
Bariatric & General Surgeon
Department of Surgery
The Permanente Medical Group
South San Francisco Kaiser Hospital
South San Francisco, CA

Chapter 10
Alice Yeh MD, FACS
Surgical Oncologist
Department of Surgery
The Permanente Medical Group
South San Francisco Kaiser Hospital
South San Francisco, CA

Chapter 12 and 13

Stanley Zaslau MD, MBA, FACS


Program Director, Associate Professor
Division of Urology
West Virginia University
Morgantown, WV

Questions and Answers


Authors: Karp, Seth J.; Morris, James P.G.; Zaslau, Stanley
Title: Blueprints Surgery, 5th Edition

Copyright ©2008 Lippincott Williams & Wilkins

> Front of Book > Abbreviations

Abbreviations

ABGs arterial blood gases

ACAS Asymptomatic Carotid Atherosclerosis Study

ACE angiotensin-converting enzyme

ACTH adrenocorticotropic hormone

ADH antidiuretic hormone

AFP alpha-fetoprotein

AI aortic insufficiency

ALT alanine transaminase

ANA antinuclear antibody

AP anteroposterior

APKD adult polycystic kidney disease

ARDS adult respiratory distress syndrome

AS aortic stenosis

ASD atrial septal defect

AST aspartate transaminase

ATLS Advanced Trauma Life Support

AUA-IPSS American Urological Association Symptom Score

AV arteriovenous

BCC basal cell carcinoma

BCG bacill (bacillus) Calmette-Guérin

BE barium enema

β-hCG beta-human chorionic gonadotropin


BP blood pressure

BPH benign prostatic hypertrophy

BRCA breast cancer gene

BUN blood urea nitrogen

CABG coronary artery bypass graft

CAD coronary artery disease

CBC complete blood count

CCK cholecystokinin

CDC Centers for Disease Control and Prevention

CEA carcinoembryonic antigen

CES cauda equina syndrome

CHF congestive heart failure

CIS carcinoma in situ

CMF cyclophosphamide, methotrexate, and 5-fluorouracil

CMV cytomegalovirus

CN cranial nerve

CNS central nervous system

COPD chronic obstructive pulmonary disease

CPAP continuous positive airway pressure

CRF corticotropin-releasing factor

CRH corticotropin-releasing hormone

CSF cerebrospinal fluid

CT computed tomography

CXR chest x-ray

DCIS ductal carcinoma in situ

DEXA dual-energy x-ray absorptiometry

DHT dihydrotestosterone

DIC disseminated intravascular coagulation

DIP distal interphalangeal


DNA deoxyribonucleic acid

DTRs deep tendon reflexes

ECG electrocardiography

EEG electroencephalogram

EGD esophagogastroduodenoscopy

EMG electromyography

ERCP endoscopic retrograde cholangiopancreatography

ESR erythrocyte sedimentation rate

EUS endoscopic esophageal ultrasound

ESWL extracorporeal shock wave lithotripsy

FDG-PET fluorodeoxyglucose positron emission tomography

FNA fine-needle aspiration

FSH follicle-stimulating hormone

G-6-PD glucose-6-phosphate dehydrogenase

GBM glioblastoma multiforme

GCS Glasgow Coma Scale

GERD gastroesophageal reflux disease

GGT gamma-glutamyl transferase

GH growth hormone

GI gastrointestinal

GU genitourinary

Hb hemoglobin

hCG human chorionic gonadotropin

HIDA hepatobiliary iminodiacetic acid

HIV human immunodeficiency virus

HLA human leukocyte antigen

HPF high-power field P

HPI history of present illness

HPV human papilloma virus


HR heart rate

ICP intracranial pressure

ID/CC identification and chief complaint

IgA immunoglobulin A

IL-2 interleukin-2

IMA inferior mesenteric artery

IMV inferior mesenteric vein

INR international normalized ratio

ITP immune thrombocytopenic purpura

IVP intravenous pyelography

JVD jugular venous distention

KUB kidneys/ureter/bladder

LAD left anterior descending coronary artery

LCA left coronary artery

LCIS lobular carcinoma in situ

LCX left circumflex

LDH lactate dehydrogenase

LES lower esophageal sphincter

LFTs liver function tests

LH luteinizing hormone

LH-RH luteinizing hormone-releasing hormone

LM left main coronary artery

LVH left ventricular hypertrophy

Lytes electrolytes

MCP metacarpophalangeal

MCV mean corpuscular volume

MELD Model for End-Stage Liver Disease

MEN multiple endocrine neoplasia

MHC major histocompatibility complex


MI myocardial infarction

MMF mycophenolate mofetil

MPA mycophenolic acid

MR mitral regurgitation

MRCP magnetic resonance cholangiopancreatography

MRI magnetic resonance imaging

MS mitral stenosis

MTC medullary thyroid carcinoma

MVA motor vehicle accident

NASCET North American Symptomatic Carotid Endarterectomy Trial

NG nasogastric

NPO nil per os (nothing by mouth)

NSAID nonsteroidal anti-inflammatory drug

NSGCT nonseminomatous germ cell tumor

Nuc nuclear medicine

OPSS overwhelming postsplenectomy sepsis

PA posteroanterior

PBS peripheral blood smear

PCNL percutaneous nephrolithotomy

PDA posterior descending coronary artery

PDS polydioxanone

PE physical examination

PEEP positive end-expiratory pressure

PET positron-emission tomography

PFTs pulmonary function tests

PIP proximal interphalangeal

PMI point of maximal impulse

PP pancreatic polypeptide

PPI proton-pump inhibitors


PSA prostate-specific antigen

PT prothrombin time

PTC percutaneous transhepatic cholangiography

PTH parathyroid hormone

PTU propylthiouracil

RA right atrium

RBC red blood cell

RCA right coronary artery

REM rapid eye movement

RPLND retroperitoneal lymph node dissection

RR respiratory rate

RV right ventricular

RVH right ventricular hypertrophy

SAH subarachnoid hemorrhage

SBFT small bowel follow-through

SBO small bowel obstruction

SCC squamous cell carcinoma

SIADH syndrome of inappropriate secretion of ADH

SLNB sentinel lymph node biopsy

SMA superior mesenteric artery

SMV superior mesenteric vein

SSI surgical site infection

STD sexually transmitted disease

STSG split-thickness skin graft

TCC transitional cell carcinoma

TIA transient ischemic attack

TIBC total iron-binding capacity

TIPS transjugular intrahepatic portosystemic shunt

TNM tumors, nodes, metastases classification


TPN total parenteral nutrition

TRAM transverse rectus abdominis myocutaneous

TRH thyrotropin-releasing hormone

TSH thyroid-stimulating hormone

TUBD transurethral balloon dilatation

TUNA transurethral needle ablation

TURP transurethral resection of the prostate P

UA urinalysis

UGI upper gastrointestinal

US ultrasound

UTI urinary tract infection

UV ultraviolet

VMA vanillylmandelic acid

VS vital signs

VSD ventricular septal defect

WBC white blood cell

XR x-ray
Authors: Karp, Seth J.; Morris, James P.G.; Zaslau, Stanley
Title: Blueprints Surgery, 5th Edition
Copyright ©2008 Lippincott Williams & Wilkins

> Front of Book > Preface

Preface

It has been 12 years since the first five books in the Blueprints series were published.
Originally intended as board review for medical students, interns, and residents who
wanted high-yield, accurate clinical content for U.S. Medical Licensing Examination
(USMLE) Steps 2 and 3, the series now also serves as a guide to students during third-year
and senior rotations. We are extremely proud that the original books and the entire
Blueprints brand of review materials have far exceeded our expectations and have been
dependable reference sources for so many students.

The fifth edition of Blueprints Surgery has been significantly revised. Reorganization of
the Table of Contents creates a more logical flow to the chapters. Every chapter includes
updates to reflect current practices in the field. A new chapter in the gastrointestinal
section explores bariatric surgery. Similar to the previous edition, sample operative
reports are included in an appendix. As Blueprints is used in a wider range of clinical
settings, students have had the opportunity to review and comment on what additional
material would be useful. In response, an increased number of figures, including
radiographic studies, photographs, and drawings, integrate with the text. This fifth
edition is the first to include a color insert, showing detailed depictions of surgical
techniques. The Questions and Answers sections include 25% more material for USMLE
Board review. Finally, suggestions for additional reading are available online, along with
an additional 50 USMLE-format questions and answers for further self-study.

We sincerely hope this edition preserves the original vision of Blueprints to provide
concise, useful information for students and that the additional material enhances this
vision.
Seth J. Karp MD

James P.G. Morris MD


Authors: Karp, Seth J.; Morris, James P.G.; Zaslau, Stanley
Title: Blueprints Surgery, 5th Edition

Copyright ©2008 Lippincott Williams & Wilkins

> Table of Contents > Part I - Introduction > Chapter 1 - Surgical Techniques

Chapter 1
Surgical Techniques

INTRODUCTION
As with most endeavors in life, the healing arts are divided into both the theoretical and the practical spheres. Surgeons are fortunate
to practice equally in both spheres by applying their intellect and technical skill to the diagnosis and treatment of sickness. The
practice of surgery is unique in the realm of medicine and correspondingly carries added responsibilities. Patients literally place their
trust in the hands of surgeons. The profound nature of cutting into another human being, and artfully manipulating his or her physical
being to achieve wellness, requires reverence, skill, and judgment.

Technologic advances in modern medicine have led to the rise and establishment of procedure-related specialties, including invasive
cardiology and radiology, dermatology, intensive care medicine, and emergency medicine, to name a few. Manipulative skills are now
required not only in the operating room but also in procedure rooms and emergency rooms for invasive treatments and repairing
traumatic injuries. Therefore, medical students and residents should master the basics of surgical technique so they are well prepared
for the challenges ahead.

PREOPERATIVE ISSUES
For well-trained and experienced surgeons, performing an operation is usually a routine affair and is relatively simple. One of the
difficulties in taking care of surgical patients, however, is actually making the decision to operate. Operating is simple; deciding not to
operate is the more difficult decision. Ultimately, the surgeon and patient must assess the risk-to-benefit ratio and decide whether the
potential benefits of surgery outweigh the potential risks. Once the decision has been made to proceed with surgery, the surgeon must
formulate a clear operative plan, taking into account and preparing for any potential deviations that may be required based on the
intraoperative findings.

The relationship between patient and doctor is based on a special trust. In the surgical sphere, individuals grant their surgeon
permission to render them unconscious, invade their body cavities, and remove or manipulate their internal structures to a degree that
the latter deems appropriate. Physicians must never minimize the importance of this special trust that underlies the surgical
relationship. A surgeon gains a patient's trust by engaging in a thorough discussion before the decision to operate is reached, outlining
the clinical situation and indications for surgery. All reasonable management options should be reviewed and the risks and potential
complications of each presented. This process of decision making is known as informed consent. Appropriate written documentation
must be obtained—usually a "request" for operation, rather than a more passive "consent"—and signed by the patient or guardian, the
person performing the procedure, and a witness.

Adequate preparation of a patient for surgery depends on examining the magnitude and nature of the intended operation in light of the
patient's general medical condition. The surgical patient must be able to endure the potential insults of surgery (hypotension,
hypoxemia, hypothermia, anemia, and postoperative pain) without being exposed to unacceptable risks of morbidity and mortality. All
patients, particularly older adult patients with multiple medical problems, should undergo an appropriate preoperative evaluation to
identify and thoroughly evaluate medical illnesses and thereby more accurately establish the degree of
perioperative risk that the proposed surgery entails. The two main goals of preoperative evaluation are to assess and maximize the P
patient's health, as well as to anticipate and avoid possible perioperative complications. Consultation with a cardiologist,
pulmonologist, endocrinologist, or internist may involve specific diagnostic tests and laboratory studies (Table 1-1).

TABLE 1-1 Preoperative Diagnostic Testing

Body
Tests Used Disorders Identified
System/Specialty
Cardiac* Electrocardiography Ischemic heart disease

Echocardiography Cardiac arrhythmias

Radionuclide Congestive heart failure


ventriculography

Thallium scintigraphy Valvular heart disease

Pathophysiologic Hypertension
limitations of testing

Respiratory Chest x-ray Chronic obstructive


pulmonary disease

Arterial blood gas Obstructive sleep apnea

Peak expiratory flow


rate

Pulmonary function test

Sleep study

Endocrine Blood test Diabetes mellitus

Adrenal disorder

Thyroid disorder

Hematology Blood test Thromboembolic disease

Bleeding disorder

Gastrointestinal Blood test Liver disease

Radiographic imaging Intestinal obstruction

Endoscopy Malnutrition
*Scoring systems include Goldman Index, Eagle Criteria, Detsky Score,
and Revised Cardiac Risk Index.

Preoperative assessment is also made by an anesthesiologist before surgery to determine the patient's fitness for anesthesia, which is
evaluated according to the American Society of Anesthesiologists Physical Status Classification System. Class I indicates a fit and healthy
patient, whereas class V indicates a moribund patient not expected to survive 24 hours with or without an operation (Table 1-2).

TABLE 1-2 American Society of Anesthesiologists Physical Status Classification System

Class Description

Class A fit and healthy patient


I

Class A patient with mild systemic


II disease
(e.g., hypertension)

Class A patient with severe systemic


III disease that
limits activity but is not
incapacitating

Class A patient with an incapacitating


IV systemic
disease that is a constant threat to
life

Class A moribund patient not expected to


V sur-
vive 24 hours with or without an
operation

Note: If the procedure is performed as an


emergency, an "E" is added
to the physical status classification. Example:
A healthy 70-year-old
male with mild hypertension undergoing
emergent appendectomy is
considered class II E.
Routine preoperative screening tests are ordered only when indicated by rational guidelines. Gone are the days when asymptomatic,
low-risk, minor surgery patients were subjected to an extensive and expensive battery of tests (blood tests, chest x-ray, urinalysis, and
electrocardiogram). The belief was that a thorough array of tests would systematically detect occult conditions, thereby avoiding
potential morbidity and mortality. Over time, such an approach has been devalued, as published studies have shown that routine
medical testing has not measurably increased
surgical safety. Therefore, modern preoperative testing relies on defined guidelines that focus on evaluating the risk arising from P
patient-specific comorbidities and conditions.

A foundational principle of the medical tradition is to do no harm: primum non nocere. In 2000, a widely publicized report from the
Institute of Medicine, To Err is Human, estimated that 98,000 people die each year in U.S. hospitals as a result of medical injuries. This
report, among others already in the literature, led to the creation of a number of national quality improvement projects that were
specifically designed to improve surgical care in hospitals. One of the best known is the Surgical Care Improvement Project (SCIP), part
of a national campaign aimed at reducing surgical complications by 25% by 2010. The multiyear project is sponsored by the Centers for
Medicare and Medicaid Services in partnership with the U.S. Centers for Disease Control and Prevention (CDC), the Joint Commission,
Institute for Healthcare Improvement, and the American Hospital Association. With a goal of saving lives and reducing patient injuries,
SCIP examines the process and outcome measures related to infectious, cardiac, venous thromboembolic, and respiratory care. As
hospitals incorporate these measures into their provision of care, it is expected that the rates of postoperative wound infection,
perioperative myocardial infarction, deep venous thrombosis and pulmonary embolism, and ventilator-related pneumonia will decrease
(Table 1-3).

TABLE 1-3 SCIP Measures

Category Outcome Measure

Infectious Prophylactic antibiotic received within 1 hour before


surgical incision.

Appropriate prophylactic antibiotic selection.

Prophylactic antibiotics discontinued within 24 hours


after surgery end time.

Appropriate method of hair removal.

Normothermia maintained in colorectal surgery


patients postoperatively.

Cardiac surgery patients have controlled 6 AM


postoperative serum glucose.

Postoperative wound infection diagnosed during


index hospitalization.

Cardiac Patients on a beta-blocker before operation receive


continued beta-blockade during the
perioperative period.
Patients with evidence of coronary artery disease
receive beta-blockers during perioperative
period.

Postoperative myocardial infarction diagnosed during


index hospitalization or within 30 days of
surgery.

Venous Recommended venous thromboembolic prophylaxis


thromboembolic ordered.

Appropriate venous thromboembolic prophylaxis


received within 24 hours before surgery to
24 hours after surgery.

Pulmonary embolism diagnosed during index


hospitalization and within 30 days of surgery.

Deep venous thrombosis diagnosed during index


hospitalization and within 30 days of surgery.

Respiratory Several process and outcome measures related to


ventilated surgery patients.

Regarding antibiotic prophylaxis for the prevention of surgical site infections, broad implementation of the measures outlined by SCIP
could decrease the overall incidence significantly. In essence, selecting the appropriate antibiotic, administering it within 60 minutes of
incision, and discontinuing it within 24 hours postoperatively is the goal.

Regarding preoperative hair removal, minimal or no hair removal is preferred. The CDC guidelines for hair removal state that only the
interfering hair around the incision site should be removed, if necessary. Removal should be done immediately before the operation,
preferably with electric clippers. Using electric clippers minimizes microscopic skin cuts, which are more common from traditional
blade razors and serve as foci for bacterial multiplication.

Maintaining patient core body temperature to avoid hypothermia should be standard practice. Both passive and active warming P
measures should be used when indicated (e.g., blankets, fluid warmer, forced-air warmer).

Traditional practice for colon surgery has included preoperative mechanical and chemical cleansing of the large intestine in an attempt
to decrease intraluminal bacterial counts and thereby minimize anastomotic leakage and postoperative infectious complications. This
established practice is based on observational studies and animal experiments and is not supported by prospective randomized trial
data. Interestingly, recent prospective randomized trials call the routine use of mechanical bowel preparation into question, and a
Cochrane Review (2005) concludes, "Mechanical bowel preparation before colorectal surgery cannot be recommended as routine." Given
the present data, mechanical bowel preparation should be used selectively depending on the clinical situation.

Regarding active infections at the time of elective surgery, CDC guidelines advise diagnosis and treatment of "all infections remote to
the surgical site before elective operation and postpone elective operations on patients with remote site infections until the infection
has resolved."

The publication of the Institute of Medicine report in 2000 brought into focus, and set as a national priority, an issue that had been
steadily growing since the mid-1990s: improving medical and surgical safety. Before the report, large medical organizations had begun
to apply a systems approach to examining medical errors. Pioneering efforts by the Veterans Health Administration to decrease medical
errors led to the establishment of the National Surgical Quality Improvement Program in 1994. The core concept of such programs is to
create systems of safety similar to the aviation and nuclear power industries. Highly visible aviation accidents have been found to
involve human error 70% of the time, as shown by National Aeronautics and Space Administration research. This statistic parallels the
less visible medical experience, as analysis of Joint Commission data on sentinel events shows that communication failures were the
primary root cause in more than 70% of events. Additional oft-cited studies indicate that surgical errors result from communication
failure, fatigue, and lack of surgical proficiency. In an effort to inculcate a culture of safety and minimize surgical misadventure
through miscommunication, many hospitals have instituted Highly Reliable Surgical Team (HRST) training. This training is modeled on
Crew Resources Management training from the aviation industry, which has been shown to enhance error reduction. Some of the HRST
training goals are creating an open and free communication environment, minimizing disruptions to patient care, improving
coordination among departments, and conducting quality preoperative briefings and verifications.

INTRAOPERATIVE ISSUES
After completing the HRST preoperative briefing communication with the anesthesiologist and operating room team, ensure that the
overall operating room environment is to your satisfaction. The operating table and overhead lights should be correctly positioned.
Room temperature and ambient noise should be adjusted as necessary. Play music if appropriate. Ensure adequate positioning and
prepping of the patient. Communicate again with the team to confirm readiness. Then scrub, gown, and drape. Before incision, again to
minimize surgical errors, many hospitals call for a final check or "time out" to ensure that the correct patient is undergoing the correct
procedure.

Deciding where to make the skin incision is usually straightforward. Thought should be taken to consider possible need for extending
the incision or possibly converting from a laparoscopic approach to open surgery. Before incising the skin, consider the skin's intrinsic
tension lines to maximize wound healing and cosmesis of the healed scar. Incisions made parallel to the natural lines of tension usually
heal with thinner scars because the static and dynamic forces on the wound are minimized. When making elective facial skin excisions
or repairs of traumatic facial lacerations, keep in mind that incisions perpendicular to these tension lines will result in wider, less
cosmetically acceptable scars (Fig. 1-1).
Figure 1-1 • Skin tension lines of the face and body.

Adapted from Simon R, Brenner B. Procedures and Techniques in Emergency Medicine. Baltimore,
MD: Williams & Wilkins; 1982.

Although general anesthetic techniques are usually the anesthesiologist's job, all invasive practitioners should have a working
knowledge of local anesthetics. Depending on the procedure being performed, the choice of local anesthetic must be tailored to each
patient. Local anesthetics diffuse across nerve membranes and interfere with neural depolarization and transmission. Each local
anesthetic agent has a different onset of action, duration of activity, and toxicity. Epinephrine is often administered concurrently with
the local anesthetic agent to induce vasoconstriction, thereby prolonging the duration of action and decreasing bleeding. The two most
commonly used local anesthetics are the shorter-acting lidocaine (Xylocaine) and
the longer-acting bupivacaine (Marcaine), the properties of which are outlined in Table 1-4. P.7 P

TABLE 1-4 Pharmacologic Properties of Local Anesthetic Agents


Onset of Action

Duration of Maximum Allowable


Agent Concentration Infiltration Block
Action Dose One Time

Lidocaine 1.0% Immediate 4–10 60–120 min 4.5 mg/kg of 1%


(Xylocaine) min (for blocks) (30 mL per average
adult)

Bupivacaine 0.25% Slower 8–12 240–480 min 3 mg/kg of 0.25%


(Marcaine) min (for blocks) (50 mL per average
adult)

Adapted from Trott A. Wounds and Lacerations: Emergency Care and Closure. 2nd ed. St. Louis,
MO: Mosby–Year Book; 1997:31.

Lidocaine (1% and 2%, with and without epinephrine) has a rapid onset of action, achieving sensory block in 4 to 10 minutes. The
duration of action is approximately 75 minutes (range, 60 to 120 minutes). The maximum allowable dosage is 4.5 mg/kg per dose
without epinephrine or 7 mg/kg per dose with epinephrine.

Bupivacaine (0.25%, 0.5%, and 0.75%, with and without epinephrine) has a slower onset of action, taking 8 to 12 minutes for a simple
block. Duration of action is approximately four times longer than that of lidocaine, lasting 2 to 8 hours, making bupivacaine the
preferred agent for longer procedures and for prolonged action. The maximum allowable dosage is 3 mg/kg per dose.

INSTRUMENTS
The basic tools of a surgeon are a knife for cutting and a needle with suture for restoring tissues to their appropriate position and
function. Additional tools and instrumentation simply allow operations to be performed with greater finesse.

The most commonly used knife blades are illustrated in Figure 1-2 and are made functional by attachment to a standard no. 3 Bard-
Parker knife handle. Choose the size and shape of the blade based on the intended indication. Abdominal or thoracic skin incisions are
typically made with no. 10, 20, or 22 blades, whereas more delicate incisions could require the smaller no. 15 blade. The sharp-tipped
no. 11 blade is ideal for entering and draining an abscess or for making an arteriotomy by incising a blood vessel in preparation for
vascular procedures.

Figure 1-2 • General surgery knife blades. Shown here are Bard-Parker knife blades nos. 10, 11,
15, 20, and 22.
Scissors are mainly used for dissecting and cutting tissues. All scissors are designed for right-handed use. Each pair of scissors should
only be used for the indication for which it was designed (Fig. 1-3). Most scissors have either straight or curved tips. Fine iris scissors
are used for delicate dissection and cutting. Metzenbaum scissors are versatile, general-use instruments. Sturdy Mayo scissors are used
for cutting thick or dense tissues, such as fascia, scar, or tendons.

Figure 1-3 • Mayo, Metzenbaum, and iris scissors.

Various forceps have been developed to facilitate manipulation of objects within the operative field, as well as to stabilize tissues and
assist in dissection. All forceps perform essentially the same function but differ in the design of their tips and their intrinsic delicacy of
form (Fig. 1-4). Toothed forceps are useful for stabilizing and moving tissues, whereas smooth atraumatic forceps are more appropriate
for delicate vascular manipulation. DeBakey forceps are good general-use instruments with
atraumatic flat tips and tiny fine serrations. Fine-toothed Adson forceps are ideal for skin closure, and stout-toothed Bonney forceps P
are excellent for facial closure.
Figure 1-4 • Smooth, Adson, and toothed forceps.

Needle and suture are used to maintain tissue apposition until healing has occurred. The array of needles and suture material is vast;
therefore, the surgeon's choice is based on the specific indication at hand.

Needles come either straight or curved. Curved needles are usually half circle or three-eighths circle. Sewing in a deep hole may
require a five-eighths circle needle, whereas microsurgery often requires quarter circle needles.

Needles can have an eye for threading the suture (French eye) or an already attached suture (swaged). Most needles today are swaged,
meaning they are a needle-suture combination. Etymologically, a swage is a blacksmithing tool used to shape metal. Thus a swaged
needle is manufactured by placing the suture into the hollow shank of a needle and then compressing the needle around the suture,
holding it firm. Some sutures are swaged to needles in such a manner that they pop off if excess tension is applied between suture and
needle.

Needle tips are either tapered or cutting. Tapered needles are circumferentially smooth and slide between the elements of tissues, P
whereas cutting needles are triangular in cross section and cut through tissues like a tiny knife (Fig. 1-5).

Figure 1-5 • Needle characteristics.

Suture material is categorized according to its permanence (absorbable or nonabsorbable), its structure (braided or monofilament), and
its caliber (Table 1-5). Absorbable suture material is made from either naturally derived, collagen-based materials or synthetic
polymers. Examples of absorbable suture material are gut (plain and chromic), polyglactic acid (Vicryl), polyglycolic acid (Dexon),
polyglyconate (Maxon), and polydioxanone (PDS). The suture is either hydrolyzed by water or undergoes enzymatic digestion, thereby
losing tensile strength over time.

TABLE 1-5 Common Suture Material

Material Loss of Inflammatory Absorbed Common Uses


Tensile Reaction
Strength

Absorbable

Gut

    Plain Sheep 7-10 Moderate 2 Superficial vessels;


intestine days months closure of
tissues that heal
rapidly
(buccal mucosa) and
require
minimal support

    Chromic Treated 10-14 Moderate 3


with days but less months
chromium
salt

Polyglactic Synthetic 4-5 wk Minimal 3 Dermis; fat muscle


acid polyfilament months
(Vicryl);
polyglycolic
acid
(Dexon)

Polyglyconate Synthetic 3-4 wk Minimal 3-4 Subcuticular close and


(Maxon, monofilament months soft
Monocryl) tissue approximation

Polydioxanone Synthetic 8 wk Minimal 6 Muscle; fascia


(PDS) monofilament months

Nonabsorbable

Polypropylene Polymer of Yearsa Minimal — Fascia; muscle;


(Prolene) propylene vessels

Nylon Polyamide Yearsa Minimal — Skin; drains;


(Surgilon, microsurgical
Nurolon) anastomoses

Silk Raw silk 1 yr Intense — Tie off vessels; bowel


spun by silk
worm

Staples Iron- — Minimal — Skin


chromium-
nickel

aWith reoperation, polypropylene and nylon remain present but decompose slightly.

From Taylor JA. Blueprints Plastic Surgery. Malden, MA: Blackwell Science; 2005.

Permanent nonabsorbable sutures are made from materials impervious to significant chemical degradation and are useful for
maintaining long-term tissue apposition. Examples of permanent nonabsorbable suture material are nylon, polypropylene, stainless
steel, and silk.

P
CLOSURE TECHNIQUES
There are many techniques for closing wounds (Fig. 1-6). Wounds can be closed using a continuous stitch that is quick to perform and
results in tension distributed along the length of the suture. Simple interrupted stitching allows for precise tissue approximation (skin or
fascia). Mattress stitches can be placed either vertically or horizontally, allowing excellent skin apposition and eversion while
minimizing tension. Subcuticular stitching using an absorbable suture at the dermal-epidermal junction is a convenient skin-closure
technique, allowing epidermal apposition so that postoperative suture removal is unnecessary. Regardless of the closure technique
used, certain basic principles must be considered to avoid wound breakdown and to achieve a well-healed, cosmetically satisfactory
scar:

Figure 1-6 • Suturing techniques: (A) Continuous, (B) simple interrupted, (C) horizontal
mattress, (D) vertical mattress, and (E) subcuticular.
(From Taylor JA. Blueprints Plastic Surgery. Malden, MA: Blackwell Publishing; 2005:7.)

skin incision along intrinsic tension lines

gentle handling of intraoperative tissue

meticulous hemostasis

tension-free closure

eversion of skin edges

An alternative to sutured skin closure of wounds is skin apposition using metal staples. Many elective surgical wounds are closed with
skin staples, because the technique allows for rapid skin closure, minimal wound inflammatory response, and near-equivalent cosmetic
results. This technique usually works best with two operators: one to evert and align the skin edges with forceps and another to fire the
stapler (Fig. 1-7).

P
Figure 1-7 • Wound closure using metal staples.

Removal of staples is performed using a simple handheld device that deforms the staple and reconfigures the staple shape, allowing for
smooth, easy withdrawal (Fig. 1-8).

Figure 1-8 • Technique for staple removal.


Authors: Karp, Seth J.; Morris, James P.G.; Zaslau, Stanley
Title: Blueprints Surgery, 5th Edition

Copyright ©2008 Lippincott Williams & Wilkins

> Table of Contents > Part I - Introduction > Chapter 2 - Care of the Surgical Patient

Chapter 2
Care of the Surgical Patient

PREOPERATIVE EVALUATION
Preoperative evaluation has two main purposes: identification of modifiable risk factors and risk
assessment. Unless the situation is emergent, every patient should have a detailed history and physical
examination. In all patients, attention should be given to a history of cerebrovascular accident, heart
disease of any kind, pulmonary disease, renal disease, liver disease and other gastrointestinal disorders,
diabetes, prior surgeries, bleeding problems, clotting problems, difficulty with anesthesia, poor nutrition,
alcohol use, and illicit drug use. Allergies, current medications, family history, social history, and a careful
and complete review of systems should be conducted. Often, the review of systems will reveal problems
that require more detailed workup.

Physical examination should be focused on identifying problems that require further workup. For example,
facial asymmetry, speech problems, weakness, or a carotid bruit may suggest prior cerebrovascular incident
that requires further workup. Cardiac disease may present with evidence of congestive heart failure,
crackles on lung examination, or jugular venous distention.

Pulmonary disease may result in, for example, a barrel chest with poor air movement in patients with
chronic obstructive pulmonary disease or wheezing in patients with asthma. Liver disease may cause
ascites, caput medusae, telangiectasias, or asterixis. Ecchymosis may be evidence of bleeding problems,
whereas extremity swelling may result from clotting disorders.

Choice of laboratory studies depends on the patient's underlying medical condition and the extent of the
surgery. There has been a trend toward less routine testing and increased reliance on the history and
physical examination. In otherwise healthy patients undergoing minor surgery, laboratory studies, including
coagulation studies, are probably not indicated. Similarly, in patients with no history of pulmonary or
cardiac disease and no significant risk factors undergoing minor or moderate surgery, electrocardiogram
(ECG) and chest x-ray (CXR) are also probably not indicated.

General guidelines for preoperative testing in patients without risk factors are as follows:

ECG: Male older than 40 years or female older than 50 years undergoing cardiovascular procedures

CXR: All patients older than 60 years or undergoing thoracic procedures

Hematocrit: All patients if the procedure is expected to cause >500 mL of blood loss

Creatinine: Patients older than 50 years or if the procedure has a high risk for generating renal failure

Pregnancy test: All women of childbearing age if pregnancy status is uncertain


MODIFICATION OF RISK FACTORS IN THE PREOPERATIVE PERIOD
Initial preoperative evaluation may suggest additional testing needed, either to determine the surgical risk
or to further identify modifiable factors. For example, chest pain or shortness of breath with mild exertion
should prompt a more thorough cardiac evaluation, including an ECG and stress test. If these tests show
cardiac disease, a decision will need to be made regarding whether the patient requires an intervention
before surgery. If preoperative
assessment demonstrates carotid artery disease, it may be best to perform an endarterectomy before the P
originally planned surgery. Other issues, such as poorly controlled diabetes, obesity, and malnutrition,
should also be addressed before surgery. This may involve modifications of diet and insulin dose, weight
loss, or inpatient admission for total parenteral nutrition. Good glucose control in diabetics as measured by
hemoglobin A1c levels is associated with improved outcomes after surgery. In patients suffering from
malnutrition scheduled for surgery, preoperative total parenteral nutrition has been shown to improve
outcomes.

TIMING OF SURGERY
Once the risk factors for surgery have been identified, a frank discussion should be held with the patient
explaining the potential risks and benefits of the surgery and which risk factors should be addressed before
the surgery. This discussion should be the basis for the informed consent for surgery. The outcome of this
discussion may be that the surgery should proceed without delay, that the procedure is too risky and should
not be attempted, or somewhere in between. For example, in a young healthy person with a symptomatic
inguinal hernia, surgery without delay is indicated. On the other hand, if the hernia is small and
asymptomatic and the patient has advanced liver disease with uncontrolled ascites, the risk of the surgery
probably outweighs the benefits. In a patient with end-stage renal disease undergoing workup for a kidney
transplant who is found to have unstable angina and coronary disease amenable to intervention, the
coronary intervention should proceed before transplantation. When operation occurs within 3 months of a
myocar-dial infarction, the risk of a subsequent one is approximately 30% in the perioperative period and
decreases to less than 5% after 6 months, so that surgery should be delayed in these patients if at all
possible. The Goldman criteria, although offered in 1977, are still widely used to evaluate surgical risk
(Table 2-1). More recent criteria from the American College of Cardiology and the American Heart
Association provide algorithms for assessing risk. Outcomes, especially for class IV risk patients, are much
better with improvements in medical and surgical care, but the classification is a useful one to help stratify
surgical risk.

TABLE 2-1 Goldman Criteria for Cardiac Risk

Point
Factor
Value

Age >70 years 5

Myocardial infarction within 6 months 10


S3 gallop or JVP >12 cm 11

Significant valvular stenosis 3

Rhythm other than sinus, or atrial 7


ectopy

Ventricular premature beats >5/min 7


at
any time

PO2 <60 or PCO2 >50 mm Hg, or 3


Potassium <3 mEq/L or HCO3 <20, or
BUN >50 mg/dL or creatinine
>3.0 mg/dL, or chronic liver disease
or debilitation

Intraperitoneal, intrathoracic, or 3
aortic
surgery

Emergency surgery 4

Risk assessed as follows: 0–5 points, class I,


1% complication risk;
6–12 points, class II, 7% complication risk; 13–
25 points, class III,
14% complication risk; >25 points, class IV,
78% complication risk.

JVP, jugular venous pressure; PO2, partial


pressure of oxygen; PCO2,
partial pressure of carbon dioxide; HCO3,
bicarbonate; BUN, blood
urea nitrogen.

DIABETES MANAGEMENT BEFORE SURGERY


There are a number of ways to manage glucose levels before and during surgery in patients with diabetes. If
possible, patients with diabetes should be scheduled as the first case of the day to simplify glucose control,
as patients will not be eating. For day surgery, in patients with type I diabetes or those with type II diabetes
requiring multiple injections, patients should take one half of their usual dose of intermediate-acting or
long-acting insulin the morning of surgery. When these patients undergo major surgery, they can hold their
insulin entirely in preparation for intravenous therapy during the operation.

For patients with type II diabetes taking once-daily or twice-daily insulin, or for patients on oral
medications undergoing day surgery, oral hypoglycemics should be held, and one half dose of intermediate-
acting insulin should be given in the morning. Patients undergoing major surgery can be managed
intraoperatively with intravenous insulin. Glucose levels should
be checked on arrival for surgery and every 2 hours while waiting. P

PERIOPERATIVE MANAGEMENT

FLUIDS AND ELECTROLYTES


Fluids and electrolytes must be provided in adequate amounts to replace intraoperative losses. This will
maintain blood pressure and ensure optimal cardiac function. Choice of fluids depends on the underlying
medical problems. For example, the use of potassium-containing fluids should be avoided in patients with
renal failure. For longer and more complicated cases, consideration of loss of other electrolytes, including
calcium and magnesium, must be addressed.

BLOOD PRODUCTS
Administration of blood products depends on the underlying health of the patient and the type of operation.
Whereas in a healthy patient with a limited intraoperative event in which 500 mL of blood is lost, resulting
in a hematocrit of 24%, a transfusion may not be indicated. However, for a patient undergoing liver
transplantation with expected ongoing blood loss, transfusion may be indicated at a level of 28%.
Transfusion of fresh-frozen plasma and platelets should be considered for patients with coagulopathy or
thrombocytopenia.

CARDIAC RISK FACTORS


In patients with known cardiac disease, aggressive intraoperative lowering of myocardial oxygen demand
with beta-blockers has been shown in randomized trials to improve outcomes and should be used.

ANTIBIOTICS
Antibiotics are of benefit in all procedures in which a body cavity is opened and are probably useful in clean
procedures. Guidelines for the use of antibiotics include administration before the incision (within 1 hour)
and redosing after 4 hours. Specific recommendations are shown in Table 2-2.

TABLE 2-2 Antibiotic Recommendations

Type of Procedure Antibiotic Choice

Clean Cefazolin or none

Cardiovascular Cefazolin

Colorectal Oral:
Neomycin/erythromycin
base plus mechanical
cleansing before
surgery
Parenteral: cefazolin/
metronidazole or
cefotetan

Prosthetic Cefazolin
joint

DIABETES MANAGEMENT
A large randomized controlled trial published in 2001 by Van den Berge et al demonstrated improved
outcomes in surgical intensive care patients maintained with strict blood sugar control between 80 and 110
mg/dL. Since then, there has been tremendous activity in defining which patients benefit from this therapy
and evaluating the complications. Many patients, including those undergoing general or cardiac surgery,
benefit from improved outcomes and decreased infectious complications, including reduced septicemia.
Patients using these regimens must be monitored carefully to prevent the development of hypoglycemia.

PREVENTION OF DEEP VENOUS THROMBOSIS


Prevention of deep venous thrombosis is of major importance. Although rare, this complication is
potentially life-threatening. Randomized clinical trials are abundantly clear. When heparin is given, it
should be done so in the preoperative area, before the incision. Administration of any anticoagulation in the
perioperative period must carefully consider the risk of bleeding, and therapy should be individualized to
the patient and the particulars of the operation (e.g., if the operation was unusually bloody). Having stated
that, there is evidence that the following regimens are efficacious in improving outcomes.

Minor Surgery
In patients younger than 40 years undergoing minor surgery with no risk factors, early ambulation decreases
risk of deep venous thrombosis, and use of subcutaneous heparin is controversial. In patients between 40
and 60 years or with risk factors, heparin 5000 U administered subcutaneously 2 times per day or enoxaparin
40 mg administered subcutaneously every day is beneficial. In patients older than 60 years or with
additional risk factors, heparin can be increased to 3 times per day, or enoxaparin 30 mg administered P
subcutaneously 2 times per day is beneficial.

Major Surgery
Patients younger than 40 years with no risk factors should receive heparin 5000 U subcutaneously 2 times
per day or enoxaparin 40 mg subcutaneously every day. Patients older than 40 years or with risk factors
should receive heparin 5000 U subcutaneously 3 times per day or enoxaparin 30 mg subcutaneously 2 times
per day.

Orthopedic Surgery
Because of the increased risk of these procedures, aggressive anticoagulation is indicated. Patients
undergoing hip fracture repair, total hip replacement, or total knee replacement should receive enoxaparin
30 mg subcutaneously 2 times per day, coumadin to keep international normalized ratio between 2 and 3, or
fondaparinux 2.5 mg.

Graded compression stockings and intermittent pneumatic compression boots are often used in combination
with pharmacologic therapy, but their benefit is less well established.

TEMPERATURE
Maintenance of normal intraoperative temperature is critical for adequate hemostasis and optimal
cardiovascular function. Use of warmed fluids and warming blankets may be necessary for long operations
or those in which large body cavities are opened.

POSTOPERATIVE MANAGEMENT
Principles of postoperative management include early mobilization, pulmonary therapy, early nutrition,
adequate fluid and electrolyte administration, management of cardiac risk factors, control of blood sugars,
and recognition of complications.

Early mobilization is important to prevent muscle wasting and weakness, reduce the risk of venous
thromboembolism, reduce the rate of pneumonia, and perhaps speed the return of bowel function. If
permitted by the type of surgery, the patient should get out of bed on the day of surgery and be walking on
the first postoperative day.

One of the most common postoperative complications is pneumonia. Deep-breathing exercises and the use
of an incentive spirometer can decrease this risk.
Early nutrition (within 24 hours of surgery) has been demonstrated in randomized controlled trials to
improve outcomes. In most patients, this will amount to having them eat, but many patients may require a
nasoenteric tube for this purpose. In patients undergoing surgery on the gastrointestinal tract, the timing of
feeds should be individualized to the patient and the type of operation. In general, enteral nutrition is
preferable to parenteral nutrition if the gut is functional.

Fluid administration is one of the most important aspects of postoperative care. Adequate resuscitation
prevents renal failure and optimizes cardiac function. Excessive fluid can cause congestive heart failure and
edema, which in turn inhibits wound healing. Fluid administration must be individualized to the patient and
the type of operation; general guidelines are to administer fluid to keep the urine output >30 mL/hr.

Decreasing cardiac risk in patients with preexisting disease requires adequate beta blockade throughout the
perioperative period. The heart rate should be kept <70 and lower if hemodynamically tolerated.

Tight control of blood sugars is clearly beneficial in reducing wound infections. This is accomplished initially
by aggressive use of sliding-scale insulin and resumption of the patient's home insulin regimen when the
patient is eating adequately.

Early recognition of surgical complications is critical to effectively managing them. Common to all but the
most minor operations are wound infections, pneumonia, urinary tract infection, catheter infections, deep
venous thrombosis, and myocardial infarction. In addition, each operation has its specific complications.
Recognition of a complication depends on detailed daily history and physical examination. Wounds should be
examined for erythema and discharge. Particularly worrisome is murky brown discharge that may represent
a dehiscence or necrotizing fasciitis. Lungs should be examined daily for decreased breath sounds and
egophony, and sputum should be examined. Thick green or brown sputum should prompt investigation for
pneumonia, including CXR and sputum gram stain and culture. Sites of catheter placement should be
examined for erythema and discharge. Urinary symptoms should prompt a urinalysis and culture. Chest pain
in the perioperative period should be taken seriously and evaluated with an ECG and cardiac enzymes in the
appropriate clinical setting.

Fever is an exceedingly common occurrence in the perioperative period, and early diagnosis is critical. The
classic "5 Ws" of wind (pneumonia), wound, walking (deep venous thrombosis), water (urinary tract P
infection), and wonder drugs (drug reaction) should be considered in all patients. Fevers on the first day
after surgery are generally attributed to the inflammatory stress of the surgery or atelectasis. If the fever is
not high and history and physical examination are unremarkable, no further workup is indicated. If, on the
other hand, the fever is high, the wound should be examined to rule out a rapidly progressive infection,
typically caused by Gram-positive cocci. Other diagnostic modalities should be used based on the type of
surgery and symptoms. An increasingly common source of fever is Clostridium difficile colitis. This should
be suspected in any patient with diarrhea after surgery. Three stool samples and a white blood cell count
should be sent. Initiation of treatment before laboratory confirmation may be indicated if the suspicion is
high.

Examples of complications related to the surgery include fascial dehiscence, breakdown or stricture of
enteric anastomoses, thrombosis of vascular grafts, deep space infections, and hernia recurrence. These
problems can usually be recognized with careful patient examination.

FLUIDS AND ELECTROLYTES


Understanding fluid and electrolyte replacement begins with knowing the composition of the various body
compartments. In a typical 70-kg person, 60% of total body weight is water. Two thirds of this water is
contained in the intracellular compartment, and one third is in the extracellular compartment. One quarter
of this extracellular water is plasma—approximately 3.5 L in a typical man. Red blood cell volume is
approximately 1.5 L. Combined with plasma, this results in a blood volume of approximately 5 L. Electrolyte
concentrations in the extracellular and intracellular space are as shown in Table 2-3.

TABLE 2-3 Electrolyte Concentrations in the Extracellular and Intracellular


Space

Extracellular Intracellular
Electrolyte
(mEq/L) (mEq/L)

Sodium 140 10

Potassium 4.0 150

Calcium 2.5 4.0

Magnesium 1.1 34

Chloride 104 4.0

Carbonate 24 12

Phosphate 2.0 40

Daily maintenance requirements for healthy adults include urinary fluid loss of 1 L, gastrointestinal loss of
200 mL, and insensible losses of 10 mL/kg. Each of these numbers can be adjusted upward in various
disease states. Fever, burns, and diarrhea are examples of processes that can dramatically increase fluid
losses.

Normal replacement in surgical patients is 1 mEq/kg of sodium and 0.5 mEq/kg of potassium per day.
Gastrointestinal losses in patients can be approximated with the information provided in Table 2-4. The
composition of various replacement fluids is as shown in Table 2-5.

TABLE 2-4 Normal Replacement of Fluids in Surgical Patients (mEq/L)


Type of Fluid Sodium Potassium H+ Bicarbonate

Gastric 20–120 12 30–100 0

Duodenal 110 15 0 10

Ileum (ileostomy) 100 10 0 40

Colon (diarrhea) 120 25 0 45

Bile 140 5 0 25

Pancreas 140 5 0 115

TABLE 2-5 Composition of Various Replacement Fluids (mEq/L)

Replacement Fluid Sodium Potassium Chloride Calcium Lactate

Normal saline 154 154 0 0 0

Half normal saline 77 77 0 0 0

Lactated Ringer's Solution 130 4 109 3 28

Common electrolyte abnormalities in the perioperative period include hyponatremia, hypernatremia,


hyperkalemia, and hypokalemia. Causes of hyponatremia should be divided into two types, depending on
whether there is reduced plasma osmolality. If plasma osmolality is normal or high, the differential
diagnosis is hyperlipidemia, hyperproteinemia, hyperglycemia, and mannitol administration. In this case,
the treatment focuses on correcting the abnormality in the osmotically active agent.

More often, the plasma osmolality is reduced. In this case, the question becomes whether the circulating P
plasma volume is high, as in congestive heart failure, cirrhosis, nephrotic syndrome, and malnutrition;
normal, as in syndrome of inappropriate secretion of antidiuretic hormone, paraneoplastic syndromes,
endocrine disorders, and various drugs (morphine, aminophylline, indomethacin); or low, with excessive
losses or inadequate replacement.

In general, patients with decreased plasma volume should be treated with hypertonic saline if the level is
low enough to warrant treatment (<120 mEq/L). In this case, it is critically important to not correct the
sodium >0.5 mEq/L per hour. If the patient's are symptomatic, however, it may be advisable to increase the
level more quickly. This should only be done in consultation with a neurologist, as faster rates can result in
central pontine myelinolysis from the osmotic shift. Patients in whom the effective plasma volume is high
should be treated with fluid restriction.

Causes of hypernatremia are divided into water loss and sodium administration. Water loss can result from
insensible losses from infection, burns, or fever; renal loss from diabetes insipidus; gastrointestinal losses;
or hypothalamic disorders. Sodium administration can be performed via ingestion or intravenously.
Treatment consists of addressing the underlying abnormality and administering fluid. Correction of
hypernatremia should not progress at a rate >0.5 mEq/L per hour, unless neurologic symptoms are present.
Rapid correction of hyponatremia can result in seizures, cerebral edema, and death.

Hypokalemia is usually due to potassium loss. Hypokalemia can result in cardiac arrhythmias, especially in
patients taking digoxin. Treatment is with exogenous replacement.

Hyperkalemia is usually due to exogenous administration or from intracellular stores. It can result in
weakness and cardiac arrhythmias. If the level is above 6 mEq/L or the patient has ECG changes, treatment
with calcium gluconate, sodium bicarbonate, insulin, and glucose can transiently decrease plasma
potassium, which may rebound, because these treatments do not alter the total body potassium. Kayexalate
decreases total body potassium but takes longer to be effective. Dialysis is extremely effective in
decreasing potassium.

KEY POINTS
Preoperative evaluation is critical to identify modifiable risk factors and provide risk
assessment.
Cardiac history is essential to the evaluation. Goldman criteria can help assess
cardiac risk.
Operation within 3 months of a myocardial infarction is associated with a 30%
complication risk.
Tight glucose control in the perioperative period improves outcomes.
Anticoagulation in the perioperative period reduces the risk of deep venous thrombosis.
Enteral nutrition is preferable to parenteral nutrition if the gut is functional.
Fever in the perioperative period is most commonly caused by the 5 Ws: wind
(pneumonia), wound, walking (deep venous thrombosis), water (urinary tract infection),
and wonder drugs (drug reaction).
Authors: Karp, Seth J.; Morris, James P.G.; Zaslau, Stanley
Title: Blueprints Surgery, 5th Edition

Copyright ©2008 Lippincott Williams & Wilkins

> Table of Contents > Part II - Gastrointestinal and Abdominal > Chapter 3 - Stomach and Duodenum

Chapter 3
Stomach and Duodenum

The stomach and duodenum are anatomically contiguous structures, share an interrelated physiology, and have similar disease
processes. Peptic ulceration is the most common inflammatory disorder of the gastrointestinal tract and is responsible for significant
disability. The stomach and duodenum are principally affected by peptic ulceration.

ANATOMY
The stomach is divided into the fundus, body, and antrum (Fig. 3-1). The fundus is the superior dome of the stomach; the body extends
from the fundus to the angle of the stomach (incisura angularis), located on the lesser curvature; and the antrum extends from the
body to the pylorus. Hydrochloric acid—secreting parietal cells are found in the fundus, pepsinogen-secreting chief cells are found in
the proximal stomach, and gastrin-secreting G cells are found in the antrum.

Figure 3-1 • Anatomy of the stomach.

Six arterial sources supply blood to the stomach: the left and right gastric arteries to the lesser curvature; the left and right
gastroepiploic arteries to the greater curvature; the short gastric arteries, branching from the splenic artery to supply the fundus; and
the gastroduodenal artery, branching to the pylorus (Fig. 3-2). The vagus nerve supplies parasympathetic innervation via the anterior
left and posterior right trunks. These nerves stimulate gastric motility and the secretion of pepsinogen and hydrochloric acid.
Figure 3-2 • Arteries and veins of the stomach and spleen. A. Arterial supply. Observe that the
stomach receives its main blood supply from branches of the celiac trunk. The fundus of the
stomach is supplied by short gastric arteries arising from the splenic artery. The spleen is
supplied by the splenic artery, the largest branch of the celiac trunk, which runs a tortuous
course to the hilum of the spleen and breaks up into its terminal (splenic) branches.

Figure 3-2 • B. Venous drainage. The drainage of the stomach is directly or indirectly into the
portal vein. The splenic vein usually receives the inferior mesenteric vein and then unites with
the superior mesenteric vein to form the portal vein as shown here. From Moore KL, Dalley AF II.
Clinically Oriented Anatomy. 4th ed. Baltimore: Lippincott Williams & Wilkins, 1999.

The duodenum is divided into four portions(Fig. 3-3). The first portion begins at the pylorus and includes the duodenal bulb. The
ampulla of Vater, through which the common bile duct and pancreatic duct drain, is located in the medial wall of the descending second
portion of the duodenum. The transverse third portion is traversed anteriorly by the superior mesenteric vessels. The ascending fourth
portion terminates at the ligament of Treitz, which defines the duodenal–jejunal junction. The arterial supply to the duodenum is via
the superior pancreaticoduodenal artery, which arises from the gastroduodenal artery, and via the inferior pancreaticoduodenal artery,
which arises from the superior mesenteric artery.
Figure 3-3 • Anatomy of the duodenum.

GASTRIC AND DUODENAL ULCERATION

PATHOGENESIS
The etiology of benign peptic gastric and duodenal ulceration involves a compromised mucosal surface undergoing acid-peptic
digestion. Substances that alter mucosal defenses include nonsteroidal anti-inflammatory drugs, alcohol, and tobacco. Alcohol directly
attacks the mucosa, nonsteroidal anti-inflammatory drugs alter prostaglandin synthesis, and tobacco restricts mucosal vascular
perfusion. However, the most important and remarkable advancement in understanding the pathogenesis of peptic ulceration was the
radical idea that infestation with the organism Helicobacter pylori was the causative factor in gastric and duodenal ulceration. This
discovery destroyed prevailing dogma and profoundly altered the medical and surgical treatment for this disease process. So profound
was this discovery that the 2005 Nobel Prize in Medicine was awarded to Drs. Marshall and Warren, two iconoclastic Australian
researchers, "for their discovery of the bacterium Helicobacter pylori and its role in gastritis and peptic ulcer disease."

P.23 P.22 P
HISTORY
Patients typically present with epigastric pain relieved by antacids. Sensations of fullness and mild nausea are common, but vomiting is
rare unless pyloric obstruction is present secondary to scarring. Physical examination is often benign except for occasional epigastric
tenderness.

DIAGNOSTIC EVALUATION
The radiographic evaluation of peptic ulcers entails barium studies that reveal evidence of crater deformities at areas of ulceration.
Serum testing determines whether there are antibodies to H. pylori, and breath testing confirms infection.

Definitive diagnosis is made by direct visualization of the ulcer using endoscopy (see Color Plate 1). For nonhealing gastric ulcers
refractory to medical therapy, it is extremely important that biopsy of the ulcer be performed to rule out gastric carcinoma. Duodenal
ulcers are rarely malignant.

TREATMENT
Medical treatment is similar for gastric and duodenal ulceration. The goals of medical therapy are to decrease production of or
neutralize stomach acid and to enhance mucosal protection against acid attack. Medications include antacids (CaCO3), H2-blockers
(cimetidine, ranitidine), mucosal coating agents (sucralfate), and proton-pump inhibitors (omeprazole). If H. pylori is present,
treatment with oral antibiotics is associated with a 90% eradication rate. Treatment regimens may consist of
tetracycline/metronidazole/bismuth subsalicylate, amoxicillin/metronidazole/ranitidine, or other combinations.
As a result of the advent of proton pump inhibitors (PPIs) and the increased understanding of the role H. pylori plays in peptic
ulceration, operations for ulcer disease have become infrequent. Indications for surgical treatment in the acute setting are either
perforation or massive bleeding. Indications for elective operation are a chronic nonhealing ulcer after medical therapy or gastric outlet
obstruction. The operation chosen must address the indication for which the procedure is performed. Historically, before the era of PPIs
and H. pylori, the goal of surgery was to permanently reduce acid secretion by removing the entire antrum. In most instances,
vagotomy and distal gastrectomy (antrectomy), with Billroth I or II anastomosis, fulfilled these criteria (Figs. 3-4 and 3-5). Because
denervation of the
stomach by truncal vagotomy alters normal patterns of gastric motility and causes gastric atony, surgical drainage procedures are P
required afterward to ensure satisfactory gastric emptying. Today, most cases of perforation are treated with closure of the defect with
omental patch, and cases of bleeding are treated with suture ligation of the bleeding vessel.

Figure 3-4 • Vagotomy and antrectomy with Billroth I anastomosis.


Figure 3-5 • Vagotomy and antrectomy with Billroth II anastomosis.

STRESS GASTRITIS AND ULCERATION

PATHOGENESIS
Critically ill patients subjected to severe physiologic stress, often in the intensive care unit setting, are at risk for developing
gastroduodenal mucosal erosion that can progress to ulceration. The commonly accepted etiology of stress gastritis and ulceration is
mucosal ischemia induced by an episode of hypotension from hemorrhage, sepsis, hypovolemia, or cardiac dysfunction. Ischemia
disrupts cellular mechanisms of mucosal protection, resulting in mucosal acidification and superficial erosion. Areas of erosion may
coalesce and form superficial ulcers. Stress ulcers may be seen throughout the stomach and proximal duodenum.

HISTORY
Patients are usually critically ill and have a recent history of hypotension. Massive upper gastrointestinal bleeding is the usual finding.

DIAGNOSTIC EVALUATION
Sites of hemorrhage can be identified by endoscopy.

TREATMENT
Endoscopy can often control bleeding by either electrocoagulation or photocoagulation. Persistent or recurrent bleeding unresponsive
to endoscopic techniques requires surgical intervention. Depending on the circumstances, operations for control of bleeding stress
gastritis or ulcer require oversewing of the bleeding vessel. Usually, vagotomy is also performed to reduce acid secretion. In many
cases, because bleeding is often diffuse and cannot be controlled by simple suture ligation, partial or total gastrectomy is performed.

PREVENTION
Prevention of stress ulceration involves maintaining blood pressure, tissue perfusion, and acid–base stability, as well as decreasing acid
production while bolstering mucosal protection. The incidence of life-threatening hemorrhagic gastritis has decreased as intravenous
H2-blocker therapy and oral cytoprotectants have been introduced to the intensive care setting.

CUSHING'S ULCER
Distinct from stress gastritis, Cushing's ulcers are seen in patients with intracranial pathology (e.g., tumors, head injury). Ulcers are
single and deep and may involve the esophagus, stomach, and duodenum. Because of the depth of ulceration, perforation is a common
complication. Neuronally mediated acid hypersecretion is thought to be the main cause of Cushing's ulcer.

ZOLLINGER-ELLISON SYNDROME AND GASTRINOMAS

PATHOGENESIS
Zollinger-Ellison syndrome occurs in patients with severe peptic ulceration and evidence of a gastrinoma (non–B-cell pancreatic tumor).
Peptic ulceration results from the production of large volumes of highly acidic gastric secretions owing to elevated serum gastrin levels.
Ninety percent of gastrinomas are found in the "gastrinoma triangle," defined by the junction of the cystic duct and the common bile
duct, the junction of the second and third portions of the duodenum, and the junction of the neck and body of the pancreas.

HISTORY
Gastrin-secreting tumors produce a clinical picture of epigastric pain, weight loss, vomiting, and severe diarrhea.

DIAGNOSTIC EVALUATION
Diagnosis is confirmed by the secretin-stimulation test, in which the injection of intravenous secretin elevates serum gastrin levels to at
least 200 pg/mL. Once diagnosed, tumor localization is performed by magnetic resonance imaging, abdominal ultrasound,
computed tomography, selective abdominal angiography, or selective venous sampling. P

TREATMENT
Acid hypersecretion can be controlled medically with H2 blockade and PPI. Somatostatin analogs (octreotide) have been found to be
effective in decreasing tumor secretion of gastrin and in controlling the growth of tumor metastases.

Gastrinoma is a curable disease, despite the malignant nature of most tumors. Complete resection of tumors results in a near 100% 10-
year survival rate. Incomplete resection or unresectability carries <50% 10-year survival rate. When simple excision or enucleation for
cure is not feasible, an attempt is made to prolong survival by debulking and performing lymph node dissection to reduce tumor burden
and acid hypersecretion.

STOMACH CANCER
Despite the decreasing incidence of gastric carcinoma in Western populations during the past decades, patient survival has not
improved. In the United States, fewer than 10% of patients with stomach cancer survive 5 years. Illustrative of geographic variation,
stomach cancer is endemic in Japan. Because of the high incidence of disease, mass screening programs are able to detect early-stage
lesions, which accounts for a 50% overall survival rate at 5 years.

RISK FACTORS
Environmental and dietary factors are thought to influence the development of gastric cancer. Smoked fish and meats contain
benzopyrene, a probable carcinogen to gastric mucosa. Nitrosamines are known carcinogens that are formed by the conversion of
dietary nitrogen to nitrosamines in the gastrointestinal tract by bacterial metabolism. Atrophic gastritis, as seen in patients with
hypogammaglobulinemia and pernicious anemia, is considered to be a premalignant condition for developing gastric cancer, because
high gastric pH encourages bacterial growth. Chronic atrophic gastritis results in achlorhydria, and 75% of patients with gastric cancer
are achlorhydric.

PATHOLOGY
Most tumors are adenocarcinomas, and spread is via lymphatics, venous drainage, and direct extension. Most tumors are located in the
antral prepyloric region.

Gastric tumors can be typed according to gross appearance. Polypoid fungating nodular tumors are usually well differentiated and carry
a relatively good prognosis after surgery. Ulcerating or penetrating tumors are the most common and are often mistaken for benign
peptic ulcers because of their sessile nature. Superficial spreading lesions diffusely infiltrate through mucosa and submucosa and have a
poor prognosis because most are metastatic at the time of diagnosis.
The pathologic staging of gastric cancer is based on depth of tumor invasion and lymph node status. Survival is closely correlated with
the pathologic stage of a specific tumor (Fig. 3-6).

Figure 3-6 • Staging system for gastric carcinoma.

HISTORY
Patients with gastric cancer usually give a history of vague and nonspecific symptoms. Upper abdominal discomfort, dyspepsia, early
satiety, belching, weight loss, anorexia, nausea, vomiting, hematemesis, or melena is common. Definite symptoms do not occur until
tumor growth causes luminal obstruction, tumor infiltration results in gastric dysmotility, or erosion causes bleeding. By the time of
diagnosis, tumors are usually unresectable. Later symptoms indicative of metastatic disease are abdominal distention owing to ascites,
resulting from hepatic or peritoneal metastases, and dyspnea and pleural effusions, resulting from pulmonary metastases.

PHYSICAL EXAMINATION
Few findings are noted on physical examination, except in advanced disease. A firm, nontender, mobile epigastric mass can be
palpated, and hepatomegaly with ascites may be present. Other distant signs of metastatic disease include Virchow supraclavicular
sentinel node, Sister Joseph umbilical node, and Blumer shelf on rectal examination.

DIAGNOSTIC EVALUATION
Anemia is often found on routine blood studies. The anemia is usually hypochromic and microcytic secondary to iron deficiency. Stool is
often positive for occult blood.

In recent years, upper endoscopy has replaced the barium-contrast upper gastrointestinal study as the imaging modality of choice.
Endoscopy allows direct visualization and biopsy of the tumor.

At least four biopsies should be made of the lesion. With 10 biopsies, the diagnostic accuracy approaches 100%. In Japan, the double- P
contrast air/barium study is used extensively for screening. Once diagnosis is made, computed tomography is performed to evaluate
local extension and to look for evidence of ascites or metastatic disease.
STAGING
Staging for stomach cancer is according to TNM classification (Table 3-1).

TABLE 3-1 American Joint Committee on Cancer (AJCC) TNM Classification of Carcinoma of the
Stomach

Primary Tumor (T)

Tis        Carcinoma in situ: intraepithelial tumor without invasion of the lamina propria

T1        Tumor invades lamina propria or submucosa

T2        Tumor invades muscularis propria or subserosaa

T2a      Tumor invades muscularis propria

T2b      Tumor invades subserosa

T3        Tumor penetrates serosa (visceral peritoneum) without invasion of adjacent structuresb

T4        Tumor invades adjacent structuresb

Regional Lymph Nodes (N)

N0        No regional lymph node metastasisc

N1        Metastasis in 1 to 6 regional lymph nodes

N2        Metastasis in 7 to 15 regional lymph nodes

N3        Metastasis in more than 15 regional lymph nodes

Distant Metastasis (M)

M0        No distant metastasis

M1        Distant metastasis

Stage Grouping

Stage 0 Tis N0 M0                    


Stage IA T1 N0 M0

Stage IB T1 N1 M0
T2a/b N0 M0

Stage II T1 N2 M0
T2a/b N1 M0
T3 N0 M0

Stage IIIA T2a/b N2 M0


T3 N1 M0
T4 N0 M0

Stage IIIB T3 N2 M0

Stage IV T4 N1–3 M0
T1–3 N1–3 M0
Any T Any N M1

aA tumor may penetrate the muscularis propria with extension into the gastrocolic or gastrohepatic
ligaments, or into the greater or lesser omentum, without
perforation of the visceral peritoneum covering these structures. In this case, the tumor is classifie
as T2. If there is perforation of the visceral peritoneum
covering the gastric ligaments or the omentum, the tumor should be classified as T3.

bThe adjacent structures of the stomach include the spleen, transverse colon, liver, diaphragm,
pancreas, abdominal wall, adrenal gland, kidney, small
intestine, and retroperitoneum. Intramural extension to the duodenum or esophagus is classified by
the depth of the greatest invasion in any of these sites,
including the stomach.

cA designation of pN0 should be used if all examined lymph nodes are negative, regardless of the
total number removed and examined.

Reprinted with permission from the American Joint Committee on Cancer (AJCC), Chicago, Illinois.
Original source: AJCC Cancer Staging Manual. 6th ed.
New York, NY: Springer-Verlag; 2002.

TREATMENT
The theory behind curative resection involves en bloc primary tumor resection with wide disease-free margins and disease-free lymph
nodes. Tumors are located either in the proximal, middle, or distal stomach. The type of operation performed for cure depends on
tumor location. Distal lesions located in the antral or prepyloric area are treated with subtotal gastrectomy and Billroth II or Roux-en-Y
anastomosis (Fig. 3-7).

Figure 3-7 • Billroth II reconstruction after antral gastric cancer resection.

Midgastric and proximal lesions are treated with total gastrectomy, with extensive lymph node dissection. The lesser and greater
omentum are removed, along with the spleen. If the body or tail of the pancreas is involved, distal pancreatectomy can be performed.
Reconstruction is achieved via Roux-en-Y anastomosis (Fig. 3-8).
Figure 3-8 • Roux-en-Y esophagojejunostomy reconstruction after total gastrectomy.

Proximal lesions carry a poor prognosis, and surgical intervention is usually palliative. If there is extension into the distal esophagus, it
is resected, along with the cardia and lesser curvature. The remaining stomach tube is closed, and the proximal aspect is anastomosed
to the midesophagus through a right thoracotomy. If extensive esophageal involvement is discovered, radical near-total gastrectomy and
a near-complete esophagectomy are performed, with continuity restored using a distal transverse colon and proximal left colon
interposition (Fig. 3-9).

P.28 P
Figure 3-9 • Colonic interposition after near-total esophagectomy and near-total
gastrectomy.

PROGNOSIS
The overall 5-year survival rate for gastric cancer in the United States is approximately 10%. Based on pathologic staging of tumors, the
survival rate for stage I is 70%; stage II, 30%; stage III, 10%; and stage IV, 0%.

KEY POINTS
Peptic ulceration involves a compromised mucosal surface undergoing acid-peptic digestion. Causes include
Helicobacter pylori infection (90% of cases), non-steroidal anti-inflammatory drugs, alcohol, and tobacco.
Treatment consists of decreasing stomach acidity and enhancing mucosal protection. H. pylori is eradicated
with oral antibiotic therapy. Surgery is reserved for perforation, massive bleeding, gastric outlet obstruction, and
nonhealing ulcers.
Stress gastritis and ulceration are secondary to mucosal ischemia caused by hypotension and hypoperfusion.
Cushing's ulcers occur in patients with intracranial pathology, most probably secondary to neuronally mediated acid
hypersecretion.
Gastrinomas cause Zollinger-Ellison syndrome, which is seen in patients with severe peptic ulceration, elevated
serum gastrin levels, or evidence of a tumor within the "gastrinoma triangle." Diagnosis is confirmed by the
secretin-stimulation test. Medical treatment includes H2 blockade, proton pump inhibition, and somatostatin
analogs. Complete surgical resection can be curative.
Pathologic staging of gastric cancer is based on the depth of invasion and lymph node status. Most tumors are
located in the antral region. En bloc resection with Billroth II or Roux-en-Y anastomosis is usually performed.
Esophageal involvement requires esophagogastrectomy.
Authors: Karp, Seth J.; Morris, James P.G.; Zaslau, Stanley
Title: Blueprints Surgery, 5th Edition

Copyright ©2008 Lippincott Williams & Wilkins

> Table of Contents > Part II - Gastrointestinal and Abdominal > Chapter 4 - Small Intestine

Chapter 4
Small Intestine

ANATOMY AND PHYSIOLOGY


The small intestine comprises the duodenum, jejunum, and ileum and extends from the pylorus
proximally to the cecum distally. Its main function is to digest and absorb nutrients. Absorption is
achieved by the large surface area of the small intestine, secondary to its long length and
extensive mucosal projections of villi and microvilli. A broad-based mesentery suspends the
small intestine from the posterior abdominal wall once the retroperitoneal duodenum emerges
at the ligament of Treitz and becomes the jejunum. Arterial blood is supplied from branches of
the superior mesenteric artery, and venous drainage is via the superior mesenteric vein (Fig. 4-
1). The mucosa has sequential circular folds, called plicae circulares. The plicae circulares are
more numerous in the proximal bowel than in the distal bowel. The mucosal villi and microvilli
create the surface through which carbohydrates, fats, proteins, and electrolytes are absorbed
(Figs. 4-2 and 4-3).
Figure 4-1 • Anatomy of the small intestine demonstrating vascular
anatomy of the varying segments. Note longer vasa recta in jejunum
versus ileum.
From Lawrence PF, Bell RM, Dayton MT, et al. Essentials of General
Surgery. 4th ed. Philadelphia: Lippincott Williams & Wilkins, 2005.
Figure 4-2 • Structure of small intestinal villi.
Figure 4-3 • Diagram of a columnar epithelial intestinal absorptive
cell with luminal microvilli.

SMALL BOWEL OBSTRUCTION


Although the potential causes of small bowel obstruction (SBO) are varied, the presentation of
this disorder is usually quite consistent because of a common mechanism. Obstruction of the
small bowel lumen causes progressive proximal accumulation of intraluminal fluids and gas.
Peristalsis continues to transport swallowed air and secreted intestinal fluid through the bowel
proximal to the obstruction, resulting in small bowel dilation and eventual abdominal distention.
Depending on the location of the obstruction, vomiting occurs early in proximal obstruction and
later in more distal blockage (Fig. 4-4). Crampy abdominal pain initially occurs as active
proximal peristalsis exacerbates bowel dilation. With progressive bowel wall edema and luminal
dilation, however, peristaltic activity decreases and abdominal pain lessens. At presentation,
patients exhibit abdominal distention and complain of mild diffuse abdominal pain.

Figure 4-4 • Variable manifestations of small bowel obstruction


depend on the level of blockage.

ETIOLOGY
The first and second most common causes of SBO are adhesions and hernias, respectively (Table
4-1). Most adhesions are caused by postoperative internal scar formation. Discovering the actual
mechanism of obstruction is important for therapeutic planning, because the mechanism of
obstruction relates to the possibility of vascular compromise and bowel ischemia. For example, a
closed-loop obstruction caused by volvulus with torsion is at higher risk for vascular compromise
than an SBO from a simple adhesive band (Fig. 4-5).

TABLE 4-1 Causes of Small Bowel Obstruction

Adhesions

Hernias—abdominal wall, internal

Neoplasms—primary, metastatic

Obturation/strictures—ischemia,
radiation,

Crohn's disease, gallstone, bezoar

Intussusception

Meckel's diverticulum

Volvulus

Superior mesenteric artery syndrome

Intramural hematoma
Figure 4-5 • Volvulus.

From Willis MC. Medical Terminology: A Programmed Learning


Approach to the Language of Health Care. Baltimore: Lippincott
Williams & Wilkins, 2002.

A second mechanism causing bowel ischemia is incarceration in a fixed space. Incarceration and
subsequent strangulation impedes venous return, causing edema and eventual bowel infarction.
Other mechanisms of obstruction that rarely compromise vascular flow are intraluminal
obstruction by a gallstone or bezoar and intussusception caused by an intramural or mucosal
lesion at the leading edge.

HISTORY
Patients usually present with complaints of intermittent crampy abdominal pain, abdominal
distention, obstipation, nausea, and vomiting. Vomiting of feculent material usually occurs later
in the course of obstruction. Constant localized pain or pain out of proportion to
physical findings may indicate ischemic bowel and is a clear indication for urgent surgical P
exploration.

PHYSICAL EXAMINATION
A distended abdomen with diffuse midabdominal tenderness to palpation is usually found on
physical examination. Typically, there are no signs of peritonitis. If constant localized tenderness
is apparent, indicating localized peritonitis, then ischemia and gangrene must be suspected. An
essential aspect of the physical examination is to check for abdominal wall hernias, especially in
postsurgical patients. Elevation in temperature should not be present in uncomplicated cases.
Tachycardia may be present from hypovolemia secondary to persistent vomiting or from toxemia
caused by intestinal gangrene.
DIAGNOSTIC EVALUATION
Upright radiographs classically demonstrate distended loops of small bowel with multiple air–
fluid interfaces. Occasionally, the radiograph shows the etiology of the obstruction, the site of
obstruction, and whether the obstruction is partial or complete. Dilated small bowel in the
presence of a dilated colon suggests the diagnosis of paralytic ileus, not SBO. A small bowel
contrast study may be necessary to demonstrate transit of contrast into the colon, thereby ruling
out SBO. Free air indicates perforation of the intra-abdominal gastrointestinal tract, whereas
biliary gas and an opacity near the ileocecal valve indicate gallstone ileus.

Abdominal computed tomography (CT) scans can often demonstrate the transition point, where
the dilated bowel proximal to the point of obstruction transitions to the decompressed bowel
more distally. Also, in cases where the bowel has twisted on its mesentery, a "swirl sign" can be
seen as the mesenteric
vasculature twists on itself, creating a distinctive swirling radiographic pattern. P

Laboratory examination often reveals a hypokalemic alkalosis owing to dehydration from


repeated emesis. White blood cell count and amylase may be mildly elevated. Lactic acidosis is
cause for concern and may indicate intestinal necrosis.

TREATMENT
In decades past, most patients presenting with an SBO were taken directly to the operating room
for exploration to rule out intestinal necrosis. However, over the past few decades it became
apparent that most patients can be safely managed medically in the absence of peritonitis or
other worrisome clinical findings. Supportive therapy allows for spontaneous resolution of the
obstruction and return of normal bowel function. For patients who are candidates for a trial of
nonsurgical therapy, initial treatment consists of nasogastric decompression to relieve proximal
gastrointestinal distention and associated nausea and vomiting. Fluid resuscitation and
supportive hydration is necessary because patients are typically intravascularly depleted from
diminished oral fluid intake and vomiting.

The decision to operate is based on the nature of the obstruction and the patient's clinical P
condition. On initial presentation, if ischemia or perforation is suspected, immediate operation
is necessary. Otherwise, patients can be observed with serial physical examinations, serum tests,
and abdominal radiographs for evidence of resolution. If the patient's condition worsens or fails
to improve with supportive therapy, operative intervention is indicated.

CROHN'S DISEASE
Crohn's disease is a transmural inflammatory disease that may affect any part of the
gastrointestinal tract, from the mouth to the anus. Ileal involvement is most common. The
disease is characterized by skip lesions that involve discontinuous segments of abnormal mucosa.
Granulomata are usually seen microscopically, but not always. Areas of inflammation are often
associated with fibrotic strictures, enterocutaneous fistulae, and intra-abdominal abscesses, all
of which usually require surgical intervention.

EPIDEMIOLOGY
Crohn's disease occurs throughout the world, although the actual incidence exhibits a geographic
and ethnic variability. The incidence in the United States is approximately 10 times that of
Japan. Ashkenazi Jews have a far higher incidence of disease than do African Americans.

ETIOLOGY
The cause of Crohn's disease remains unknown. Because of the presence of granulomata,
mycobacterial infection has been postulated as the causative agent. Recent investigations with
Mycobacterium paratuberculosis have proved inconclusive. An immunologic basis for the disease
has also been advanced; however, although humoral and cellular immune responses are involved
in disease pathogenesis, no specific immunologic disturbance has been identified.

PATHOLOGY
The small intestine is affected in at least 70% of all patients with Crohn's disease. The ileum is
typically diseased, with frequent right colon involvement. On gross inspection, the serosal
surface of the bowel is hypervascular, and the mesentery characteristically shows signs of
"creeping fat." The bowel walls are edematous and fibrotic. The mucosa has a cobblestone
appearance, with varying degrees of associated mucosal ulceration (see Color Plates 2 and 3).

Histologically, a chronic lymphocytic infiltrate in an inflamed mucosa and submucosa is seen.


Fissure ulcers penetrate deep into the mucosa and are often associated with granulomata and
multinucleated giant cells. Granulomata are seen more frequently in distal tissues, which
explains why granulomata are seen more often in colonic disease than in ileal disease.

HISTORY
Patients with Crohn's disease of the small bowel present complaining of diarrhea, abdominal
pain, anorexia, nausea, and weight loss. The diarrhea is usually loose and watery, without frank
blood. Dull abdominal pain is usually in the right iliac fossa or periumbilical region.
Children often present with symptoms of malaise and have noticeable growth failure. Strictures P
may cause partial SBO, resulting in bacterial overgrowth and subsequent steatorrhea, flatus, and
bloating.

PHYSICAL EXAMINATION
Patients may appear to be either generally healthy or may have significant cachexia. Abdominal
examination may reveal right iliac fossa tenderness. In acutely ill patients, a palpable abdominal
mass may be present, indicating abscess formation. Enterocutaneous fistulae may be present.
Oral examination may reveal evidence of mucosal ulceration, whereas perianal inspection may
show skin tags, fissures, or fistulae. Extraintestinal manifestations include erythema nodosum,
pyoderma gangrenosum, ankylosing spondylitis, and uveitis.

DIAGNOSTIC EVALUATION
Blood studies often show a mild iron-deficiency anemia and a depressed albumin level. If
hypoalbuminemia is severe, peripheral edema may be present.

Small intestine Crohn's disease is diagnosed by bariumcontrast enteroclysis. This small intestine
enema technique provides better mucosal definition than standard small bowel follow-through
studies. This technique illustrates aphthoid ulcers, strictures, fissures, bowel wall thickening,
and fistulae. Fistulograms are helpful to define existing fistula tracks, and CT can localize
abscesses. Once radiographic evidence of disease is found, colonoscopy should be performed to
evaluate the colonic mucosa and to obtain biopsies of the terminal ileum.

DIFFERENTIAL DIAGNOSIS
In addition to the diagnosis of Crohn's disease, one should consider the possibility of acute
appendicitis, Yersinia infection, lymphoma, intestinal tuberculosis, and Behçet disease.

COMPLICATIONS
Crohn's disease carries a high morbidity and low mortality. Small bowel strictures secondary to
inflammation and fibrosis are common complications that present as obstructions. Fistulae from
small bowel to adjacent loops of small bowel, large bowel, bladder, vagina, or skin also occur.
Ileal Crohn's disease can result in gallstone formation because of the interruption of the
enterohepatic circulation of bile salts. Kidney stones may also form because of hyperoxaluria.
Normally, calcium and oxalate bind in the intestine and are excreted in the feces. With ileal
Crohn's disease, steatorrhea causes ingested fat to bind intraluminal calcium, thus allowing free
oxalate to be absorbed. Finally, adenocarcinoma is a rare complication that usually arises in the
ileum.

TREATMENT
Mild disease can be controlled with a 4- to 6-week course of sulfasalazine or mesalazine.
Alternatively, oral corticosteroids can be used with equivalent results. Metronidazole may also be
useful. Patients with bile salt–induced diarrhea after ileal resection may benefit from
cholestyramine.

Severe disease is treated with hospitalization, bowel rest, hydration, intravenous nutrition,
corticosteroids, and metronidazole. Patients with chronic active disease may benefit from a
course of mercaptopurine.
Surgery for Crohn's disease should only be performed for complications of the disease (Table 4-
2). Operation should be conservative and should address only the presenting indication, using
gentle surgical technique. Resections should be avoided, as overly aggressive intervention can
produce surgically induced short bowel syndrome and malnutrition. Some common surgical
problems encountered in Crohn's disease and its treatments include ileocolic disease, which is
managed by conservative ileocecal resection to grossly normal margins (Fig. 4-6); stricture,
managed by stricturoplasty, which entails incising the antimesenteric border of the stricture
along the intestinal long axis and then closing the enterotomy transversely (Fig. 4-7); and
abscess/fistula, which is managed by open or percutaneous drainage of the abscess and resection
of the small bowel segment responsible for initiating the fistula with primary anastomosis (Fig. 4-
8).

TABLE 4-2 Indications for Surgery in Crohn's Disease

Stricture with Obstructive Symptoms

Fistula

Abscess

Perforation

Bleeding

P
Figure 4-6 • Ileocecal resection for Crohn's disease.
Figure 4-7 • Stricturoplasty of a localized stricture with transverse
closure.

Figure 4-8 • Fistula resection with end-to-end anastomosis.

MECKEL'S DIVERTICULUM
This most common congenital anomaly of the small intestine is an antimesenteric remnant
arising from a failure of vitelline duct obliteration during embryonic development. Meckel's
diverticula are true diverticula affecting all three intestinal muscle layers. Diverticula are
usually <12 cm in length and are found within 100 cm of the ileocecal valve.

Associated abnormalities of the vitelline duct depend on the degree of duct obliteration that
occurs during development. Complete ductal obliteration leaves a thin fibrous band connecting
ileum to umbilicus, whereas complete duct persistence results in a patent ileoumbilical fistula.
Partial obstruction results in cyst or blind sinus formation (Fig. 4-9). Heterotopic tissue (gastric,
pancreatic) is found in 30% to 50% of diverticula.
Figure 4-9 • Vitelline duct remnants.

In the United States, Meckel's diverticulum is associated with the "Rule of 2s": it occurs in 2% of
the population, is located within 2 ft of the ileocecal valve, is usually 2 in long, contains two
types of heterotopic tissue (gastric, pancreatic, duodenal, or intestinal), and is the most
common cause of rectal bleeding in infants younger than 2 years.

COMPLICATIONS
Bleeding within the diverticulum may occur from peptic ulceration arising from heterotopic
gastric mucosa. In infants, Meckel's diverticulum is the most common cause of major lower
gastrointestinal bleeding.

Bowel obstruction may result from one of two mechanisms. Intussusception can occur when an
inverted diverticulum functions as a lead point, or small bowel volvulus can occur around a fixed
obliterated vitelline duct extending from the ileum to the umbilicus.

DIAGNOSTIC EVALUATION
For Meckel's diverticula containing heterotopic gastric mucosa, the technetium 99 (99Tc) scan is
helpful for diagnosis: pertechnetate anions are taken up by ectopic gastric parietal cells and
indicate diverticulum location. Diverticula that do not contain heterotopic gastric tissue can
occasionally be visualized using standard barium-contrast studies.

TREATMENT
Definitive treatment for Meckel's diverticulum complications is surgical resection. In adult
patients incidentally
found to have an asymptomatic Meckel's diverticulum during laparotomy, the diverticulum should P
be left in situ, as the chance of producing surgical morbidity and mortality are respectively 23
and five times higher for resection than when only symptomatic diverticula are removed.
SMALL BOWEL TUMORS
Tumors of the small bowel are rare, accounting for 1% to 5% of all gastrointestinal tumors. Most
tumors are benign. Common benign neoplasms of the small bowel include tubular and villous
adenomas, lipomas, leiomyomas, and hemangiomas. Telangiectasias of Rendu- Osler-Weber
syndrome, neurofibromas of neurofibromatosis, hamartomatous polyps of Peutz-Jeghers
syndrome, and heterotopic tissue as in Meckel's diver-ticulum are also found. Possible
explanations for this lack of malignancy include short exposure to ingested carcinogens
secondary to rapid transit time, low bacterial counts resulting in fewer endogenously produced
carcinogens, and the intraluminal secretion of IgA by small bowel mucosa.

Benign lesions are usually asymptomatic and are incidental findings. Of symptomatic lesions,
obstruction is the most common presentation, followed by hemorrhage. In the workup of
gastrointestinal bleeding, however, unless other evidence exists, small bowel lesions should be
low on the list of differential diagnoses, because >90% of bleeding lesions occur between the
esophagus and distal duodenum and between the ileocecal valve and anus. Small bowel lesions
should be suspected if careful skin examination reveals café-au-lait spots (neurofibromatosis),
telangiectasia (Rendu- Osler-Weber syndrome), or mucocutaneous pigmentation (Peutz-Jeghers
syndrome).

Malignant tumors of the small bowel typically present with obstruction or bleeding. The four
major malignant tumors are adenocarcinoma, gastrointestinal stromal tumors, carcinoid, and
lymphoma.

DIAGNOSTIC EVALUATION
Visual endoscopic identification of small bowel tumors is usually possible for lesions of the
proximal duodenum and terminal ileum. The remainder of the small bowel requires examination
by barium-contrast studies. For larger lesions, CT may be helpful.

In situations involving active hemorrhage, 99Tc sulfur colloid or 99Tc-labeled red blood cell
studies may show the bleeding site. However, a bleeding rate of 1 mL/min is required for
accurate localization.

When available diagnostic modalities are insufficient, exploratory laparotomy may be necessary.
In addition to external inspection at laparotomy, operative endoscopy can be used for
intraluminal evaluation.

CARCINOID TUMORS
Carcinoid tumors are the most common endocrine tumors of the gastrointestinal tract,
constituting more than half of all such lesions. They account for up to 30% of all small bowel
tumors. Carcinoid tumors arise from neuroendocrine enterochromaffin cells. Hence tumors can
secrete serotonin and other humoral substances, such as histamine, dopamine, tachykinins,
peptides, and prostaglandins. The metabolite of serotonin, 5-hydroxyindoleacetic acid, is
excreted in the urine and is easily detected.

All carcinoids are considered malignant because of their potential for invasion and metastasis.
Patients with metastatic disease manifest the carcinoid syndrome, which consists of the systemic
effects (flushing, diarrhea, sweating, and wheezing) of secreted vasoactive substances. Presence
of the carcinoid syndrome indicates hepatic metastasis, because systemic
effects occur when venous drainage from a tumor escapes hepatic metabolism of vasoactive P
substances.

Approximately 85% of carcinoid tumors are found in the intestine; of these, approximately 50%
are found in the appendix, making it the most common site of occurrence, followed by the
ileum, jejunum, rectum, and duodenum (see Color Plate 4). Other sites of disease include the
lungs and occasionally the pancreas and biliary tract. Appendiceal carcinoids rarely metastasize,
whereas lesions of the ileum have the highest association with carcinoid syndrome. Jejunoileal
carcinoids are frequently multicentric.

HISTORY
The clinical presentation of patients with carcinoid tumors differs depending on tumor location.
Primary tumors may present as SBO, because tumors can incite an intense local fibrosis of the
bowel that causes angulation and kinking of the involved segment. As noted, metastatic disease
with hepatic spread manifests as the carcinoid syndrome. Occult primary lesions do not cause
systemic effects because 5-hydroxytryptamine (serotonin) is metabolized by the liver. Other
presenting symptoms can include abdominal pain, upper intestinal or rectal bleeding,
intussusception, weight loss, or a palpable abdominal mass.

DIAGNOSTIC EVALUATION
Laboratory studies should include plasma and urine analysis to evaluate for elevated levels of
plasma serotonin and urinary 5-hydroxyindoleacetic acid. Barium-contrast studies are also useful
for diagnosing carcinoid tumors. Barium enemas can demonstrate lesions of the rectum and large
bowel, whereas small bowel enteroclysis may show a discrete lesion or a stricture secondary to
fibrosis. Because primary tumors are usually small, CT is usually helpful only for detecting
hepatic metastases. Colonoscopy can show tumors from the terminal ileum to the rectum.

Because neuroendocrine tumors often express functional receptors, radiolabeled octreotide


imaging can be useful in detecting occult disease. Octreotide scanning is based on physiologic
function, rather than on detectable anatomic alterations, and may have better diagnostic
sensitivity than conventional imaging modalities.

TREATMENT
Surgical resection of the primary tumor is always undertaken, even in cases of metastatic
disease. If the tumor is left in situ, bowel obstruction and intussusception ultimately result. At
laparotomy, adequate bowel and mesenteric margins must be obtained, as with any cancer-
related operation. Depending on tumor size and the degree of spread, lesions can be treated
with simple local excision for small primaries to wide en bloc resection for metastatic disease.

Patients who have carcinoid syndrome can achieve symptomatic relief with subcutaneous
injections of somatostatin analogs (e.g., octreotide). Induction with general anesthesia may
provoke a life-threatening carcinoid crisis characterized by hypotension, flushing, tachycardia,
and arrhythmias. Intravenous somatostatin or octreotide rapidly reverses the crisis.

PROGNOSIS
Carcinoid tumors are relatively indolent, slow-growing neoplasms. Prognosis for patients with
carcinoid tumors is directly related to the size of the primary tumor and to the presence of
metastasis.

For noninvasive lesions of the appendix and rectum <2 cm in size, the 5-year survival rate nears
100%. As the tumor size increases, the survival rate decreases. The presence of muscle wall
invasion and positive lymph nodes are poor prognostic signs.

Patients with hepatic metastases have an average survival of approximately 3 years. Liver lesions
are usually multiple. Because incapacitating symptoms of the carcinoid syndrome are
proportional to tumor bulk, cytoreductive surgery can ameliorate symptoms, as well as prolong
survival. Nonsurgical palliation is achieved with somatostatin analog therapy or
chemoembolization of the tumor.

KEY POINTS
Small bowel obstruction is commonly caused by adhesions and hernias.
Bowel infarction occurs with closed-loop obstruction and strangulation.
Patients with peritonitis require immediate surgery. Many patients are
successfully managed with supportive therapy alone. Surgery is indicated if the
obstruction fails to resolve spontaneously.
Crohn's disease is a transmural inflammatory process that affects any part of
the gastrointestinal tract, from mouth to anus. The ileum is most commonly
involved. Surgical treatment is reserved mainly for complications. P.3
Meckel's diverticulum is the most common congenital abnormality of the small
intestine and arises from a failure of vitelline duct obliteration. It is a true
diverticulum and may contain heterotopic gastric and pancreatic tissue. Peptic
ulceration with hemorrhage may develop. It is the most common cause of major
lower gastrointestinal bleeding in infants.
Small bowel tumors are rare and usually benign, often presenting as small
bowel obstructions. Benign tumors include adenomas, lipomas, leiomyomas, and
hemangiomas. Malignant tumors include adenocarcinoma, gastrointestinal
stromal tumors, carcinoid, and lymphoma.
Carcinoid tumors are the most common endocrine tumors of the gastrointestinal
tract and most frequently occur in the appendix. All carcinoid tumors are
considered malignant because of their potential for invasion and metastasis.
Carcinoid tumors secrete serotonin, which is broken down in the liver to the
metabolite 5-hydroxyindoleacetic acid, which is, in turn, excreted in the urine.
Carcinoid syndrome, manifested by flushing, diarrhea, sweating, and wheezing,
invariably indicates hepatic metastases, because vasoactive substances have
escaped hepatic metabolism.
Carcinoid syndrome is treated with somatostatin analogs and
chemoembolization to provide symptomatic relief
Authors: Karp, Seth J.; Morris, James P.G.; Zaslau, Stanley
Title: Blueprints Surgery, 5th Edition

Copyright ©2008 Lippincott Williams & Wilkins

> Table of Contents > Part II - Gastrointestinal and Abdominal > Chapter 5 - Colon

Chapter 5
Colon

ANATOMY AND PHYSIOLOGY


The colon begins at the ileocecal valve and extends distally to the anal canal. Its primary
function is the reabsorption of water and sodium, secretion of potassium and bicarbonate, and
storage of fecal material. The ascending and descending colon are fixed in a retroperitoneal
location, whereas the transverse and sigmoid colon are intraperitoneal.

Arterial supply to the cecum, ascending colon, and transverse colon is from the superior
mesenteric artery by way of the ileocolic, right colic, and middle colic arteries. The remainder
of the colon is supplied by the inferior mesenteric artery by way of the left colic, sigmoid, and
superior hemorrhoidal arteries and the middle and inferior hemorrhoidal arteries that arise from
the internal iliac artery. The inconstant anastomotic artery between the middle colic of the
superior mesenteric artery and left colic of the inferior mesenteric artery is called the
anastomosis (or arc) of Riolan. The interconnecting arcades in closer proximity to the mesenteric
border of the colon are referred to as the marginal artery of Drummond (Fig. 5-1). This
amalgamation of anastomotic branches runs around the medial margin of the entire colon, from
the ileocolic artery to the sigmoid arteries. Venous drainage from the colon includes the superior
and inferior mesenteric veins. The inferior mesenteric vein joins the splenic vein, which joins
the superior mesenteric vein to form the portal vein. In this way, mesenteric blood flow enters
the liver, where it is detoxified before entering the central circulation. Lymphatic drainage
follows the arteries and veins.
Figure 5-1 • The blood supply to the colon originates from the
superior and inferior mesenteric arteries.

From Corman RL. Colon and Rectal Surgery. 5th ed. Philadelphia:
Lippincott Williams & Wilkins, 2004.

ULCERATIVE COLITIS
Ulcerative colitis is an inflammatory disease of the colon with unknown cause. An autoimmune
basis is suspected. Inflammation almost always involves the rectum and extends proximally
toward the cecum to varying degrees. The small bowel is uninvolved except in cases of
"backwash ileitis" that may occur in proximal colonic disease. Extracolonic manifestations
include inflammatory eye and skin disorders, arthritis, blood disorders, and sclerosing
cholangitis.

PATHOLOGY
Inflammation is confined to the mucosa and submucosa. Superficial ulcers, thickened mucosa,
crypt abscesses, and pseudopolyps may also be present.

EPIDEMIOLOGY
The incidence is six per 100,000. It is more common in developed countries, especially among
Caucasians and the Jewish population. There is no predilection for sex. Approximately 20% of
patients have first-degree relatives who are affected, suggesting a genetic basis. Linkage
analysis has identified an association with HLA-DR2.

HISTORY
Most patients usually present in the second through fourth decade of life. Patients commonly
complain
of bloody diarrhea, mucus/pus per rectum, fever, abdominal pain, and weight loss. A history of P
repeated attacks is common. Numerous diseases are associated with ulcerative colitis, including
sclerosing cholangitis in 1% of patients, as well as arthritis, iritis, cholangitis, aphthous ulcers,
pyoderma gangrenosum, erythema nodosum, hemolytic anemia, and ankylosing spondylosis.
These diseases may be part of the initial presentation.

PHYSICAL EXAMINATION
Abdominal pain is common. Rectal tenderness may occur with rectal fissures. The disease may
present with abdominal distention as evidence of massive colonic distention, a situation known
as toxic megacolon. This may progress to frank perforation with signs of peritonitis.

DIAGNOSTIC EVALUATION
Plain films may show massive colonic dilation, indicating toxic megacolon. Perforation will result
in air under the diaphragm. Barium enema may reveal a "stovepipe colon" owing to loss of
haustral folds, as well as mucosal ulcerations.

Endoscopy demonstrates thickened friable mucosa. Fissures and pseudopolyps, if present, almost
always involve the rectum and varying portions of the colon. Biopsy shows ulceration limited to
the mucosa and submucosa. Crypt abscesses arising from the crypts of Lieberkuhn coalesce to
form ulcerations.
COMPLICATIONS
Perforation and hemorrhage may occur during a severe attack. Obstruction may develop from
stricture as a
result of chronic inflammation. Toxic megacolon is uncommon but is life-threatening if not P
controlled with medical therapy. Severe inflammation causes destruction of the myenteric plexus
and muscular layer, leading to massive distention and perforation. Patients are invariably septic
and mortality high unless emergent subtotal colectomy is performed. Colon cancer occurs
frequently, with a risk of approximately 10% within 20 years. Once the diagnosis of ulcerative
colitis is made, routine colonoscopic surveillance is mandatory.

TREATMENT
Initial therapy is medical, with fluid administration, electrolyte correction, and parenteral
nutrition if necessary. Corticosteroids, other immunosuppressives, and sulfasalazine are all
effective. Topical mesalamine, in the form of enemas, is effective for mild and moderate
disease. Newer immunosuppressive agents—including infliximab, a monoclonal antibody against
tumor necrosis factor—may be useful. High-fiber diet and bulking agents are often useful.

Indications for surgery include colonic obstruction, massive blood loss, failure of medical
therapy, toxic megacolon, and cancer. The recommendation of prophylactic colectomy for these
patients is being reconsidered on the basis of recent data that suggest the incidence of cancer is
not as high as once thought. When elective surgery is performed, sphincter-sparing operations
allow the ileum to be anastomosed to the rectal stump or anus, preserving continence and bowel
movement. The ileum is fashioned into a J-pouch, which serves the fecal reservoir role of the
removed rectum.

DIVERTICULOSIS
Diverticulosis refers to the presence of diverticula, outpouchings of the colon that occur at
points where the arterial supply penetrates the bowel wall (singular, diverticulum; plural,
diverticula) (Fig. 5-2). These are acquired or false diverticula because not all layers of the bowel
wall are included. Most diverticula occur in the sigmoid colon (Figs. 5-3 and 5-4). Diverticulosis is
the most common cause of lower gastrointestinal hemorrhage, usually from the right colon. Of
people with diverticulosis, 15% will have a significant episode of bleeding.
Figure 5-2 • Blood supply to the colon (A) and formation of the
diverticulum (B). Note the passage of the mucosal diverticulum
through the muscle coat along the course of the artery.
From Snell RS. Clinical Anatomy. 7th ed. Philadelphia: Lippincott
Williams & Wilkins, 2003.
Figure 5-3 • Diverticulosis, diverticulitis.
Reprinted with permission from Willis MC. Medical Terminology: The
Language of Health Care. 1st ed. Baltimore: Williams & Wilkins;
1996:374.
Figure 5-4 • Extensive sigmoid diverticular disease with slight
spasm but no stigmata of acute inflammation. In the absence of
classic symptoms and signs of diverticulitis, surgery is not advised
solely on the basis of this radiographic appearance.
From Corman RL. Colon and Rectal Surgery. 5th ed. Philadelphia:
Lippincott Williams & Wilkins, 2004.

EPIDEMIOLOGY
Diverticular disease is common in developed nations and is likely related to low-fiber diets.
Because of reduced intraluminal stool volume, the normal segmental colonic peristaltic
contractions are extra forceful, which increases intraluminal pressure and causes herniation of
the mucosa through the circular muscles of the bowel wall where the marginal artery branches
penetrate. Men and women are equally affected, and the prevalence increases dramatically with
age. Approximately one third of the population has diverticular disease, but this number
increases to more than half of those older than 80 years of age.

HISTORY
Patients usually present with bleeding from the rectum without other complaints. They may have
had previous episodes of bleeding or crampy abdominal pain, commonly in the left lower
quadrant.

DIAGNOSTIC EVALUATION
For patients who stop bleeding spontaneously, elective colonoscopy should be performed to
determine the cause of the bleeding. If bleeding continues, diagnostic
and therapeutic modalities include radioisotope bleed- ing scans, which have variable success P
rates, and mesenteric angiography, which has an excellent success rate in the presence of active
bleeding.

TREATMENT
Asymptomatic individuals require no treatment. In the event of a bleed, 80% will stop
spontaneously. Elective segmental or subtotal colectomy is not usually recommended at first
episode. However, depending on the ability to accurately determine the site of bleeding, the
severity of the initial bleeding episode, and the general status of the patient, it may be
indicated. Patients with recurrent bleeding are usually offered surgical resection. Active
bleeding is treated colonoscopically if the colon can be cleaned and the bleeding site identified.
Embolization of the bleeding vessel may be possible using selective angiography. In the face of
massive bleeding, if the above methods fail and no bleeding site is identified, emergent subtotal
colectomy is performed. Before embarking on such an irreversible procedure, which involves
removing most of the colon, it is of utmost importance to ensure that the bleeding source is not
from hemorrhoids or a rectal source. If a colonic bleeding site is identified, segmental colectomy
can be performed, usually based on the arterial branch feeding the bleeding site.
P
DIVERTICULITIS
The narrow neck of a diverticulum predisposes it to infection, which occurs either from
increased intraluminal pressure or inspissated food particles. Infection leads to localized or free
perforation into the abdomen. Diverticulitis most commonly occurs in the sigmoid and is rare in
the right colon. Approximately 20% of patients with diverticula experience an episode of
diverticulitis. Each attack makes a subsequent attack more likely and increases the risk of
complications.

HISTORY
Patients usually present with left lower quadrant pain; right-sided diverticulitis causes right-
sided pain but is less common. The pain is usually progressive over a few days and may be
associated with diarrhea or constipation.

PHYSICAL EXAMINATION
Abdominal tenderness, usually in the left lower quadrant, is the most common finding. Local
peritoneal signs of rebound and guarding may be present. Significant colonic inflammation may
present as a palpable mass. Diffuse rebound tenderness and guarding as evidence of generalized
peritonitis suggests free intra-abdominal perforation.

DIAGNOSTIC EVALUATION
Elevation of the white blood cell count is usual. Radiographs of the abdomen are typically
normal, except for cases of perforation or obstruction. In cases of perforation, free air is seen
under the diaphragms on chest x-ray. Computed tomography (CT) may demonstrate pericolic fat
stranding, bowel wall thickening, or abscess. Colonoscopy and barium enema should not be
performed during an acute episode because of the risk of causing or exacerbating an existing
perforation.

COMPLICATIONS
Stricture, perforation, or fistulization with the bladder, skin, vagina, or other portions of the
bowel may develop.

TREATMENT
Most episodes of diverticulitis are mild and can be treated on an outpatient basis with broad-
spectrum oral antibiotics. Combination treatment with cipro-floxacin and metronidazole (Flagyl)
is appropriate to cover aerobic and anaerobic organisms. For severe cases or cases in older adult
patients or debilitated patients, hospitalization with bowel rest and broad-spectrum intravenous
antibiotics (e.g., ampicillin, ciprofloxacin, and metronidazole) are required. For patients who do
not improve in 24 to 48 hours, repeat CT scan with percutaneous drainage of any identifiable
abscess cavity may obviate the need for emergency operation. In the event of free perforation or
failure of medical management, surgical exploration with resection and colostomy is usually
required (Hartmann procedure; Fig. 5-5). In addition, surgical intervention is indicated in the
presence of the complications previously described. With repeated attacks of diverticulitis, the
risk of developing complications increases significantly.

Figure 5-5 • The Hartmann procedure for diverticulitis: primary


resection for diverticulitis of the colon. The affected segment
(clamp attached) has been divided at its distal end. In a primary
anastomosis, the proximal margin (dotted line) is transected and
the bowel attached end-to-end. In a two-stage procedure, a
colostomy is constructed at the proximal margin with the distal
stump oversewn (Hartmann procedure, as shown) or brought to the
outer surface as a mucous fistula. The second stage consists of
colostomy takedown and anastomosis.
From Smeltzer SC, Bare BG. Brunner & Suddarth's Textbook of
Medical-Surgical Nursing. 9th ed. Philadelphia: Lippincott Williams
& Wilkins, 2000.
COLONIC NEOPLASMS
Recent evidence suggests that colon cancer follows an orderly progression in which adenomatous
polyps undergo malignant transformation over a variable time
period (Fig. 5-6). For this reason, these polyps are considered premalignant lesions. Fifty percent P
of carcinomas have a ras gene mutation, whereas 75% have a p53 gene mutation.
Figure 5-6 • Model of colorectal carcinogenesis. (Redrawn from
Fearon ER, Vogelstein B. A genetic model of colorectal cancer
tumorigenesis. Cell 1990;61:759.)
From Corman RL. Colon and Rectal Surgery. 5th ed. Philadelphia:
Lippincott Williams & Wilkins, 2004.

EPIDEMIOLOGY
Colon cancer is the second most common cause of cancer-related death in the United States.
Risk factors include high-fat and low-fiber diets, age, and family history. Ulcerative colitis,
Crohn's disease, and Gardner syndrome all predispose to cancer, and cancer develops in all
patients with familial polyposis coli if they are not treated.

PATHOLOGY
Adenomatous polyps are either tubular or villous, with some lesions exhibiting features of both.
The higher the villous component, the higher the risk of malignancy. As the lesion grows in size,
the likelihood of its having undergone malignant transformation increases significantly. Although
tubular adenomas <1 cm contain malignancy in only 1% of cases, lesions >2 cm contain
malignancy 25% of the time. For villous adenomas, the numbers are 10% and 50%, respectively.
Ninety percent of colon cancers are adenocarcinomas, and 20% of these are mucinous, carrying
the worst prognosis. Other types include squamous, adenosquamous, lymphoma, sarcoma, and
carcinoid. Three percent of tumors are synchronous (occurring simultaneously), and
metachronous tumors (multiple primary cancers developing at intervals) also occur in 3% of
cases.

SCREENING
Screening is aimed at detecting polyps and early malignant lesions. In theory, colon cancer is a
preventable disease, because if all patients underwent thorough screening and timely polyp
removal, the mortality rate from colon cancer would be drastically reduced. The current
screening recommendations from the American Gastroenterological Association divide people
into two groups: average risk and increased risk. Average-risk persons lack any identifiable risk
factors. Increased-risk persons have either a personal history of adenomatous polyps or
colorectal cancer, first-degree relatives with colorectal cancer or adenomatous polyps, a family
history of multiple cancers, or a history of inflammatory bowel disease. Screening should begin
at age 50 years for average-risk patients and age 40 years for increased-risk patients. American
Cancer Society guidelines for the early detection of colorectal cancer include the following:

yearly fecal occult blood test or fecal immunochemical test


flexible sigmoidoscopy every 5 years P

yearly fecal occult blood test or fecal immunochemical test, plus flexible sigmoidoscopy every
5 years

double-contrast barium enema every 5 years

colonoscopy every 10 years

Patients with positive test results should be followed up with colonoscopy.

STAGING
Generations of medical students have been confused by the various staging systems used for
classifying colon cancer. Although the Dukes classification system devised in 1932 was simple and
uncomplicated, it was eventually found to be inferior with respect to prognostication than the
subsequently developed Astler-Coller system. Since 1991, the American Society of Colon and
Rectal Surgeons have endorsed the TNM staging system, which has become the standard for
modern cancer staging (Fig. 5-7).

Figure 5-7 • Understanding cancer: the stages of cancer (I-IV).


The TNM (tumor, nodes, metastases) classification system is as follows:

Tis: Carcinoma in-situ

T1: Tumor invades submucosa.

T2: Tumor invades muscularis propria.

T3: Tumor invades through the muscularis propria into the subserosa or into the pericolic or
perirectal tissue.

T4: Tumor directly invades other organs or structures and/or perforates the visceral
peritoneum.

N0: No regional lymph node metastasis.

N1: Metastasis in one to three regional lymph nodes.

N2: Metastasis in four or more regional lymph nodes.

M0: No distant metastasis or residual tumor.

M1: Distant metastasis present.

Staging is based on a combined evaluation of characteristics involving the tumor, lymph nodes,
and presence of metastasis.

HISTORY
The clinical presentation of colon cancer is often dependent on the location of the lesion. Small
proximal ascending colonic neoplasms are often asymptomatic. Occult blood in the stool and
weight loss from metastatic disease may be the only signs. As the size of a lesion increases, right
colon cancers usually cause bleeding that is more significant, whereas lesions in the left colon
typically present with obstructive symptoms, including a change in stool caliber, tenesmus, or
constipation. In general, this is due to fecal matter entering the right colon in liquid form and
easily transiting a large cecal lesion, whereas desiccated stool in the left colon tends to obstruct
when confronted with malignant luminal narrowing.

Rectal bleeding from a low rectal cancer should never be mistakenly explained away as
symptomatic hemorrhoids. Simple digital rectal examination will demonstrate the tumor and
prevent delay in diagnosis. Rectal cancer also can present with passage of mucus per rectum,
arising from tumor surface secretions.

Complete acute large bowel obstruction may also occur. Any older adult patient who lacks a
history of prior abdominal surgery or recent colonoscopy who presents with a large bowel
obstruction must be considered to have obstructing colon cancer until proven otherwise. Any
sizable lesion may produce abdominal pain. Perforation typically causes frank peritonitis.
Constitutional symptoms, including weight loss, anorexia, and fatigue, are common with
metastatic disease.
PHYSICAL EXAMINATION
Rectal examination may reveal occult or gross blood, and for low rectal cancers, the lesion can
be directly palpated. For large bulky tumors, a mass may be noted on abdominal examination.
Stigmata of hereditary disorders, including familial polyposis syndrome or Gardner syndrome,
may be present.

DIAGNOSTIC EVALUATION
Laboratory evaluation should include a hematocrit, which often reveals microcytic anemia from
chronic occult blood loss. The liver is the most common site
for metastases, and liver function tests may be abnormal. Carcinoembryonic antigen may also be P
obtained; although it is not a useful screening test, it is valuable as a marker for recurrent
cancer.

Colonoscopy has the advantage of examining the entire colon while also performing confirmatory
biopsy for tissue diagnosis. Flexible sigmoidoscopy reaches up to 70 cm of the most distal large
intestine, whereas colonoscopy can examine the entire colon and even intubate the distal ileum.
Approximately 70% of lesions should be detected by flexible sigmoidoscopy. Rigid sigmoidoscopy
is only useful for examining the lower 25 cm and is therefore often used to evaluate rectal
cancers.

Radiologic evaluation can be performed with double-contrast barium enema, which uses both a
radio-opaque contrast medium (barium) to coat the colon wall and air to provide luminal
distention. The classic finding on barium enema is a constricting filling defect, known as an
apple core lesion (Fig. 5-8). CT is useful for evaluating extent of disease and the presence of
metastases, particularly in the liver.
Figure 5-8 • Adenocarcinoma of the colon presenting as an "apple
core" lesion. Image from a barium enema study demonstrates a
circumferential mass (arrows). This mass has disrupted the normal
mucosal pattern and has irregular overhanging edges.

From Kelsen DP, Daly JM, Kern SE, et al. Gastrointestinal Oncology:
Principles and Practice. Philadelphia: Lippincott Williams & Wilkins,
2002.
Magnetic resonance imaging may be better for evaluating liver metastases but usually does not
add more overall information than that which is obtained with CT.

Positron-emission tomography scan is useful for showing metastatic disease or else late
recurrence in a patient who previously underwent resection and who has an increasing
carcinoembryonic antigen level. For rectal lesions, endorectal ultrasound is the standard of care
for assessing the depth of tumor invasion and the presence of lymph node metastases.

TREATMENT
Surgical therapy of colon cancer is based on complete removal of the malignant lesion and
associated lymph nodes. The oncologic principles underlying segmental colon resection for
malignancy are based on the blood supply of the segment of colon containing the lesion, as well
as the distribution of the parallel draining lymph node network. For cancers of the cecum and
ascending colon, right hemicolectomy is indicated. Tumors of the transverse colon require
transverse colectomy, with removal of the hepatic and splenic flexures. Descending colon tumors
require left colectomy, and sigmoid tumors are treated with sigmoidectomy. Most rectal tumors
are treated with low anterior resection, whereas the very low rectal cancers near the anus
occasionally require abdominoperineal resection, which entails resection of the anus with
closure of the perianal skin and creation of a permanent end colostomy, because anastomosis
may not be technically feasible. Examples of the extent of resection for different types of
colectomy are shown in Figure 5-9.
Figure 5-9 • Types of colectomy. (A) Sigmoid colectomy; (B)
transverse colectomy; (C) left colectomy; (D) right colectomy.

Adapted from Kelsen DP, Daly JM, Kern SE, et al. Gastrointestinal
Oncology: Principles and Practice. Philadelphia: Lippincott Williams
& Wilkins, 2002.

Historically, open surgery has been the standard approach for colon resection; however, the
laparoscopic technique has gained rapid acceptance, given the reduced morbidity compared with
open surgery, in addition to studies showing the less invasive approach to be equally effective as
open surgery in terms of survival. The widely quoted randomized trial results by the Clinical
Outcomes of Surgical Therapy (COST) Study Group, published in 2004, showed no difference in
either recurrence or 3-year survival between laparoscopic or open groups. These findings were
subsequently supported by other randomized trials, such as the Conventional Versus
Laparoscopic-Assisted Surgery in Colorectal Cancer (CLASSIC) trial from the United Kingdom,
published in 2007. In summary, despite the development of new surgical techniques, the basic
oncologic
goals and extent of resection should be identical, regardless of the approach. P

With respect to the extent of resection, guidelines from the American Joint Committee on
Cancer, the American College of Pathology, and the National Comprehensive Cancer Network
recommend 12 or more lymph nodes to be sampled during surgery. Thorough sampling and
examination of the draining lymph nodes is thought to improve staging accuracy, which allows
more appropriate adjuvant chemotherapy administration. By upstaging patients, some
investigators believe patients will therefore be offered more aggressive treatment that will likely
result in improved overall survival.

COLECTOMY: THE OPERATION


Traditional preoperative preparation has included mechanical and antimicrobial bowel cleansing;
however, this practice is currently undergoing critical review (see Preoperative Issues in Chapter
1 for expanded discussion). Most open resections are performed via a midline incision. The
rationale and extent of excision for various tumors is described above and in Figure 5-9.

Mobilization of the right or left colon involves incising the white line of Toldt on the respective
side. Care is taken to avoid the ureter, which can be injured as the colon is mobilized.
Consideration of a
ureteral stent should be made if the tumor is bulky and there is concern about identifying the P
ureter intraoperatively. The transverse colon is intraperitoneal and does not require
mobilization. Once adequate length of colon has been mobilized, the peritoneum overlying the
mesentery is incised to its root, and all the mesenteric vessels in the specimen are li-gated. In
the open technique, noncrushing clamps are usually placed alongside the resection margin to
reduce spillage, and the ends of the bowel are usually stapled and the specimen removed.
Reconstruction of bowel continuity is performed with either hand-sewn or stapled anastomosis.
For low colon or rectal anastomosis, use of an end-to-end anastomosis stapler placed through the
anus is a preferred technique.

ANGIODYSPLASIA
Angiodysplasia is being recognized with increasing frequency as a significant source of lower
gastrointestinal hemorrhage. These anomalous vascular lesions are histologically similar to
telangiectasia and arise most commonly in the cecum and right colon.

EPIDEMIOLOGY
Angiodysplasia is one of the most common causes of lower gastrointestinal bleeding. The
prevalence increases with age, to an incidence of approximately one fourth of the older adult
population. Age and resulting bowel wall strain are thought to cause vascular tissue
proliferation, leading to angiodysplastic lesions.
HISTORY
Patients usually present with multiple episodes of low-grade bleeding. In 10% of cases, patients
present with massive bleeding.

DIAGNOSTIC EVALUATION
Diagnosis can be made with arteriography, nuclear scans, or endoscopy.

TREATMENT
Endoscopic treatment includes electrocautery and argon plasma coagulation. Angiography with
highly selective embolization or vasopressin infusion is often effective. Because many
angiodysplastic lesions rebleed, definitive treatment may occasionally require segmental
colectomy.

VOLVULUS
Volvulus occurs when a portion of the colon rotates on the axis of its mesentery, compromising
blood flow and creating a closed-loop obstruction (Fig. 5-10). The sigmoid colon (75%) and cecum
(25%) are most commonly involved. The relative redundancy of the sigmoid loop causes torsion
around the mesenteric axis, whereas poor fixation of the cecum in the right iliac fossa leads to
either axial torsion (cecal volvulus) or anteromedial folding (cecal bascule).

Figure 5-10 • Volvulus.


EPIDEMIOLOGY
The incidence of volvulus is approximately two in 100,000. Risk factors include age, chronic
constipation, previous abdominal surgery, and neuropsychiatric disorders.

HISTORY
The patient usually relates the acute onset of crampy abdominal pain and distention.

PHYSICAL EXAMINATION
The abdomen is tender and distended, and peritoneal signs of rebound and involuntary guarding
may be present. Frank peritonitis and shock may follow.

DIAGNOSTIC EVALUATION
Abdominal radiographs may reveal a massively distended colon with a "corkscrew" or "bird's beak"
at
the point of torsion. The distended colonic loop has the appearance of a bent tire or large coffee P
bean.

TREATMENT
Sigmoid volvulus may be reduced by enemas or endo-scopy. Rectal tubes are sometimes used to
prevent acute recurrence and aid decompression. Because of the high rate of recurrence,
operative repair after resolution of the initial episode is recommended. In the acute setting and
depending on the operative findings, fixation of the untwisted loop to the respective fossa may
suffice for cases of viable bowel; otherwise, resection is performed with either primary
anastomosis or end colostomy (Hartmann procedure) in cases of sepsis and gangrene. Treatment
of cecal volvulus is usually operative at the outset, because nonoperative intervention is rarely
successful, and the incidence of gangrenous ischemic changes is high.

APPENDICITIS
Appendicitis is the most common reason for urgent abdominal operation. The causes of
appendiceal inflam- mation and infection are related to processes that obstruct the appendiceal
lumen, thereby causing distal swelling, decreased venous outflow, and ischemia. The most
common extraluminal cause of obstruction is the swelling of submucosal lymphoid tissue in the
wall of the appendix in response to a viral infection. This is illustrated by the incidence of viral
syndromes often seen in pediatric patients shortly before developing appendicitis. The most
common intraluminal cause of obstruction is from a fecalith (small, firm ball of stool). Cases of
obstruction with fecaliths have a higher incidence of perforation.
EPIDEMIOLOGY
Children and young adults between ages 5 and 35 years are most commonly affected.
Appendicitis will develop in approximately 5% of people over their lifetime. Perforation at the
time of surgery is more often seen in very young children and in older adults as a result of
delayed diagnosis.

HISTORY
Patients typically complain of epigastric pain that subsequently migrates to the right lower
quadrant. The initial discomfort is thought to be due to obstruction and swelling of the appendix
and the latter due to peritoneal irritation. Retrocecal appendicitis may cause pain higher in the
right abdomen, whereas appendicitis located in the pelvis may cause vague pelvic discomfort.
Anorexia is an almost universal complaint. Nausea and emesis may occur after the onset of pain.
Up to 20% of patients report experiencing diarrhea, which often leads the examiner to make an
incorrect diagnosis of gastroenteritis. Generalized abdominal pain may signify rupture and
diffuse peritonitis.

PHYSICAL EXAMINATION
Low-grade fever is typical. Nearly all patients have right lower quadrant tenderness, classically
located at McBurney's point, two thirds the distance from the umbilicus to anterior superior iliac
spine. Rebound and guarding develop as the disease progresses and the peritoneum becomes
inflamed. Signs of peritoneal irritation include the obturator sign (pain on external rotation of
the flexed thigh) and the psoas sign (pain on right thigh extension). Rovsing's sign is eliciting pain
in the right lower quadrant on palpation of the left lower quadrant. In cases of contained
perforation, the omentum walls off the infectious process, occasionally resulting in a palpable
mass in thin patients. If the perforation is free and not contained, then diffuse peritonitis and
septic shock may develop. Rectal examination may reveal tenderness if the appendix hangs low
in the pelvis.

DIAGNOSTIC EVALUATION
The white blood cell count is usually mildly to moderately elevated. Urinalysis should be
performed to rule out a urinary tract infection.

Depending on a patient's age, presenting history and physical examination, and available
resources, radiologic studies may include ultrasound or CT scanning. Plain abdominal x-rays
(supine and upright) usually provide no useful information in confirming the diagnosis of
appendicitis. Ultrasonographic evidence of appendicitis includes appendiceal wall thickening,
luminal distention, and lack of compressibility. Ultrasound is also useful in female patients for
demon- strating ovarian or other gynecologic pathology. CT scanning may show appendiceal
enlargement, periappendiceal inflammatory changes, free fluid, or right lower quadrant abscess
(Fig. 5-11).

Figure 5-11 • Computed tomographic appearance of appendicitis.

From Harwood-Nuss A, Wolfson AB, Linden CH, et al. The Clinical


Practice of Emergency Medicine. 3rd ed. Philadelphia: Lippincott
Williams & Wilkins, 2001.

CT scanning is also useful for ruling in or out alternative diagnoses, thereby reducing the
negative appendectomy rate in many hospitals.

P
TREATMENT
Uncomplicated appendicitis requires appendectomy. Both open and laparoscopic techniques are
appropriate. Laparoscopic appendectomy is associated with less postoperative pain, a shorter
hospital course, better cosmesis, and faster return to work. Selected advanced cases with
appendiceal abscess may initially be managed nonoperatively with antibiotics and percutaneous
CT-guided abscess drainage. Once the infection has abated and the inflammatory process
resolved, interval appendectomy may be performed at a later date.

KEY POINTS
Surgery for ulcerative colitis is indicated for intractable bleeding,
obstruction, failure of medical therapy, toxic megacolon, and risk of
cancer.
Diverticulosis is the most common cause of lower gastrointestinal bleeding.
Prevalence of diverticula increases with age. Cause is related to low-fiber dietary
intake.
Elective surgical therapy for diverticulitis is indicated for repeated attacks
because of the high recurrence and complication rate. Emergent surgical therapy
is indicated for free perforation and usually requires segmental colon resection
and end colostomy (Hartmann procedure).
Colon cancer follows a progression from adenoma to carcinoma. Adenomatous
polyps are considered premalignant and must be removed entirely. Screening
for colon cancer should begin at age 50 for normal-risk patients.
Surgical therapy for colon cancer is predicated on removal of the malignant
lesion and the draining lymph nodes. Both open and laparoscopic techniques are
accepted surgical therapies for resection.
Angiodysplasia is common in older adult patients and is one of the most
common causes of lower gastrointestinal bleeding.
Volvulus is a life-threatening condition that pre-sents with abdominal pain and
distention.
Appendicitis is the most common reason for urgent abdominal operation. Right
lower quadrant abdominal pain, fever, and leukocytosis are hallmarks of the
disease. Appendectomy can be performed with either open or laparoscopic
techniques.
Authors: Karp, Seth J.; Morris, James P.G.; Zaslau, Stanley
Title: Blueprints Surgery, 5th Edition

Copyright ©2008 Lippincott Williams & Wilkins

> Table of Contents > Part II - Gastrointestinal and Abdominal > Chapter 6 - Liver

Chapter 6
Liver

ANATOMY AND PHYSIOLOGY


The liver is located in the right upper quadrant of the abdomen and is bounded superiorly and
posteriorly by the diaphragm, laterally by the ribs, and inferiorly by the gallbladder, stomach,
duodenum, colon, kidney, and right adrenal. It is covered by Glisson capsule and peritoneum.
The right and left lobes of the liver are defined by the plane formed by the gallbladder fossa and
the inferior vena cava. The falciform ligament between the liver and diaphragm is a landmark
between the lateral and medial segments of the left lobe. The coronary ligaments continue
laterally from the falciform and end at the right and left triangular ligaments. These ligaments
define the bare area of the liver, an area devoid of peritoneum. The liver parenchyma is divided
into eight segments on the basis of arterial and venous anatomy (see Color Plate 5). Segment 1 is
also known as the caudate lobe. It is not visible from the ventral surface of the liver, being
tucked behind segment 4. The caudate is juxtaposed to the inferior vena cava and has venous
branches that drain directly into the cava. These branches are quite fragile and must be
carefully controlled if resection of the caudate is required. Segments 2, 3, and 4 form the left
lobe of the liver, whereas segments 5, 6, 7, and 8 comprise the right lobe. Segment 4 may be
divided into cranial segment 4a and caudal segment 4b.

The hepatic circulation is based on a portal circulation that provides the liver with first access to
all intestinal venous flow. Seventy-five percent of total hepatic blood flow is derived from the
portal vein, which is formed from the confluence of the splenic and superior mesenteric veins.
The remaining blood supply comes from the hepatic artery via the celiac axis. The right hepatic
artery arises from the superior mesenteric artery in 15% of patients. When this occurs, the artery
will run posterior to the bile duct on the right side of the hilum, and it is termed a replaced right
hepatic artery. The left hepatic artery arises from the left gastric in 15% of patients, called a
replaced left hepatic artery. In this instance, the artery will run in the cranial portion of the
gastrohepatic ligament. Other arterial variants include a completely replaced hepatic artery,
which arises from the superior mesenteric artery, and a middle hepatic artery, which occurs
when the segment 4 branch arises in the hilum. Blood leaving the liver enters the inferior vena
cava via the right, middle, and left hepatic veins. Often there is an accessory right hepatic vein
that leaves the liver caudad to the principle right hepatic vein. This vein must be controlled
separately during right hepatic lobectomy.

The hepatic hilum can be palpated by placing a finger through the foramen of Winslow (epiploic
foramen) into the lesser sac (Fig. 6-1). This is an important maneuver because it provides control
of the hepatic hilum (hepatoduodenal ligament), within which runs the hepatic artery, portal
vein, and bile duct. A Pringle maneuver, which involves placing a clamp on the hilum, disrupts
most blood flow to the liver and can greatly reduce bleeding during liver resection (Fig. 6-2).
This maneuver also makes the liver ischemic and can cause arterial thrombosis. As a result, it
should be used for a limited amount of time and only when necessary.

Figure 6-1 • The lesser sac. The lesser sac is behind the
hepatoduodenal and hepatogastric ligaments. Entry is through the
epiploic foramen (foramen of Winslow).
From Sadler T. Langman's Medical Embryology, Ninth Edition Image
Bank. Baltimore: Lippincott Williams & Wilkins, 2003.
Figure 6-2 • Pringle maneuver. Occlusion of the porta hepatis
decreases blood flow to the liver to slow bleeding during liver
surgery.
From Blackbourne LH. Advanced Surgical Recall. 2nd ed. Baltimore:
Lippincott Williams & Wilkins, 2004.

The liver is the site of many critical events in energy metabolism and protein synthesis. Glucose
is taken up and stored as glycogen, and glycogen is broken down, as necessary, to maintain a
relatively constant level of serum glucose. The liver is able to initiate gluconeogenesis during
stress, and the liver can oxidize fatty acids to
ketones, which the brain can use as an energy source. Proteins synthesized in the liver include P
the coagulation factors fibrinogen, prothrombin, prekallikrein, high-molecular-weight kininogen,
and factors V, VII, VIII, IX, X, XI, and XII. Of these, prothrombin and factors VII, IX, and X are
dependent on vitamin K. The anticoagulant warfarin (Coumadin) affects these vitamin K–
dependent pathways, resulting in an increased pro- thrombin time. Albumin and alpha globulin
are pro-duced solely in the liver.
The digestive functions of the liver include bile synthesis and cholesterol metabolism. Heme is
used to form bilirubin, which is excreted in the bile after conjugation with glycine or taurine.
Bile emulsifies fats to aid their digestion and plays a role in vitamin uptake. Bile salts excreted
into the intestine are reabsorbed into the portal circulation. This cycle of bile excretion and
absorption is termed the enterohepatic circulation. Total body bile circulates approximately
10 times per day in this loop. More than 95% of excreted bile is reabsorbed, and the remainder P
must be resynthesized. The rate-limiting step of cholesterol synthesis involving the enzyme 3-
hydroxy-3-methyl-glutaryl–coenzyme A reductase occurs in the liver, as does cholesterol
metabolism to bile salts.

Detoxification occurs in the liver through two pathways. Phase I reactions involve cytochrome
P450 and include oxidation, reduction, and hydrolysis. Phase II reactions consist of conjugation.
These reactions are critical to destruction or renal clearance of toxins. The dosing of all oral
drugs is determined only after considering the first-pass effect of the drug through the liver. The
initial hydroxylation of vitamin D occurs in the liver. Immunologic functions are mediated by
Kupffer cells, the resident liver macrophages.

BENIGN LIVER TUMORS

PATHOLOGY
Benign liver tumors include hepatocellular adenoma (see Color Plate 6), focal nodular
hyperplasia, hemangioma, and lipoma. Hemangiomas are categorized into capillary and
cavernous types, the former being of no clinical consequence and the latter capable of attaining
large size and rupturing.

EPIDEMIOLOGY
Only 5% of liver tumors are benign, with hemangioma being the most common. Approximately 7%
of people have a cavernous hemangioma at autopsy. The incidence of adenoma is one per million
in women without a history of oral contraceptive use. These medicines increase the risk by a
factor of 40. This lesion most commonly occurs in women between 30 and 50 years of age.
Adenoma and focal nodular hyperplasia are five times more common in female patients.

History
Patients with adenomas and hemangiomas can be asymptomatic or present with dull pain;
rupture can produce sudden onset of severe abdominal pain. These lesions can also become large
enough to cause jaundice or symptoms of gastric outlet obstruction, including nausea and
vomiting. Focal nodular hyperplasia is rarely symptomatic.

PHYSICAL EXAMINATION
Large lesions can be palpated. Jaundice may occur in patients if the tumor causes bile duct
obstruction.

DIAGNOSTIC EVALUATION
These lesions are most often found incidentally at laparotomy or on imaging studies requested
for other reasons. Laboratory evaluation is often unremarkable, although hemorrhage in an
adenoma can lead to hepatocellular necrosis and a subsequent increase in transaminase levels.
Hemangioma can cause a consumptive coagulopathy. Ultrasound differentiates cystic from solid
lesions. Triple-phase computed tomography (CT) is the best study for distinguishing between
various types of benign and malignant lesions, but in certain cases, this determination is not
possible. Adenomas are typically low-density lesions; focal nodular hyperplasias may appear with
a filling defect or central scar, whereas hemangiomas have early peripheral enhancement after
contrast administration. Hemangiomas should not be biopsied because of the risk of bleeding.

TREATMENT
Patients with adenoma who are using oral contraceptives should stop. If the lesion does not
regress, resection should be considered in otherwise healthy individuals because of the risk of
malignant degeneration or hemorrhage. Relative contraindications to resection include a tumor
that is technically difficult to resect or tumors of large size in which a large portion of the liver
would need to be removed. Symptomatic hemangiomas should be resected, if possible. Because
focal nodular hyperplasia is not malignant and rarely causes symptoms, it should not be resected
unless it is found incidentally at laparotomy and is small and peripheral enough to be wedged out
easily.

LIVER CANCER

PATHOLOGY
Liver cancers are hepatomas, also known as hepatocellular carcinoma, or metastases from other
primaries (see Color Plate 7).

EPIDEMIOLOGY
Ninety-five percent of liver tumors are malignant. Hepatoma is one of the most common
malignancies
in the world, but rates in the United States are relatively low (approximately two per 100,000). P
It is more common in male than in female patients.

ETIOLOGY
Cirrhosis is a predisposing factor to hepatoma; as such, hepatitis B, the leading cause of
cirrhosis, and alcoholism are associated with hepatoma development. Fungal-derived aflatoxins
have been implicated as causes of hepatoma, as have hemochromatosis, smoking, vinyl chloride,
and oral contraceptives.

HISTORY
Patients with hepatoma may complain of weight loss, right upper quadrant or shoulder pain, and
weakness. Hepatic metastases are often indistinguishable from primary hepatocellular
carcinoma.

PHYSICAL EXAMINATION
Hepatomegaly may be appreciable, and signs of portal hypertension, including splenomegaly and
ascites, may be present. Jaundice occurs in approximately half of patients.

DIAGNOSTIC EVALUATION
Laboratory examination may reveal abnormal liver function tests. α-Fetoprotein is a specific
marker for hepatoma but can also be elevated in embryonic tumors. Radiographic studies are
used to differentiate benign and malignant lesions. Ultrasonography can distinguish cystic from
solid lesions, whereas CT or magnetic resonance imaging can reveal multiple lesions and clarify
anatomic relationships (Fig. 6-3). They can also demonstrate nodularity of the liver,
hypersplenism, and portal hypertension, indicative of underlying liver disease. Hepatic
arteriography can diagnose a hemangioma. Because most cancers occur in the setting of liver
disease and cirrhosis, it is important to perform viral studies for hepatitis.
Figure 6-3 • Hepatocellular carcinoma with portal venous
thrombosis. Computed tomography image demonstrates portal vein
thrombus (black arrows on thrombosed right and left portal veins).
A mass (curved white arrows) is present in the right lobe of the
liver.

From Kelsen DP, Daly JM, Kern SE, et al. Gastrointestinal Oncology:
Principles and Practice. Philadelphia, PA: Lippincott Williams &
Wilkins; 2002:10–16.

TREATMENT
Before consideration of resection, the underlying health of the liver should be assessed using the
Child-Turcotte-Pugh scoring system (Table 6-1). Patients with Child class C disease will generally
not tolerate a resection; patients with Child class B disease may tolerate a limited resection.

TABLE 6-1 Child-Turcotte-Pugh Scoring System for Cirrhosis

Clinical
1 Point 2 Points 3 Points
Variable

Encephalopathy None Stage Stage 3–4


1–2

Ascites Absent Slight Moderate

Bilirubin <2 2–3 >3


(mg/dL)

Bilirubin <4 4–10 10


in PBC or
PSC (mg/dL)

Albumin (g/dL) >3.5 2.8–3.5 <2.8

Prothrombin <4 sec 4–6 sec >6 sec or


time (seconds or or INR >2.3
prolonged INR INR
or INR) <1.7 1.7–2.3

Child class A = 5–6 points; Child class B = 7–9


points; Child class
C = 10–15 points.

PBC, primary biliary cirrhosis; PSC, primary


sclerosing cholangitis; INR,
international normalized ratio.

If the patient is a surgical candidate, treatment involves resection of the tumor. Survival without
treatment averages 3 months; resection can extend survival to 3 years, with a 5-year survival
rate of 11% to 46%.
The decision to resect the tumor depends on comorbid disease and the location and size of the P
tumor. When possible, wedge resection should be performed, because formal hepatic lobectomy
does not provide any additional survival benefit. Patients with small tumors who are not
candidates for resection because of tumor location or concomitant cirrhosis should be considered
for liver transplantation. Liver transplantation is becoming an increasingly attractive option for
these patients, providing good long-term survival. Patients who meet the Milan criteria (tumor <5
cm or no more than three tumors, the largest of which is <3 cm) will receive extra points on the
liver transplant list.

Metastatic disease occurs in decreasing frequency from lung, colon, pancreas, breast, and
stomach. When colon cancer metastasizes to the liver, resection of up to three lesions has been
shown to improve survival and should be attempted as long as the operative risk is not
prohibitive. In general, liver metastases from other tumors should not be resected.

LIVER ABSCESSES

ETIOLOGY
Liver abscesses are most frequently due to bacteria, amebas, or the tapeworm Echinococcus.
Bacterial abscesses usually arise from an intra-abdominal infection in the appendix, gallbladder,
or intestine but may be due to trauma or a complication of a surgical procedure. Causative
organisms are principally gut flora, including Escherichia coli, Klebsiella, enterococci, and
anaerobes (including Bacteroides). Amebic abscesses owing to Entamoeba histolytica are an
infrequent complication of gastrointestinal amebiasis.

EPIDEMIOLOGY
Pyogenic abscesses are responsible for fewer than one in 500 adult hospital admissions. Amebic
abscesses occur in 3% to 25% of patients with gastrointestinal amebiasis (Fig. 6-4). Risk factors
include HIV, alcohol abuse, and foreign travel. Echinococcus is most commonly seen in Eastern
Europe, Greece, South Africa, South America, and Australia; although rare in the United States,
it is the most common cause of liver abscesses worldwide (Fig. 6-5).

Figure 6-4 • Amebic abscesses of the liver. The cut surface of the
liver shows multiple abscesses containing "anchovy paste" material.

From Rubin E, Farber JL. Pathology. 3rd ed. Philadelphia, PA:


Lippincott Williams & Wilkins; 1999:9–75.
Figure 6-5 • A computed tomography scan shows a multilocular cyst
in the liver of a patient with hydatid disease.
From Sun, Tsieh MD. Parasitic Disorders: Pathology, Diagnosis, and
Management. 2nd ed. Baltimore: Lippincott Williams & Wilkins,
1999.

HISTORY
Patients with pyogenic or amebic abscesses usually have nonspecific complaints of vague
abdominal pain, weight loss, malaise, anorexia, and fever. Travel to an endemic region may
suggest Echinococcus.

PHYSICAL EXAMINATION
The liver may be tender or enlarged, and jaundice may occur. Rupture of an abscess can lead to
peritonitis, sepsis, and circulatory collapse.

DIAGNOSTIC EVALUATION
The white blood cell count and transaminase levels are elevated. Antibodies to ameba are found
in 98% of patients with amebic abscesses but in fewer than 5% of those with pyogenic abscesses.
Echinococcal infection
produces eosinophilia and a positive heme agglutination test. Ultrasonography is approximately P
90% sensitive for demonstrating a lesion; CT is slightly better. The presence of multiple cysts, or
"sand," on CT is suggestive of Echinococcus. Sampling of the cyst contents with CT or ultrasound
guidance reveals the causative organism in the case of pyogenic abscesses but does not usually
lead to a diagnosis in amebic abscesses. Aspiration of echinococcal cysts is contraindicated
because of the risk of contaminating the peritoneal cavity.

TREATMENT
Pyogenic abscesses require antibiotics alone or in combination with percutaneous or open
drainage. Amebic abscesses are treated with metronidazole (Flagyl), with or without
chloroquine, and surgical drainage is reserved for complications, including rupture. Echinococcal
abscesses require an open procedure. Scolecoidal agents (e.g., ethanol or 20% sodium chloride)
are instilled directly into the cyst, followed by drainage, with care not to spill the organisms into
the peritoneum.

PORTAL HYPERTENSION

ETIOLOGY
Portal hypertension is caused by processes that impede hepatic blood flow, either at the
presinusoidal, sinusoidal, or postsinusoidal levels. Presinusoidal causes include schistosomiasis
and portal vein thrombosis. The principal sinusoidal cause in the United States is cirrhosis,
usually caused by alcohol but also by hepatitis B and C. Cirrhosis develops in approximately 15%
of alcoholics. Postsinusoidal causes of portal hypertension include Budd-Chiari syndrome (hepatic
vein occlusion), pericarditis, and right-sided heart failure.

COMPLICATIONS
Bleeding varices are a life-threatening complication of portal hypertension. When portal
pressures increase, flow through the hemorrhoidal, umbilical, or coronary veins becomes the
low-resistance route for blood flow. The coronary vein empties into the plexus of veins draining
the stomach and esophagus (Fig. 6-6). Engorgement of these veins places the patient at risk of
bleeding into the esophagus or stomach.
Figure 6-6 • Selected collateral circulation in portal hypertension.

HISTORY
Alcoholism, hepatitis, or previous variceal hemorrhage are common.

PHYSICAL EXAMINATION
A variety of physical findings, including ascites, jaundice, "cherubic face," spider angioma,
testicular atrophy, gynecomastia, and palmar erythema, may suggest the diagnosis.

DIAGNOSTIC EVALUATION
Laboratory examination may reveal increased liver enzymes, which may return to normal with
advanced cirrhosis as the amount of functioning hepatic parenchyma decreases. Tests of liver
synthetic function, including clotting times and serum albumin, may be abnormal.
TREATMENT
Patients with portal hypertension are placed on beta-blockers to decrease the risk of bleeding.
Endoscopic surveillance and banding are useful in preventing bleed- ing episodes.

For patients with upper gastrointestinal bleeds, large-bore intravenous lines and volume
resuscitation should be started immediately. A nasogastric tube should be placed to confirm the
diagnosis. If the patient cannot be lavaged clear, suggesting active bleeding, emergency
endoscopy is both diagnostic and therapeutic. Endoscopy is >90% effective in controlling acute
bleeding from esophageal varices. Should this fail, balloon tamponade with a Sengstaken-
Blakemore tube and vasopressin infusion should be considered. Use of the Sengstaken-Blakemore
tube involves passing the gastric
balloon into the stomach, exerting gentle traction on the tube, and then inflating the esophageal P
balloon to tamponade bleeding. Although effective in stopping life-threatening hemorrhage, the
tube can produce gastric and esophageal ischemia and must be used with extreme caution.
Transjugular intrahepatic portosystemic shunting has a high rate of success in controlling acute
bleeding and is usually preferred to an emergent surgical shunt, although this is also an option
(Fig. 6-7).
Figure 6-7 • Transjugular intrahepatic portosystemic shunting. A
metallic shunt is placed from the hepatic vein to the right portal
vein via a catheter introduced through the internal jugular vein.
The shunt will typically continue into the main portal vein.

From Blackbourne LH. Advanced Surgical Recall. 2nd ed. Baltimore,


MD: Lippincott Williams & Wilkins; 2004.

Approximately 40% of patients with varices will develop a bleeding complication. Seventy
percent of patients with a first episode will rebleed. For this reason, a definitive procedure
should be considered after the initial episode is controlled.

Patients with bleeding varices and cirrhosis will ordinarily be considered for liver
transplantation. If there is no cirrhosis, or if the patient has good residual liver function, surgical
shunts have better long-term patency than transjugular intrahepatic portosystemic shunting.
Surgical shunts are divided into nonselective and selective shunts. Nonselective shunts divert the
entire portal blood flow into the systemic circulation. An example is an end-to-side portacaval
shunt, in which the portal vein is divided and drained directly into the inferior vena cava.
Selective shunts divert only a portion of the portal blood away from the liver. The most common
is the distal splenorenal shunt, in which portal blood is shunted through the renal vein and into
the cava. Because there is still blood going to the liver to be detoxified, patients with selective
shunts have less encephalopathy and equivalent success in preventing rebleeding.

As a last resort in patients with bleeding esophageal varices, a Sugiura procedure can be
performed. During this procedure, the varices are disconnected from the portal circulation by
complete esophageal transection and reanastomosis. This procedure also includes splenectomy,
proximal gastric devascularization, vagot-comy, and pyloroplasty.

In patients with Budd-Chiari syndrome, side-to-side portacaval shunt can be life-saving.

KEY POINTS
The liver performs an array of functions involving energy metabolism,
protein synthesis, digestion, and detoxification.
Only 5% of liver tumors are benign.
Hepatocellular carcinoma is extremely common worldwide but is relatively rare
in the United States.
Causes of hepatocellular carcinoma include cirrhosis, aflatoxin, smoking, and
vinyl chloride.
The prognosis for hepatocellular carcinoma is poor.
Liver abscesses are most commonly caused by bacteria, amebas, or
Echinococcus.
Portal hypertension has presinusoidal, sinusoidal, and postsinusoidal causes.
Variceal hemorrhage is life-threatening, but endo-scopy is usually successful in
controlling bleeding.
Because of the high recurrence rate, a definitive procedure should be considered
after the first episode of variceal bleeding.
Surgical shunts may have better long-term patency than transjugular
intrahepatic portosystemic shunt- ing and should be considered in patients with
preserved liver function.
Authors: Karp, Seth J.; Morris, James P.G.; Zaslau, Stanley
Title: Blueprints Surgery, 5th Edition

Copyright ©2008 Lippincott Williams & Wilkins

> Table of Contents > Part II - Gastrointestinal and Abdominal > Chapter 7 - Gallbladder

Chapter 7
Gallbladder

ANATOMY AND PHYSIOLOGY


The gallbladder is located in the right upper quadrant of the abdomen beneath the liver. The
cystic duct exits at the neck of the gallbladder and joins the common hepatic duct to form the
common bile duct, which empties into the duodenum at the ampulla of Vater. This is surrounded
by the sphincter of Oddi, which regulates bile flow into the duodenum (Fig. 7-1).
Figure 7-1 • Duct system.
From Anatomical Chart Company.

Bile produced in the liver is stored in the gallbladder. Bile is important for the absorption of fat-
soluble vitamins (A, D, E, and K). Cholecystokinin stimulates gallbladder contraction and release
of bile into the duodenum. The spiral valves of Heister in the cystic duct prevent bile reflux into
the gallbladder. Arterial supply is from the cystic artery, which most commonly arises from the
right hepatic artery and courses through the triangle of Calot, which is bounded by the cystic
duct laterally, the common hepatic duct medially, and the edge of the liver superiorly (Fig. 7-2).
Figure 7-2 • The hepatocystic triangle (of Calot) is defined by three
structures: cystic duct, common hepatic duct and lower liver edge.
From Blackbourne LH. Advanced Surgical Recall. 2nd ed. Baltimore:
Lippincott Williams & Wilkins, 2004.

GALLSTONE DISEASE
Cholelithiasis is the presence of gallstones within the gallbladder. Choledocholithiasis refers to
stones in the common bile duct (Fig. 7-3). Biliary colic is pain produced when the gallbladder
contracts against a transiently obstructing stone in the neck of the gallbladder. There is no
inflammatory or infectious process in biliary colic. Acute cholecystitis produces a constant pain
and refers to inflammation and infection of the gallbladder; total or partial occlusion of the
cystic duct is thought to be required. The most common organisms cultured during acute
cholecystitis are Escherichia coli, Klebsiella, enterococci, Bacteroides fragilis, and
Pseudomonas. Gallstones within the common bile duct are a major cause of pancreatitis, known
as gallstone pancreatitis.
Figure 7-3 • Biliary pathology.

PATHOGENESIS
Gallstones are composed of cholesterol, calcium bilirubinate (pigment), or a mixture of both.
Cholesterol stones make up approximately 75% of gallstones in Western countries. Stone
formation occurs when bile becomes supersaturated with cholesterol. Cholesterol crystals then
precipitate out of solution and agglomerate to form stones.

A high-cholesterol diet causes increased concentrations of cholesterol and may have a role in the
pathogenesis of cholesterol stones. Pigment stones are composed of calcium bilirubinate and are
either black or brown. Black pigment stones are usually found in the gallbladder and are
associated with cirrhosis and hemolytic processes, such as sickle cell anemia, thalassemia, and
spherocytosis. Brown stones are associated with chronic biliary tract infection and are often
found in the bile ducts. Patients with indwelling biliary stents or with intraluminal nonabsorbable
sutures in the ducts from prior surgery are prone to developing brown stones.

EPIDEMIOLOGY
Approximately 10% of the U.S. population has gallstones. The vast majority of people with stones
are asymptomatic. Nevertheless, more than 600,000 chole- cystectomies are performed in the
United States annually.

Gallstones are found more commonly in women. Risk factors include obesity, multiparity, chronic
total
parenteral nutrition use, high-dose estrogen oral contraceptives, rapid weight loss, diabetes, and P
increasing age. Some ethnic groups such as American Indians have very high prevalence rates.
Spinal cord injury predisposes to cholesterol stones.

HISTORY
As stated, most patients with gallstones are asymptomatic. Patients with biliary colic usually
complain of right upper quadrant or epigastric pain, often radiating around the right flank to the
back. The pain is usually postprandial (occurring after eating). Pain episodes may be precipitated
by fatty food intake and last several hours before resolving spontaneously. Associated nausea and
vomiting are common.

Cholecystitis implies infection and inflammation of the gallbladder. The pain of cholecystitis is
usually constant, with progressive worsening. Patients may have fever, chills, or sweats.

Choledocholithiasis can result in transient or complete blockage of the common bile duct.
Patients may relate episodes of passing dark urine or light-colored stools caused by the inability
of bile pigments to reach the gastrointestinal tract and from subsequent renal clearance.
Choledocholithiasis can also lead to ascending cholangitis, demonstrated by right upper quadrant
abdominal pain, fever, and chills.

Pancreatitis owing to choledocholithiasis (gallstone pancreatitis) typically manifests with P


epigastric pain radiating to the back.

PHYSICAL EXAMINATION
Physical examination in simple biliary colic reveals right upper quadrant tenderness but no fever.
Cholecystitis may be associated with fever and signs of peritoneal irritation, including right
upper quadrant rebound and guarding. The classic finding in acute cholecystitis is the arrest of
inspiration on deep right upper quadrant palpation as pressure from the examiner's hand
contacts the inflamed gallbladder and peritoneum (Murphy sign). Choledocholithiasis may be
associated with jaundice, in addition to signs of biliary colic. Cholangitis is classically marked by
fever, right upper quadrant pain, and jaundice (Charcot triad). Progression of cholangitis to
sepsis defines Reynolds pentad by adding hypotension and mental status changes to the triad.
Patients with gallstone pancreatitis have epigastric tenderness. A palpable, nontender, distended
gallbladder in the clinical setting of jaundice indicates malignant biliary obstruction (Courvoisier
law).

DIAGNOSTIC EVALUATION
Laboratory examination in biliary colic is usually unremarkable. Cholecystitis usually manifests
with increased white blood cell count and minor liver function test abnormalities.
Choledocholithiasis is classically associated with increased serum bilirubin and alkaline
phosphatase. Cholangitis usually causes elevated serum bilirubin and transaminase levels, as well
as leukocytosis. Gallstone pancreatitis is accompanied by elevations in serum amylase and
lipase.

Ultrasound is the best modality for imaging the gallbladder and bile ducts, having a sensitivity
and specificity of 98% for the detection of gallstones. On ultrasound, the gallstones appear as
opacities, with echoless shadows posteriorly (Fig. 7-4). Moving the patient during ultrasound
examination often demonstrates migration of the stones to the dependent portion of the
gallbladder. Ultrasound is also used for diagnosing acute cholecystitis. Fluid around the
gallbladder (pericholecystic fluid), a thickened gallbladder wall, and an ultrasonographic Murphy
sign all support the diagnosis of acute cholecystitis.
Figure 7-4 • Gallstone. There is a hyperechoic stone (large arrow)
in the dependent portion of the gallbladder, with a posterior shadow
(small arrow). There is no thickening of the gallbladder wall or fluid
around the gallbladder.

From Harwood-Nuss A, Wolfson AB, Linden CH, et al. The Clinical


Practice of Emergency Medicine. 3rd ed. Philadelphia: Lippincott
Williams & Wilkins, 2001.
When ultrasound findings are equivocal or acalculous cholecystitis is suspected,
cholescintigraphy (e.g., hepatobiliary iminodiacetic acid scan) is almost 100% sensitive and 95%
specific for acute cholecystitis.
In this test, a radionucleotide that is injected intravenously is taken up in the liver and excreted P
into the biliary tree. If the cystic duct is obstructed, as in acute cholecystitis, the gallbladder
does not fill, and the radionucleotide passes directly into the duodenum.

Choledocholithiasis can be diagnosed by intraoperative cholangiography at the time of surgery or


preoperatively or postoperatively by endoscopic retrograde cholangiopancreatography (ERCP).
ERCP is performed by using a specialized side-viewing endoscope to visualize the ampulla, where
the pancreatic and biliary ducts enter the duodenum. Using a catheter passed through the
endoscope, contrast media is injected retrograde and outlines the biliary tree and pancreatic
duct (Fig. 7-5). Magnetic resonance cholangiopancreatography can noninvasively detect common
bile duct stones; however, it lacks the therapeutic advantage of ERCP for stone extraction.
Figure 7-5 • Endoscopic retrograde cholangiopancreatography
removal of common bile duct stone using balloon-tip catheter.

COMPLICATIONS
Most gallstones are quiescent. When cholecystitis develops, however, delayed diagnosis may
result in gangrenous necrosis of the gallbladder wall with perforation, leading to localized
abscess or frank biliary peritonitis. Emphysematous cholecystitis owing to Clostridium
perfringens can be seen in diabetic patients.

Gallstone pancreatitis may occur as a result of a common duct stone causing blockage of the
ampulla, theoretically resulting in bile reflux into the pancreatic duct or increased intraductal
pressure. (See Chapter 9, Pancreas.)

Chronic perforation may result in a bilioenteric fistula. This occurs in older adult patients when a
large gallstone erodes through the gallbladder wall and causes a fistula to form between the
gallbladder and bowel (usually duodenum, rarely colon). The large stone can then pass out of the
gallbladder, through the fistula and into the bowel, resulting in distal bowel obstruction
(gallstone ileus). Stone obstruction of the small bowel typically occurs at the terminal ileum,
whereas the large bowel obstruction typically occurs at the sigmoid colon. Pneumobilia and a
smooth obstructing mass on imaging studies are classic findings.

TREATMENT
For patients with asymptomatic stones found on workup for other problems, the incidence of
symptoms or complications is approximately 2% per year. Cholecystectomy is usually not advised
for these patients. For individuals with biliary colic, laparoscopic cholecystectomy is a safe and
effective procedure. This is performed electively in most cases. If the preoperative workup
suggests that common duct stones may be present, either ERCP or intraoperative
cholangiography should be performed. (See Appendix for a description of a typical laparoscopic
cholecystectomy.)

Patients with acute cholecystitis should be adequately fluid resuscitated before surgery, as
vomiting and diminished oral intake often results in dehydration. Intravenous antibiotics should
be administered.

Laparoscopic cholecystectomy is the procedure of choice for removal of the gallbladder (Fig. 7-
6). In the acute setting with gallbladder inflammation and infection, the procedure tends to be
technically more difficult and has a higher rate of conversion to the open technique when
compared with elective operations for biliary colic. Rarely, when patients are too ill to tolerate
surgery, a cholecystostomy tube may be considered. This involves placing a percutaneous drain
into the gallbladder lumen for decompression and drainage of pus. Cholecystectomy can then be
performed when the patient is stable.
Figure 7-6 • Laparoscopic cholecystectomy: four small incisions
allow insertion of laparoscope and instruments for retraction &
dissection.

From Smeltzer SC, Bare BG. Textbook of Medical-Surgical Nursing.


9th ed. Philadelphia: Lippincott Williams & Wilkins, 2000.

Patients with gallstone pancreatitis require aggressive fluid resuscitation and close observation.
Fortunately, mild episodes account for 80% of cases; however, severe fulminant cases can be
lethal. Intravenous antibiotics are
only indicated in severe cases with pancreatic necrosis, infected necrosis, or infectious P
complications. Early ERCP is indicated in patients with signs of common bile duct obstruction
(cholangitis, jaundice, dilated common duct on imaging studies) and in patients with severe
disease. Once pancreatic inflammation subsides, cholecystectomy with intraoperative
cholangiography should be performed during the same hospitalization to reduce the risk of
recurrent pancreatitis and to rule out residual common duct stones. If stones are present,
intraoperative common duct exploration or postoperative ERCP is performed. The risk of
recurrent pancreatitis is approximately 40% within 6 weeks (see Chapter 9, Pancreas).

Cholangitis, usually caused by choledocholithiasis, requires rapid diagnosis and treatment. Gram-
negative organisms are the most common cause. Prompt treatment with intravenous antibiotics,
fluid resuscitation, and urgent biliary decompression and drainage are indicated. ERCP with
sphincterotomy is the primary intervention. If the obstructing stone is unable to be extracted, an
indwelling biliary stent can be passed proximal to the stone to allow decompression and drainage
of infected bile into the duodenum. Other methods of decompression include percutaneous
transhepatic drainage or open surgical drainage with common bile duct exploration and T-tube
placement. Overall mortality is approximately 15%.

GALLBLADDER CANCER

EPIDEMIOLOGY
Carcinoma of the gallbladder is the most common malignancy of the biliary tract. Cancer of the
gallbladder is three times more common in females. The incidence is 2.5 in 100,000. Risk factors
include gallstones, porcelain gallbladder, and adenoma. One percent of all patients who undergo
cholecystectomy for gallstones will be found to have gallbladder carcinoma.

PATHOLOGY
Adenocarcinoma is the most common histologic type. Approximately 80% are adenocarcinomas,
10% are anaplastic, and 5% are squamous cell.

HISTORY
Unfortunately, most patients usually present with late-stage disease complaining of vague right
upper quadrant pain. Weight loss, anorexia, and nausea may also be present.

PHYSICAL EXAMINATION
A right upper quadrant mass may be present. Obstructive jaundice represents invasion or
compression of the common bile duct. Ascites is seen in advanced cases.

STAGING
Staging of gallbladder carcinoma is as follows:

Stage I: confined to mucosa/submucosa

Stage II: involvement of muscle layer of gallbladder wall

Stage III: lymph node involvement or extension into the liver (<2 cm liver invasion)

Stage IV: liver invasion >2 cm or distant metastasis.

TREATMENT
The mainstay of treatment for gallbladder cancer is surgical, because early detection and
complete resection
provide the only chance for cure. The extent of resection is controversial but is based on the P
stage of disease. Options include simple cholecystectomy, radical resection of the gallbladder
including partial hepatic resection, or palliative operation as symptoms arise.

Most cases of early-stage disease are found incidentally after elective laparoscopic
cholecystectomy. If the cancer is stage I and confined to the mucosa/submucosa, then no
additional surgery is indicated. For stage II and III lesions, where the muscle layer is involved,
the en-bloc resection of the gallbladder with hepatic segments 4 and 5 as well as radical lymph
node dissection is indicated. Overall, stage IV cancers show no benefit from attempts at radical
resection (except perhaps in patients with T4N0 disease).

PROGNOSIS
Unless cancer is found incidentally at cholecystectomy for symptomatic gallstones, the chance of
long-term survival is low. For all patients diagnosed with gallbladder cancer, only 4% will be alive
after 5 years. Patients found to have incidentally noted in situ disease on cholecystectomy have
survival rates of 88%. Overall survival rates of patients with American Joint Committee on Cancer
stage I disease is 60%; stage II, 24%; stage III, 9%; and stage IV, 1%.

BILE DUCT CANCER

EPIDEMIOLOGY
Bile duct cancer (cholangiocarcinoma) is rare. Risk factors include ulcerative colitis, sclerosing
cholangitis, and infection with Clonorchis sinensis. Patients with sclerosing cholangitis should be
observed closely for evidence of cancer.

HISTORY
Patients with advanced disease typically complain of right upper quadrant pain. Biliary
obstruction may lead to jaundice and pruritus.
PHYSICAL EXAMINATION
The patient may be jaundiced, with a palpable nontender distended gallbladder. If so,
Courvoisier law states that the site of obstruction is in the common bile duct, distal to the
confluence of the hepatic and cystic ducts.

DIAGNOSTIC EVALUATION
Laboratory studies are consistent with the chemical findings of obstructive jaundice. Ultrasound
and computed tomography show evidence of biliary obstruction with dilated ducts, but
percutaneous transhe-patic cholangiography or ERCP is usually necessary to demonstrate the
lesion. With access to the biliary tree, brushings or biopsies can be performed for cytologic
diagnosis.

TREATMENT
Unresectable tumors are usually treated with either endoluminal stenting to relieve obstruction
or else with percutaneous catheter biliary drainage. Surgical treatment mostly depends on the
location of the tumor within the extrahepatic bile duct. Tumors of the proximal and middle
thirds of the duct are best treated with resection and reconstruction with Roux-en-Y
hepaticojejunostomy (Fig. 7-7). Tumors of the distal lower third are best treated with
pancreaticoduodenectomy (Whipple procedure; Fig. 7-8).
Figure 7-7 • Choledochojejunostomy. Anastomosis of common bile
duct to jejunum.

From Blackbourne LH. Advanced Surgical Recall. 2nd ed. Baltimore:


Lippincott Williams & Wilkins, 2004.
Figure 7-8 • Classic pancreaticoduodenectomy. Top left: The
structures resected include the distal stomach; pylorus; entire
duodenum; head, neck, and uncinate process of the pancreas with
tumor (black); gallbladder; and distal extrahepatic biliary tree. Top
right: The structures retained include the proximal stomach, body
and tail of the pancreas, proximal biliary tree, and jejunum distal
to ligament of Treitz. Bottom: The reconstruction is shown as a
proximal end-to-end pancreaticojejunostomy, end-to-side
hepaticojejunostomy, and a distal gastrojejunostomy.

From Yeo C, Cameron JL. The pancreas. In: Hardy J, ed. Hardy's
Textbook of Surgery. 2nd ed. Philadelphia: JB Lippincott Co,
1988:717-718.

PROGNOSIS
Overall mortality is 90% at 5 years. The 5-year survival rate after proximal duct resection is
approximately 5%, after middle duct resection is 10%, and after Whipple procedure for distal
duct lesions is 30%.

P
KEY POINTS
Cholelithiasis is asymptomatic gallstones within the gallbladder.
Biliary colic is symptomatic gallstones causing transient right upper
quadrant pain without inflammation or infection.
Cholecystitis is inflammation and often infection of the gallbladder; symptoms
include persistent right upper quadrant pain and signs of infection.
Choledocholithiasis is stones in the common bile duct; serum studies often show
elevated liver function tests.
Cholangitis is infection in the bile ducts extending into the liver; patients have
right upper quadrant pain, fever, and jaundice.
Gallstone pancreatitis is a complication of gallstone disease
(choledocholithiasis).
Gallbladder and bile duct cancers are rare and usually fatal.
A jaundiced patient with a distended palpable nontender gallbladder indicates
distal common bile duct obstruction (Courvoisier law).
Surgical therapy for cholangiocarcinoma (bile duct cancer) depends on tumor
location: resection with Roux-en-Y reconstruction (proximal tumors) or Whipple
operation (distal tumors).
Authors: Karp, Seth J.; Morris, James P.G.; Zaslau, Stanley
Title: Blueprints Surgery, 5th Edition

Copyright ©2008 Lippincott Williams & Wilkins

> Table of Contents > Part II - Gastrointestinal and Abdominal > Chapter 8 - Spleen

Chapter 8
Spleen

The spleen is a lymphatic organ located in the left upper abdominal quadrant. It contains the largest accumulation of lymphoid cells in
the body. In addition to filtering the blood, it plays an important role in host defense. Splenic lymphocytes are involved in antigen
recognition and plasma cell production, whereas splenic endothelial macrophages extract bacteria and damaged red blood cells from
circulation by phagocytosis. White pulp of the spleen contains high concentrations of antigen-presenting cells and receives arterial
blood supply first. Blood then travels through an open-type circulation known as the red pulp, where foreign particles are filtered and
antibody-coated cells are removed. Removal of red blood cell imperfections is another function of the spleen. These include Howell-
Jolly bodies (Fig 8-1), which are nuclear remnants, Pappenheimer bodies, which are iron inclusions, and Heinz bodies, which are
denatured hemoglobin. Splenectomy or hyposplenism is associated with the presence of these abnormalities in the peripheral blood.

Figure 8-1 • Howell-Jolly bodies. These are round nuclear fragments that are chromosomes
separated from the mitotic spindle during abnormal mitosis.

From Anderson SC. Anderson's Atlas of Hematology. Philadelphia: Lippincott Williams & Wilkins,
2003.

Surgical issues regarding the spleen are multiple and varied. Life-threatening hemorrhage from a lacerated spleen resulting from
trauma is a common problem, requiring swift surgical intervention. Certain disease states, such as immune thrombocytopenic purpura
(ITP) and the hemolytic anemias, are often treated with splenectomy when medical management fails. Splenectomy may be necessary
as part of another operation, such as distal pancreatectomy. In addition, the traditional staging workup for Hodgkin disease has involved
removal of the spleen to determine extent of disease, although this is now rarely performed.

ANATOMY
The spleen is embryologically derived from condensations of mesoderm in the dorsal mesogastrium of the developing gastrointestinal
tract. In the mature abdomen, the spleen is found attached to the stomach by the gastrosplenic ligament and to the left kidney by the
splenorenal ligament. Other supporting attachments include the splenocolic and splenophrenic ligaments (Fig. 8-2).

Figure 8-2 • Normal anatomic relations of the spleen. The spleen lies behind the stomach and
above the left kidney. It is an intra-abdominal organ, although its blood supply courses through
the retroperitoneum.

Accessory spleens are present in approximately 25% of patients. They are most often found in the splenic hilum and in the supporting
splenic ligaments and greater omentum.

Arterial blood is mostly supplied via the splenic artery, which is one of three branches of the celiac axis (splenic, left gastric, common
hepatic). At the hilum, the splenic artery divides into smaller branches that supply the several splenic segments. Additional arterial
blood is supplied via the short gastric and left gastroepiploic vessels (Fig. 8-3).
Figure 8-3 • Arterial supply of the spleen.

Venous drainage is from segmental veins that join at the splenic hilum to form the splenic vein. Running behind the upper edge of the
pancreas, the splenic vein joins with the superior mesenteric vein to form the portal vein.

SPLENIC HEMORRHAGE
The most common cause of splenic hemorrhage is blunt abdominal trauma. Nonpenetrating injury may cause disruption of the splenic
capsule or frank laceration of the splenic parenchyma. Displaced rib fractures
of the left lower chest often cause splenic laceration. Splenic injuries are graded from I to V based on the extent of laceration and P
hematoma (Table 8-1).

TABLE 8-1 Grades of Splenic Injury

Grade Description

I Hematoma: Subcapsular <10%


   surface area
Laceration: <1 cm deep

II Hematoma: Subcapsular 10%–50%


   surface area
Parenchymal <5 cm diameter
Laceration: 1–3 cm deep, not involving
   trabecular vessels

III Hematoma: Subcapsular >50% surface area


Parenchymal >5 cm diameter
Any expanding or ruptured
Laceration: >3 cm deep or involving
   trabecular vessels

IV Laceration: Segmental vessels involved


   with devascularization <50%

V Completely shattered spleen or hilar


   vascular injury with devascularization.

From PF Lawrence. Essentials of General Surgery. 4th ed. Philadelphia: Lippincott Williams &
Wilkins, 2006.

Splenic hemorrhage may also be iatrogenic. Intraoperative damage to the spleen may occur during unrelated abdominal surgery that
results in bleeding controlled only by splenectomy. Estimates are that 20% of splenectomies result from iatrogenic causes. Infectious
diseases (e.g., mononucleosis, malaria) may damage the spleen to the point that unnoticed blunt trauma can cause "spontaneous"
splenic rupture and hemorrhage.

HISTORY
Patients typically present with a recent history of trauma, usually to the left upper abdomen or left flank.

P
PHYSICAL EXAMINATION
Depending on the degree of splenic injury and hemo-peritoneum, a physical examination may reveal left upper quadrant abdominal
tenderness, left lower rib fractures, abdominal distention, peritonitis, and hypovolemic shock. Pain on the left shoulder is referred to as
Kerr's sign and is referred pain from peritoneal irritation.

DIAGNOSTIC EVALUATION
Computed tomography (CT) scan, abdominal ultrasound, and peritoneal lavage (see Chapter 27, Trauma) can be used to detect
intraperitoneal blood. In hemodynamically stable patients, CT can demonstrate the degree of both splenic injury and hemoperitoneum.

TREATMENT
For patients with splenic injury who are hemodynamically stable and without evidence of ongoing hemorrhage, nonoperative
management with close hemodynamic monitoring has become the accepted treatment of choice. In children, nonoperative management
is widely applied as a result of the incidence of overwhelming postsplenectomy sepsis seen in the pediatric population. Recent advances
in interventional radiology have made it possible to embolize the spleen, or a portion, to obviate the need for laparotomy. This can
have the advantage of salvaging enough of the spleen to prevent major complications, such as postsplenectomy sepsis, but splenic
abscess may occur. For patients with known or suspected splenic injury who are hemodynamically unstable, operative intervention is
indicated to control ongoing hemorrhage.

Once in the operating room, the decision to perform splenic repair (splenorrhaphy) versus splenectomy is based on the degree of injury
to the par-enchyma and blood supply of the organ. Relatively minor injuries, such as a small capsular laceration with minor oozing, may
be repaired, whereas a fragmented spleen with involvement of the hilar vessels necessitates surgical removal. If splenic repair is
contemplated, the spleen must be mobilized completely to determine the extent of injury and the likelihood of success. If there are
other organs injured in addition to the spleen, splenectomy is generally recommended.

IMMUNE THROMBOCYTOPENIC PURPURA


ITP is an autoimmune hematologic disease in which antiplatelet IgG antibodies, produced largely in the spleen, are directed against a
platelet-associated antigen, resulting in platelet destruction by the reticuloendothelial system and subsequent thrombocytopenia. The
disease is typically seen in young women, who may present with complaints of menorrhagia, easy bruising, mucosal bleeding, and
petechiae. Men may present with complaints of prolonged bleeding after shaving trauma.
TREATMENT
Initial therapy is with corticosteroids, which improve platelet counts after 3 to 7 days of therapy. Initial doses are 1 mg/kg orally until
the platelet count is normal, followed by a 4- to 6-week taper. High-dose dexamethasone or methylprednisolone is also an option.
Intravenous immunoglobulin can be used to support the platelet count, as can anti-Rh(D) if patients have Rh(D) positive platelets. For
prolonged active bleeding, platelet transfusions should be administered to achieve hemostasis. If patients are asymptomatic, platelet
counts of <30,000 can be tolerated. Once patients become symptomatic or the platelet count decreases to <10,000, therapy should be
considered. Patients with chronic refractory ITP may benefit from rituximab or thrombopoiesis-stimulating agents, but these are
considered experimental at this time.

Few patients enjoy complete and sustained remission with corticosteroid treatment alone. Patients typically become refractory to
medical treatment, and thrombocytopenia recurs. Splenectomy is then indicated. After splenectomy, normal platelet counts develop in
approximately 80% of patients because the organ of both significant antiplatelet antibody production and platelet destruction is
removed.

HYPERSPLENISM
Hypersplenism describes a state of increased splenic function that results in various hematologic abnormalities, which can be
normalized by splenectomy. Elevated splenic function causes a depression of the formed blood elements, leading to a compensatory
hyperplasia of the bone marrow.

P
HISTORY
As in ITP, most patients are women who present with signs of anemia, recurrent infections, or easy bruising.

PHYSICAL EXAMINATION
Abdominal examination reveals splenomegaly.

DIAGNOSTIC EVALUATION
Peripheral-blood smear may reveal leukopenia, anemia, thrombocytopenia, or pancytopenia. Bone marrow biopsy shows pancellular
hyperplasia.

TREATMENT
Splenectomy may produce hematologic improvement.

HEMOLYTIC ANEMIAS
Hemolytic anemias are characterized by an elevated rate of red blood cell destruction from either a congenital or acquired etiology.
Congenital hemolytic anemias result from basic defects of the cell membrane (hereditary spherocytosis), hemoglobin synthesis
(thalassemia), hemoglobin structure (sickle cell anemia), or cellular metabolism (glucose-6-phosphate dehydrogenase deficiency).
Acquired autoimmune hemolytic anemias result when antibodies are produced that are directed against the body's own red blood cells.

DIAGNOSTIC EVALUATION
A positive direct Coombs test demonstrates complexed antibodies on the red blood cell membrane. Warm-reactive antibodies are IgG,
and cold-reactive antibodies are IgM.

TREATMENT
The role of splenectomy in treating hemolytic anemias depends on the particular disease process. For example, red blood cell survival
normalizes after splenectomy for hereditary spherocytosis, whereas operative intervention has no role in the treatment of anemia of
glucose-6-phosphate dehydrogenase deficiency that is secondary to a defect of metabolism, not cellular structure. Occasionally,
splenectomy may be useful in selected patients with sickle cell anemia and thalassemia. Patients with autoimmune hemolytic anemias
undergo initial corticosteroid treatment and progress to splenectomy only after medical treatment failure.

HODGKIN DISEASE STAGING


Because of a greater reliance on CT scans and the favorable success of salvage chemotherapy in the treatment of Hodgkin lymphoma,
the need for determining whether disease is present across the diaphragm by means of laparotomy and splenectomy has sharply
declined. Treatment with salvage chemotherapy after local radiation failure still carries a highly favorable outcome in most cases.
Therefore, splenectomy for staging Hodgkin disease is now rarely performed.

OVERWHELMING POSTSPLENECTOMY SEPSIS


Asplenic individuals are at greater risk for developing fulminant bacteremia because of decreased opsonic activity, decreased levels of
IgM, and decreased clearance of bacteria from the blood after splenectomy. As a rule, children are at greater risk for development of
sepsis than are adults, and fatal sepsis is more common after splenectomy for hematologic disorders than after trauma. The risk of
sepsis is higher in the first postoperative year, and, for adults, each subsequent year carries approximately a 1% chance of developing
sepsis. Other data show a mortality of less than one per 1,000 patient-years. The clinical picture of overwhelming postsplenectomy
sepsis is the onset of high fever followed by circulatory collapse from septic shock. Disseminated intravascular coagulation often occurs.
The offending pathogens are the encapsulated bacteria Streptococcus pneumoniae, Haemophilus influenzae, and Neisseria
meningitidis.

Splenectomy Considerations
Any patient scheduled to undergo splenectomy should receive immunization against Steptococcus pneumoniae, Haemophilus influenzae,
and Neisseria meningitidis, preferably at least 2 weeks before surgery. Straightforward splenectomy can be performed laparoscopically,
whereas complicated splenectomy in patients with massive spleens or severe thrombocytopenia are more likely to require laparotomy.
During surgery, the spleen is mobilized from the splenorenal, phrenosplenic, and lienocolic ligaments. The main artery is found on the
superior border of the pancreas and should generally
be taken early. Care must be taken to control the short gastric arteries entering the greater curve of the stomach. The tail of the P
pancreas nestles into the splenic hilum and can be damaged, causing a pancreatic leak or fistula. Patients generally have nasogastric
decompression to prevent the stomach from distending and hemorrhage along the greater curve. Patients with compromised immune
systems, including young children and transplant recipients, should be considered for long-term oral penicillin prophylaxis.

SPLENIC ABSCESS
Splenic abscesses most commonly occur as a consequence of other intra-abdominal infection. They can also occur after arterial
embolization of either the liver or spleen. Diabetes seems to be a risk factor. Patients generally present with left upper quadrant pain,
fever, and leucocytosis. Antibiotics, catheter-based drainage, and splenectomy are all appropriate management options.

KEY POINTS
The spleen is a lymphatic organ that plays roles in antigen recognition and blood filtering.
Accessory spleens occur in 25% of patients and are most commonly found in the splenic hilum.
Arterial blood is supplied via the splenic artery, the short gastric arteries, and branches of the left
gastroepiploic artery.
Hemorrhage secondary to trauma is the most common indication for splenectomy.
Nonoperative management or organ-sparing splenorrhaphy can be attempted to avoid the risk of overwhelming
postsplenectomy sepsis, especially in children.
Hemodynamically stable patients with splenic injury can be managed nonoperatively.
Immune thrombocytopenic purpura, hypersplenism, and specific hemolytic anemias are disease states for which
splenectomy may be indicated.
Splenectomy for staging Hodgkin disease is now rarely performed, because of improved imaging modalities
(computed tomography scan) and the success of chemotherapy.
The risk of overwhelming postsplenectomy sepsis is greater in children than in adults. High fever and septic shock
are often accompanied by disseminated intravascular coagulation.
Vaccination against Streptococcus pneumoniae, Haemophilus influenzae, and Neisseria meningitidis should be
administered to all surgically and functionally asplenic patients, because these encapsulated organisms are
responsible for causing overwhelming postsplenectomy sepsis.
Authors: Karp, Seth J.; Morris, James P.G.; Zaslau, Stanley
Title: Blueprints Surgery, 5th Edition

Copyright ©2008 Lippincott Williams & Wilkins

> Table of Contents > Part II - Gastrointestinal and Abdominal > Chapter 9 - Pancreas

Chapter 9
Pancreas

The pancreas is a key regulator of digestion and metabolism through both endocrine and exocrine functions.
Disorders of surgical importance include acute pancreatitis, chronic pancreatitis, and pancreatic cancer.

EMBRYOLOGY
Formation of the pancreas begins during the first few weeks of gestation, with the development of the
ventral and dorsal pancreatic buds. Clockwise migration of the ventral bud allows fusion with the larger
dorsal bud, creating the duct of Wirsung, which is the main pancreatic duct. Failure of this process results
in pancreas divisum, wherein the duct of Santorini drains a portion of the exocrine pancreas through a
separate minor duodenal papilla (Fig. 9-1). This anatomic variant is associated with pancreatitis. Annular
pancreas occurs when the ventral bud fails to rotate, resulting in pancreatic tissue completely or partially
encircling the second portion of the duodenum. This situation may result in duodenal obstruction, requiring
duodenojejunostomy or gastrojejunostomy in some cases.
Figure 9-1 • Pancreas divisum. Instead of draining through a common
pancreatic duct, the ducts of Wirsung (major duct) and Santorini (minor
duct) enter the duodenum separately.

ANATOMY AND PHYSIOLOGY


The pancreas is a retroperitoneal structure located posterior to the stomach and anterior to the inferior
vena cava and aorta. This yellowish, multilobed gland is divided into four portions: head, which includes the
uncinate process; neck; body; and tail (Fig. 9-2). It lies in a transverse orientation, with the pancreatic
head in intimate association with the C loop of the duodenum, the body draped over the spine, and the tail
nestled in the splenic hilum.
Figure 9-2 • Regional anatomy of the pancreas.

The arterial blood supply to the pancreatic head is derived from the anterior and posterior
pancreaticoduodenal arteries (Fig. 9-3). These arteries arise from the superior pancreaticoduodenal artery,
which is a continuation of the gastroduodenal artery, and from the inferior pancreaticoduodenal artery,
which arises from the superior mesenteric artery. The body and tail are supplied from branches of the
splenic and left gastroepiploic arteries. Venous drainage follows arterial anatomy and enters the portal
circulation.

Figure 9-3 • Blood supply of the pancreas. The celiac axis gives rise to the
common hepatic artery and splenic artery. The common hepatic artery gives
rise to the gastroduodenal artery, which supplies the head of the pancreas,
and the splenic artery gives rise the pancreatic magna and many small
branches into the body and tail. In addition, the superior mesenteric artery
gives rise to the inferior pancreaticoduodenals arteries, which supply the
head and the uncinate process.

Sympathetic innervation is responsible for transmitting pain of pancreatic origin, whereas efferent
postganglionic parasympathetic fibers innervate islet, acini, and ductal systems. In patients with intractable
pain from chronic pancreatitis who have failed operative drainage or resection, splanchnicectomy
(sympathectomy) can be performed to interrupt sympathetic nerve fibers.

The functional units of the endocrine pancreas are the islets of Langerhans, which are multiple small
endocrine glands scattered throughout the pancreas that make up only 1% to 2% of the total pancreatic cell
mass. The bulk of the pancreatic parenchyma is exocrine tissue. Four islet cell types have been identified: A
cells (alpha), B cells (beta), D cells (delta), and F cells (pancreatic polypeptide [PP] cells).

Alpha cells produce glucagon, which is secreted in response to stimulation by amino acids, cholecystokinin,
gastrin, catecholamines, and sympathetic and parasympathetic nerves. The role of alpha cells is to ensure
an ample supply of circulating nutritional fuel during periods of fasting. It promotes hepatic
gluconeogenesis and glycogenolysis and inhibits gastrointestinal motility and gastric acid secretion.

The largest percentage of islet volume is occupied by the insulin-producing beta cells. The main function of P
insulin is to promote the storage of ingested nutrients. Insulin is released into the portal circulation in
response to glucose, amino acids, and vagal stimulation. Insulin has both local and distant anabolic and
anticatabolic activity. Local paracrine function is the inhibition of glucagon secretion by alpha cells. In the
liver, insulin inhibits gluconeogenesis, promotes the synthesis and storage of glycogen, and prevents
glycogen breakdown. In adipose tissue, insulin increases glucose uptake by adipocytes, promotes
triglyceride storage, and inhibits lipolysis. In muscle, it promotes the synthesis of glycogen and protein.

Somatostatin is secreted by islet delta cells in response to the same stimuli that promote insulin release.
Pancreatic somatostatin slows the movement of nutrients from the intestine into the circulation by
decreasing pancreatic exocrine function, reducing splanchnic blood flow, decreasing gastrin and gastric acid
production, and reducing gastric emptying time. Somatostatin also has paracrine-inhibitory effects on
insulin, glucagon, and PP secretion.

F cells secrete PP after ingestion of a mixed meal. The function of PP is unknown; however, it may be
important in priming hepatocytes for gluconeogenesis. Patients with pancreatic endocrine tumors have been
noted to have elevated levels of circulating PP.

The basic functional unit of the exocrine pancreas is the acinus. Acinar cells contain zymogen granules in
the apical region of the cytoplasm. Acini are drained by a converging ductal system that terminates in the
main pancreatic excretory duct. The centroacinar cells of individual acini form the origins of the ducts, with
intercalated duct cells lining the remainder.

Exocrine pancreatic secretions are products of both ductal and acinar cells. Ductal cells contribute a clear, P
basic-pH, isotonic solution of water and electrolytes, rich in bicarbonate ions. Secretion of pancreatic fluid
is principally controlled by secretin, a hormone produced in the mucosal S cells of the crypts of Lieberkühn
in the proximal small bowel. The presence of intraluminal acid and bile stimulates secretin release, which
binds pancreatic ductal cell receptors, causing fluid secretion.
Pancreatic digestive enzymes are synthesized by and excreted from acinar cells after stimulation by
secretagogues (cholecystokinin, acetylcholine). Excreted enzymes include endopeptidases (trypsinogen,
chymotrypsinogen, and proelastase) and exopeptidases (procarboxypeptidase A and B). Other enzymes
produced are amylase, lipase, and colipase. All peptidases are excreted into the ductal system as inactive
precursors. Once in the duodenum, trypsinogen is converted to the active form, trypsin, by interaction with
duodenal mucosal enterokinase. Trypsin, in turn, serves to activate the other excreted peptidases. In
contrast to the peptidases, the enzymes amylase and lipase are excreted into the ductal system in their
active forms.

ACUTE PANCREATITIS

PATHOGENESIS
Acute pancreatitis is a disease of glandular enzymatic autodigestion that has varying presentations, ranging
from mild parenchymal edema to life-threatening hemorrhagic pancreatitis. Multiple causes have been
identified, with alcoholism and gallstone disease accounting for 80% to 90% of cases among Western
populations. The remaining cases are attributed to hyperlipidemia, hypercalcemia, trauma, infection,
ischemia, trauma from endoscopic retrograde cholangiopancreatography (ERCP), and cardiopulmonary
bypass (Table 9-1). The exact pathogenesis of acute pancreatitis remains unclear. One possibility is that
obstruction of the ampulla of Vater by gallstones, spasm, or edema causes elevated intraductal pressure
and bile reflux into the pancreatic duct. Activation and extravasation of intraparenchymal enzymes results
in tissue destruction and ischemic necrosis of the pancreas and retroperitoneal tissues.

TABLE 9-1 Causes of Acute Pancreatitis

Alcohol

Biliary tract disease

Hyperlipidemia

Hypercalcemia

Familial

Trauma—external, operative, ERCP

Ischemic—hypotension, cardiopulmonary bypass


Pancreatic duct obstruction—tumor, pancreas divisum, ampullary
stenosis, Ascaris infestation

Duodenal obstruction

Infection—mycoplasma, mumps, Coxsackie

ERCP, endoscopic retrograde cholangiopancreatography.

History
Because of the different degrees of pancreatic tissue destruction seen in cases of pancreatitis, the
presentation of acute disease is varied, and diagnosis may be difficult. Important past medical history
includes information regarding prior episodes of pancreatitis, alcoholism, and biliary colic. Patients present
with upper abdominal pain (often radiating to the back), nausea, vomiting, and a low-grade fever. A severe
attack of pancreatitis is manifested by hypotension, sepsis, and multiorgan failure. Patients with an
alcoholic cause usually experience pain 12 to 48 hours after alcohol ingestion.

PHYSICAL EXAMINATION
Patients have upper abdominal tenderness, usually without peritoneal signs. The abdomen may be slightly
distended secondary to a paralytic ileus. Low-grade fever and tachycardia are common.

DIFFERENTIAL DIAGNOSIS
Acute pancreatitis is often difficult to differentiate from other causes of upper abdominal pain. The clinical
presentation may mimic that of a perforated peptic ulcer or acute biliary tract disease. Other conditions
that may have similar presentations are acute intestinal obstruction, acute mesenteric thrombosis, and a
leaking abdominal aortic aneurysm.

DIAGNOSTIC EVALUATION
More than 90% of patients who present with acute pancreatitis have an elevated serum amylase. However,
amylase levels are relatively nonspecific, because many other intra-abdominal conditions, including P
intestinal obstruction and perforated peptic ulcer, may cause amylase elevation. If the diagnosis is unclear,
a lipase level should also be measured, because it is solely of pancreatic origin.

Leukocytosis >10,000/mL is common, and hemoconcentration with azotemia may also be present because of
intravascular depletion secondary to significant third-space fluid sequestration. Hyperglycemia frequently
occurs as a result of hypoinsulinemia, and hypocalcemia occurs from calcium deposition in areas of fat
necrosis.
Routine chest x-ray may reveal a left pleural effusion, known as a sympathetic effusion, secondary to
peripancreatic inflammation. Air under the diaphragm indicates perforation of a hollow viscus, such as a
perforated peptic ulcer.

The classic radiographic finding on abdominal x-ray is a sentinel loop of dilated mid- to distal duodenum or
proximal jejunum located in the left upper quadrant, adjacent to the inflamed pancreas. In cases of
gallstone pancreatitis, radiopaque densities (gallstones) may be seen in the right upper quadrant.

Ultrasonography is the preferred modality for imaging the gallbladder and biliary ductal system, because it
is more sensitive as compared with computed tomography (CT) scan. Ultrasound is the study of choice for
the detection of cholelithiasis during the workup of gallstone pancreatitis.

CT is the most sensitive radiologic study for confirming the diagnosis of acute pancreatitis. Virtually all
patients show evidence of either parenchymal or peripancreatic edema and inflammation. CT is also
valuable in defining parenchymal changes associated with pancreatitis, such as pancreatic necrosis and
pseudocyst formation. For severe cases, CT scanning with intravenous contrast is important for determining
the percentage of pancreatic necrosis, which is a predictor of infectious complications. CT-guided
interventional techniques can also be performed to tap peripancreatic fluid collections to rule out
infection.

ERCP is useful for imaging the biliary ductal system and can be a diagnostic, as well as a therapeutic,
modality. In the case of gallstone pancreatitis, the presence of common bile duct stones
(choledocholithiasis) can be confirmed and the stones extracted endoscopically. Magnetic resonance
cholangiopancreatography is a newer noninvasive technique that is a diagnostic, but not therapeutic,
modality.

DISEASE SEVERITY SCORES


Because the clinical course of pancreatitis can vary from mild inflammation to fatal hemorrhagic disease,
prompt identification of patients at risk for development of complications may improve final outcomes. The
Ranson criteria are well-known prognostic signs used for predicting the severity of disease on the basis of
clinical and laboratory results (Table 9-2). The ability to predict a patient's risk of infectious complications
and mortality at the time of admission and during the initial 48 hours allows appropriate therapy to be
instituted early in hospitalization. Mortality is correlated with the number of criteria present at admission
and during the initial 48 hours after admission: 0 to two criteria, 1% mortality; three to four criteria, 16%;
five to six criteria, 40%; and seven to eight criteria, 100%. Since the publication of the Ranson criteria in
1974, newer severity scores have been developed (Acute Physiology and Chronic Health Evaluation II score)
to estimate mortality risk in critically ill patients. This calculation uses 12 variables in intensive care unit
patients to predict mortality.

TABLE 9-2 Ranson Criteria for Acute Pancreatitis

At Admission During Initial 48 Hours

Age >55 years Hematocrit decrease >10%


At Admission During Initial 48 Hours

WBC >16,000/µL Blood urea nitrogen increase


>5 mg/dL

Serum glucose Calcium decrease to


>200 mg/dL <8 mg/dL

Serum LDH Arterial PO2 <60 mm Hg


>350 U/L

AST >250 U/mL Base deficit <4

Fluid sequestration <6 L

WBC, white blood cells; LDH, lactate dehydrogenase; AST, aspartate


aminotransferase.

TREATMENT
Medical treatment of pancreatitis involves supportive care of the patient and treatment of complications as
they arise. No effective agent exists to reverse the inflammatory response initiated by the activated
zymogens. With adequate care, however, most cases are self-limited and resolve spontaneously.

Hydration is the most important early intervention in treating acute pancreatitis, because significant third- P
spacing occurs secondary to parenchymal and retroperitoneal inflammation. Hypovolemia must be avoided
because pancreatic ischemia may quickly develop secondary to inadequate splanchnic blood flow.

Traditional treatment calls for putting the pancreas "to rest" by not feeding the patient (NPO). The goal is to
decrease pancreatic stimulation, thereby suppressing pancreatic exocrine function. Nasogastric suction can
be instituted to treat symptoms of nausea and vomiting.

Antibiotics should be initiated if there is infected pancreatic necrosis, as confirmed by biopsy. In the
absence of this, antibiotics are widely used for pancreatitis, but their efficacy is controversial.

If the severity of disease necessitates a prolonged period of remaining NPO, an alternative method of
administering nutrition must be instituted. Intravenous nutrition (total parenteral
nutrition/hyperalimentation) is commonly initiated. Once pancreatic inflammation resolves, gradual
advancement of oral intake proceeds, beginning with low-fat, high-carbohydrate liquids to avoid pancreatic
stimulation.

Oxygen therapy may be necessary for treatment of hypoxia, which often occurs secondary to pulmonary
changes thought to be due to circulating mediators. Evidence of atelectasis, pleural effusion, pulmonary
edema, and adult respiratory distress syndrome may be seen on chest radiograph.

Surgical treatment of acute pancreatitis is directed at complications that develop secondary to the
underlying disease process. During the early phase of pancreatitis, areas of necrosis may form because of
tissue ischemia from enzyme activation, inflammation, and edema. Necrotic areas eventually liquefy and
may become infected if they are unable to reabsorb and heal. CT scanning with intravenous contrast is the
key test for defining the extent of pancreatic necrosis. Nonenhancement of 50% or more of the pancreas on
CT scan is a strong predictor for the development of infectious complications. Infected collections require
surgical debridement and drainage to avoid fatal septic complications.

Initially, collections around the pancreas during episodes of pancreatitis are termed acute pancreatic fluid
collections. Peripancreatic collections that persist after the inflammatory phase has subsided may develop a
thickened wall, or "rind." Such collections are called pancreatic pseudocysts. To alleviate symptoms or
prevent major complications, surgical drainage is usually required for cysts >6 cm in diameter that have
persisted for more than 6 weeks. Standard therapy is internal drainage into the stomach, duodenum, or
small intestine.

During the later stage of disease, abscess formation may occur. The pathogenesis is a progression: an
ischemic parenchyma progresses to necrosis and is seeded by bacteria, with eventual abscess formation.
Most bacteria are of enteric origin, and standard antibiotic therapy is insufficient treatment. If surgical
drainage and debridement are not performed, the mortality nears 100%. Percutaneous drainage is usually
inadequate, because only the fluid component is removed and the necrotic infected tissue remains. These
patients are often remarkably sick. Multiple debridements may be required when the episode is severe. It is
not uncommon for anasarca to result from massive volume resuscitation, and it may be difficult to close the
abdomen. In this case, a temporary closure should be used. Definitive closure is performed as early as
possible after the infection is controlled and the edema has improved. Operations to remove infected
pancreatic tissue can be extremely bloody and adequate access and blood should be available.

Hemorrhage secondary to erosion of blood vessels by activated proteases can be a life-threatening


complication. Often it is the main hepatic, gastroduodenal, or splenic artery that bleeds. All efforts should
be made to control this with angiography, because it can be exceedingly difficult to control this lesion in
the operating room.

CHRONIC PANCREATITIS
Of patients with acute pancreatitis, a small number progress to chronic pancreatitis. The chronic form of
disease is characterized by persistent inflammation that causes destructive fibrosis of the gland. The
clinical picture is of recurring or persistent upper abdominal pain with evidence of malabsorption,
steatorrhea, and diabetes.

PATHOGENESIS
Chronic pancreatitis can be categorized into two forms: calcific pancreatitis, usually associated with
persistent alcohol abuse, and obstructive pancreatitis, secondary to pancreatic duct obstruction. Alcohol-
induced calcific pancreatitis is the most common form of disease in Western populations. Proposed
mechanisms of disease include ductal plugging and occlusion by protein and mineral precipitates. The
resulting inflammation and patchy fibrosis
subsequently lead to parenchymal destruction and eventual atrophy of the gland. Obstructive chronic P
pancreatitis is due to ductal blockage secondary to scarring from acute pancreatitis or trauma, papillary
stenosis, pseudocyst, or tumor. This blockage results in upstream duct dilatation and inflammation.

HISTORY
Abdominal pain is the principal presenting complaint and the most frequent indication for surgery. The pain
is upper abdominal, is either intermittent or persistent, and frequently radiates to the back. Patients are
often addicted to narcotic pain relievers. Other symptoms result from exocrine insufficiency
(malabsorption) and endocrine insufficiency (diabetes mellitus).

DIAGNOSTIC EVALUATION
Given the functional reserve of the pancreas, the diagnosis of chronic pancreatitis is best made using
imaging techniques that detect pancreatic morphologic changes rather than tests of glandular function.
Exocrine function may be evaluated by the secretin cholecystokinin test, which is now rarely used.

The radiologic signs of chronic pancreatitis include a heterogeneously inflamed or atrophied gland, a dilated
and strictured pancreatic duct, and the presence of calculi. Ultrasonography and CT are useful initial
imaging procedures; however, ERCP is the most accurate means of diagnosing chronic pancreatitis, because
it clearly defines the pathologic changes of the pancreatic ductal system and the biliary tree.

P
TREATMENT
Effective treatment of chronic abdominal pain is often the focus of care for patients with chronic
pancreatitis. Opiates are useful for controlling visceral pain; however, many patients become opiate depen-
dent over the long term. Alcohol nerve blocks of the celiac plexus have only moderate success.

Pancreatic exocrine insufficiency is treated with oral pancreatic enzymes, and insulin is used to treat
diabetes mellitus. Ethanol intake by the patient must cease.

Surgical intervention is undertaken only if medical therapy has proved unsuccessful in relieving chronic
intractable pain. Functional drainage of the pancreatic duct and the resection of diseased tissue are the
goals of any procedure. Given ERCP and CT findings, the correct operation can be planned.

For patients with a "chain of lakes"–appearing pancreatic duct, caused by sequential ductal scarring and
dilatation, a longitudinal pancreaticojejunostomy (Puestow procedure) is indicated to achieve adequate
drainage. A Roux-en-Y segment of proximal jejunum is anastomosed side-to-side with the opened pancreatic
duct, facilitating drainage (Fig. 9-4). Distal pancreatic duct obstruction causing localized distal parenchymal
disease is best treated by performing a distal pancreatectomy.
Figure 9-4 • Longitudinal pancreaticojejunostomy used in the treatment of
chronic pancreatitis.

ADENOCARCINOMA OF THE PANCREAS

EPIDEMIOLOGY
Pancreatic adenocarcinoma is a leading cause of cancer death, trailing other cancers such as lung and
colon. Men are affected more than women by a two-to-one ratio. Risk factors for development of
pancreatic cancer are increasing age and cigarette smoking. The peak incidence is in the fifth and sixth
decades. Ductal adenocarcinoma accounts for 80% of the cancer types and is usually found in the head of
the gland. Local spread to contiguous structures occurs early, and metastases to regional lymph nodes and
liver follow.

HISTORY
The signs and symptoms of carcinoma of the head of the pancreas are intrinsically related to the regional
anatomy of the gland. Patients classically complain of obstructive jaundice, weight loss, and constant deep
abdominal pain owing to peripancreatic tumor infiltration. Patients may present with jaundice and a
palpable nontender gallbladder, indicating tumor obstruction of the distal common bile duct (Courvoisier
sign). Pruritus often accompanies the development of jaundice.

DIFFERENTIAL DIAGNOSIS
The differential diagnosis of malignant obstructive jaundice includes carcinomas of the ampulla of Vater,
pancreatic head, distal common bile duct, or duodenum.

DIAGNOSTIC EVALUATION
The most common laboratory abnormalities are elevated alkaline phosphatase and direct bilirubin levels,
indicating obstructive jaundice. The average bilirubin level in neoplastic obstruction is typically higher than
that seen in bile duct obstruction from gallstone disease.

CT and ERCP are the modalities of choice for evaluating pancreatic cancer. CT reveals the location of the
mass, the extent of tumor invasion or metastasis, and the degree of ductal dilatation (Fig. 9-5). ERCP
defines
the ductal anatomy and the extent of ductal obstruction and provides biopsy specimens for tissue diagnosis. P
Drainage stents can be placed into the common bile duct during ERCP for biliary tree decompression.
Imaging information suggesting unresectability includes local tumor extension, contiguous organ invasion,
superior mesenteric vein or portal vein invasion, ascites, and distant metastases.
Figure 9-5 • Pancreatic adenocarcinoma. Computed tomography image
demonstrates an endoscopically placed biliary stent (curved arrow), passing
through a mass (M) in the head of the pancreas. The pancreatic
adenocarcinoma has almost obliterated the superior mesenteric vein
(arrowhead) and abuts the superior mesenteric artery (open arrow). A
retrocaval lymph node is noted (thick arrow).
From Kelsen DP, Daly JM, Kern SE, et al. Gastrointestinal Oncology:
Principles and Practice. Philadelphia: Lippincott Williams & Wilkins, 2002.

TREATMENT
The operation for resectable tumors in the head of the pancreas is pancreaticoduodenectomy (Whipple
procedure; Fig. 9-6). This major operation entails the en bloc resection of the antrum, duodenum, proximal
jejunum, head of pancreas, gallbladder, and distal common bile duct.
Figure 9-6 • Pancreaticoduodenectomy (Whipple procedure). Preoperative
anatomic relationships (A) and postoperative reconstruction (B).

PROGNOSIS
Long-term survival for pancreatic cancer remains dismal, and most patients die within 1 year of diagnosis.
The 5-year survival rate for all patients with tumors of the head of the pancreas is approximately 3%. For
individuals with tumors amenable to Whipple resection, the 5-year survival rate is only 10% to 20%. Tumors
of the body and tail are invariably fatal, because diagnosis is usually made at a more advanced stage as a
result of the lack of early obstructive findings.

OTHER PANCREATIC NEOPLASMS


Approximately 20% of all pancreatic cystic lesions will not be pseudocysts. These lesions may be benign or
malignant, but even the malignant ones carry better prognosis than adenocarcinoma. Simple cysts are
usually congenital. When they appear in children, no treatment is necessary. In patients with polycystic
kidney disease, 10% will also have pancreatic cysts. They do not require treatment.

More complicated cystic disease of the pancreas includes serous cystadenoma and mucinous cystic
neoplasms. Serous cystadenomas most often occur in women between 30 and 50 years of age. They are
generally asymptomatic. Ultrasound reveals a complex low-density mass with fine septae. The fluid is
generally clear. Resection is generally curative. Mucinous cystic neoplasms have malignant potential and
should be resected with a margin, usually requiring formal partial pancreatectomy. Survival is excellent at 5
years at approximately 70%.

Solid and cystic papillary neoplasms tend to present as large lesions in young women. Resection is often
curative.

Intraductal papillary mucinous neoplasms are distinguished from the other cystic lesions of the pancreas in
that the pancreatic duct is enlarged because of mucin deposition. ERCP demonstrates mucin in the duct.
These lesions have malignant potential and require resection. Long-term prognosis is excellent with
resection.

P
FUNCTIONAL TUMORS OF THE PANCREAS

INSULINOMA
This is the most common tumor of pancreatic islet cells. It is more common in women than men and
commonly affects middle-aged patients. Hypersecretion of insulin causes symptoms. Diagnosis is made from
the classic Whipple triad of low fasting blood sugars (less than 45 mg/dL); symptoms of hypoglycemia,
including palpitations, tachycardia, and shaking; and resolution by administration of exogenous glucose.
Patients can lose consciousness or experience seizures. Although most are sporadic, any patient with this
syndrome should be suspected of having multiple endocrine neoplasia (MEN) type I, especially if the lesions
are multiple. Patients will have an immunoreactive insulin-to-glucose ratio of greater than 0.3 and elevated
proinsulin and C-peptide levels. These lesions may be difficult to find because of their small size. Workup
includes CT or magnetic resonance imaging to localize the lesion and determine whether metastases are
present. Somatostatin receptor scintigraphy may be useful but often fails to localize the lesion. Even if the
lesion is not localized, surgical exploration is indicated to try to locate and resect the tumor. Intraoperative
ultrasound should be used in this case. Even when metastatic disease is present, resection may be indicated
for symptomatic relief.

Gastrinoma
This tumor is defined by gastrin hypersecretion, causing the severe peptic ulcer disease of Zollinger-Ellison
syndrome. The syndrome is slightly more common in men than in women and generally affects patients in
middle age. Eighty percent of cases are sporadic, whereas 20% will occur in association with MEN type I.
These lesions most commonly occur in the pancreas or duodenum, but can also occur in other areas.
Patients present with symptoms of acid hypersecretion, ulcers, and secretory diarrhea. This lesion accounts
for less than 1% of all ulcer disease. Patients will have elevated gastrin levels and basal acid output.
Secretin stimulation test is used to confirm the diagnosis. CT, magnetic resonance imaging, and ultrasound
all have some utility in localizing the tumor, but somatostatin scintigraphy is the test of choice. In sporadic
cases not associated with MEN type I, treatment is medical management of gastric hypersecretion and then
resection, given the malignant potential of the lesion. In patients with MEN type I, management must be
coordinated with treatment of the other abnormalities.

OTHER TUMORS
Other, rarer types of neuroendocrine tumors that may affect the pancreas may elaborate vasoactive
intestinal peptide, pancreatic polypeptide, adrenocorticotropic hormone, and growth hormone–releasing
factor.

Key Points
The pancreas is a retroperitoneal structure consisting of a head, neck, body, and
tail.
The duct of Wirsung drains the mature pancreas. Occasionally, a duct of Santorini
drains through a separate minor papilla.
Congenital variants arise from aberrant pancreatic bud migration.
The islets of Langerhans of the endocrine pancreas include alpha cells (glucagon), beta
cells (insulin), delta cells (somatostatin), and pancreatic polypeptide cells.
In Western populations, alcohol ingestion and gallstone disease are primarily responsible
for acute pancreatitis, and alcohol use is primarily responsible for chronic pancreatitis.
Ranson criteria are used to predict the severity of the disease and to estimate mortality.
Pancreatitis is usually self-limiting and resolves with supportive care; however,
complications such as chronic pseudocyst, abscess, necrosis, or hemorrhage are treated
surgically.
Surgical treatment includes drainage procedures (longitudinal pancreaticojejunostomy
[Puestow procedure]) or pancreatic resection (distal pancreatectomy).
In pancreatic cancer, obstructive jaundice, weight loss, and abdominal pain are common
findings.
The Courvoisier sign is jaundice and a nontender palpable gallbladder, indicating tumor
obstruction of the distal common bile duct.
Computed tomography and endoscopic retrograde cholangiopancreatography are used to
determine tumor resectability.
Resectable tumors of the head of the pancreas are removed by pancreaticoduodenectomy
(Whipple procedure). Prognosis is generally poor.
Authors: Karp, Seth J.; Morris, James P.G.; Zaslau, Stanley
Title: Blueprints Surgery, 5th Edition

Copyright ©2008 Lippincott Williams & Wilkins

> Table of Contents > Part II - Gastrointestinal and Abdominal > Chapter 10 - Bariatric Surgery

Chapter 10
Bariatric Surgery

Bariatric surgery, or weight loss surgery, limits the amount of food the stomach can hold by reducing the
stomach's capacity to a few ounces. In addition to reducing food intake, some weight loss surgeries also
alter the digestive process, which curbs the amount of calories and nutrients absorbed.

ANATOMY
The relevant anatomy involves the stomach and proximal small bowel. These are well-vascularized organs,
and although they can withstand significant transections and bypass, it remains critical not to compromise
the blood supply. See Chapter 3, Stomach and Duodenum, and Chapter 4, Small Intestine.

MORBID OBESITY

EPIDEMIOLOGY
The increased risk for morbidity and mortality is proportional to the degree of a person's excess weight. The
most common way to quantify obesity is by the body mass index (BMI), which is calculated as [weight
(kilograms)/height (meters)2]. The BMI associated with lowest mortality is between 20 and 25 kg/m2. An
adult with a BMI >25 kg/m2 is considered overweight; an adult with BMI >30 kg/m2 is considered obese.
Morbid obesity, approximately equivalent to a person being 100 pounds overweight, can be a life-
threatening condition. A person is classified as being morbidly obese if he or she has a BMI ≥40 kg/m2, or a
BMI of ≥35 kg/m2 with an obesity-related disease, such as type 2 diabetes, heart disease, or sleep apnea.
Approximately 100 million Americans are obese, and 15 million are morbidly obese. Obesity is rapidly
becoming an epidemic in the United States, with prevalence rates of 15% in 1980, and increasing to 33% in
2004. Obesity costs the U.S. healthcare system an estimated $117 billion annually, according to the National
Institute of Diabetes and Digestive and Kidney Diseases. After tobacco, obesity is the second leading cause
of preventable death in the United States.

PATHOPHYSIOLOGY
The ultimate biologic basis of obesity is unknown. A sedentary lifestyle and the availability of high-caloric
foods certainly contribute to this disease process. This disorder, nevertheless, is accompanied by a
reduction in life expectancy, which is due in large part to the complications associated with diabetes,
hypertension, and sleep apnea.

HISTORY
The approach to the morbidly obese patient has to take into account the weight history, dietary habits,
lifestyle, exercise tolerance, and medical comorbidities of the individual. Previous attempts at nonsurgical
weight loss should be documented. The dietary and weight history should focus on identifying eating
disorders, as well as any emotional or psychiatric conditions that may be linked. Counseling a patient to
engage in an active exercise regimen is also critical.

The medical history should aim at ruling out any metabolic causes of obesity, such as hypothyroidism and
Cushing's syndrome. Obesity is also related to a host of medical comorbidities that should be identified and
stabilized preoperatively. These are listed in Table 10-1. As with any other major operation, surgical risk P
should be individually assessed on the basis of the medical history.

TABLE 10-1 Medical Comorbidities of Obesity

Diabetes mellitus Hypertension

Obstructive sleep apnea Hyperlipidemia

Coronary artery disease Asthma

Pseudotumor cerebri Infertility (men and


women)

Polycystic ovary Thromboembolism


syndrome

Osteoarthritis Low back pain

Gout Gastroesophageal reflux

Gallstones Nonalcoholic
steatohepatosis

Urinary incontinence Skin infections/ulcerations

Depression Cancer (colorectal, breast,


prostate, endometrial)

Pulmonary hypertension

PHYSICAL EXAMINATION
A general physical examination should be performed preoperatively. Specific to the bariatric population, the
airway should be evaluated for difficulty of intubation as well as risk for obstructive sleep apnea. The
abdomen should be assessed for the degree of central obesity, as well as prior surgical incisions, which may
make laparoscopic surgical approaches difficult.

DIAGNOSTIC EVALUATION
Patients are considered suitable candidates for bariatric surgery if they meet criteria set by the 1991
National Institutes of Health consensus conference on bariatric surgery. Patients must:

Have a BMI >40, or a BMI >35 with significant weight-related comorbidities, such as diabetes,
hypertension, or sleep apnea.

Have no metabolic abnormalities that could cause weight gain.

Have attempted and failed at nonsurgical weight loss.

Be psychologically stable with no identifiable eating disorders.

As for any major surgical procedure, the preoperative workup should include an ECG and possibly stress
testing to rule out cardiac disease. A chest radio-graph may show an enlarged heart or pulmonary
congestion. A sleep study may be necessary to evaluate for sleep apnea, which may necessitate the use of a
continuous positive airway pressure machine during sleep. Patients with a significant history of heartburn or
reflux should undergo an upper gastrointestinal radiograph to rule out a hiatal or paraesophageal hernia,
which may alter the surgical plan.

TREATMENT
Nonsurgical Options
Before surgery, all potential candidates should have attempts at lifestyle modifications, including
supervised diet and exercise plans. Pharmacologic options include sympathomimetic drugs, such as
phentermine or sibutramine (Meridia), or drugs that alter fat digestion, such as orlistat (Xenical). However,
most studies have shown that medical management of obesity fails in up to 95% of cases and that most
patients regain a substantial portion of their excess weight as soon as medications are discontinued.

Surgical Options
Bariatric surgery has been recognized by the National Institutes of Health as the most effective method to
achieve long-term weight loss. A myriad of bariatric surgical procedures have been devised over the years
and may be classified as either being restrictive, malabsorptive, or a combination thereof. Restrictive
procedures include adjustable gastric banding (AGB), sleeve gastrectomy, and vertical banded gastroplasty.
Malabsorptive procedures such as the jejunal-ileal bypass or biliopancreatic diversion have largely fallen out
of favor because of issues with malnutrition and organ failure.

The two most commonly performed bariatric surgical procedures in modern practice are AGB and the Roux-
en-Y gastric bypass (RYGB). RYGB, the most popular operation in the United States, is both restrictive and
malabsorptive. Both AGB and RYGB can be performed either in traditional open fashion or laparoscopically,
although the latter approach has significant advantages in decreasing pain, recovery time, and wound
complications. Performing these procedures laparoscopically is technically challenging and associated with a
significant learning curve.

RYGB involves the creation of a proximal gastric pouch of approximately 30 mL capacity. Intestinal P
continuity is restored by attaching a limb of proximal jejunum to this gastric pouch with biliopancreatic
continuity established via a jejunojejunostomy (Fig. 10-1).
Figure 10-1 • Roux-en-Y gastric bypass. In a gastric bypass, the stomach is
transected unevenly, creating a small proximal pouch. A Roux-en-Y
gastrojejunostomy is then created. Weight loss occurs as a result of
decreased food intake as well as some malabsorption. Gastric bypass is
currently the most common bariatric procedure performed in the United
States.
From Lawrence PF. Essentials of General Surgery. 4th ed. Philadelphia:
Lippincott Williams & Wilkins, 2006.

Patients may experience dumping syndrome postoperatively with RYGB, especially with consumption of
highly concentrated sweets. Dumping syndrome is manifested by abdominal cramps, nausea, vomiting, and
flushing. In a way, this may be used as an effective form of negative reinforcement to limit the consumption
of sweets. Other risks of this procedure include leakage from the intestinal anastomoses, as well as ulcers,
strictures, and internal hernias.

Morbidly obese patients are inherently high risk given their propensity for deep venous thrombosis formation
and higher incidence of diabetes, hypertension, obstructive sleep apnea, and undiagnosed heart conditions.
These all have to be taken into consideration when planning for RYGB. Mortality rate averages 0.5%.

AGB involves placing a silicone band around the upper portion of the stomach (Fig. 10-2). A catheter
connects the band to an injection chamber, which is implanted subcutaneously. In the postoperative period,
this chamber is used to inflate the band gradually to progressively narrow the gastric inlet and limit caloric
intake by controlling portion size. Because no intestines are bypassed, dumping syndrome does not occur.
Complications of this procedure include slippage of the stomach around the band, erosion of the band into
the lumen of the stomach, and infection, leakage, and migration of the band and injection chamber.
Mortality rate averages 0.05%.
Figure 10-2 • Adjustable gastric band. A band is placed laparoscopically
around the stomach with a subcutaneus port to adjust constriction; this
results in a smaller gastric reservoir.
From Blackbourne LH. Advanced Surgical Recall. 2nd ed. Baltimore:
Lippincott Williams & Wilkins, 2004.

RESULTS
Bariatric surgical procedures achieve variable weight loss results. RYGB, whether performed
laparoscopically or open, is associated with 75% to 80% excess weight loss (EWL). Excess weight is defined as
preoperative weight minus a person's ideal body weight. More importantly, this is associated with resolution
of diabetes in approximately 80% to 85% of patients, hypertension in 70% to 80%, and obstructive sleep
apnea in 75%. Significant improvements are also seen in lipid profiles and other cardiac risk factors. Long
term, there is a 40% decrease in all causes of mortality. Patients need to be monitored
long term for any signs of mineral and vitamin deficiency. There are case reports of comatose patients as a P
result of B-complex vitamin deficiencies, as well as problems with calcium deficiencies and transient hair
loss; patients usually recover after the first 6 months postoperatively. In the long term, up to 50% of
patients who undergo RYGB may have some weight regain, such that the effective long-term excess weight
loss is approximately 65% EWL.

AGB achieves 30% to 40% EWL within a year. However, long-term weight loss is approximately 50% to 55%
EWL at 5 to 10 years postoperatively. Diabetes improves in approximately 60% of patients, as do most other
comorbidities. Good weight loss results with AGB are particularly dependent on compliance with healthy
dietary habits, as patients will not develop any dumping syndrome to dissuade them from consuming
inordinate amounts of sweets.

Regardless of the surgical procedure, success in terms of postoperative weight loss is still highly dependent
upon patient behavior in terms of pursuing healthy dietary and exercise behavior. Patients who continue to
overeat and disregard restrictions on portion size run the risk of dilating the gastric pouch in either a RYGB
or AGB. Weight regain in the long term can often be attributed not to any technical surgical failure, but to
the fact that patients may revert back to unhealthy lifestyle habits.

Key Points

Morbid obesity is defined as a body mass index >40 kg/m2, or a body mass index of
35 to 40 kg/m2 with comorbidities such as hypertension, diabetes, or sleep apnea.
Pharmacologic options such as phentermine, sibutramine, or orlistat may result in
some weight loss but rarely achieve sustained results in the long term.
Surgical options are generally categorized into restrictive and malabsorptive options.
The two most commonly performed bariatric procedures are the Roux-en-Y gastric bypass
and the adjustable gastric band.
Excess weight loss is approximately 65% to 80% with Roux-en-Y gastric bypass and 50%
with adjustable gastric band.
Both procedures are associated with significant improvements in medical comorbidities.
Authors: Karp, Seth J.; Morris, James P.G.; Zaslau, Stanley
Title: Blueprints Surgery, 5th Edition

Copyright ©2008 Lippincott Williams & Wilkins

> Table of Contents > Part II - Gastrointestinal and Abdominal > Chapter 11 - Hernias

Chapter 11
Hernias

HERNIAS
A hernia occurs when a defect or weakness in a muscular or fascial layer allows tissue to exit a
space in which it is normally contained. Hernias are categorized as reducible, incarcerated, or
strangulated. Reducible hernias can be returned to their body cavity of origin. Incarcerated
hernias cannot be returned to their body cavity of origin. Strangulated hernias contain tissue
with a compromised vascular supply. These are particularly dangerous because they lead to
tissue necrosis. If the bowel is involved, this can progress to perforation, sepsis, and death.

EPIDEMIOLOGY
Between 500,000 and 1,000,000 hernia repairs are performed each year. Five percent of people
have an inguinal hernia repair during their lifetime. Half of all hernias are indirect inguinal, and
one fourth are direct inguinal. In decreasing incidence are incisional and ventral (10%), femoral
(6%), and umbilical hernias (3%). Obturator hernias are rare. Indirect inguinal hernias are the
most common in both males and females; overall, hernias have a five:one male predominance.
Femoral hernias are more common in females than in males.

INGUINAL HERNIAS

ANATOMY
The abdominal contents are kept intraperitoneal by fascial and muscular layers: the innermost
layer is the transversalis fascia, with the three more superficial musculofascial layers being the
transversus abdominis, the internal oblique, and the external oblique (see Color Plate 8).

During normal development, the testes begin in an intra-abdominal position and descend through
the internal ring, taking with them a layer of peritoneum that is stretched into a hollow tube
called the processus vaginalis. This invagination of peritoneum, exiting through the deep and
superficial inguinal rings and extending into the scrotum to where the testicle has terminated its
descent, will result in an indirect inguinal hernia unless the following occurs: the tubular
processus vaginalis must collapse upon itself, fusing opposing peritoneal surfaces and obliterating
the tube into a cord-like structure. If this obliteration/fusion of the peritoneal layers does not
occur completely, an indirect inguinal hernia will result (Fig. 11-1A).

Figure 11-1 • A. Indirect inguinal hernia. B. Direct inguinal hernia.


Note that the neck of the indirect inguinal hernia lies lateral to the
inferior epigastric artery, and the neck of the direct inguinal hernia
lies medial to the inferior epigastric artery.
From Snell RS. Clinical Anatomy. 7th ed. Lippincott Williams &
Wilkins, 2003.

A direct inguinal hernia results from a weakness in the abdominal wall (specifically, the
transversalis) in the area just deep to the superficial ring where the spermatic cord exits the
inguinal canal before traveling down into the scrotum (Fig. 11-1B). This area is medial to the
epigastric vessels and is called Hesselbach triangle (defined as the edge of the rectus sheath
medially, the inguinal ligament inferiorly, and the inferior epigastric vessels laterally; Fig. 11-2).

Figure 11-2 • An inguinal hernia is a protrusion of parietal


peritoneum and viscera, such as part of the intestine, through a
normal or abnormal opening from the abdominal cavity. There are
two major categories of inguinal hernia: indirect and direct;
approximately 75% are indirect hernias.
From Moore KL, Agur A. Essential Clinical Anatomy. 2nd ed.
Philadelphia: Lippincott Williams & Wilkins, 2002.
Progressive structural weakening and loss of integrity of the transversalis at this location allows
bulging of the weakened abdominal wall through the area of Hesselbach triangle, the most
medial aspect and back wall of the inguinal canal itself. Once the defect is large enough, bowel
or other abdominal contents can protrude directly through the fascia and out the superficial
ring. Direct inguinal hernias are typically seen in older individuals, whereas indirect inguinal
hernias predominate in the pediatric population. Also, the
external oblique, which inserts onto the pubic tubercle and bounds the external superficial ring, P
has no function in the pathogenesis of hernias.

HISTORY
Patients with reducible inguinal hernias describe an intermittent bulge in the groin or scrotum.
Persistence of the bulge with nausea or vomiting raises concern for incarceration. Severe pain at
the hernia site with nausea or vomiting may occur with strangulation.

PHYSICAL EXAMINATION
With the patient in a standing position, a fingertip is directed upward to find the superficial ring.
This is facilitated in male patients by entering the loose scrotum at its base and following the
course of the
spermatic cord until the superficial ring is encountered. With the fingertip inserted into the ring, P
a bulge is felt in small hernias as the patient coughs or bears down (Fig. 11-3).
Figure 11-3 • Inguinal canal palpation. Gloved hand palpates
inguinal canal by invaginating loose scrotal skin with right index
finger at bottom of scrotal sack. Image shows finger following
spermatic cord.
LifeART image copyright © 2009 Lippincott Williams & Wilkins. All
rights reserved.

In larger hernias, the herniated sac can be palpated without the aid of Valsalva maneuvers. With
the patient in the supine position, reducible hernias can be reduced into the abdomen, whereas
incarcerated hernias cannot. Strangulated hernias are tender as a result of peritoneal
inflammation. Abdominal distention is often encountered as a result of bowel obstruction.

TREATMENT
Traditionally, the simple presence of an inguinal hernia was indication enough for a surgeon to
advise elective repair. Conventional thinking was that the benefit of preventing a hernia accident
(i.e., acute incarceration with bowel obstruction or strangulation of abdominal contents) clearly
outweighed the potential risks and complications of elective hernia repair. However, the natural
history of untreated inguinal hernias and the risks they posed had never been precisely
determined until recently. In 2006, a landmark clinical trial involving 720 men with minimally
symptomatic inguinal hernias randomly assigned patients to either watchful waiting or surgery
with standard open tension-free repair. The key study finding was that the rate of hernia
accidents are extremely rare (1.8 per 1,000 patient-years), leading to the study's conclusion
that, "A strategy of watchful waiting is a safe and acceptable option for men with asymptomatic
or minimally symptomatic inguinal hernias. Acute hernia incarcerations occur rarely, and
patients who develop symptoms have no greater risk of operative complications than those
undergoing prophylactic hernia repair."

Modern hernioplasty is based on the idea of tension-free repair using an implantable


biocompatible prosthe-sis to reconstruct the fascial hernia defect (Figs. 11-4 and 11-5). The
superiority of the concept of mesh tension-free repair versus traditional nonmesh tension-
producing repair was validated by a Cochrane Group review published in 2002. The use of mesh
was found to significantly reduce the risk of hernia recurrence by an astounding 50% to 75%.
Reducible inguinal hernias can be repaired on an elective basis depending on the degree of
patient symptoms. Both open and laparoscopic techniques are acceptable, although a recent
Veterans Affairs study (2004) showed a higher recurrence rate with laparoscopic repair (10%)
when compared with open repair (4%) of primary hernias. The usual indications for laparoscopic
repair are bilaterality and recurrence. Interestingly, the Veterans Affairs study showed the rates
of recurrence after repair of recurrent hernias as being similar with laparoscopic versus open
repairs (10.0% versus 14.1%). Inguinal hernias in adult men should typically be repaired with
mesh to avoid recurrence. In women, the round ligament is ligated and the ring closed so mesh
use is variable.
Figure 11-4 • Plug and patch inguinal hernia repair. Plug.

From Blackbourne LH. Advanced Surgical Recall. 2nd ed. Baltimore:


Lippincott Williams & Wilkins, 2004.
Figure 11-5 • Plug and patch inguinal hernia repair. Patch.

From Blackbourne LH. Advanced Surgical Recall. 2nd ed. Baltimore:


Lippincott Williams & Wilkins, 2004.

When a hernia is not reducible with gentle pressure, a trial of Trendelenburg position, sedation, P
and more forceful pressure can be attempted. If the hernia is thought to be strangulated, then
reduction is contraindicated, because reducing necrotic bowel into the abdomen may produce
bowel perforation and subsequent lethal sepsis. Emergency surgery is indicated in this situation.
A sample operative note for an inguinal hernia is provided in the Appendix.

UMBILICAL HERNIAS
Umbilical hernias occur at the umbilicus and are congenital. They result from incomplete closure
of the fetal umbilical defect. Most resolve spontaneously by the age of 4 years.

EPIDEMIOLOGY
The incidence is 10% of Caucasians and 40% to 90% of African Americans.

HISTORY
The patient may have a bulge at the umbilicus.

TREATMENT
Indications for operation include incarceration, strangulation, or cosmetic concerns.

OTHER HERNIAS
Femoral hernias occur through the femoral canal, located below the inguinal ligament and
defined by the femoral vein laterally, the inguinal ligament superiorly, the lacunar ligament
medially, and Cooper ligament inferiorly.

Incisional hernias occur through a previous surgical incision. Ventral hernias occur in the midline
along the linea alba, usually between the xiphoid and umbilicus. The herniation is through a
weakness of the decussating fibers of the linea alba. Spigelian hernias are found at or below the
junction between the vertically oriented semilunar line lateral to the rectus abdominus muscle
and the transversely oriented arcuate line (linea semicircularis). A pantaloon hernia is a
combined direct and indirect inguinal hernia where both hernias straddle each side of the
inferior epigastric vessels and protrude like pant legs (pantaloon). Richter hernia occurs when a
knuckle of bowel protrudes into a hernia defect, but only a portion of the circumference is
involved and the bowel lumen remains patent. Typically, the result is gangrenous necrosis of the
herniated tissue. A sliding hernia is any hernia that contains intra-abdominal organs. Internal
hernias may occur in patients after abdominal operations when bowel gets trapped as a result of
new anatomic relationships. Obturator hernias are typically found in thin, older adult women
who present with bowel obstruction caused by small bowel herniation into the obturator canal.
Obturator nerve compression by the hernia can result in paresthesias or pain radiating down the
medial thigh (the Howship-Romberg sign). Littre hernia is any groin hernia that contains a
Meckel's diverticulum.

Something that is not a hernia but is often confused as one is diastasis recti. An upper midline
bulge develops when the patient performs a Valsalva maneuver, and herniation is suspected. On
close physical examination, however, no actual defect or "hole" in the fascia is found. Rather, the
linea alba has become attenuated and weak, resulting in widening of the distance between the
rectus muscles. It is this thin, stretched linea alba that bulges out and mimics a large hernia.
Surgical repair is not indicated.
Key Points P

Hernias are extremely common. Inguinal hernias are the most common,
and 5% of people require repair during their lifetime.
Indirect inguinal hernias arising from a patent processus vaginalis are
more common than direct hernias from abdominal wall weakness.
Hernias that become incarcerated should be operated on urgently.
Hernias that become strangulated are a surgical emergency.
Umbilical hernias are congenital, more common in African Americans, and
frequently resolve spontaneously.
Diastasis recti is not a true herniation and does not require surgical repair.
Modern hernioplasty is based on the principle of tension-free repair using an
implantable biocompatible prosthesis to reconstruct the fascial hernia defect.
Authors: Karp, Seth J.; Morris, James P.G.; Zaslau, Stanley
Title: Blueprints Surgery, 5th Edition

Copyright ©2008 Lippincott Williams & Wilkins

> Table of Contents > Part III - Endocrine > Chapter 12 - Thyroid Gland

Chapter 12
Thyroid Gland

Surgical thyroid disease encompasses those conditions in which partial or complete removal of the thyroid
gland is required as a result of goiter and hyperthyroid conditions that are unresponsive to medical
management or benign and malignant neoplastic disease.

ANATOMY
The thyroid gland develops embryologically from the first and second pharyngeal pouches, migrating
caudally to form a butterfly-like structure anterior to the trachea at the level of the second tracheal ring.
The thyroglossal duct is the remnant of the tract. Neural crest cells, which are the source of the C cells that
produce calcitonin, are also involved in the process of thyroid formation. The normal thyroid gland is
bilobed, weighing 15 to 25 g. There is usually an isthmus connecting the two lobes, as well as a pyramidal
lobe, which is superior to the isthmus. The thyroid gland receives its main blood supply from the superior
and inferior thyroidal arter-ies, the latter of which is shared with its neighboring parathyroid glands (Figs.
12-1 and 12-2).
Figure 12-1 • Anatomy of the thyroid gland.
Figure 12-2 • Course of the recurrent laryngeal nerve.

In close proximity to the thyroid are the paired recurrent laryngeal nerves (RLNs), which control the
cricopharyngeus muscle as well as the vocal cords. The RLNs originate from the vagus nerves in the chest.
The right RLN runs behind the subclavian artery at the base of the neck more lateral than the left RLN at
this level. Occasionally the right RLN may be nonrecurrent. The right RLN tends to course lateral to medial
as it approaches the inferior thyroidal artery from the base of the neck, whereas the left RLN courses with
less angulation, running parallel to the tracheoesophageal groove. The RLN enters the larynx at the level of
the inferior constrictor muscles.

The superior laryngeal nerves originate from the vagus nerves as well. It is the external branch of the
superior laryngeal nerves which holds significance, given its motor innervation to the cricothyroid muscle.
This latter structure controls the high pitch of voice. It lies within the superior pole vessels, and care must
be taken when the superior pole of the thyroid is being removed.

PHYSIOLOGY
The purpose of the thyroid gland is to maintain the body's metabolism via the hormone thyroxine (T4) or
triiodothyronine (T3), produced by the follicular cells. A negative feedback loop with the hypothalamus and
pituitary controls the state of the gland via thyroid-stimulating hormone (TSH; Fig. 12-3).
Figure 12-3 • Regulation of thyroid gland secretion. TRH, thyroid-releasing
hormone; TSH, thyroid-stimulating hormone.
From Premkumar K. The Massage Connection Anatomy and Physiology.
Baltimore: Lippincott Williams & Wilkins, 2004.

GRAVE'S DISEASE AND TOXIC NODULAR GOITER


Grave's disease and toxic nodular goiter (TNG) encompass the majority of hyperthyroidism cases. TNG has
also been referred to as Plummer's disease. Whereas Grave's disease is an autoimmune process in which the
body's own antibodies stimulate the TSH receptor, causing excess T4 and T3 in addition to gland growth, the
mechanism of action of TNG is through autoproduction of thyroid hormones regardless of TSH control.

P
HISTORY AND PHYSICAL EXAMINATION
Both diseases clinically can lead to similar symptoms and signs, such as anxiety, heat sensitivity,
nervousness, weight loss, fatigue, palpitations, tachycardia, and palpable goiter, but findings unique to
Grave's disease are eye proptosis, which is irreversible, and pretibial myxedema (see Color Plate 9).

DIAGNOSTIC EVALUATION
Laboratory tests consist of TSH and free T4. One finds the TSH to be suppressed to near zero, with elevated
free T4. Radioactive iodine uptake testing will differentiate thyroiditis from Grave's disease but is not
necessary for confirmation if the clinical picture is consistent with Grave's disease. In TNG, diffuse areas of
uptake will be seen.

P
TREATMENT
Hyperthyroidism treatment can be medical, with use of radioactive iodine or, in the case of Grave's disease,
drugs that inhibit synthesis of thyroid hormones, such as methimazole and propylthiouracil. In addition,
beta-blockers can be used for symptomatic relief in Grave's disease as well. Surgery is the best option when
patients are symptomatic from their goiters (e.g., dysphagia or pressure-like sensations in the neck), when
malignancy cannot be excluded in the enlarged thyroid, and when radioactive iodine is not a suitable option
(children, pregnant women, resistant cases, and patients not agreeable to risks of radioactive iodine).
Specifically, bilateral subtotal thyroidectomy is performed to minimize risk of hyperthyroidism recurrence.
However, surgery also puts the patient at risk for long-term hypothyroidism, particularly in Grave's disease,
where a euthyroid state rarely is achieved after surgery alone. All patients preoperatively must be made
euthyroid using methimazole or propylthiouracil. Iodide preparations are avoided in patients preoperatively
as they may worsen the patient's condition, whereas iodides are often used preoperatively in Grave's
disease to minimize vascularity of the thyroid.

THYROID NODULE
Evaluation of the thyroid nodule is what leads a patient eventually to surgery. Although thyroid nodules are
common in the general population, with a higher incidence in women than in men, only a small portion are
clinically evident, and within this group, nearly 90% of the nodules are benign, with the rest being
malignant. However, risk factors that would lead one to suspect malignancy include history of head/neck
radiation, family history of thyroid cancer (familial medullary thyroid carcinoma, multiple endocrine
neoplasia syndrome), male sex (because more female patients present with benign thyroid nodules than
male patients), very young age (<20 years) and old age (>60 years), rapid enlargement of the nodule, voice
changes, and presence of thyroid disease. The palpable nodule on examination or, as is now being seen
more often, incidental nodules on radiologic studies for other purposes, can be definitively tested with fine-
needle aspiration (FNA). The technique of FNA involves sampling the contents of the nodule in several
planes via a 25-gauge needle on a syringe. The cytologic specimen can be evaluated immediately for
adequacy and morphology, determining whether the
lesion is benign, malignant, or indeterminate. Light microscopy remains the gold standard, although P
molecular markers, microarrays, and associated gene mutations are coming into play. These latter tools may
further differentiate the numerous classification of thyroid cancers involved in a continuum from well
differentiated to poorly differentiated, allowing appropriate treatment.

ALGORITHM
After taking an adequate history and conducting a physical examination, the patient's TSH level should be
checked, after which point an ultrasound should be performed. The ultrasound can characterize the nodule
by echogenicity, borders, vascularity, and calcifications. In addition, ultrasound evaluation of any nearby
lymphadenopathy can further predict risk of malignancy. Nodules 1 to 1.5 cm are considered biologically
significant and should undergo FNA.

Repeat FNA is recommended in the event of an inadequate specimen. Presence of follicular or Hürthle cells
without malignancy is an indication for surgical intervention, because cytological examination alone is
insufficient for determining presence of malignancy. Presence of capsular invasion or angioinvasion can only
be seen on histological examination where the thyroid architecture is intact. In addition, frozen section of
the thyroid specimen will not necessarily confirm the diagnosis, as multiple sections must be investigated.
Thus in many instances, the diagnosis will be deferred to review of permanent sections on pathological
examination.

GOITERS
In most parts of the world, enlarged thyroid gland, or goiter, is due to iodine deficiency; however, in the
United States, it is most likely due to Hashimoto thyroiditis. Classically, the mechanism of action has been
TSH stimulation, but now consideration is given to nodular growth that occurs with age and is made worse
with other exposures, such as excess iodine, Hashimoto thyroiditis, environmental goitrogens, Grave's, and
lithium.

Indications for surgery include neck compression syndromes affecting swallowing, breathing, and speaking,
where malignancy cannot be excluded; hyperthyroidism and progressive growth in the presence of
suppression medically; and cosmesis (see Color Plate 10).

Rather than classify goiters by size or weight criteria, the World Health Organization in 1960 developed a
grading system for clinical evaluation, with grade 0 representing no enlargement and grade 3 representing
enlargement that is evident from a distance (Color Plate 10). The majority of goiters remain in the neck;
however, a minority have substernal extension into the anterior mediastinum.

THYROID CANCER
Thyroid cancer incidence is greater in women than men (3% versus 1%, respectively) for unknown reasons.
Compared with other cancers with greater incidence, such as breast, lung, and prostate, thyroid cancer
mortality is favorable overall. Multiple types of thyroid tumors exist, which can be distinguished by cell
origin. Most common are the follicular cell neoplasms, which include papillary and its variants; follicular
and Hürthle cell; poorly differentiated; and anaplastic. These occur on a spectrum ranging from well
differentiated to poorly differentiated, determining biologic behavior and subsequent treatment. Other
cancers include medullary cancer derived from C cells and lymphoma. Papillary and follicular cancers tend
to have a more favorable prognosis than the medullary and anaplastic variants.

STAGING
Staging for thyroid cancer is according to the TNM classification (Table 12-1).

TABLE 12-1 American Joint Committee on Cancer (AJCC) TNM Classification


of Thyroid Carcinoma

Primary Tumor (T)

Note: All categories may be subdivided: (a) solitary tumor, (b) multifocal
tumor (the largest determines the classification).

TX      Primary tumor cannot be assessed

T0      No evidence of primary tumor

T1      Tumor 2 cm or less in greatest dimension limited to the thyroid

T2      Tumor >2 cm but not >4 cm in greatest dimension limited to the


thyroid

T3      Tumor >4 cm in greatest dimension limited to the thyroid or any


tumor with minimal extrathyroid
           extension (e.g., extension to sternothyroid muscle or perithyroid
soft tissues)

T4a     Tumor of any size extending beyond the thyroid capsule to invade
subcutaneous soft tissues, larynx, trachea, esophagus, or recurrent
laryngeal nerve

T4b     Tumor invades prevertebral fascia or encases carotid artery or


mediastinal vessels

All anaplastic carcinomas are considered T4 tumors.

T4a      Intrathyroidal anaplastic carcinoma—surgically resectable

T4b      Extrathyroidal anaplastic carcinoma—surgically unresectable

Regional Lymph Nodes (N)

Regional lymph nodes are the central compartment, lateral cervical, and
upper mediastinal lymph nodes.

NX       Regional lymph node(s) cannot be assessed

N0        No regional lymph node metastasis

N1        Regional lymph nodes metastasis

N1a      Metastasis to level VI (pretracheal, paratracheal, and


prelaryngeal/Delphian lymph nodes)

N1b      Metastasis to unilateral, bilateral, or contralateral cervical or


superior mediastinal lymph nodes

Distant Metastasis (M)

MX       Distant metastasis cannot be assessed


M0        No distant metastasis

M1        Distant metastasis

Stage Grouping

Separate stage groupings are recommended for follicular, medullary, and


anaplastic (undifferentiated) carcinoma.

Papillary or Follicular

Under 45 years

Stage I      Any T      Any N      M0

Stage II     Any T      Any N      M1

Papillary or Follicular

45 years and older

Stage I        T1          N0           M0

Stage II       T2          N0           M0

Stage III      T3          N0           M0

                   T1          N1a         M0

                   T2          N1a         M0

                   T3          N1a         M0
Stage IVA   T4a         N0           M0

                   T4a        N1a          M0

                   T1          N1b         M0

                   T2          N1b         M0

                   T3          N1b         M0

                   T4a         N1b         M0

Stage IVB   T4b        Any N       M0

Stage IVC   Any T     Any N       M1

Medullary carcinoma

Stage I         T1          N0           M0

Stage II        T2          N0           M0

Stage III       T3          N0           M0

                    T1          N1a         M0

                    T2          N1a         M0

                    T3          N1a         M0

Stage IVA    T4a        N0           M0
                     T4a       N1a         M0

                     T1         N1b        M0

                     T2         N1b         M0

                     T3         N1b        M0

                     T4a        N1b          M0

Stage IVB     T4b      Any N        M0

Stage IVC     Any T   Any N        M1

Anaplastic Carcinoma

All anaplastic carcinomas are considered stage IV.      

Stage IVA       T4a      Any N      M0

Stage IVB      T4b      Any N      M0

Stage IVC      Any T      Any N      M1

Histopathologic Type

There are four major histopathologic types:

• Papillary carcinoma (including follicular variant of papillary carcinoma)

• Follicular carcinoma (including Hürthle cell carcinoma)


• Medullary carcinoma

• Undifferentiated (anaplastic) carcinoma

Stage 0      Tis      N0      M0

Stage IA      T1      N0      M0

Stage IB      T1      N1      M0

                 T2a/b      N0      M0

                    T3      N0      M0

Stage II     T1      N2      M0

                T2a/b      N1      M0

                   T3      N0      M0

Stage IIIA      T2a/b      N2      M0

                     T3        N1      M0

                     T4        N0      M0

Stage IIIB      T3      N2      M0

Stage IV      T4      N1-3      M0

                   T1-3      N1-3      M0
                  Any T      Any N      M1

Used with permission of the American Joint Committee on Cancer


(AJCC), Chicago, Ill. Original source: AJCC Cancer Staging Manual. 6th
ed. New York, NY: Springer-Verlag, 2002.

TREATMENT

Papillary
Surgery is the mainstay of treatment for papillary carcinoma and its variants. There has been much
discussion regarding extent of surgery, which can range from lobectomy and isthmectomy to total
thyroidectomy. Papillary cancer can be multicentric, spreading into nearby lymph nodes. Lymph node
dissection is performed selectively. In selected patients where the cancer is confined to one lobe and is <1.5
cm without extracapsular involvement, minimal lobectomy with isthmectomy may be sufficient. However,
total thyroidectomy is advocated to minimize risk of local recurrence if injury to the parathyroids and
recurrent laryngeal nerves are minimal. Postoperatively, TSH is suppressed with thyroidal hormone
replacement, and radioactive iodine may be used to ablate any residual thyroidal tissue. Measurement of
serum thyroglobulin, which is a tumor marker for well-differentiated thyroid
cancers, can be performed after total thyroidectomy and at follow-up to monitor signs of P.94 P.93 P
recurrence.

Follicular
A follicular lesion on FNA is not definitive in distinguishing benign from malignant, because the components
of the capsule and blood vessels must be evaluated via histological examination. The minimal approach is to
perform lobectomy and await permanent pathology reading. Obtaining frozen section of a follicular lesion
usually is not meaningful or cost effective. However, if one highly suspects follicular carcinoma based on
size (>4 cm), selective frozen section may be helpful or one may proceed with total thyroidectomy. Also,
evaluation of the contralateral lobe at the time of surgery is important in deciding extent of surgery. After
lobectomy with pathology showing minimally invasive follicular carcinoma, completion thyroidectomy is not
necessarily indicated in low-risk patients. Follicular cancer does not metastasize to lymph nodes as much as
papillary cancer. Thus elective nodal dissection is not performed in the absence of adenopathy. In contrast
to papillary cancer, follicular cancer tends to metastasize via the bloodstream. Radioactive iodine may be
used in patients with unresectable, gross disease.

Medullary
In the familial form of the disease, whether isolated or associated with multiple endocrine neoplasia 2A or
2B, medullary thyroid cancer is known to metastasize early to surrounding lymph nodes. Patients should be
genetically tested given their risk profile and undergo total thyroidectomy with central neck dissection at
an early age. Nearly 70% of medullary cancers are sporadic. These patients may present with a thyroid
nodule associated with cervical adenopathy. Staining of FNA for calcitonin and absence of thyroglobulin is
diagnostic. As with the familial forms, total thyroidectomy with central neck dissection is performed (see
Color Plate 11).

Anaplastic/Lymphoma
Rarely are anaplastic thyroid cancer and lymphoma treated surgically. Both processes present with a rapidly
enlarging neck mass. The undifferentiated anaplastic cancer is considered lethal with a short survival time
of months, whereas the prognosis for lymphoma is better. FNA with appropriate immunohistochemical
staining is used for appropriate diagnosis, but sometimes open biopsy for tissue histology is confirmatory.
Non-Hodgkin B-cell type is the most common lymphoma pathology for thyroid. Airway management rather
than thyroidectomy is the surgical intervention of choice.

COMPLICATIONS OF THYROID SURGERY


There are unique risks involved in thyroid surgery that increase with extent of surgery. The most important
is injury to the recurrent laryngeal nerve, which not only
controls voice but airway function. The best prevention is identification of the nerve by visualization and P
assistance with use of intraoperative nerve monitoring. Even if the nerve is not known to be injured, vocal
cord paresis can occur postoperatively. Temporary paresis/ paralysis of the nerve can last up to 3 months.
Symptoms include voice changes and aspiration of liquids. Bilateral vocal cord paralysis requires
tracheostomy. Long-term paralysis may be treated with synthetic injection such as Teflon, gel foam, or
collagen; this technique is known as laryngoplasty.

Injury to the external branch of the superior laryngeal nerve can occur during the takedown of the superior
pole of the thyroid. The result is minimization of high pitch, which may not be apparent except when
singing and yelling.

Calcium balance disruption occurs postoperatively from parathyroid manipulation and devascularization. It
is not uncommon to see hypocalcemia immediately after surgery, which can be supplemented with oral as
well as intravenous calcium. Vitamin D is added if calcium addition alone is insufficient. This condition is
temporary. However, if the parathyroids are known to be removed or permanently devascularized and no
reimplantation occurs, permanent hypoparathyroidism is likely to occur. These patients will need lifelong
calcium and Vitamin D supplementation.

Postoperative bleeding in the wound bed given the location is life-threatening. The expanding hematoma
causes compression on the airway, and this surgical emergency should be addressed in the operating room.

Key Points
The thyroid gland is a bilobed structure connected by an isthmus and derived from
the migration of the first and second pharyngeal pouches from the base of the
tongue.
Grave's disease and toxic nodular goiter are the most common causes of hyperthyroidism.
Grave's disease is an autoimmune disorder where the body's own immune system
stimulates the thyroid-stimulating hormone receptor, whereas autoproduction of thyroid
hormone not responding to thyroid-stimulating hormone is the mechanism of action in
toxic nodular goiter.
Patients must be made euthyroid before surgery.
Evaluation of the thyroid nodule includes ultrasound and fine-needle aspiration.
Fine-needle aspiration can be used to determine whether the lesion is benign, malignant,
or indeter-minate.
Follicular lesions on fine-needle aspiration require minimal lobectomy for definitive
classification. Elements of capsular or angioinvasion determine malignancy, which can only
be seen on histology.
Goiters are classified by a grading system developed by the World Health Organization.
Patients with disease that is symptomatic or for which malignancy cannot be excluded,
causes hyperthyroidism, or is progressively enlarging or cosmetically deforming are
candidates for thyroidectomy.
Thyroid cancer has a spectrum of well-differentiated to poorly differentiated cell types,
with follicular cell origin being the most common. In descending order of incidence:
papillary and its variants, follicular, and anaplastic. Medullary is from the C cells and has
isolated familial form as well as association with multiple endocrine neoplasia 2A and 2B.
Papillary and follicular cancers tend to have more favorable outcomes than medullary and
anaplastic cancers.
Surgical treatment is the mainstay for differentiated thyroid cancers but is rarely effective
for aggressive anaplastic and lymphoma.
Complications of surgery are related to the extent of surgery. These include injuries to the
recurrent laryngeal nerve, external branch of superior laryngeal nerve, and parathyroids.
Postoperative bleeding in the wound bed can be life-threatening.
Authors: Karp, Seth J.; Morris, James P.G.; Zaslau, Stanley
Title: Blueprints Surgery, 5th Edition

Copyright ©2008 Lippincott Williams & Wilkins

> Table of Contents > Part III - Endocrine > Chapter 13 - Parathyroid Gland

Chapter 13
Parathyroid Gland

The surgical treatment of parathyroid disease relates mainly to hyperparathyroidism.

ANATOMY
Parathyroid glands are normally small, yellowish-brown tissues measuring 2 × 3 × 5 mm. Each gland on
average weighs 30 to 40 mg. Pathologic glands, whether they are parathyroid adenomas or hyperplasia,
appear grossly enlarged and reddish-brown. Both are hypercellular microscopically.

At least four parathyroid glands are present in all individuals, residing posterior to the thyroid.
Embryologically, the upper or superior paired glands arise from the fourth branchial pouch, located in a
plane posterior to the recurrent laryngeal nerve (RLN), usually within 1 cm where the RLN and inferior
thyroidal artery cross. The lower or inferior paired glands along with the thymus arise from the third
branchial pouch, located in a plane anterior to the RLN within 2 cm of the lower thyroid pole. Superior
glands can also be found more lateral than inferior glands. All four glands receive their blood supply from
the inferior thyroidal artery, sharing it with the thyroid gland (Figs. 13-1, 13-2, and 13-3).
Figure 13-1 • The pharyngeal pouches. The inferior parathyroid arises from
the third pouch and the superior arises from the fourth.
Figure 13-2 • Normal siting of the upper parathyroid glands.
Figure 13-3 • Normal siting of the lower parathyroid glands.

Ectopic locations of parathyroid glands are based on embryology and are the key to localization. Aberrant
lower glands rarely are intrathyroidal but can be found undescended, with thymic tissue high in the neck
near the hyoid bone (Fig. 13-4).
Figure 13-4 • Ectopic parathyroid gland locations secondary to aberrant
migration.

PHYSIOLOGY
Parathyroid glands are endocrine organs regulating calcium and phosphate metabolism. The parathyroid
hormone (PTH), which can be biochemically detected in its intact 84 amino acid form, works on bone as
well as kidney tubules to increase blood calcium. PTH is regulated through a feedback system.
Hyperparathyroidism exists in several forms. Primary hyperparathyroidism (HPT) results from excess PTH,
which causes mobilization of calcium deposits from bone, inhibition of renal phosphate reabsorption, and
stimulation of renal tubular absorption of calcium. Overall, both total body calcium and phosphate wasting
occur, leading to osteoporosis and bony mineral loss.

Secondary HPT is from hyperplasia of the parathyroid glands, occurring in those with chronic hypocalcemia,
as is usually seen in patients with renal disease. The hyperphosphatemia suppresses calcium levels, leading
to excess PTH production.

Tertiary HPT is the consequence of secondary HPT becoming autonomous. Long after a renal dialysis patient
undergoes successful renal transplantation, the hyperplastic parathyroid glands will eventually function
normally. Therefore, it is rare to intervene surgically.

Pseudohyperparathyroidism or humoral hypercalcemia of malignancy leads to hypercalcemia as well as


hypophosphatemia similar to primary HPT; however, it is PTH-related protein that is the cause. This latter
molecule is not detected in the assays for PTH. The malignancy causes hypercalcemia as a result of these
mechanisms: PTH-related protein, lytic bone metastases, and ectopic calcitriol secretion.

EPIDEMIOLOGY
HPT is the most common cause of hypercalcemia, followed by malignant disease. The majority of HPT cases
occur sporadically or in a nonfamilial form, often from a single hyperfunctioning gland or adenoma.

Patients with multiple endocrine neoplasia 1 (involvement of the three Ps: parathyroid, pituitary, pancreas) P
and multiple endocrine neoplasia 2A (hyperparathyroidism, pheochromocytoma, and medullary cancer of
the thyroid) have HPT involving multigland hyperplasia of the parathyroids.

PATHOLOGY
Solitary parathyroid adenoma accounts for 80% of primary HPT. Nearly 15% of cases are due to hyperplasia,
where up to all four glands are involved. It is thought that single-gland disease and multigland disease lie
within a spectrum. Double adenoma lies between solitary adenoma and hyperplasia. Parathyroid carcinoma
is rare, occurring in fewer than 2% of cases. Typically the hypercalcemia is more
pronounced in value and presentation. The lesions can be encapsulated like their benign counterparts but P
tend to be more ill defined, possibly invading the thyroid or nearby structures.

HISTORY
Historically, patients presented with long-term complications related to "stones, bones, groans, and moans."

Renal stones/nephrocalcinosis

Bones: aches and arthralgias

Groans: abdominal pain from constipation, pancreatitis, or peptic ulcer disease

Psychic moans: mood swings, fatigue, anxiety, or memory loss

Today's surgical treatment of parathyroid disease is seen more in asymptomatic individuals rather than those
sustaining long-term complications. Presently it is not uncommon to see postmenopausal women who have
abnormal bone mineral densitometry to detect osteoporosis be screened on calcium blood tests and are
found to be normal or hypercalcemic. This then prompts a workup for hypercalcemia where the PTH is
elevated. The 2002 updated National Institutes of Health Consensus Development Conference on the
Management of Asymptomatic Primary Hyperpara-thyroidism issued guidelines for surgery of the
hyperparathyroid patient:

Serum calcium >1.0 mg/mL above upper limit of normal

>400 mg/d hypercalciuria

Sequelae of hypercalcemia resulting from primary hyperparathyroidism, such as nephrolithiasis, osteitis


fibrosa cystica, classic neuromuscular disease, severe psychoneurologic disorder

Cortical bone density reduction with T score <2.5 standard deviation (SD) of lumbar spine, hip, or wrist

<30% reduction of renal function in the absence of other cause

Inability to have appropriate follow-up

Age <50 years

PHYSICAL EXAMINATION
Rarely will any abnormalities be found on the physical examination, as the early diagnosis is captured
biochemically. Should a palpable neck mass be associated with HPT, one would then suspect parathyroid
carcinoma.

DIFFERENTIAL DIAGNOSIS
In addition to medical conditions associated with hypercalcemia that must be excluded, it is important to
distinguish benign familial hypocalciuric hypercalcemia (BFHH) from HPT, as the former is not treated. Thus
surgical intervention will be of no benefit. BFHH can present with hypercalcemia associated with mildly
elevated PTH levels. The test of choice for exclusion of BFHH is presence of low urinary calcium (Table 13-
1).

TABLE 13-1 Disease and Factors Causing Hypercalcemia

Hyperparathyroidism

Multiple myeloma

Sarcoid and other granulomatous diseases

Milk-alkali syndrome

Vitamin D intoxication
Vitamin A intoxication

Paget disease

Immobilization

Thiazide diuretics

Addisonian crisis

Familial hypocalciuric hypercalcemia

DIAGNOSTIC EVALUATION
A combination of elevated calcium and PTH levels is highly suggestive of primary HPT. Again, BFHH can be
ruled out by presence of low urinary calcium.

TREATMENT
Primary HPT is a surgical disease, and only parathyroidectomy can successfully treat the disease. The
success of the operation is measured by the return of
calcium to normal limits. Those with hypercalcemic crisis such as coma, delirium, anorexia, vomiting, and P
abdominal pain must be initially stabilized with vigorous intravenous hydration and forced diuresis with
furosemide. Associated hypokalemia and hyponatremia must also be corrected.

Once HPT is confirmed, preoperative localization of the suspecting gland may be performed in an attempt
to perform minimally invasive surgery or assist in determining whether the source is from adenoma versus
hyperplasia. Moreover, localization studies are indicated in those undergoing reoperative surgery for
persistent or recurrent HPT. The preferred modality is technetium (Tc)-sestamibi scanning, including oblique
and single photon emission computed tomography scans, but ultrasound and computed tomography may also
be complementary

To maximize the success of surgery, preoperative Tc-sestamibi scanning and intraoperative rapid PTH assay
are recommended. The intraoperative rapid PTH assay is a means to monitor adequate removal of the
abnormal parathyroid gland. Once the tumor is removed, PTH levels should decrease to <50% of the baseline
value in as short as 10 minutes. These two modalities also allow for a minimally invasive approach,
decreasing operative time and length of stay.

SURGERY
Traditionally, the four gland bilateral neck exploration has been implemented without prior localization
studies. Recently, with the assistance of preoperative Tc-sestamibi where a single adenoma is localized, a
patient may undergo minimally invasive surgery, during which the gland of concern is targeted via a smaller
incision, sometimes under regional anesthesia and as an outpatient procedure under experienced hands. If
level of experience is sufficient, endoscopic parathyroidectomy is also used.

If there is no localization preoperatively and ultrasound is also nonrevealing, the likely finding is
hyperplasia. Four-gland exploration should be used. All four glands are identified in their usual locations
before any removal; their size, appearance, and color will usually determine presence of abnormality
without resorting to biopsy and pathologic confirmation. If all glands are hyperplastic, a subtotal
parathyroidectomy, which is the equivalent to 3.5-gland removal, or total parathyroidectomy with
autotransplantation is performed.

If a missing gland is not identified in its usual location, the search for ectopic sites should be performed.
These ectopic sites are related to the embryologic development. Thymic and thyroidal tissues may need to
be encompassed as well.

After surgery, although calcium levels may return to normal, the patient may still develop symptoms of
hypocalcemia, such as perioral numbness/tingling, distal extremity paresthesias, and signs of a positive
Chvostek, where the facial muscle twitches with tapping of the facial nerve. Calcium should be replenished,
even if normal, in these instances.

KEY POINTS
Hyperparathyroidism is the most common cause of hypercalcemia for which surgical
intervention can successfully treat the disease.
There are many forms of hyperparathyroidism; however, primary HPT is usually due
to single-gland adenoma.
Embryologic migration of parathyroid tissue determines resting location as well as ectopic
sites.
Hyperparathyroidism is diagnosed biochemically using blood studies as well as urine tests.
Minimally invasive surgery is possible as a result of localization studies and rapid
parathyroid hormone assay availability.
Authors: Karp, Seth J.; Morris, James P.G.; Zaslau, Stanley
Title: Blueprints Surgery, 5th Edition

Copyright ©2008 Lippincott Williams & Wilkins

> Table of Contents > Part III - Endocrine > Chapter 14 - Breast

Chapter 14
Breast

ANATOMY AND PHYSIOLOGY


The breast extends from the clavicle (approximately the second rib) to the sixth rib and from the sternum
to the mid axillary line. Approximately 10 to 100 alveoli (secretory units) form lobules that drain into ducts,
which eventually lead to the nipple. The breast is surrounded by fascia connected by the suspensory
ligaments of Cooper.

The blood supply to the breast is mostly from anterior perforating branches of the internal mammary artery,
various branches of the axillary artery, and posterior intercostal arteries. The lymphatic drainage is
primarily to axillary lymph nodes, internal mammary lymph nodes, and interpectoral nodes of Rotter.
Axillary lymph nodes are stratified into three levels depending on their location relative to the pectoralis
minor muscle. Level I lymph nodes are lateral to the muscle. Level II lymph nodes are deep to (behind) the
muscle, and level III lymph nodes are medial to the muscle (Fig. 14-1).
Figure 14-1 • Breast anatomy with blood supply and lymphatics.

The nipple is innervated by T4, no matter how pendulous the breast. Although they do not innervate the
breast, the long thoracic and thoracodorsal nerves are important because of their proximity to axillary
lymph nodes. Injury to the long thoracic and thoracodorsal nerves during axillary nodal dissection may
result in winged scapula or weakness in shoulder adduction, respectively. Additionally, the intercostal
brachial cutaneous nerves pierce the tail of the breast parenchyma and, when divided, will cause
paresthesias of the medial aspect of the upper arm.

Cyclic hormonal changes affect the breast. The breasts may feel lumpy and tender before menses.
Pregnancy causes marked hypertrophy of the alveoli, lobules, and ducts in preparation for lactation. With
menopause, the lobules become atrophic.

PATHOLOGY
Benign conditions of the breast include fibrocystic changes, fibroadenomas, simple cysts, intraductal
papillomas, and gynecomastia. Phyllodes tumors can be benign, borderline, or malignant. High-risk and
premalignant lesions include atypical ductal and lobular hyperplasia and lobular carcinoma in situ (LCIS).
The most common malignancies are intraductal carcinoma (also known as ductal carcinoma in situ [DCIS],
which is noninvasive because it does not penetrate the basement membrane), invasive ductal carcinoma,
and invasive lobular carcinoma. Inflammatory breast cancer is characterized by skin involvement (invasion
of the subdermal lymphatics). Paget disease of the nipple is an intraepithelial neoplasm that may be
associated with an underlying breast cancer (invasive or in situ).

EPIDEMIOLOGY
Lifetime risk of American women for developing breast cancer is one in eight (12%). It is the most
frequently diagnosed cancer in women and the second most frequent cause of cancer-related death among
women. Incidence increases with age and varies among various ethnic groups: incidence of breast cancer in
Hawaiians is greater than that in Whites, which is greater than that in Blacks, which is greater than that in
Asians and Hispanics, which is greater than that in American Indians. Significant risk factors are female
sex, age, BRCA gene mutations, hormone replacement therapy, personal history of breast cancer, radiation P
to the chest at age younger than 40 years, first-degree relative with breast cancer (higher if the relative
was premenopausal), and prior biopsy-proven LCIS or atypical hyperplasia (ductal or lobular). The incidence
of breast cancer in men is 1%. The most common tumor in young women is fibroadenoma.

HISTORY
Most women report finding a breast lump while showering or with breast self-examination. Breast pain is
usually associated with benign lesions, but may occasionally be present with malignant lesions. Lumps that
increase before menses and decrease after menses are usually benign simple cysts or fibrocystic changes.
Spontaneous or bloody nipple discharge is associated with intraductal papillomas. Malignant lesions do not
vary in size with the menstrual cycle. Patients with advanced stages may have weight loss, odor from
ulcerating or fungating lesions, pain from bone metastasis (back, chest, or extremities), nausea or
abdominal pain from liver metastasis, or headaches from brain metastasis.

PHYSICAL EXAMINATION
When examining the breasts, one looks for skin changes, asymmetry (visible bulge or dimpling of skin),
nipple retraction, palpable masses, and lymphadenopathy in the axilla and supraclavicular fossa. Well-
circumscribed, mobile, nontender lumps in young women are usually fibroadenomas or phyllodes tumors.
Breast tenderness is associated with fibrocystic changes or simple cysts. Vague masses or firm lumps with
indistinct borders are suggestive of malignancy. With advanced stages, there may be skin changes such as
dimpling, peau d'orange (edema of the skin, making it look like an orange peel), ulceration, erythema, or
fixation to the skin or chest
wall. Enlarged or matted lymph nodes may indicate metastasis. Paget disease usually presents with nipple P
or areolar rash or excoriation. Inflammatory breast cancer may have erythema, peau d'orange, or skin
thickening.

DIAGNOSTIC EVALUATION
With the increase in breast cancer awareness, screening mammograms initially led to an increase in breast
cancer diagnoses. This increased incidence has subsequently reached a plateau. Screening mammograms
have resulted in diagnosing breast cancer at an earlier stage, with improved survival and decreased
mortality.

Mammograms miss 15% of palpable breast cancers and should be done in conjunction with clinical breast
examination. Mammographic signs that suggest malignancy include a density with indistinct margins,
spiculated mass (Fig 14-2), and clusters of or linear/branching microcalcifications. Lesions not seen on prior
mammograms need further work up. Also, if the patient reports a palpable lump, then a diagnostic
mammogram (additional views to magnify and/or compress the breast are obtained to identify or
characterize the lesion) and breast ultrasound are performed.

Figure 14-2 • Craniocaudal (CC) (A) and oblique (MLO) (B) views of
mammogram showing a breast cancer (circle). Image courtesy of Rhona
Chen, MD.

Ultrasound can differentiate between solid and cystic masses (Fig. 14-3). It is not optimal for screening
because it rarely detects microcalcifications and is extremely dependent on the experience of the person
performing the examination.
Figure 14-3 • Solid (fibroadenoma) and cystic (simple cyst) lesions on
ultrasound. Image courtesy of Rhona Chen, MD.

Breast magnetic resonance imaging (MRI) is rapidly increasing in popularity. Current indications include
identifying occult primary cancer with axillary metastasis in women with no clinical breast mass and normal
mammogram, screening women at very high risk (>25%) for developing breast cancer, determining
extent of breast cancer to help with surgical planning, and measuring response to neoadjuvant therapy. P
Advantages of breast MRI include creation of three-dimensional images, minimal or no breast compression,
effectiveness in women with dense breasts, and high sensitivity for detecting breast cancer. Disadvantages
are moderate to low specificity (false-positive results lead to unnecessary biopsies and patient anxiety),
expensive cost, long time to perform examination (30 to 40 minutes, as compared with 5 to 10 minutes for
mammogram), requirement of contrast, and inability to demonstrate microcalcifications.

Table 14-1 includes the American Cancer Society recommendations for breast cancer screening.

TABLE 14-1 American Cancer Society Guidelines for Breast Cancer


Screening

Yearly mammograms starting at the age of 40 years and continuing for as


long as a woman is in good health.

Clinical breast examination should be part of a periodic health


examination, about every 3 years for women in their 20s and 30s and
every year for women 40 years and older.

Women should know how their breasts normally feel and report any
breast change promptly to their health care providers. Breast self-
examination is an option for women starting in their 20s.

Women at high risk (greater than 20% lifetime risk) should have magnetic
resonance imaging (MRI) and a mammogram every year. Women at
moderately increased risk (15%–20% lifetime risk) should talk with their
doctors about the benefits and limitations of adding MRI screening to
their yearly mammogram. Yearly MRI screening is not recommended for
women whose lifetime risk of breast cancer is less than 15%.

Adapted from the American Cancer Society (www.cancer.org ).

Lesions suggestive of malignancy on diagnostic examination are biopsied with image guidance:
mammographic, ultrasonographic, or MRI guided core needle biopsy. Palpable masses not seen with imaging
can be biopsied with fine-needle aspiration (FNA), core needle biopsy, excisional open biopsy, or incisional
biopsy. A benign lesion can be observed radiographically and clinically. Malignant lesions require surgical
consultation. If the pathology report shows atypia or LCIS or is discordant with radiologic or clinical
findings, then an open biopsy should be performed. This may require wire localization by the radiologist. If
the patient presents with skin changes over the breast mass, an incisional biopsy should be considered,
taking an ellipse of skin with the mass to check for cancer involvement of dermal lymphatics.

If DCIS or invasive breast cancer is diagnosed pathologically, additional pathologic tests are performed. The
tumor is checked for hormone receptors, HER-2/neu receptor, and, occasionally, various other biologic
markers. Tumors with better prognostic indicators are well differentiated (low-grade cancer), have
overexpression of estrogen receptors or progesterone receptors, and do not overexpress HER-2/neu
receptor. The Oncogene DX test helps determine whether a woman will benefit from chemotherapy in
borderline situations.

P
TREATMENT
Simple cysts may be aspirated if large or symptomatic. If the fluid is bloody, it should be sent for cytology
to check for malignancy. If the cyst does not resolve completely after aspiration or recurs after three
aspirations, it should be excised to rule out malignancy.

Solid lumps should be excised if enlarging or symptomatic or if other diagnostic studies have been
inconclusive (pathology from image-guided biopsy is discordant with radiographic or clinical findings).

Treatment recommendations for LCIS have evolved from bilateral mastectomies to the current
recommendation of close observation with annual mammogram and frequent clinical breast examination.
LCIS is usually an incidental finding on a biopsy performed for other reasons. Although it does increase the
risk for developing subsequent cancer in either breast, it is no longer thought to be a precursor of breast
cancer.

For intraductal (noninvasive, in situ) or invasive breast cancers, surgical treatment options are breast-
conserving therapy (lumpectomy with or without radiation therapy) or mastectomy. Lumpectomy is the
removal of the cancer with a rim of normal breast tissue to obtain clear or negative margins to ensure the
cancer has been completely removed. If there is cancer at the surgical margin, the patient should undergo
re-excision to obtain clear margins or consider mastectomy. Mastectomy is the removal of the breast and
nipple/areolar complex (from clavicle to rectus muscle and sternum to latissimus dorsi, taking pectoralis
fascia with the breast tissue). Statistically, long-term survival is approximately the same, but local
recurrence is slightly higher with breast-conserving therapy (BCT): 7% to 10% with radiation and up to 25%
without radiation, as compared with mastectomy (3%). If the patient elects BCT and develops a recurrent
cancer in the same breast, then mastectomy is usually recommended, especially if the breast has been
previously irradiated.

Although BCT is being performed more often as a result of earlier stage at diagnosis, there are still some
circumstances when mastectomy should be recommended: multicentric cancers (cancer in more than one
quadrant of the breast), extensive high-grade DCIS, large tumor relative to size of breast where
lumpectomy would result in poor cosmetic outcome, and when clear margins have not been obtained after
re-excision.

In addition, lymph nodes should be checked for metastasis for staging purposes. Sentinel lymph node (SLN)
biopsy is rapidly replacing the complete axillary lymph node dissection (CALND) for stage I and II breast
cancer as the standard of care. SLN biopsy involves injection of a radioactive isotope and/or a vital blue dye
(isosulfan blue or methylene blue) into the breast (subareolar, intradermal or intraparenchymal) to locate
the first few draining lymph nodes the cancer is most likely to involve. The "hot" (radioactive) and/or blue
lymph nodes are removed. The axilla is also palpated and any enlarged lymph nodes are also removed. The
average number of sentinel lymph nodes removed is 2.8. The procedure is 97% accurate. If no SLN is found
or if the SLN has metastasis, then CALND is performed, removing level I and II axillary lymph nodes. When
CALND is performed with mastectomy, the procedure is called a modified radical mastectomy. The
advantage of SLN biopsy is more intensive pathologic evaluation of fewer lymph nodes for more accurate
staging and avoidance of lymphedema, which occurs in approximately 15% of patients who undergo CALND.

Radiation therapy is most commonly performed with external-beam irradiation to the breast and may
include the axillary and supraclavicular nodal regions. Whole-breast radiation usually involves daily
treatments over 4 to 6 weeks. Sometimes a boost dose is given to the lumpectomy site. Interest in partial
breast irradiation has re-emerged with the advent of the Mammosite balloon. The balloon is inserted into
the lumpectomy cavity and radioactive beads are implanted into the balloon (similar to brachytherapy).
This permits high doses of radiation to be administered to the adjacent breast tissue in a shorter period of
time (usually 4 to 5 days). The balloon is subsequently removed as a minor procedure.

Antiestrogen hormonal therapy is given to patients if breast cancer is estrogen or progesterone receptor
positive. It can reduce the risk of recurrence by approximately 50%. Tamoxifen is given to premenopausal
women. Aromatase inhibitors (e.g., anastrozole, letrozole, and exemestane) are now the recommended
hormonal therapy for postmenopausal women because of the lower incidence of endometrial cancer and
thromboembolic events. However, there is a higher incidence of osteoporosis and fractures. Ongoing clinical
trials will determine whether premenopausal women will benefit from aromatase inhibitors. Sometimes
premenopausal women are treated with chemical or surgical oophorectomy.

Chemotherapeutic options have also evolved. Doxorubicin (Adriamycin) and cyclophosphamide is used
predominantly over the combination of cyclophosphamide, methotrexate, and fluorouracil. In addition,
taxanes (taxoids) or anthracyclines may be added to the regimen. Trastuzumab (Herceptin), a monoclonal
antibody, has been shown to improve survival in patients
P
whose tumors test positive for the HER-2/neu receptor. Other monoclonal antibody drugs are currently
being evaluated in clinical trials.

For patients who have mastectomy, reconstruction can be done immediately (at the same operation as the
mastectomy) or delayed (requiring a second or third anesthetic). Reconstructive options include saline or
silicone implants and a wide assortment of flaps. Flaps composed of muscle, subcutaneous fat, and skin can
be transferred to the chest to recreate the breast mound. These can be pedicle flaps or free flaps.
Latissimus dorsi muscle or rectus muscle flaps are the most common. The nipple/areolar complex can also
be recreated and tattooed, resulting in excellent cosmesis. If the patient is not interested in
reconstruction, then an external prosthesis should be prescribed. This prevents neck, shoulder, and upper
back pain from the body compensating for the uneven weight on the chest from the remaining breast.

Surgery and radiation therapy are performed for local control of breast cancer, whereas chemotherapy and
hormonal therapy are initiated for systemic control. A select group of patients with small, low-grade DCIS
may be treated by lumpectomy without radiation. Stage 0, I, and II breast cancers are usually treated with
surgery first. The resulting pathology then guides subsequent adjuvant therapy (chemotherapy, hormonal
therapy, radiation therapy, or a combination). Stage III and IV breast cancers are usually treated with
neoadjuvant chemotherapy (and sometimes hormonal and/or radiation therapy) followed by surgery and
radiation. Sometimes an advanced-stage breast cancer can be reduced in size to permit breast-conserving
surgery. If the patient is a candidate, hormonal therapy may also be given preoperatively (neoadjuvant) or
postoperatively once the chemotherapy and/or radiation therapy has finished.

Tables 14-2 and 14-3 show TNM Staging and American Joint Committee on Cancer Classification for Breast
Cancer, respectively.

TABLE 14-2 TMN Staging for Breast Cancer

Stage Description

Tumor

TX Primary tumor not assessable

T0 No evidence of primary tumor

Tis Carcinoma in situ

T1 Tumor ≤2 cm in greatest dimension

T2 Tumor >2 cm but not >5 cm in


greatest dimension
T3 Tumor >5 cm in greatest dimension

T4 Tumor of any size with direct


extension into chest wall (not
including pectoral muscles) or skin
edema or skin ulceration or satellite
skin nodules confined to the same
breast or inflammatory carcinoma

Regional
Lymph Nodes

NX Regional lymph nodes not assessable

N0 No regional lymph node involvement

N1 Metastasis to movable ipsilateral


axillary lymph node(s)

N2 Metastasis to ipsilateral axillary


lymph node(s) fixed to one another
or to other structures

N3 Metastasis to ipsilateral internal


mammary lymph nodes

Distant
Metastasis

MX Presence of distant metastasis cannot


be assessed

M0 No distant metastasis
M1 Distant metastasis present (includ
ing ipsilateral supraclavicular lymph
nodes)

TABLE 14-3 AJCC Classification for Breast Cancer Based on TNM Criteria

Stage Tumor Nodes Metastases

0 Tis N0 M0

I T1 N0 M0

IIA T0, 1 N1 M0

T2 N0 M0

IIB T2 N1 M0

T3 N0 M0

IIIA T0, 1, 2 N2 M0

T3 N1, 2 M0

IIIB T4 N1, 2 M0

Any T N3 M0

IV Any T Any N M1
AJCC, American Joint Committee on Cancer.
Adapted from AJCC Cancer Staging Manual. 6th ed. New York:
Springer-Verlag, 2002.

As mentioned earlier, screening mammography has resulted in the diagnosis and treatment of earlier-staged P
breast cancers. In addition, improved understanding of molecular biology of breast cancer has resulted in
the development and use of new hormonal and chemotherapeutic agents. All of this has resulted in longer
survival rates and lower mortality (Table 14-4).

TABLE 14-4 2004 Breast Cancer Prognosis Based on Stage

Stage 5 Years 10 Years

Localized 97% 94%

Regional 80% 66%

Distant 25% <15%

From National Cancer Institute web site (www.cancer.gov ),


Surveillance, Epidemiology and End Results (SEER) fast stat results: SEER
relative survival rates by stage at diagnosis for breast cancer, SEER 9
registries for 1988–2003.

PROPHYLAXIS
In women with breast atypia and LCIS, clinical trials have shown that tamoxifen decreases the risk of
developing breast cancer. Tamoxifen is usually administered for 5 years. Some experts postulate that it
should be given longer. The Study of Tamoxifen and Raloxifene (STAR) trial showed that raloxifene is as
effective as tamoxifen and is associated with 30% fewer thromboembolic events and 36% fewer uterine
cancers.

For high-risk women who do not want to take drugs to reduce the risk of breast cancer, an alternative is
prophylactic mastectomy. This reduces the risk of breast cancer by 97%.

KEY POINTS
Breast cancer is the most frequently diagnosed cancer in women and the second
most frequent cause of cancer death among women.
Significant risk factors are female sex, age, BRCA gene mutations, hormone
replacement therapy, personal history of breast cancer, radiation to the chest at age
<40 years, first-degree relative with breast cancer (higher if the relative was
premenopausal), and prior biopsy-proven LCIS or atypical hyperplasia (ductal or lobular).
Screening mammography has resulted in earlier detection and treatment, improved
survival, and decreased mortality. Screening magnetic resonance imaging is
recommended in very high-risk women with mammography.
Even if the mammogram is normal, a palpable abnormality should be evaluated further.
Treatment options for breast cancer are lumpectomy and radiation (breast-conserving
therapy) or mastectomy. Sentinel lymph node biopsy is rapidly replacing the axillary
dissection for evaluation of lymph nodes.
Chemoprophylaxis may be used in high-risk women to decrease the risk of breast cancer.
Authors: Karp, Seth J.; Morris, James P.G.; Zaslau, Stanley
Title: Blueprints Surgery, 5th Edition

Copyright ©2008 Lippincott Williams & Wilkins

> Table of Contents > Part III - Endocrine > Chapter 15 - Pituitary, Adrenal, and Multiple Endocrine Neoplasias

Chapter 15
Pituitary, Adrenal, and Multiple Endocrine Neoplasias

PITUITARY

Anatomy and Pathophysiology


The pituitary gland is located at the base of the skull within the sella
turcica, a hollow in the sphenoid bone. The optic chiasm lies anterior,
the hypothalamus lies above, and cranial nerves III, IV, V, and VI and the
carotid arteries lie in proximity. These structures are all at risk for
compression or invasion from a pituitary tumor. Visual field defects can
occur when a tumor encroaches on the optic chiasm. This most
commonly presents as a bitemporal hemianopsia (Fig. 15-1). The gland
weighs less than 1 g and is divided into an anterior lobe, or
adenohypophysis (anterior- adeno), and a posterior lobe, or
neurohypophysis. The anterior pituitary produces its own hormones–-
prolactin, growth hormone (GH), follicle-stimulating hormone (FSH),
luteinizing hormone, adrenocorticotropin (ACTH), and thyrotropin–-all
under the control of hypothalamic hormones that travel directly from
the hypothalamus through a portal circulation to the anterior pituitary
(Fig. 15-2). The hormones of the posterior pituitary, vasopressin and
oxytocin, are produced in the hypothalamus and are transported to the
posterior lobe (Fig. 15-3).
Figure 15-1 • Visual disturbances from compressive pituitary lesions.
Figure 15-2 • Pituitary hormones.
Figure 15-3 • Stimulation, production, control, and metabolism of
hormones. (1) Stimulation of the cerebral cortex resulting in (2) stimulation
of the hypothalamus to release tropic hormones from the anterior pituitary
(3). The hormone circulates to a target gland (4), where a second hormone
can be produced. This hormone is bound to transport or binding proteins (5)
and can undergo peripheral metabolism (6) or bind to receptors in target
tissues (7), which results in the specific hormone effect (8). Hormones can
then be cleared metabolically (9) or excreted from the system (10). If the
tropin acts on the hypothalamus, this is an example of the short-loop
feedback, whereas if the hormone or peripheral metabolic products
circulate to act on the hypothalamus, the control is termed long-loop
negative feedback.

From Gornall AG, Luxton AW. Endocrine disorders. In: Gornall AG, ed.
Applied Biochemistry of Clinical Disorders. 2nd ed. Philadelphia: Lippincott,
1986.

PROLACTINOMA
Pathology
Most prolactin-secreting tumors are not malignant. Prolactin-secreting tumors are divided into
macroadenomas (size >10 mm) and microadenomas (size <10 mm). Macroadenomas are characterized by
gland enlar-gement, whereas microadenomas do not cause gland enlargement.

Epidemiology
Prolactinoma is the most common type of pituitary neoplasm. Macroadenomas are more common in men,
whereas microadenomas are 10 times more common in women.

History
Macroadenomas usually produce headache as the tumor enlarges. Women may describe irregular menses,
amenorrhea, or galactorrhea.

Physical Examination
Defects of extraocular movements occur in 5% to 10% of patients and reflect compromise of cranial nerves
III, IV, or VI. Women may have galactorrhea, whereas only 15% of men have sexual dysfunction or
gynecomastia.

Diagnostic Evaluation
A serum prolactin level of >300 µg/L establishes a diagnosis of pituitary adenoma, whereas a level >100
µg/L is suggestive. Magnetic resonance imaging (MRI) differentiates microadenomas from macroadenomas
and allows characterization of local tumor growth (Fig. 15-4).
Figure 15-4 • Pituitary adenoma. A magnetic resonance sagittal view of the
brain shows a distinct pituitary tumor (arrow). V, lateral ventricle; P, pons;
C, cerebellum.
From Rubin E, Farber JL. Pathology. 3rd ed. Philadelphia: Lippincott
Williams & Wilkins, 1999.

P.109 P
Treatment
Asymptomatic patients with microadenomas can be observed without treatment. When symptoms of
hyperprolactinemia occur, a trial of bromocriptine or cabergoline should be initiated. In the event of
failure, transsphenoidal resection provides an 80% short-term cure rate, although long-term relapse rate
may be as high as 40%. For patients who desire children, transsphenoidal resection provides a 40% success
rate for childbearing.

Management options for macroadenomas with compressive symptoms include bromocriptine, which may
decrease the size of the tumor, and surgical resection, often in combination. Resection is associated with
high recurrence rates. Radiation therapy is effective for long-term control but is associated with
panhypopituitarism.
GROWTH HORMONE HYPERSECRETION

Pathogenesis
GH stimulates production of growth-promoting hormones, including somatomedins and insulin-like GH.
Overproduction results in acromegaly, which is almost exclusively due to a pituitary adenoma, although
abnormalities in hypothalamic production of GH-releasing hormone can also occur.

Epidemiology
Acromegaly has a prevalence of 40 per million.

History
Patients may complain of sweating, fatigue, headaches, voice changes, arthralgias, and jaw malocclusion.
Symp-toms usually develop over a period of years. The patient may have a history of kidney stones.

Physical Examination
The hallmark of the disease is bony overgrowth of the face and hands, with roughened facial features and
increased size of the nose, lips, and tongue (Fig. 15-5). Signs of left ventricular hypertrophy occur in more
than half of all patients, and hypertension is common.
Figure 15-5 • Acromegaly is characterized by enlargement of the facial
features (nose, ears) and the hands and feet.
From Weber J, Kelley J. Health Assessment in Nursing. 2nd ed. Philadelphia:
Lippincott Williams & Wilkins; 2003:D10.1a.

Diagnostic Evaluation
Serum GH levels are elevated, and GH is not suppressed by insulin challenge. Insulin resistance may be
present. An MRI should be obtained to delineate the extent of the lesion.

Treatment
Treatment options include resection, radiation, and bromocriptine. Surgical cure rates are approximately
75% in patients with lower preoperative GH levels but
only 35% in patients with high preoperative GH levels. Radiation is effective but slow and may result in P
panhypopituitarism. Bromocriptine can suppress GH production in combination with other treatment
modalities; it is not usually effective as a single therapy.

FOLLICLE-STIMULATING HORMONE AND LUTEINIZING HORMONE


HYPERSECRETION
Epidemiology
These tumors comprise approximately 4% of all pituitary adenomas.

History
Patients usually complain of headache or visual field changes from compression. Symptoms of
panhypopituitarism may be present, as the tumors often grow to large size. Women have no symptoms that
are attributable to oversecretion of FSH or LH. Men with FSH-secreting tumors may complain of depressed
libido.

Physical Examination
The patient may have signs of compression of the structures surrounding the sella turcica.

Diagnostic Evaluation
Hormone levels are elevated.

Treatment
Surgery is necessary to relieve compression if it occurs.

THYROTROPIN AND ADRENOCORTICOTROPIN EXCESS


These diseases are discussed in their respective sections.
ADRENAL HYPERSECRETION

ANATOMY AND PHYSIOLOGY


The adrenal glands lie just above the kidneys, anterior to the posterior portion of the diaphragm. The right
gland is lateral and just posterior to the inferior vena cava, whereas the left gland is inferior to the stomach
and near the tail of the pancreas. The blood supply derives from the superior supra-adrenal, the middle
supra-adrenal, and the inferior supra-adrenal coming from the inferior phrenic artery, the aorta, and the
renal artery, respectively. Venous drainage on the right is to the inferior vena cava and on the left is to the
renal vein.

The gland is divided into cortex and medulla. The cortex secretes glucocorticoids (cortisol),
mineralocorticoids (aldosterone), and sex steroids, whereas the medulla secretes catecholamines
(epinephrine, norepinephrine, and dopamine; Fig. 15-6). Cholesterol is the precursor for both
glucocorticoids and mineralocorticoids through a variety of pathways, beginning with the formation of
pregnenolone, the rate-limiting step for corticoid synthesis.

Figure 15-6 • Adrenal hormones.

Cortisol is secreted in response to ACTH from the pituitary, which is, in turn, controlled by corticotropin-
releasing factor secretion from the hypothalamus. Hypovolemia, hypoxia, hypothermia, and hypoglycemia
stimulate cortisol production. Cortisol has many actions, including stimulation of glucagon release and
inhibition of insulin release.
Exogenous glucocorticoids suppress the immune system and impair wound healing. They block inflammatory
cell migration and inhibit antibody production, histamine release, collagen formation, and fibroblast
function. These effects are significant causes of morbidity in patients maintained on corticosteroid therapy.

Aldosterone secretion is controlled by the renin-angiotensin system. In response to decreased renal blood
flow or hyponatremia, juxtaglomerular cells secrete renin. This causes cleavage of angiotensinogen to
angiotensin I, which in turn is cleaved to angiotensin II. Angiotensin II causes vasoconstriction and
stimulates
aldosterone secretion. Aldosterone stimulates the distal tubule to reabsorb sodium. This increases water P
retention and works to restore circulating blood volume and pressure.

CUSHING'S SYNDROME
Pathogenesis
Cushing's syndrome is due to overproduction of cortisol. In approximately 80% of patients, cortisol
overproduction is secondary to ACTH hypersecretion. A pituitary adenoma is the cause in 80% of these
patients (strictly termed "Cushing's disease"), whereas the remainder derive from other tumors, including
small-cell carcinoma of the lung and carcinoid tumors of the bronchi and gut. Adrenal adenoma is the cause
of cortisol hypersecretion in 10% to 20% of patients, whereas adrenal carcinoma and excess corticotropin-
releasing factor production from the hypothalamus are unusual sources for increased cortisol production.

History
Patients may complain of weight gain, easy bruising, lethargy, and weakness.

Physical Examination
Patients have a typical appearance, with truncal obesity, striae, hirsutism, buffalo hump (accumulation of
fat at the base of the neck), and moon facies (full, rounded face; Fig. 15-7). Hypertension, proximal muscle
weakness, impotence or amenorrhea, osteoporosis, glucose intolerance, and ankle edema may be present.
Figure 15-7 • Cushing's syndrome demonstrating hirsutism, moon facies, and
a buffalo hump.

Diagnostic Evaluation
Increased cortisol production is most reliably demonstrated by 24-hour urine collection. Low ACTH levels
suggest an adrenal source, as the autonomously secreted cortisol suppresses ACTH production. The
dexamethasone suppression test is useful in differentiating among pituitary microadenomas,
macroadenomas, and ectopic sources of ACTH. Dexamethasone is a potent inhibitor of ACTH release. In
patients with pituitary microadenomas, dexamethasone is able to suppress ACTH production, whereas in
other patient groups, this effect is not seen. Response to corticotropin-releasing hormone stimulation is
accentuated when the source is pituitary but not when the source is adrenal or ectopic.

Treatment
Therapy is directed toward removing the source of increased cortisol production. For pituitary sources,
resection is preferred. For an adrenal source, adrenalectomy is curative if the lesion is an adenoma.
Resection should be attempted for adrenal carcinoma.

HYPERALDOSTERONISM (CONN'S SYNDROME)

Pathogenesis
Causes of excess secretion of aldosterone include adrenal adenoma (80%), idiopathic bilateral hyperplasia
(15%), adrenal carcinoma (rare), or ectopic production (rare).
Epidemiology
The prevalence among patients with diastolic hypertension is one in 200.

History
Symptoms are usually mild and include fatigue and nocturia.

Physical Examination
Hypertension is the most common finding.

Diagnostic Evaluation
Hypokalemia occurs as sodium is preferentially reabsorbed in the distal tubule, causing kaliuresis.
Aldosterone
levels in serum and urine are increased, and serum renin levels are decreased. If hyperaldosteronism is P
demonstrated, computed tomography (CT) or MRI is used to evaluate the adrenals. In this setting, the
presence of a unilateral adrenal mass >1 cm strongly suggests the diagnosis of adrenal neoplasm.

Treatment
Surgical excision is indicated for adenoma, whereas excision or debulking, or both, and chemotherapy are
the treatments of choice for carcinoma. Pharmacologic therapy for patients with idiopathic bilateral
hyperplasia usually includes a trial of potassium-sparing diuretics and dexamethasone.

EXCESS SEX STEROID PRODUCTION


Adrenal neoplasms can secrete excess sex steroids. Virilization suggests the lesion is malignant. Treatment
is surgical removal.

ADRENAL INSUFFICIENCY
Pathogenesis
Long-term corticosteroid use can lead to suppression of the adrenal cortex. In the setting of surgical stress,
the cortex may not be able to respond with the appropriate release of glucocorticoids and
mineralocorticoids. These patients are at risk for Addison's disease or acute adrenal insufficiency, which is
life-threatening.

History
Patients complain of abdominal pain and vomiting.

Physical Examination
Obtundation may occur. Hypotension, hypovolemia, and hyperkalemia can lead to shock and cardiac
arrhythmias.

Treatment
Preoperative identification of patients at risk for adrenal suppression is critical, and perioperative cortico-
steroids are necessary. The corticosteroids should be continued if the patient is in critical condition.

PHEOCHROMOCYTOMA

Pathophysiology
This tumor produces an excess of catecholamines.

Epidemiology
Pheochromocytoma is a rare tumor. It occurs most commonly in the third and fourth decades, with a slight
female predominance. Approximately 5% to 10% occur in association with syndromes, including the multiple
endocrine neoplasias types IIa and IIb. Approximately 10% are malignant. Pheochromocytoma is the cause of
hypertension in fewer than 0.2% of patients. The catecholamine source is most commonly the adrenals but
can occur elsewhere in the abdomen (10%) or outside the abdomen (2%).

History
Patients may complain of headaches, tachycardia or palpitations, anxiety, sweating, chest or abdominal
pain, and nausea either in paroxysms or constant in nature. Physical exertion, tyramine-containing foods,
nicotine, succinylcholine, and propranolol can precipitate attacks.

Diagnostic Evaluation
Systolic blood pressure can be marked by peaks approaching 300 mm Hg but may be normal on a single
reading. Diagnosis is established by elevated urinary epinephrine and norepinephrine, as well as their
metabolites, metanephrine, normetanephrine, and vanillylmandelic acid. CT or MRI yields information
about tumor size and location (Fig. 15-8). Nuclear medicine scan using radioactive metaiodobenzylguanidine
is especially useful for finding extra-adrenal tumors.
Figure 15-8 • Computed tomography scan demonstrating a
pheochromocytoma.

Treatment
Pheochromocytomas are removed surgically (see Color Plate 12). Preoperative preparation is critical to
ensure that the patient does not have a hypertensive crisis in the operating room. Alpha blockade with
phenoxybenzamine or phentolamine is usually combined with beta blockade. It is important to establish
alpha blockade first. Isolated beta blockade in the setting of catecholamine surge can produce shock, as
cardiac function is prevented from increasing while systemic vascular resistance increases.

P
INCIDENTAL ADRENAL MASS
Approximately 1% of CT scans obtained for any reason reveal an adrenal mass, making the incidental
adrenal mass a common clinical scenario. Workup includes a thorough history to find symptoms of Cushing's
syndrome, hyperaldosteronism, or pheochromocytoma. Laboratory evaluation includes urine for 24-hour
urinary-free cortisol, dexamethasone suppression test, serum sodium and potassium, and epinephrine,
norepinephrine, and their metabolites. Resection is recommended for evidence of metabolite activity either
by symptoms or by laboratory evaluation or if the mass is larger than 4 cm.
MULTIPLE ENDOCRINE NEOPLASIAS
Multiple endocrine neoplasia (MEN) I consists of the three P's: parathyroid hyperplasia, pancreatic islet cell
tumors, and anterior pituitary adenomas. It is a rare genetic disorder, inherited in an autosomal dominant
manner, affecting approximately 20 people per million. Parathyroid hyperplasia is the most common
manifestation of the syndrome and occurs in 90% of cases. Pancreatic neoplasms occur in 50%. These are
most commonly gastrinoma, but tumors of cells producing insulin, glucagon, somatostatin, and vasoactive
intestinal peptide can also occur. The anterior pituitary tumor is most commonly prolactin-secreting and
occurs in approximately 25% of patients.

A consensus panel established the diagnosis as the presence of at least two of these tumor types. When one
relative exhibits at least one of the tumor types the diagnosis of familial MEN I is made. The syndrome is
caused by a defect in the MENI gene, also called menin, which resides on the long end of chromosome 11.
The exact pathogenesis is unknown.

Compared with sporadic hyperparathyroidism, in these patients, this aspect of the disease generally
presents earlier and with no female predilection. The defect driving hyperparathyroidism seems to be
persistent and diffuse, causing multifocality and high recurrence rates after parathyroidectomy.

Patients generally present with hypercalcemia, and most commonly complain of gastrointestinal symptoms,
including constipation and vague abdominal discomfort. Renal colic may be caused by nephrolithiasis.
Psychiatric complaints of depression, anxiety, or cognitive defects may occur. Gout, pseudogout, and
chondrocalcinosis may also be present. Hypertension and accelerated atherosclerosis are common. Diagnosis
is based on hypercalcemia in the presence of elevated parathyroid hormone levels.

Pituitary adenomas in these patients are most commonly prolactinomas, but different kindreds may have
other tumors that predominate. When associated with MEN I, these tumors tend to be larger and more
aggressive than sporadic cases.

Because treatments for hyperparathyroidism and pituitary adenomas are well established, pancreatic
tumors are the major threat to long-term survival for these patients. Major tumor types are gastrinoma,
insulinoma, and other nonfunctioning pancreatic tumors. These lesions are discussed in their respective
sections in Chapter 9, Pancreas.

Treatment of MENI generally follows treatment recommendations for the individual tumors. Subtotal
parathyroidectomy should be performed for marked hypercalcemia, nephrolithiasis, or decreased bone
density. Pituitary tumors are treated as described for sporadic cases. Survival with gastrinoma may not be
improved with resection; medical therapy only may be appropriate in select patients. Patients with
insulinoma should generally be offered resection depending on tumor location; options include local
resection of pancreatic head tumors and subtotal distal pancreatectomy.

MEN II is a rare disease, with a prevalence of approximately 25 people per million. It is divided into MEN IIa,
IIb, and familial medullary thyroid cancer. The syndrome is inherited in an autosomal dominant fashion and
is due to a defect in a protein on chromosome 10 that codes for a RET proto-oncogene. This is a
receptor tyrosine kinase that is involved in cell growth and differentiation. Because the hallmark of this P
disease is medullary thyroid carcinoma (MTC), a potentially fatal lesion that can be prevented by early
thyroidectomy when the patient is still a child, screening is essential in high-risk populations to make the
diagnosis early.

MEN IIa consists of MTC, pheochromocytoma, and parathyroid hyperplasia. MTC occurs in almost all affected
patients. Diagnosis is often started with a thyroid nodule or cervical lymphadenopathy, although it is also
commonly found as a result of screening in patients with the appropriate family history. Pheochromocytoma
occurs in 40% of patients with MEN IIa. This entity should be treated first because it can complicate treating
other aspects of this syndrome, although it is unusual for this to occur before the onset of MTC.

MEN IIb consists of MTC, pheochromocytoma, and mucosal neuromas, with characteristic body habitus,
including thick lips, kyphosis, and pectus excavatum. Diagnosis and treatment follow treatment for the
individual lesions (Table 15-1).

TABLE 15-1 Multiple Endocrine Neoplasias (MEN)

MEN I

Parathyroid hyperplasia

Pancreatic islet cell tumors

Anterior pituitary adenoma

MEN IIa

MTC

Pheochromocytoma

Parathyroid hyperplasia

MEN IIb

MTC

Pheochromocytoma

Mucosal neuromas
MTC, medullary thyroid carcinoma.

KEY POINTS
Prolactinoma is the most common pituitary tumor and is usually not malignant.
The diagnosis of acromegaly is based on characteristic appearance and elevated
growth hormone levels; treatment options include surgery, radiation, and
bromocriptine.
Cushing's syndrome results from overproduction of cortisol, most commonly due to
adrenocorticotropin overproduction from a pituitary tumor.
Adrenal adenoma is the most common cause of hyperaldosteronism.
Patients on corticosteroids preoperatively must be identified and perioperative
corticosteroids considered to avoid life-threatening adrenal insufficiency.
Patients with pheochromocytoma may present with paroxysms of headache, flushing, and
anxiety. Diagnosis is made on urine examination for catecholamines and catecholamine
metabolites.
Establishing alpha blockade before beta blockade is imperative to prevent cardiovascular
collapse.
Incidental adrenal masses should be excised if they have symptomatic or biochemical
evidence of activity or if they are >4 cm in diameter.
Multiple endocrine neoplasia (MEN) I consists of parathyroid hyperplasia, pancreatic islet
cell tumors, and anterior pituitary adenomas. MEN IIa consists of medullary thyroid
carcinoma, pheochromocytoma, and parathyroid hyperplasia. MEN IIb consists of
medullary thyroid carcinoma, pheochromocytoma, and mucosal neuromas.
Authors: Karp, Seth J.; Morris, James P.G.; Zaslau, Stanley
Title: Blueprints Surgery, 5th Edition

Copyright ©2008 Lippincott Williams & Wilkins

> Table of Contents > Part IV - Cardiac, Thoracic, and Vascular > Chapter 16 - Vascular Surgery

Chapter 16
Vascular Surgery

ANEURYSMS AND DISSECTIONS


An aneurysm is an abnormal dilation of an artery. Saccular aneurysms occur when a portion of the artery
forms an outpouching, or "mushroom." Fusiform aneurysms occur when the entire arterial diameter grows.
True aneurysms involve all layers of the arterial wall: intima, media, and adventitia. An artery is considered
aneurysmal if the diameter is >1.5 times its normal size. Otherwise, an enlarged artery is considered
ectatic.

In contrast, a dissection occurs when a defect in the intima allows blood to enter between layers of the wall
(Fig. 16-1). Blood pressure then causes the layers of the wall to separate from one another. The serious
nature of aneurysms and dissections is due to the weakened vessel wall and the potential for catastrophic
events. In the case of an aneurysm, this includes rupture or vascular compromise; dissections can result in
the occlusion of the ostia of visceral arteries or progress into the heart and can affect the coronary
circulation or lead to tamponade.

Figure 16-1 • Aneurysms and dissections.


ABDOMINAL AORTIC ANEURYSM

Anatomy
The abdominal aorta lies below the diaphragm and above the iliac arteries. Branches include the celiac
trunk, superior mesenteric artery, inferior mesenteric artery, renal arteries, and gonadal arteries.
Approximately 95% of abdominal aneurysms begin distal to the takeoff of the renal arteries.

Etiology
Ninety-five percent of aneurysms of the abdominal aorta are associated with atherosclerosis. Other causes
include trauma, infection, syphilis, and Marfan's syndrome. Protease activity in the vessel wall is commonly
increased.

EPIDEMIOLOGY
Abdominal aortic aneurysms are responsible for 15,000 deaths per year. The incidence is approximately
0.05%, but in selected high-risk populations, the incidence increases to 5%. Men are affected 10 times more
frequently than women, with an age of onset usually between 50 and 70 years. Risk factors include
atherosclerosis, hypertension, hypercholesterolemia, smoking, and obesity. The disease is associated with
peripheral vascular disease, heart disease, and carotid artery disease.

HISTORY
Most aneurysms are asymptomatic. Pain usually signifies a change in the aneurysm–-commonly enlargement,
rupture, or compromise of vascular supply–-and should therefore be considered an ominous symptom. Pain
may occur in the abdomen, back, or flank. The legs could be involved if the aneurysm includes the iliac
arteries or if an embolic event occurs. The pain is usually sudden in onset and does not remit.

P
Physical Examination
Abdominal examination may reveal a pulsatile abdominal mass. Enlargement, rupture, or compromise of
vascular supply may manifest by tenderness, hypotension, tachycardia, or a change in the location or
intensity of pain. In addition, the lower extremities may have pallor, cool temperature or pulses that are
diminished or unequal.

Diagnostic Evaluation
Ultrasound is an accurate, noninvasive method to assess the size of the aneurysm and the presence of clot
within the arterial lumen. Computed tomography (CT) or magnetic resonance imaging provides anatomic
detail and precise localization of the aneurysm. An aortogram may be helpful in planning surgical
intervention to demonstrate involvement of other vessels, specifically the renal, mesenteric, and iliac
arteries.

Treatment
If the patient is asymptomatic, workup can proceed electively. Treatment of asymptomatic abdominal aortic
aneurysms depends on the size of the lesion, which is directly proportional to its propensity to grow, leak,
or rupture. Aneurysms <4 cm in diameter are unlikely to rupture, and medical management with
antihypertensives, preferably beta-blockers, is advocated. When the aneurysm reaches approximately 4 to 5
cm, two options are available: early operation or close follow-up. A recent randomized trial suggests that
mortality is the same in both options. When the aneurysm reaches 5 cm in diameter, the incidence of
rupture is >25% at 5 years, and repair is recommended, unless the patient is at prohibitive operative risk.
Table 16-1 lists rupture rates per year based on aneurysm size. Treatment options have recently expanded
with the advent of stent grafts that can be placed through the femoral artery. In selected patients, these
stents carry less morbidity than traditional operative repair. Concerns include stent migration and leaks
around the prosthesis, but in general, these complications can be managed effectively. Technologic
advances and clinical experience are widening the versatility of these stents, allowing the placement of
fenestrated stents with orifices for visceral vessels and stents with limbs that can be placed at arterial
bifurcations.

TABLE 16-1 Ankle-Brachial Index

Aneurysm Size Risk of Rupture per Year

<5 cm 4%

5–7 cm 7%

>7 cm 19%

Any patient presenting on physical examination with symptoms that suggest a catastrophic aortic event
should undergo emergent diagnostic workup or intervention. Once the diagnosis of ruptured or leaking
abdominal aortic aneurysm is determined, arrangements should be made for fluid resuscitation and
immediate operative intervention.

Repair of Abdominal Aortic Aneurysm: The Operation


Consistent with the size of the operation, preoperative preparation includes large-bore intravenous lines,
central monitoring, and intravenous antibiotics. Blood, either autologous or cross-matched, should be
available. Abdominal aortic aneurysms can be approached via either a midline incision or an oblique incision
over the left 11th intercostal space. Using a midline incision requires mobilization of the small bowel to the
patient's right. Incision of the posterior peritoneum to the left of the aorta allows exposure of the entire
aorta. The oblique incision is reserved for a retroperitoneal approach, in which the entire contents of the
peritoneal cavity are mobilized to the right, allowing exposure of the aorta. Proximal and distal control
around the aneurysm is obtained, and heparin is administered before clamping. A graft is placed using
permanent sutures. If a transabdominal
approach is used, the peritoneum is closed over the graft if possible (Fig. 16-2). P
Figure 16-2 • Aneurysm, aortic: abdominal aneurysm of the descending
aorta.
LifeART image copyright (c) 2006 Lippincott Williams & Wilkins. All rights
reserved.
THORACIC AORTIC ANEURYSM

Anatomy
The thoracic aorta lies between the heart and the diaphragm. It gives rise to the brachiocephalic, left
common carotid, left subclavian, bronchial, esophageal, and intercostal arteries.

Etiology
Thoracic aortic aneurysms are caused by cystic medial necrosis, atherosclerosis, or, less commonly, trauma,
dissection, or infection.

EPIDEMIOLOGY
Males are affected three times as often as females. Risk factors include atherosclerosis, smoking,
hypertension, and family history.

History
Most aneurysms are asymptomatic. Rupture usually presents with chest pain or pressure. Expansion of the
aneurysm can compress the trachea, leading to cough, or erode into the trachea or bronchus, causing
massive hemoptysis. An aneurysm close to the aortic valve can cause dilation of the annulus, resulting in
aortic valve insufficiency and chest pain, dyspnea, or syncope.

Physical Examination
Hypotension and tachycardia may be present. If the aneurysm involves the aortic annulus, it can lead to
aortic regurgitation and congestive heart failure. Pulse examination may be abnormal if distal embolization
occurs.

Diagnostic Evaluation
Chest radiography may show a widened thoracic aorta. Electrocardiography may demonstrate myocardial
ischemia, especially if the aneurysm compromises the coronary supply. In the asymptomatic patient with a
thoracic aneurysm, CT or echocardiography is helpful in establishing the diagnosis. Echocardiography can
also determine the extent of involvement of the aortic valve and possible cardiac tamponade. Aortography
may be useful for planning operative intervention, because it defines the aneurysm's relation to a number of
critical structures.

Treatment
As with abdominal aortic aneurysms, operative repair should be considered when the maximum diameter
approaches 5 cm. Symptomatic presentation
is an indication for immediate operative intervention. As with abdominal aortic aneurysms, the indications P
for stent grafts for thoracic aortic aneurysms are being carefully evaluated.

AORTIC DISSECTION
PATHOGENESIS
Dissections can be caused by hypertension, trauma, Marfan syndrome, or aortic coarctation.

Epidemiology
Aortic dissections are more common than either thoracic or abdominal aneurysms. Incidence increases with
age, and men are more commonly affected than women.

History
Patients usually complain of the immediate onset of severe pain, often described as tearing, usually in the
chest, back, or abdomen. Nausea or light-headedness may also be present.

Physical Examination
Patients may be hypotensive. Rales on chest auscultation or a new murmur suggest that the dissection
continues retrograde into the aortic root. Peripheral pulses are diminished if distal blood flow is
compromised. If the dissection continues into the visceral arteries, compromise of mesenteric vessels can
produce abdominal pain, compromise of renal arteries can cause oliguria, and compromise of spinal blood
supply can produce neurologic deficits.

Diagnostic Evaluation
A chest radiograph may show a widened mediastinum. CT may show the dissection or clot in the arterial
lumen. Diagnosis can be made with transesophageal ultrasound, magnetic resonance imaging, or aortogram.
Dissections are classified according to the DeBakey classification: type I involves both the ascending and the
descending aorta, type II involves only the ascending aorta, and type III involves only the descending aorta.

Treatment
Dissection of the ascending thoracic aorta usually requires surgery, because of the potential for retrograde
progression into the aortic root and subsequent compromise of the coronary circulation or tamponade from
rupture into the pericardium. Eighty percent of patients with involvement of the ascending aorta die
without treatment. Antihypertensive therapy is used preoperatively in an attempt to halt the progression of
the dissection.

In contrast, dissections limited to the descending aorta are best managed medically, with antihypertensives,
including sodium nitroprusside and beta blockade. Invasive monitoring with fluid resuscitation should be
instituted immediately. Surgery is reserved for lesions that progress or cause distal ischemia. Stent grafts
have been shown to be safe in selected patients. Determination of who should receive a stent graft is being
studied.

CAROTID ARTERY DISEASE

ANATOMY
The common carotid artery on the right arises from the brachiocephalic artery, and on the left, from the
aorta. The common carotid then bifurcates into internal and external branches. The internal carotid gives
off the ophthalmic artery before continuing to the circle of Willis to supply the brain. The area around the
bifurcation is clearly "high-rent" territory, which contains a number of vital structures which can be injured
during surgery. The marginal mandibular branch of the facial nerve lies deep and inferior to the horizontal
ramus of the mandible. Injury to this nerve during dissection results in lower lip paralysis. Branches of the
cervical plexus, including the greater auricular nerve, may be at the cranial aspect of the incision and
should be avoided to preserve sensation to the ear and the angle of the mandible. Posterolateral to the
carotid artery in the carotid sheath lies the vagus nerve. Injury results in vocal cord paralysis. One or two
centimeters above the carotid bifurcation is the hypoglossal nerve; injury results in dysphagia and tongue
deviation. In contrast, the ansa cervicalis, which courses inferiorly from the hypoglossal to innervate the
strap muscles, can be sacrificed as needed for exposure. Laterally and deep to the carotid artery lies the
jugular vein, a branch
of which, the facial vein, is generally divided to facilitate exposure of the carotid. P

PATHOGENESIS
Symptoms are the result of atherosclerosis. Mechanisms of morbidity include plaque rupture, ulceration,
hemorrhage, thrombosis, and low flow states. Because of the rich collateralization of the cerebral
circulation through the circle of Willis, thrombosis and low flow states may be asymptomatic.

Epidemiology
Atherosclerotic occlusive disease of the carotid artery is a major cause of stroke. In the United States,
400,000 people are hospitalized for stroke each year, and cerebrovascular events are the third most
common cause of death. The incidence of stroke increases with age. Other risk factors include
hypertension, diabetes, smoking, and hypercholesterolemia. Markers for carotid disease include evidence of
other atherosclerotic disease and prior neurologic events.

History
Patients often relate previous neurologic events, including focal motor deficits, weakness, clumsiness, and
expressive or cognitive aphasia. These could occur as a transient ischemic attack, which resolves in 24
hours; a reversible ischemic neurologic deficit, which resolves in greater than 24 hours; or a fixed
neurologic deficit. One characteristic presentation for carotid disease is amaurosis fugax, or transient
monocular blindness, usually described as a shade being pulled down in front of the patient's eye. This is
due to occlusion of a branch of the ophthalmic artery.

PHYSICAL EXAMINATION
Patients may exhibit a fixed neurologic deficit. Hollen-horst plaques on retinal examination are evidence of
previous emboli. A carotid bruit is evidence of turbulence in carotid blood flow, but the presence of a bruit
does not unequivocally translate into a hemodynamically significant lesion, and the absence of a bruit does
not unequivocally indicate the absence of significant disease.

DIAGNOSTIC EVALUATION
Carotid duplex scanning is both sensitive and specific for carotid disease. Conventional or magnetic
resonance angiography is more accurate for assessing the degree of stenosis.

TREATMENT
Treatment depends on the history, degree of stenosis, and characteristics of the plaque. Antiplatelet
therapy with aspirin is effective in preventing neurologic events. When dealing with an acute event, heparin
should be considered after head CT determines that the event is not hemorrhagic. Indications for carotid
endarterectomy are controversial. Results of two large randomized controlled trials–-the Asymptomatic
Carotid Atherosclerosis Study (ACAS) and the North American Symptomatic Carotid Endarterectomy Trial
(NASCET)–-suggest surgery is best reserved for the following patients: those with >75% stenosis, those with
70% stenosis and symptoms, those with bilateral disease and symptoms, or those with >50% stenosis and
recurring transient ischemic attacks despite aspirin therapy. The role of stenting is controversial. Despite
the Systolic and Pulse Pressure Hemodynamic Impro-vement by Restoring Elasticity (SAPPHIRE) trial, which
suggested stenting is not inferior to endarterectomy in high-risk patients, there were problems with the
study design, and the procedure is still being evaluated as an alternative to open surgery, which has low
morbidity and mortality in large centers.

CAROTID ENDARTERECTOMY: THE OPERATION


Perioperative monitoring is surgeon-dependent. Techniques include keeping the patient awake through the
procedure, using local or regional anesthesia, using continuous electroencephalogram monitoring, or using
no monitoring at all. Administration of intravenous antibiotics, usually a first-generation cephalosporin,
precedes the incision. After site verification, the neck is extended, and an incision is made over the
anterior border of the sternocleidomastoid muscle. Dissection continues through the platysma and along the
sternocleidomastoid. Ligation of the facial vein allows complete exposure of the carotid, which lies just
medial to the jugular vein. Care is taken not to injure the hypoglossal nerve at the superior aspect of the
dissection or the spinal accessory nerve (Fig. 16-3). Proximal and distal control of the carotid is obtained,
the patient
is heparinized, and the artery is opened after clamps are applied. Use of a stent may provide cerebral P
protection. Plaque is carefully dissected out of the artery, and the artery is usually closed with a patch.
Figure 16-3 • Carotid endarterectomy: pertinent anatomical structures.

ACUTE AND CHRONIC MESENTERIC VASCULAR DISEASE

ANATOMY
Acute and chronic mesenteric vascular disease includes disease of the celiac axis, which is the arterial
supply to the liver, spleen, pancreas, and stomach; the superior mesenteric artery, which supplies the
pancreas, small bowel, and proximal colon; and inferior mesenteric artery, which supplies the distal colon
and rectum. In addition, thrombosis of the superior mesenteric vein can cause visceral ischemia.

The celiac axis arises from the aorta below the crus of the diaphragm. It travels ventrally and is surrounded
by a dense network of nerves and connective tissue. After a short distance, it gives rise to the splenic
artery, which travels to the left behind the pancreas to the spleen; the common hepatic artery, which
travels to the right toward the hepatic hilum; and the smaller left gastric artery, which travels cranial and
toward the left. The superior mesenteric artery courses below the pancreas and slightly to the right toward
the mesentery of the bowel. The inferior mesenteric artery arises below the renal arteries and above the
aortic bifurcation and travels to the left toward the bowel. The superior mesenteric vein runs above and to
the right of the superior mesenteric artery. Its location can be determined by palpating the artery at the
root of the mesentery in thin patients. The vein then joins the splenic and inferior mesenteric veins to form
the portal vein.
PATHOGENESIS
Acute ischemia is caused by embolization, thrombo-sis, nonocclusive ischemia, and mesenteric vein throm-
bosis. Embolization is associated with atherosclerotic disease or mural cardiac thrombus. Acute thrombosis
is
associated with atherosclerosis and hypercoagulable states. Vasopressor agents can produce acute ischemia. P
Chronic ischemia usually requires severe atherosclerotic disease in at least two major arterial trunks among
the superior and inferior mesenteric arteries and the celiac axis, because of the extensive collateralization.

EPIDEMIOLOGY
The incidence of acute mesenteric ischemia is estimated at one in 1,000 hospital admissions, and mortality
is >50%. Prevalence of chronic ischemia increases with age, and risk factors include hypertension, smoking,
hypercholesterolemia, and diabetes.

HISTORY
Patients with acute ischemia may have a history of previous embolic events, atrial fibrillation, or congestive
failure. Abdominal pain is usually sudden in onset and is severe, with diarrhea or vomiting. History in
chronic mesenteric ischemia usually reveals crampy abdominal pain after eating. This results in decreased
oral intake and weight loss. Nausea, vomiting, constipation, or diarrhea may occur. The disease can be
mistaken for malignant disease or cholelithiasis.

PHYSICAL EXAMINATION
In episodes of acute ischemia, the classic finding is "pain out of proportion to physical examination." The
abdomen may be distended. Rectal examination often reveals guaiac-positive stool. Atrial fibrillation may
be present. Physical findings in chronic ischemia include abdominal bruits, guaiac-positive stool, and
evidence of peripheral vascular disease or coronary artery disease.

DIAGNOSTIC EVALUATION
In acute ischemia, there could be an elevated white blood cell count, metabolic acidosis, or an elevated
hematocrit as fluid is sequestered in the infarcting bowel. Abdominal radiographs are often normal in the
early phase of the disease, but as the intestine becomes edematous, "thumbprinting" of the bowel wall
occurs. Evaluation in chronic ischemia includes selective visceral angiography to identify the site of the
lesion.

TREATMENT
Once the diagnosis of acute ischemia is made, laparot-omy with examination and resection of any infarcted
bowel should be considered. In selected cases, angiography can be therapeutic, as well as diagnostic, with
catheter-based therapies. Aggressive surgical intervention should not be delayed if there is evidence
suggestive of dead bowel. Despite aggressive intervention, mortality is extremely high. For chronic
ischemia, angiography can define the lesion and allow consideration of surgical options. Acute mesenteric
vein thrombosis is treated with anticoagulation and laparotomy if necrotic bowel is suspected.

PERIPHERAL VASCULAR DISEASE


ANATOMY
Lesions may occur in the iliac, common and superficial femoral, popliteal, peroneal, anterior tibial, and
posterior tibial arteries. The common iliac arteries arise at approximately the level of the umbilicus. They
course in the retroperitoneal space and give rise to the internal iliac artery, which runs to the pelvis, and
the external, which becomes the femoral artery in the femoral canal as it passes under the inguinal
ligament. It enters the adductor canal, below the Sartorius, and continues through the adductor hiatus,
where is becomes the popliteal artery. Below the knee, the popliteal artery branches into the anterior
tibial, the posterior tibial, and dorsalis pedis arteries, which supply the distal leg. In general, of the three
vessels that supply the distal ankle and foot, a single direct arterial supply is adequate to prevent limb loss
and rest pain.

PATHOGENESIS
In acute disease, the most common cause is an embolus that causes a sudden decrease in blood flow. The
most common sources are the aorta and the heart. Rarer causes include acute arterial thrombosis, acute
venous thrombosis, and arterial spasm. In chronic disease, progressive atherosclerotic disease causes
narrowing of the arterial lumen and decreased blood flow. Pain occurs as decreased blood flow is unable to
meet the metabolic and waste-removal demand of the tissue.

EPIDEMIOLOGY
Acute disease occurs in patients with cardiac thrombus, atrial fibrillation, or atherosclerosis. Risk factors
for chronic disease include atherosclerosis, smoking, diabetes, hypertension, and advanced age. P

HISTORY
Acute ischemia causes sudden and severe lower-extremity pain and paresthesias. Patients with chronic
ischemia typically present with claudication, defined as reproducible pain on exercise relieved by rest. The
site of claudication provides a clue to the level of disease. Buttock claudication usually indicates aortoiliac
disease, whereas calf claudication suggests femoral atherosclerosis. Pain at rest is indicative of severe
disease and a threatened limb. Slow or nonhealing ulcers may be present.

PHYSICAL EXAMINATION
In acute disease, the patient may exhibit pulselessness, pallor, and poikilothermia (coolness). Taken
together with pain and paresthesia, these form the five P's of acute vascular compromise (Fig. 16-4). In
chronic disease, the lower extremity may reveal loss of hair, pallor on elevation, rubor on placing the
extremity in a dependent position, wasting of musculature, thick nails, and thin skin. The extremity may be
cool to the touch, and pulses may be diminished or absent. Ulcers or frank necrosis may be present.
Figure 16-4 • Signs and symptoms of acute embolus.

DIAGNOSTIC EVALUATION
Angiography is necessary in cases of acute ischemia to identify the lesion. Evaluation for chronic ischemia
includes Doppler flow measurement of distal pulses. The normal signal is triphasic; as disease progresses,
the signal becomes biphasic, monophasic, and then absent. Ankle-brachial indices are calculated as the
systolic blood pressure at the ankle divided by the systolic pressure in the brachial artery. A value of <0.5 is
indicative of significant disease (Table 16-1). Arteriography is the gold standard for defining the level and
extent of disease and for planning surgery.

TREATMENT
Acute ischemic embolus can be treated with heparin, thrombolysis, or embolectomy. For chronic ischemia,
patients with claudication have a low rate of limb loss, and initial therapy is based on smoking cessation
and a graded exercise program. Success rates with nonoperative therapy are good. In patients with disabling
claudication, threatened limbs, nonhealing ulcers, or gangrene, angioplasty or revascularization should be
considered (Table 16-2). It is always prudent to consider amputation in selected patients with long-standing
acute ischemia or chronic ischemia, though this decision is often a difficult one and requires experience and
careful discussions with the patient.

TABLE 16-2 Progression of Peripheral Vascular Disease

Claudication Rest Pain Gangrene

Blood Decreased Markedly Minimal


flow decreased

ABI ~0.5 0.3–0.5 <0.3

Treatment Smoking Revascularization Amputation


cessation

Graded Angioplasty Revascularization


exercise

Angioplasty

PERIPHERAL BYPASS: THE OPERATION


Patients usually have concurrent coronary artery disease, and preoperative evaluation and intraoperative
beta blockade should be considered. The entire leg is usually prepped. Intravenous antibiotics are often a
first-generation cephalosporin, and central monitoring should be considered. The most common techniques
for anesthesia include general and regional. If the proximal vessels are open, inflow is usually from the
femoral artery, which is dissected via an infrainguinal incision directly over the artery. If the target is the
popliteal, it is isolated via an incision over the medial aspect of the knee, at the level where the outflow
will be targeted. If the target is the dorsalis pedis, tibialis anterior, or peroneal, the incision is made
directly over the target. Above-knee popliteal reconstruction can be accomplished with synthetic grafts,
but infrapopliteal reconstructions should use either in situ or reversed saphenous vein.

P
VENOUS DISEASE

DEEP VENOUS THROMBOSIS


First-line therapy for patients with deep venous thrombosis or pulmonary embolism is systemic
anticoagulation. In patients for whom this therapy is contraindicated (e.g., patients requiring a planned
major surgery, those with active bleeding, or those facing significant complication of anticoagulation), filter
placement in the inferior vena cava may decrease the short-term risk of pulmonary embolism. The use of
filters, though common, is generally not well supported by randomized controlled trials. They have not
been shown to decrease mortality and are associated with significant complications, including inferior vena
cava thrombosis, recurrent deep venous thrombosis, and postphlebitic syndrome. Recently, removable
filters increasingly are being used to try to decrease the risk of long-term complications, but their use is
still being studied.

VARICOSE VEINS
This is an extremely common lesion, affecting up to 15% of men and 25% of women. Over time, the pressure
of gravity in the lower extremity may cause enlargement of the veins in the leg as valves fail. In most cases,
the lesions are of cosmetic concern only, though they can become large and painful and cause ulcers. In
deep veins this can lead to venous thrombosis. Treatment options include sclerotherapy, laser or catheter-
based ablation, and vein stripping.

KEY POINTS
Aneurysms and dissections can be rapidly fatal.
Repair via open operation or stent graft should be considered for asymptomatic
aneurysms with diameter >4 or 5 cm.
Symptomatic aneurysms or dissections require emergency diagnosis and treatment.
Dissections that involve the ascending aorta usually require surgery, whereas dissections
that involve the descending aorta are best managed medically.
Carotid artery disease is a major cause of stroke in the United States.
Indications for operation include 75% stenosis, 70% stenosis and symptoms, bilateral
disease and symptoms, or >50% stenosis and recurring transient ischemic attacks despite
aspirin therapy.
Stenting is an option for selected patients.
Patients with acute mesenteric ischemia present with "pain out of proportion to
examination," and a mechanism for embolic disease is usually present.
Chronic mesenteric ischemia results in weight loss and abdominal pain and is frequently
mistaken for malignant disease.
Acute peripheral embolus is marked by the five P's: pulselessness, pallor, poikilothermia,
pain, and paresthesia.
Symptoms of chronic peripheral vascular disease usually follow a well-defined progression.
Operation should be considered only in patients with severe chronic peripheral vascular
disease.
Authors: Karp, Seth J.; Morris, James P.G.; Zaslau, Stanley
Title: Blueprints Surgery, 5th Edition

Copyright ©2008 Lippincott Williams & Wilkins

> Table of Contents > Part IV - Cardiac, Thoracic, and Vascular > Chapter 17 - Heart

Chapter 17
Heart

ANATOMY
Coronary circulation begins at the sinus of Valsalva, where the right and left coronary arteries
(RCA, LCA) arise. The left main artery branches into the left anterior descending and the left
circumflex arteries. The left anterior descending artery supplies the anterior of the left
ventricle, the apex of the heart, the intraventricular septum, and the portion of the right
ventricle that borders the intraventricular septum (Fig. 17-1). The left circumflex artery travels
in the groove separating the left atrium and ventricle and gives off marginal branches to the left
ventricle. The RCA travels between the right atrium and ventricle to supply the lateral portion of
the right ventricle (Fig. 17-2). The posterior descending artery (PDA) comes from the RCA in 90%
of patients and supplies the arteriovenous node. Patients whose PDA arises from the RCA are
termed right dominant. If the PDA arises from the left circumflex, the system is left dominant.
Figure 17-1 • Schematic showing the origin of the coronary
circulation. LMA, left main artery; LAD, left anterior descending;
RCA, right coronary artery; RPD, right posterior descending; LCX,
left circumflex; AV, aortic valve; L, left; R, right.

LifeART image copyright (c) 2009 Lippincott Williams & Wilkins. All
rights reserved.
Figure 17-2 • Schematic showing the path of the coronary
circulation. The left main artery gives rise to the left anterior
descending and left circumflex arteries.

The aortic valve is located between the left ventricle and the aorta. It usually has three leaflets,
which form three sinuses. One sinus gives rise to the RCA, another to the LCA, and the third
forms the noncoronary sinus. The mitral valve is located between the left atrium and ventricle.
It normally has two leaflets, with the anterior protruding farther across the valve. Chordae
tendineae attach the leaflets to the papillary muscles, which in turn serve to tether the leaflets
to the ventricular wall.

CONGENITAL HEART DISEASE


Approximately 0.6% of live births will be complicated by congenital heart defects. This risk more
than doubles in subsequent siblings. Most cases are sporadic, although there are well-known
associations of genetic syndromes and congenital heart defects. These include atrioventricular
canal defects in children with Down syndrome, coarctation of the aorta in children with Turner's
syndrome, and supravalvular aortic stenosis in children with William's syndrome.

Congenital heart defects are best understood by the pathophysiology they cause, either
congestive heart failure or cyanosis. Most anomalies fit into one of these categories.

Congestive heart failure can be caused either by a left-to-right shunt or an obstructive lesion.
Left-to-right shunts result in a certain fraction of blood leaving the left side of the heart to the
right side, increasing pulmonary pressure and necessitating large volume of left ventricular
output to satisfy peripheral demands. Among these lesions are patent ductus arteriosus, in which
the normal fetal connection between the pulmonary artery and aorta is not obliterated at birth.
Depending on the size of the communication, it can cause severe failure. Premature infants with
pulmonary compromise and/or children with an audible murmur should undergo closure, which
can be achieved via catheter-based therapies.

Atrial and ventricular septal defects also result in left-to-right shunting. Patients with atrial
septal defects classically have a second heart sound in which the split is of fixed duration. Over
time such patients can develop pulmonary hypertension, right and left ventricular failure, and
atrial arrhythmias. Most atrial septal defects can be closed using percutaneous methods.
Ventricular septal defects may present in a similar fashion with pulmonary hypertension and
congestive heart failure, culminating in Eisenmenger syndrome when the shunt reverses to a
right-to-left shunt, causing cyanosis. These lesions should be fixed.

Atrioventricular canal defects are complicated lesions involving the atrial and ventricular septae P
and the atrioventricular valves. When diagnosed, these lesions should be fixed. In its extreme
form, a single arterial trunk arises from a joined ventricular chamber; this is termed a truncus
arteriosus.

Obstructive lesions can also cause congestive heart failure. Aortic stenosis most commonly
occurs as a result of a bicuspid aortic valve, but can also occur at the supravalvular or
subvalvular location, or as a result of subvalvular ventricular hypertrophy, termed hypertrophic
muscular subaortic stenosis. Although many procedures may be used to treat this condition, of
particular interest is the Ross operation, which involves transposing the pulmonary valve to the
aortic position and using a prosthetic graft for a new pulmonary valve.

Mitral stenosis commonly occurs in association with other congenital heart defects. Pulmonic
stenosis is often mildly symptomatic and is generally treated with percutaneous methods.

The most common obstructive lesion in children is coarctation of the aorta. This occurs when the
aorta becomes narrowed, usually just after the takeoff of the left subclavian artery (Fig. 17-3).
Often, blood flow to the lower extremities will depend on a patent ductus arteriosus, closure of
which can cause severe heart failure. Because of the risk of endocarditis, severe hypertension,
congestive heart failure, and stroke, these lesions are repaired.
Figure 17-3 • Coarctation of the aorta. The aorta is narrowed distal
to the takeoff of the left subclavian artery. The increased pressure
caused by this lesion frequently causes the ductus arteriosus to
remain open.
From Pillitteri A. Maternal and Child Nursing. 4th ed. Philadelphia,
PA: Lippincott Williams & Wilkins, 2003.
Right-to-left shunts occur when deoxygenated blood makes its way into the peripheral
circulation; this is also referred to as cyanotic heart disease. The most common cause is
Tetralogy of Fallot (Fig. 17-4). This abnormality has four components: ventricular septal defect,
pulmonic stenosis, right ventricular hypertrophy, and an overriding aorta.

Figure 17-4 • Tetralogy of Fallot. Coronal view of heart in tetralogy


of Fallot. Characteristics include pulmonary trunk stenosis, right
ventricular hypertrophy, dextroposition of the aorta, and ventricular
septal defect.
LifeART image copyright (c) 2009 Lippincott Williams & Wilkins. All
rights reserved.

Tricuspid atresia prevents flow from the right atrium to the right ventricle, and instead, P
deoxygenated blood traverses an atrial septal defect into the left atrium. Other lesions that can
cause a right-to-left shunt include pulmonic atresia, transposition of the great arteries, and total
anomalous pulmonary venous connection.

CORONARY ARTERY DISEASE

EPIDEMIOLOGY
Atherosclerosis of the coronary arteries is the most common cause of mortality in the United
States, responsible for one third of all deaths. Approximately 5 million Americans have coronary
artery disease, which is five times more prevalent in men than in women. Risk factors include
hypertension, family history, hypercholesterolemia, smoking, obesity, diabetes, and physical
inactivity.

PATHOPHYSIOLOGY
Coronary artery stenosis is a gradual process that begins in the second decade. When the lumen
decreases to 75% of the native area, the lesion becomes hemodynamically significant.

HISTORY
Patients with ischemic heart disease usually complain of substernal chest pain or pressure that
may radiate down the arms or into the jaw, teeth, or back. Typically, the pain occurs during
periods of physical exertion or emotional stress. Episodes that are reproducible and resolve with
rest are termed stable angina. If the pain occurs at rest or does not improve with rest, is new
and severe, or is progressive, it is termed unstable angina and suggests impending infarction.
Presentation can be variable, with patients complaining of indigestion, nausea, vomiting,
diaphoresis, cough, new onset arrhythmia, syncope, and, in older adult patients, confusion or
delirium. Ischemic heart disease can also be asymptomatic, classically in patients with diabetes.

PHYSICAL EXAMINATION
The patient may have evidence of peripheral vascular disease, including diminished pulses. Signs
of ventricular failure, including cardiomegaly, congestive heart failure, an S3 or S4, or murmur of
mitral regurgitation (MR), may occur.

DIAGNOSTIC EVALUATION
Electrocardiogram (ECG) may show signs of ischemia or an old infarct. A chest radiograph may
show an enlarged heart or pulmonary congestion. An exercise stress test is sensitive in
identifying myocardium at risk. These areas can be localized using nuclear medicine scans,
including thallium imaging. Echocardiography is extremely useful in evaluating myocardial
function and valvular competence. Angiography is the gold standard for identifying lesions in the
coronary arteries, assessing their severity, and planning operative intervention.

TREATMENT
Patients with severe disease of the left main artery or with severe disease in the three major
coronary arteries have decreased mortality after coronary artery bypass surgery. Pain is reliably
relieved in >85% of patients. Surgical options include bypass using the internal mammary arteries
or saphenous veins. Internal mammary bypass is preferred because of higher patency rates.

Percutaneous coronary interventions include balloon angioplasty and stent placement. Despite
the tremendous volume of procedures performed in the United States, the exact indications for
interventions
are not well known. A large study in 2005 in the New England Journal of Medicine by Hannon et P
al. demonstrated that in patients with two or more diseased vessels, stents were associated with
an increased need for further procedures and increased mortality compared with bypass
grafting. Because early mortality was higher with bypass grafting, care must be individualized to
the patient.

One common problem with coronary stents is in-stent restenosis. Benefits of drug-eluting stents,
which elaborate various substances including sirolimus and paclitaxel, remain unclear.

It is important to note that nearly all risk factors for coronary artery disease are modifiable
before and after an event. Preventive medicine used to aggressively lower blood pressure and
cholesterol, combined with diet and exercise regimens, is critical.

AORTIC STENOSIS

ETIOLOGY
Aortic stenosis (AS) can present early in life–-for example, when the valve is unicuspid–-but more
commonly occurs in the older population. A congenitally bicuspid valve usually causes AS by the
time the patient reaches 70 years of age. Other causes include rheumatic fever, which results in
commissural fusion and subsequent calcification, and degenerative stenosis, in which
calcification occurs in the native valve (see Color Plate 13).

PATHOPHYSIOLOGY
The initial physiologic response to AS is left ventricular hypertrophy to preserve stroke volume
and cardiac output. Left ventricular hypertrophy and increasing resistance at the level of the
valve result in decreased cardiac output, pulmonary hypertension, and myocardial ischemia.

HISTORY
Patients often complain of angina, syncope, and dyspnea, with dyspnea being the worst
prognostic indicator.

PHYSICAL EXAMINATION
A midsystolic ejection murmur, as well as cardiomegaly and other signs of congestive heart
failure, may be present. Pulsus tardus et parvus, a delayed, diminished impulse at the carotid,
may be apparent.

DIAGNOSTIC EVALUATION
Echocardiography or cardiac catheterization reliably studies the valve. A decrease in the aortic
valve area from the normal 3 or 4 cm to <1 cm signifies severe disease.

TREATMENT
Patients who are symptomatic should undergo aortic valve replacement unless other medical
conditions make it unlikely that the patient could survive the operation. In asymptomatic
individuals< progressive cardiomegaly is an indication for operation, because surgical therapy is
superior to medical therapy.

AORTIC INSUFFICIENCY

ETIOLOGY
Aortic insufficiency can be caused by rheumatic fever, connective tissue disorders including
Marfan and Ehlers-Danlos syndromes, endocarditis, aortic dissection, and trauma.

PATHOPHYSIOLOGY
The incompetent valve causes a decrease in cardiac output, and left ventricular dilatation
occurs. The larger ventricle is subject to higher wall stress, which increases myocardial oxygen
demand.

HISTORY
Patients complain of angina or symptoms of systolic dysfunction.

PHYSICAL EXAMINATION
Typically, there is a crescendo-decrescendo diastolic murmur and a wide pulse pressure with a
water hammer quality. The point of maximal impulse may be displaced or diffuse.

DIAGNOSTIC EVALUATION
Echocardiography is a sensitive and specific means of making the diagnosis.

P
TREATMENT
Symptomatic patients should undergo replacement surgery if their medical condition allows them
to tolerate a major procedure.

MITRAL STENOSIS

ETIOLOGY
Mitral stenosis (MS) develops in 40% of patients with rheumatic heart disease. Rheumatic heart
disease occurs after pharyngitis caused by group A streptococcus. A likely autoimmune
phenomenon causes pancarditis, resulting in fibrosis of valve leaflets. Histologic findings include
Aschoff nodules. MS may also be caused by malignant carcinoid and systemic lupus
erythematosus.

PATHOPHYSIOLOGY
Fibrosis progresses over a period of two or three decades, causing fusion of the leaflets, which
take on a characteristic "fish mouth" appearance, significantly impeding blood flow through the
valve. Increased left atrial pressures lead to left atrial hypertrophy, which in turn may cause
atrial fibrillation or pulmonary hypertension. Pulmonary hypertension can further progress to
right ventricular hypertrophy and right-sided heart failure.

EPIDEMIOLOGY
MS has a female predominance of 2:1.

HISTORY
Characteristic complaints include dyspnea and fatigability. Occasionally, pulmonary hypertension
leads to hemoptysis.

PHYSICAL EXAMINATION
Cachexia or symptoms of congestive heart failure may be present, with pulmonary rales and
tachypnea. Jugular venous distention, peripheral edema, ascites, and a sternal heave of right
ventricular hypertrophy may be appreciable. Heart sounds are usually characteristic, consisting
of an opening snap followed by a low rumbling murmur. The splitting of the second heart sound is
decreased, and the pulmonary component is louder. The heart rate may demonstrate the
irregular pattern of atrial fibrillation.

DIAGNOSTIC EVALUATION
Chest x-ray may show cardiomegaly, including signs of left atrial hypertrophy. Pulmonary edema
may be present. ECG may show atrial fibrillation. Broad, notched P waves are an indication of
left atrial hypertrophy. Right axis deviation is evidence of right ventricular hypertrophy.
Echocardiography with Doppler flow measurement is extremely useful for demonstrating MS,
estimating flow, and assessing the presence of thrombi. Cardiac catheterization gives a direct
mea-surement of transvalvular pressure gradient, from which the area of the mitral annulus can
be calculated.

THERAPY
Surgical options include valvulotomy or replacement. Therapy is indicated for symptomatic
patients.

MITRAL REGURGITATION

ETIOLOGY
Approximately 40% of cases are caused by rheumatic fever; other causes include idiopathic
calcification associated with hypertension, diabetes, AS, and renal failure. Mitral valve prolapse
progresses to MR in 5% of affected individuals. Less common causes include myocardial ischemia,
trauma, endocarditis, and hypertrophic cardiomyopathy.

PATHOPHYSIOLOGY
As regurgitation becomes hemodynamically significant, the left ventricle dilates to preserve
cardiac output. A significant volume is ejected retrograde, increasing cardiac work, left atrial
volumes, and pulmonary venous pressure. This, in turn, may lead to left atrial enlargement and
fibrillation or cause pulmonary hypertension, which could result in right ventricular failure.

EPIDEMIOLOGY
MR is more common than MS and has a male predominance.

HISTORY
Patients commonly complain of dyspnea, orthopnea, and fatigue.

P
PHYSICAL EXAMINATION
Patients may appear cachectic. Frequently, there is an irregular pulse, pulmonary rales, and a
sternal heave. The pulse characteristically has a rapid upstroke, and vwaves may be present. A
holosystolic murmur that radiates to the axilla or back is common. The point of maximal impulse
is often displaced.

DIAGNOSTIC EVALUATION
Chest x-ray may show cardiomegaly and pulmonary edema. ECG commonly demonstrates left
ventricular or biventricular hypertrophy, left atrial enlargement, and P mitrale.
Echocardiography is extremely useful in establishing the diagnosis and the underlying lesion.
Cardiac catheterization is useful in establishing pulmonary pressures and cardiac output.

TREATMENT
Medical therapy consists of afterload reducing agents, such as angiotensin-converting enzyme
inhibitors, nitroglycerin, and diuretics. Surgical intervention is indicated if congestive failure
interferes with daily life, if pulmonary hypertension or left ventricular dilation worsens, or if
atrial fibrillation develops. If life-threatening MR develops from endocarditis, ischemia, or
trauma, aggressive treatment with afterload reduction, a balloon pump if necessary, and
antibiotics if indicated should be used to convert an emergency operation to an elective one.
Because of the severe hemodynamic instability that can occur, operative intervention involving
repair or replacement may be necessary in the acute setting. These emergency operations carry
>15% mortality rate.

AORTIC BALLOON PUMPS AND VENTRICULAR ASSIST DEVICES


In patients with markedly decreased cardiac output insufficient to sustain end organ perfusion,
an intra-aortic balloon pump may be useful to decrease afterload, increase coronary perfusion,
decrease myocardial oxygen demand, and increase cardiac output (Figs. 17-5 and 17-6). These
are temporary devices, generally placed via a femoral approach. Correct placement is critical.
Occlusion of the left subclavian artery during inflation must be avoided, and proximal location
above the renal arteries is also critical. Vascular complications, including arterial thrombosis and
embolism, must be
constantly checked for and may necessitate balloon removal. P
Figure 17-5 • How the intra-aortic balloon pump works: Balloon
inflation. The balloon inflates as the aortic valve closes and diastole
begins. Diastole increases perfusion to the coronary arteries.
From Nursing Procedures. 4th ed. Ambler, PA: Lippincott Williams &
Wilkins, 2004.
Figure 17-6 • How the intra-aortic balloon pump works: Balloon
deflation. The balloon deflates before ventricular ejection, when
the aortic valve opens. This permits ejection of blood from the left
ventricle against a lowered resistance. As a result, aortic end-
diastolic pressure and afterload decrease and cardiac output
increases.
From Nursing Procedures. 4th ed. Ambler, PA: Lippincott Williams &
Wilkins, 2004.
When medical therapy and intra-aortic balloon pumps are insufficient to provide adequate
cardiac output, left ventricular assist devices may be used. These are divided into pulsatile flow
pumps and nonpulsatile flow pumps, which use axial or centrifugal flow. These pumps can be
used in the short term, generally in patients with temporary heart failure. These patients are
expected to improve, and a common indication is postcardiotomy syndrome. Left ventricular
assist devices can also be useful longer term as a bridge to transplantation. Thromboembolism,
bleeding, and infection are common complications.

KEY POINTS
Coronary artery disease is the leading cause of mortality in the United
States.
Risk factors include hypertension, smoking, obesity, diabetes,
hypercholesterolemia, inactivity, and family history.
Coronary artery disease is treated surgically if all three coronary arteries or the
left main coronary artery are diseased or if patients have debilitating symptoms.
Aortic stenosis can be caused by a congenital bicuspid valve or rheumatic fever;
symptoms include angina, syncope, and dyspnea.
Aortic insufficiency can be caused by rheumatic fever, endocarditis, connective
tissue disorders, aortic dissection, and trauma; symptoms include angina and
dyspnea.
Mitral stenosis is most commonly caused by rheumatic fever; symptoms include
fatigue and dyspnea.
Mitral regurgitation is caused by rheumatic fever, idiopathic calcification, mitral
valve prolapse, myocardial ischemia, trauma, endocarditis, and hypertrophic
cardiomyopathy; symptoms include fatigue and dyspnea.
Congenital heart disease can be broadly divided into lesions causing congestive
heart failure or cyanosis.
Tetralogy of Fallot has four components: ventricular septal defect, pulmonic
stenosis, right ventricular hypertrophy, and an overriding aorta.
Authors: Karp, Seth J.; Morris, James P.G.; Zaslau, Stanley
Title: Blueprints Surgery, 5th Edition

Copyright ©2008 Lippincott Williams & Wilkins

> Table of Contents > Part IV - Cardiac, Thoracic, and Vascular > Chapter 18 - Lung

Chapter 18
Lung

In this chapter, common surgical diseases and disorders of the lung, tracheobronchial tree, and pleura are
discussed. An important key point is to understand the scope of the "silent epidemic" of smoking-related
deaths from lung cancer, as it is the leading cause of cancer-related deaths for both men and women in
North America, killing more people annually than breast, prostate, and colorectal cancer combined. The
reader should ponder why more societal and governmental efforts are not directed at combating the main
cause of this lethal disease, namely, tobacco products.

ANATOMY
The lungs are divided into three lobes with 10 segments on the right and two lobes with nine segments on
the left (Fig. 18-1). The decreased number of divisions on the left can be thought of as space taken up by
the heart. The right mainstem bronchus forms a gentler curve with the trachea than does the left mainstem
bronchus (Fig. 18-2). Therefore, aspirated foreign bodies are more likely to lodge in the right mainstem
bronchus. In aspiration pneumonia, the aspirated material is most likely to deposit in the most depen-dent
portions of the lungs. For a supine individual, these are the posterior segments of the upper lobes and the
superior segments of the lower lobes. The arterial supply to the lungs is through the pulmonary arteries as
well as the bronchial arteries, which arise from the aorta and intercostal vessels.

Figure 18-1 • Lung lobes. Anterior view.

From Anatomical Chart Company.


Figure 18-2 • Transverse section of the trachea and the esophagus and
trachea (anterior views).
From Premkumar K. The Massage Connection Anatomy and Physiology.
Baltimore, MD: Lippincott Williams & Wilkins, 2004.

BENIGN TUMORS OF THE TRACHEA AND BRONCHI

PATHOLOGY
Types of benign tumors include squamous papilloma, angioma, fibroma, leiomyoma, and chondroma.
Squamous papillomatosis is associated with human papilloma viruses 6 and 11.

EPIDEMIOLOGY
Truly benign neoplasms of the trachea and bronchi are rare.
HISTORY
Patients commonly present with recurrent pneumonias, cough, or hemoptysis.

PHYSICAL EXAMINATION
Patients may have decreased breath sounds on the affected side, with the additional signs and symptoms
owing to postobstructive pneumonia.

P.134 P
DIAGNOSTIC EVALUATION
Chest radiography may demonstrate a mass, and there is often a postobstructive pneumonia if the lesion
significantly narrows the bronchial lumen.

TREATMENT
Angiomas frequently regress, and observation is recommended. Other lesions require surgical removal to
relieve symptoms and establish a diagnosis. This may require partial lung resection or sleeve resection,
where a segment of bronchus is removed and pri-marily reanastomosed. Squamous papillomatosis has a high
recurrence rate.

TRACHEOBRONCHIAL TUMORS WITH MALIGNANT POTENTIAL


Tumors with malignant potential include bronchial carcinoids, adenoid cystic carcinoma, and
mucoepidermoid tumors. Carcinoid tumors, which are malignant in approximately 10% of patients, may
cause paraneoplastic syndromes through release of various substances, including histamine, serotonin,
vasoactive intestinal peptide, gastrin, growth hormone, insulin, glucagon, and catecholamines.

EPIDEMIOLOGY
These tumors make up fewer than 5% of all pulmonary neoplasms and have no obvious age or sex
predilection. Carcinoids make up approximately 1% of all lung tumors; adenoid cystic carcinoma,
approximately 0.5%; and mucoepidermoid, approximately 0.2%.

HISTORY
Patients most commonly complain of cough, dyspnea, hemoptysis, or recurrent pneumonia. Less frequently,
carcinoid tumors may produce carcinoid syndrome, with complaints of flushing and diarrhea, as well as
manifestations of specific hormone excess. This syndrome only occurs in approximately 3% of patients with
carcinoid tumors.

PHYSICAL EXAMINATION
The patient may have respiratory compromise or decreased breath sounds. Carcinoid tumors may cause
valvular heart disease with signs of pulmonic stenosis or tricuspid regurgitation.

DIAGNOSTIC EVALUATION
Chest radiography may reveal a lesion or postobstructive pneumonia. Bronchoscopy is useful to obtain tissue
diagnosis and define bronchial anatomy. Computed tomography (CT) of the chest is routine for preoperative
planning.

TREATMENT
These tumors should all be resected. Long-term survival for carcinoid tumors is 80%; for adenoid cystic
carcinoma and mucoepidermoid tumors, the prognosis is also favorable.

LUNG CANCER

EPIDEMIOLOGY
Lung cancer is the leading cause of cancer-related death for both men and women in North America,
responsible for >150,000 deaths each year in the United States and accounting for almost 30% of all cancer-
related deaths. More than 80% of lung cancers are smoking-related (Fig. 18-3). In the United States, more
people die each year from lung cancer than from breast, prostrate, and colorectal cancers combined. Lung
cancer kills more men than prostate cancer and more women
than breast cancer. Lung cancer incidence rates among women continue to increase. Deaths from lung P
cancer in women have increased 400% between 1960 and 1990. Smoking cessation significantly reduces an
individual's risk of developing lung cancer, although the level of risk remains greater than for nonsmokers.
Asbestos, formaldehyde, radon gas, arsenic, uranium, chromates, and nickel have been identified as
carcinogens, especially when combined with smoking.
Figure 18-3 • Death rate from lung cancer among smokers and nonsmokers.
Nonsmokers exhibit a small, linear increase in the death rate from lung
cancer from the age of 50 years onwards. By contrast, those who smoke
more than one pack per day show an exponential increase in the annual
death rate from lung cancer starting at approximately age 35 years. By age
70, heavy smokers have an approximately 20-fold greater death rate from
lung cancer than nonsmokers.
From Rubin E, Farber JL. Pathology. 3rd ed. Philadelphia, PA: Lippincott
Williams & Wilkins, 1999.

PATHOLOGY
Lung cancer is divided into small-cell lung cancer (SCLC; 20% to 25%) and non–small-cell lung cancer (NSCLC;
75% to 80%). NSCLC is further divided into squamous cell carcinoma (30%), adenocarcinoma (35%), and
large-cell carcinoma (10%). SCLC is usually centrally located and may be associated with paraneoplastic
syndromes. Approximately 5% of patients have symptoms of inappropriate secretion of antidiuretic
hormone, whereas 3% to 5% have Cushing's syndrome from adrenocorticotropin production. Squamous cell
cancer usually occurs centrally and can be associated with symptoms of hypercalcemia secondary to
production of a substance similar to parathyroid hormone. Adenocarcinoma typically occurs at the
periphery.

HISTORY
Most patients come to seek medical attention as a result of signs and symptoms indicating advanced
disease. Ninety percent of patients with lung cancer are symptomatic at the time of diagnosis. Worsening
cough with increased sputum production and hemoptysis often indicates airway obstruction by tumor. A
history of recurrent pneumonia requiring antibiotic therapy is common. Persistent chest, back, or shoulder
pain is related to nerve involvement or direct tumor invasion. Bone pain indicates distant skeletal
metastases, whereas neurologic symptoms indicate brain metastases. Systemic symptoms include fatigue,
loss of appetite, and unintentional weight loss.

PHYSICAL EXAMINATION
Chest auscultation may reveal diminished breath sounds owing to pneumonia or malignant pleural effusion.
Supraclavicular lymphadenopathy may be present. Recent onset of hoarseness indicates involvement of the
recurrent laryngeal nerve. Horner syndrome (ptosis, myosis, and anhydrosis) results from a superior sulcus
tumor causing neural invasion. Superior vena cava syndrome and Pancoast syndrome (shoulder and arm pain
on the affected side) may occur. Paralysis of the diaphragm indicates phrenic nerve involvement. Patients
with advanced disease are usually ill-appearing and exhibit significant weight loss.

DIAGNOSTIC EVALUATION
Chest x-ray is often the modality first used to diagnose a malignant pulmonary lesion. Chest CT including
the liver and adrenal glands often follows to delineate tumor size, presence of lymphadenopathy and
pleural effusion, and evidence of the likelihood of distant disease (Fig. 18-4).

Figure 18-4 • Solitary pulmonary nodule evaluated by computed


tomography (CT). A. Chest radiograph shows nodule in the left upper lobe
(arrow). B. CT shows extensive emphysema and a noncalcified nodule in the
posterior segment, left upper lobe, with a "tail" extending to the pleura.
The nodule is noncalcified and was proven to be a non–small-cell lung
cancer.
From Crapo JD, Glassroth JL, Karlinsky JB, et al. Baum's Textbook of
Pulmonary Diseases. 7th ed. Philadelphia, PA: Lippincott Williams & Wilkins,
2004.

Bone scan and brain imaging may also be obtained, if necessary. Noninvasive functional testing for P.137 P
distant disease can be conducted via positron emission tomography scan. This information allows for clinical
staging and decision making.

Invasive testing is usually required for definitive diagnosis. Diagnostic thoracentesis is used to evaluate for
malignant pleural effusion. Flexible bronchoscopy allows for tissue biopsy and bronchial washings. Trans-
thoracic CT-guided fine-needle biopsy can also provide diagnostic tissue sampling. Mediastinoscopy with
lymph node biopsy can be diagnostic while also providing information for accurate nodal staging (Fig. 18-5).

Figure 18-5 • Mediastinoscopy. The passive angle of the mediastinoscope


within the pretracheal fascia aims the tip of the scope directly at the
trachea or subcarinal space.
From Shields TW, LoCicero J III, Ponn RB, et al. General Thoracic Surgery.
6th ed. Philadelphia, PA: Lippincott Williams & Wilkins, 2004.

STAGING
The TNM system is used for staging of lung cancer (Tables 18-1 and 18-2). T1 lesions are <3 cm; T2 lesions
are >3 cm or involve the main bronchus >2 cm from the carina or involve the visceral pleura. T3 lesions
invade the chest wall, diaphragm, mediastinal pleura, or pericardium or involve the main bronchus within 2
cm of the carina. T4 lesions invade the heart, great vessels, mediastinum, trachea, esophagus, vertebral
bodies, or carina or have malignant effusions or satellite tumors.

TABLE 18-1 Current International Staging System for Non–Small-Cell Lung


Cancer Staging

Primary Tumor (T)

T1 A tumor that is ≤3 cm in greatest dimension, surrounded by lung or


visceral pleura, and with
out evidence of invasion proximal to a lobar bronchus at
bronchoscopy.

T2 A tumor >3 cm in greatest dimension, or a tumor of any size that


either invades the visceral
pleura or has associated atelectasis or obstructive pneumonitis
extending to the hilar bronchus
or at least 2 cm distal to the carina. Any associated atelectasis or
obstructive pneumonitis must
involve less than an entire lung.

T3 A tumor of any size with direct extension into the chest wall
(including superior sulcus
tumors), diaphragm, or the mediastinal pleura or pericardium
without involving the heart, great
vessels, trachea, esophagus, or vertebral body, or a tumor in the
main bronchus within 2 cm of
the carina without involving the carina.

T4 A tumor of any size with invasion of the mediastinum or involving


the heart, great vessels,
trachea, esophagus, vertebral body, or carina or presence of
malignant pleural effusion.*

Nodal Involvement (N)

N0 No metastasis to regional lymph nodes

N1 Metastasis to lymph nodes in the peribronchial or the ipsilateral


hilar region

N2 Metastasis to ipsilateral mediastinal lymph nodes and subcarinal


lymph nodes

N3 Metastasis to contralateral mediastinal lymph nodes, contralateral


hilar lymph nodes, ipsilateral
or contralateral scalene lymph nodes, or supraclavicular lymph
nodes

Distant Metastasis (M)

M0 No distant metastasis

M1 Distant metastasis

*Most pleural effusions associated with lung cancer are due to tumor.
There are, however, a few patients in whom cytopathologic examination
of pleural
fluid (on more than one specimen) is negative for tumor and in whom the
fluid is nonbloody and is not an exudate. In cases in which these
elements and
clinical judgment dictate that the effusion is not related to the tumor,
the patient should be staged T1, T2, or T3, excluding effusion as a
staging element.

TABLE 18-2 Lung Cancer Staging

Stage 1A T1, N0, M0

Stage 1B T2, N0, M0

Stage IIA T1, N1, M0

Stage IIB T2, N1, M0

T3, N0, M0

Stage IIIA T3, N1, M0

T1, N2, M0

T2, N2, M0

T3, N2, M0

Stage IIIB T4, N0, M0

T4, N1, M0
T4, N2, M0

T1, N3, M0

T2, N3, M0

T3, N3, M0

T4, N3, M0

Stage IV Any T, Any N, M1

N1 lesions have positive nodes in the ipsilateral peribronchial or hilar region. N2 lesions have positive nodes
in the ipsilateral mediastinal or subcarinal region. N3 lesions have metastases either to contralateral nodes
or ipsilateral scalene or supraclavicular regions.

Stage IA lesions are T1N0M0 lesions. Stage IB lesions are T2N0M0 lesions. Stage IIA lesions are T1N1M0
lesions. Stage IIB lesions are T2N1M0 or T3N0M0 lesions. Stage IIIA lesions are T3N1M0, T1N2M0, T2N2M0, or
T3N2M0 lesions. Stage IIIB lesions are tumors without distant metastases, including primary tumors to T4
and N3 regional lymph node metastases. Stage IV patients have evidence of distant metastases.

TREATMENT
Since the first successful pneumonectomy for lung cancer in 1933, surgical resection has been considered
the standard therapy for patients with potentially curable disease. For patients with NSCLC, those with
clinical stage I and II disease are referred for an attempt at curative surgical resection. For patients with
stage III disease, some may be considered for resection as part of a multimodality therapeutic program
usually involving neoadjuvant chemoradiotherapy. Patients with N3 disease and clinical stage IV are not
typically candidates for surgical resection.

Standard surgical treatment of NSCLC consists of complete resection of the disease. In addition to
parenchymal resection, the operation should also include either sampling or removal of all intrathoracic
ipsilateral lymph node stations into which the tumor could potentially drain. For tumors confined to a single
lung lobe, the most common surgical procedure performed is lobectomy. For tumors invading an adjacent
lobe, a bilobectomy is required. If complete resection requires removal of all lobes on the effected side,
then pneumonectomy is performed. For a patient with poor pulmonary function and limited pulmonary
reserve, then segmentectomy or wedge resection may be performed. However, the decision to perform a
smaller resection provides poorer overall survival because studies show higher locoregional recurrence rates
after segmentectomy versus the more oncologically complete lobectomy (23% versus 5%).

Results of clinical trials now support the use of adjuvant platinum-based chemotherapy in patients with P
completely resected lung cancer (stage I, II, and IIIA). The use of adjuvant chemotherapy improves 5-year
survival rates by approximately 5% to 15% when compared with surgical therapy alone. Standard
chemotherapy regimens for NSCLC include a platinum agent (cisplatin or carboplatin) combined with a
nonplatinum agent (etoposide, irinotecan, paclitaxel, gemcitabine, and others). Radiotherapy is often
added if mediastinal lymph nodes are involved (stage III).

As opposed to NSCLC, SCLC is usually widely disseminated at the time of diagnosis, so surgery is rarely
indicated. Only very early-stage small-cell tumors (T1 to T2, N0) are considered for potential resection. The
standard treatment for this aggressive disease is combined chemotherapy and radiotherapy. Chemotherapy
usually consists of combination therapy (including cyclophosphamide, doxorubicin, vincristine, cisplatin,
carboplatin, or etoposide), with triplet combinations often used. Radiation therapy, usually in combination
with chemotherapy, is an important modality for treating SCLC. Various radiation treatment plans exist,
with differing doses, timing, and fractionation schedules. Elective whole-brain irradiation is used in many
centers to minimize the risk of developing brain metastases. The principle of brain irradiation is to destroy
any hidden tumor cells that may potentially lurk in the brain, because asymptomatic cerebral metastases
are theoretically protected from systemic chemotherapy by the blood-brain barrier. By undergoing so-called
prophylactic cranial irradiation, it is argued that survival rates are increased and symptomatic metastases
are prevented, because approximately 25% to 50% of untreated patients will develop brain metastases.

PROGNOSIS
Lung cancer remains a lethal disease, despite recent advances. The 5-year survival rate for all patients is
slightly >10%. For early-stage asymptomatic NSCLC (stage IA), usually detected incidentally on chest
radiograph, the 5-year survival rate is approximately 80%. This figure rapidly decreases to 55% for stage IB
and to 30% for stage II disease. Given the overall poor survival rates, there has been a resurgence of
interest in screening tests using spiral CT scans for early detection of lung cancer. This is an active area of
ongoing research. Long-term survival in SCLC is rare.

MESOTHELIOMA

PATHOLOGY
Mesothelioma is a malignant lesion derived most commonly from the visceral pleura.

EPIDEMIOLOGY
Mesothelioma is rare. Asbestos is the major risk factor. Cigarette smoking markedly increases the incidence
of mesothelioma in patients exposed to asbestos.

HISTORY
Chest pain from local chest wall extension, dyspnea secondary to pleural effusion, and lung entrapment,
weight loss, and unexplained night sweats may occur.

PHYSICAL EXAMINATION
The patient may have decreased breath sounds on the side of the tumor as a result of pleural effusion and
lung entrapment.

DIAGNOSTIC EVALUATION
Chest radiography often demonstrates a pleural effusion. Thoracocentesis typically yields bloody fluid, and
cytology is often negative for malignant disease. CT scan shows a chronic effusion with irregular thickened
visceral pleura. Patients with a suggestive history and a recurrent pleural effusion with no clear cause
should undergo thoracoscopy and pleural biopsy, even in the presence of negative fluid cytology.

TREATMENT
Overall prognosis is poor, with few survivors living beyond 2 years. Early-stage lesions may be resectable but
require induction chemotherapy followed by extrapleural pneumonectomy, a significantly morbid
procedure. Chemotherapy and radiotherapy are used for nonoperative patients.

P
PNEUMOTHORAX
The lung is covered by visceral pleura, and the inner chest wall is covered by parietal pleura. These two
continuous surfaces form a potential space. Simple pneumothorax occurs when air enters this space and the
lung falls away from the chest wall resulting in a "dropped" lung (Fig. 18-6). Open pneumothorax occurs
when a defect in the chest wall allows continuous entry of air from the outside (Fig. 18-7). Tension
pneumothorax occurs when air enters the potential space but cannot escape. A ball valve–like effect
allows pressure to increase within the hemithorax, thereby forcibly collapsing the ipsilateral lung and P
compressing mediastinal structures.

Figure 18-6 • Pneumothorax. Injury to lung tissue allows air to leak into the
pleural space and put pressure on the lung.
From Cohen BJ. Medical Terminology. 4th ed. Baltimore, MD: Lippincott
Williams & Wilkins, 2003.
Figure 18-7 • Open pneumothorax with element of tension causing midline
shift of mediastinal structures.
From Willis MC. Medical Terminology: A Programmed Learning Approach to
the Language of Health Care. Baltimore, MD: Lippincott Williams & Wilkins,
2002.

ETIOLOGY
Spontaneous pneumothorax is usual seen in young thin males with rupture of congenital apical blebs or in
older patients with bullous emphysema from smoking. It can also occur in patients on mechanical
ventilation, especially if high inspiratory pressures are required. Infection, specifically tuberculosis or
Pneumocystis carinii, can cause pneumothorax, as can lung tumors on rare occasion. Placement of central
venous catheters results in pneumothorax in 1% of cases. The use of ultrasound guidance for central line
placement in the internal jugular position reduces the chance of pneumothorax to nearly zero.
Thoracocentesis, needle biopsy, and operative trauma are other iatrogenic causes. Open pneumothorax is
caused by penetrating trauma, whereas tension pneumothorax can occur by any of the above mechanisms.

HISTORY
Patients can be entirely asymptomatic, or they may complain of dyspnea or pleuritic chest pain.

P
PHYSICAL EXAMINATION
Simple pneumothorax may result in decreased breath sounds and hyperresonance on the affected side.
Tension pneumothorax may cause tachycardia, hypotension, and hypoxia, and the trachea may be displaced
away from the affected side.

DIAGNOSTIC EVALUATION
Chest radiography reveals absence of lung markings in the affected area, usually in the apex, in an upright
film. A visible line corresponding to the visceral pleural surface of the lung is evident. Tracheal deviation or
mediastinal shift suggests tension pneumothorax.

TREATMENT
Simple pneumothoraces of less than 20% can be observed if no increase in size is demonstrated on serial
chest x-rays and the patient is hemodynamically stable. Indications for chest tube placement include size
>20% or those that demonstrate increase in size during serial radiographic observation. Open pneumothorax
requires repair of the defect and tube thoracostomy. Symptomatic tension pneumothorax is a surgical
emergency and requires immediate needle thoracostomy, usually in the midclavicular line in the second
intercostal space on the affected side. This will decompress the chest and allow normalization of
hemodynamics and oxygenation. Tube thoracostomy should then follow (Fig. 18-8).
Figure 18-8 • Tube thoracostomy. A. The site for insertion of the tube at the
anterior axillary line. The skin incision is usually made over the intercostal
space one below the space to be pierced. B. The various layers of tissue
penetrated by the scalpel and later the tube as they pass through the chest
wall to enter the pleural cavity (space). The incision through the intercostal
space is kept close to the upper border of the rib to avoid injuring the
intercostal vessels and nerve. C. The tube advancing superiorly and
posteriorly in the pleural space.
From Snell RS. Clinical Anatomy. 7th ed. Philadelphia, PA: Lippincott
Williams & Wilkins, 2003.

EMPYEMA
Empyema is an infection within the pleural space.

ETIOLOGY
Empyema is most commonly caused by pneumonia, lung abscess, recent thoracic surgery, or esophageal
perforation. The most common organisms are those that cause primary lung infection, including
Staphylococcus, Streptococcus, Pseudomonas, Klebsiella, Escherichia coli, Proteus, and Bacteroides.

HISTORY
The patient may have a history of previous pneumonia, thoracic surgery, or esophageal instrumentation.
Fatigue, lethargy, and shaking chills may occur.

PHYSICAL EXAMINATION
The patient is often systemically ill. Fever and decreased breath sounds at the affected lung base are
common.

DIAGNOSTIC EVALUATION
The serum white blood cell count is elevated. Chest radiography reveals a pleural effusion. Chest CT shows
a defined fluid collection (Fig. 18-9). Aspiration of the pleural fluid by thoracentesis shows an exudative
effusion, characterized by a high white blood cell count with predominantly polymorphonuclear cells, a low
pH, a low glucose, and high lactate dehydrogenase. Bacteria on Gram stain and culture may be present.
Figure 18-9 • Empyema. Air fluid level (arrowhead) in the pleural space in
this patient with an empyema and bronchopleural fistula. An elliptical fluid
collection is present, and the inner margin of the collection is smooth and
thin. The inner margin also appears dense (straight arrows), indicating
enhancement, with a split-pleura sign.
From Shields TW, LoCicero J III, Ponn RB, et al. General Thoracic Surgery.
6th ed. Philadelphia, PA: Lippincott Williams & Wilkins, 2004.

TREATMENT
On rare occasions, antibiotics and needle aspiration alone are successful, but usually tube thoracostomy is
required. For patients who have inadequate re-expansion owing to "trapped" lung after chest tube insertion,
surgical decortication may be required to remove the restricting pleural peel and facilitate lung re-
expansion.
P
KEY POINTS
Benign lesions of the trachea and bronchi are rare. Sleeve resection or partial lung
resection may be required for treatment.
Carcinoid tumors can release a variety of substances, causing paraneoplastic
syndromes.
Lung cancer is the leading cause of cancer-related death in North America. More than
80% of lung cancers are smoking-related. Most patients are symptomatic at the time of
presentation and are not surgical candidates. Accurate preoperative staging is required to
determine appropriate treatment.
Early-stage lung cancer is treated with surgery and chemotherapy.
Asbestos is the major risk factor for developing mesothelioma. Cigarette smoking greatly
increases the risk of developing mesothelioma. Prognosis is poor.
Tension pneumothorax is a surgical emergency and requires rapid decompression.
Empyema is treated with tube thoracostomy and antibiotics. Surgical decortication may be
required.
Authors: Karp, Seth J.; Morris, James P.G.; Zaslau, Stanley
Title: Blueprints Surgery, 5th Edition

Copyright ©2008 Lippincott Williams & Wilkins

> Table of Contents > Part IV - Cardiac, Thoracic, and Vascular > Chapter 19 - Esophagus

Chapter 19
Esophagus

ANATOMY AND PHYSIOLOGY


The esophagus is a muscular tube that functions pri-marily as a conduit for transporting ingested
solids and liquids from the mouth into the intra-abdominal gastrointestinal tract. It extends from
the pharynx to the stomach, traversing the posterior mediastinum, and is bounded posteriorly by
the vertebral column and thoracic duct, anteriorly by the trachea, laterally by the pleura, and
on the left by the aorta (Figs. 19-1 and 19-2). It begins at the cricoid cartilage in the neck and
courses downward to the left, then to the right, and back to the left to pierce the diaphragm
and join the cardia of the stomach. The intra-abdominal portion of the esophagus measures <3
cm. The upper one third of the esophageal musculature is skeletal, whereas the lower two thirds
is smooth muscle. Unlike the intra-abdominally located stomach, small intestine, and colon,
there is no true serosal layer covering the esophagus.
Figure 19-1 • Clinical divisions of the esophagus.

From Lawrence PF. Essentials of General Surgery. 4th ed.


Philadelphia, PA: Lippincott Williams & Wilkins, 2006.
Figure 19-2 • Esophagus and associated structures. A. Schematic
drawing of a lateral view of the head, neck, and trunk showing the
esophagus and the structures associated with it. The esophagus
descends posterior to the trachea and leaves the thorax through the
esophageal hiatus in the diaphragm. B. Transverse section of the
esophagus showing the muscular layers and microscopic structure of
its wall. C. Coronal section of the esophagus, diaphragm, and
stomach (superior part). Observe the phrenicoesophageal ligament
that connects the esophagus flexibly to the diaphragm; it limits
upward movement of the esophagus while permitting some
movement during swallowing and respiration.
From Moore KL, Dalley AF II. Clinically Oriented Anatomy. 4th ed.
Baltimore, MD: Lippincott Williams & Wilkins, 1999.

The vagus nerves form a plexus around the esophagus, which condenses distally to form two
trunks on the lateral esophagus. These trunks, in turn, rotate so that the left trunk moves
anteriorly, whereas the right trunk moves posteriorly.

The esophageal mucosa is lined by squamous epithelium that becomes columnar near the gastro-
esophageal junction. The next layer encountered moving radially outward is the submucosa,
which contains the submucous (Meissner) plexus. Next are two muscular layers, the inner circular
muscle layer and the outer longitudinal muscle layer. Sandwiched in between these two muscular
layers is the myenteric (Auerbach) plexus.

The arterial supply of the esophagus involves vessels from the neck, chest, and abdomen. The
superior and inferior thyroid arteries supply the upper esophagus, whereas the intercostals, left
gastric, and phrenic arteries supply the lower esophagus. Venous drainage of the upper
esophagus is into the inferior thyroid and vertebral veins, whereas the mid and lower esophagus
drains into the azygous, hemiazygous, and left gastric veins. Submucosal veins can become
engorged in patients with portal hypertension, causing varices and potentially life-threatening
bleeding. Lymphatics drain into cervical, mediastinal, celiac, and gastric nodes. Innervation is
from the vagus, cervical sympathetic ganglion, splanchnic ganglion, and celiac ganglion. These
nerves are responsible for esophageal motility.

Peristasis conveys food into the stomach. Gastric reflux is prevented by increased tone in the
lower portion of the esophagus; there is no true sphincter. Air ingestion is prevented by resting
tone in the upper esophagus.

ESOPHAGEAL NEOPLASMS

PATHOLOGY
Esophageal neoplasms are almost always malignant. Benign tumors account for fewer than 1% of
cases and are usually leiomyomas or congenital cysts. Symptomatic benign esophageal neoplasms
are treated with local resection or enucleation. Most malignant esophageal neoplasms seen
worldwide are of squamous cell histologic type, whereas adenocarcinoma histology predominates
in the United States and other industrialized nations. Most cases of adenocarcinoma arise in the
distal third or gastroesophageal junction (80%). Metastases are usually to liver, lungs, and bones,
with at least 35% of patients with distant metastases at the
time of diagnosis. Early-stage cancers are usually diagnosed only incidentally or else discovered P
on screening studies. Prognosis is poor, with an overall 5-year survival rate of only 5%.
PATHOGENESIS
Mucosal insult seems to be a common pathway toward the genesis of esophageal cancer. Chronic
ingestion of extremely hot liquids, esophageal burns from acid or base ingestions, radiation-
induced esophagitis, and reflux esophagitis are all implicated in causing esophageal cancer.
Alcohol, smoking, nitrosamines, and malnutrition/vitamin deficiency also play a role in cancer
development. Barrett esophagus, which occurs when the normal squamous epithelium becomes
columnar in response to injury from acid reflux, is considered a premalignant lesion.
Approximately 10% of patients with Barrett esophagus will develop adenocarcinoma.

In patients with achalasia, approximately 6% will develop squamous cell carcinoma of the
esophagus.

EPIDEMIOLOGY
The incidence of esophageal cancer varies according to the presence of the etiologic factors
described previously. For example, in places with high soil nitrosamine content, the prevalence
of esophageal cancer is almost 1% of adults. In the United States, the incidence of esophageal
cancer is four in 100,000 White men and 12 in 100,000 Black men. It is most commonly a disease
of men between 50 and 70 years of age.

HISTORY
At the time of diagnosis, most patients have advanced disease and are not surgical candidates
for curative resection. The lack of a serosal lining and the rich submucosal lymphatic network of
the esophagus allows early extension of tumor into adjacent mediastinal structures as well as
early local lymph node spread. Approximately 75% of patients have mediastinal or extrathoracic
lymph node metastases at the time of diagnosis.

The classic presentation of distal esophageal adenocarcinoma is an older man with a history of
gastro-esophageal reflux disease complaining of progressive dysphagia to solids initially, and then
to liquids. Typically, patients feel well and have no other symptoms. Some report noting mild
weight loss, more often related to diminished caloric intake resulting from obstructive symptoms
than to cachexia from metastatic disease. Chest pain and odynophagia (pain with swallowing)
may also occur. Invasion of a recurrent laryngeal nerve may cause hoarseness from vocal cord
paralysis.

Patients with esophageal squamous cell carcinoma often have a history of heavy alcohol and
tobacco use and present with more pronounced symptoms owing to more advanced disease.

PHYSICAL EXAMINATION
Signs are nonspecific, and patients appear well unless significant metastatic disease is present.
Supraclavicular lymphadenopathy at presentation is rare.
DIAGNOSTIC EVALUATION
Barium esophagogram detects malignant lesions in 96% of patients. Findings range from small
mucosal defects to "apple core" lesions to complete obstruction (Fig. 19-3). This is usually the
initial study for the evaluation of new onset dysphagia. Definitive diagnosis of cancer requires
confirmation by flexible endoscopy with tissue biopsy. If Barrett esophagitis alone is noted, the
extent of disease can be determined and biopsies performed to look for dysplasia or carcinoma
in situ.
Figure 19-3 • A. Contrast radiograph showing the typical "apple
core" lesion of carcinoma of the middle one third of esophagus. B.
Ragged edge seen in carcinoma of the distal esophagus.

From Lawrence PF. Essentials of General Surgery. 4th ed.


Philadelphia, PA: Lippincott Williams & Wilkins, 2006.

To determine the stage of the primary tumor (T) and regional lymph node status (N), endoscopic
ultrasound
(EUS) is used. Ultrasound is best for evaluating local staging: the intramural extent of the tumor P
as well as assessing for lymph node enlargement or abnormality. Ultrasound depicts the normal
five layers of the esophagus and can accurately determine the T status in 85% to 90% of patients.
Fine-needle aspiration biopsy of abnormal-appearing lymph nodes under ultrasound guidance
may be performed.

Evaluation of regional and distant disease is best done by computed tomography (CT) scan and
positron emission tomography scan. CT scanning should be performed first after initial diagnosis,
because lung and liver metastases are often detected. If CT is negative for metastases, then EUS
is performed. Combined preoperative CT and EUS have an accuracy rate of 86% for TNM staging.

P
STAGING
The TNM system is used for staging esophageal cancer. Clinical stage (cTNM) is determined by
evaluation of all information derived from physical examination, imaging studies, endoscopy,
biopsy, and occasionally laparoscopy or thoracoscopy. Once the clinical stage is determined,
rational treatment plans can be proposed to the patient (Table 19-1 and Fig. 19-4).

TABLE 19-1 AJCC TNM Classification of the Esophagus

Primary Tumor (T)

Tis Carcinoma in situ

T1 Tumor invades lamina propria


or submucosa
T2 Tumor invades muscularis
propria

T3 Tumor invades adventitia

T4 Tumor invades adjacent


structures

Regional Lymph Nodes (N)

T0 No regional lymph node


metastasis

T1 Regional lymph node


metastasis

Distant Metastasis (M)

M0 No distant metastasis

M1 Distant metastasis

Stage Grouping

Stage Tis N0 M0
0

I T1 N0 M0

IIA T2 N0 M0, T3 N0 M0
IIB T1 N1 M0, T2 N1 M0

III T3 N1 M0, T4 Any N M0

IV Any T Any N M1

Used with permission of the American


Joint Committee on Cancer (AJCC),
Chicago, IL. Original source: AJCC
Cancer Staging Manual, 6th ed. New
York, NY: Springer-Verlag, 2002.
Figure 19-4 • Visual representation of TNM staging of esophageal
cancer.
Reprinted with permission of The Cleveland Clinic Foundation.

TREATMENT
Because there is no serosal layer covering the esophagus, disease is often locally advanced or
metastatic on presentation, leading to poor overall survival statistics. Surgical cure rates are 20%
to 30% at best. The goals of surgical treatment are removal of the tumor and regional lymph
nodes followed by re-establishment of gastrointestinal tract continuity. For early-stage disease,
esophagectomy provides the possibility of cure. Preoperatively, induction chemoradiotherapy
may be administered, especially to downstage patients with resectable bulky tumors. Some
evidence has shown that induction chemotherapy and radiotherapy preoperatively, followed by
surgery, can improve long-term survival, but this remains an area of controversy. The extent of
lymphadenectomy is also an area of ongoing controversy.

Various operations with varying degrees of invasiveness have been developed for treating
esophageal cancer. In deciding which procedure to perform, the surgeon must take into
consideration the location of the tumor, the proposed esophageal substitute, and the physical
condition and body habitus of the patient. Generally, the most common surgical approaches are
transthoracic (thoracotomy) and transhiatal without thoracotomy. Incisions may be required in
the neck, chest, and abdomen.

The most common transthoracic esophagectomy is one called the Ivor Lewis esophagectomy, P
named after the distinguished Welsh surgeon who developed the approach. In brief, it involves
upper midline laparot-omy with mobilization of the stomach and is followed by right
thoracotomy with esophageal resection and anastomosis. It is useful for patients with tumors of
the middle and lower esophagus. For tumors of the upper esophagus, many surgeons use a
modified approach, called the McKeown modification, which involves additional intrathoracic
esophageal mobilization and anastomosis in the right neck through a separate cervical incision
(three-hole technique). The tubularized stomach is used as the esophageal substitute in most
cases. Other possible substitutes are the colon and small bowel (Fig. 19-5).
Figure 19-5 • Three-hole Mckeown modification (using additional
cervical incision) of the standard Ivor Lewis abdominal and right
thoracic approach.

Esophagectomy without thoracotomy is termed the transhiatal approach. This technique uses
laparot-omy for gastric and esophageal mobilization, followed by left cervical incision for the
anastomosis. The major benefit of this approach is avoiding the complications that arise from
thoracotomy.

Although open surgical approaches remain the standard for esophagectomy, recent advances in
minimally invasive surgery have allowed esophagectomy to be performed using laparoscopic and
thoracoscopic techniques. A cervical incision is also used (Fig. 19-6). This is a complex and
challenging procedure that has a steep learning curve. Early results of laparoscopic
esophagectomy seem to compare favorably with open surgery, but long-term data are presently
not available.
Figure 19-6 • Completed cervical anastomosis after laparoscopic
transhiatal esophagectomy without thoracotomy.

From Shields TW, LoCicero J III, Ponn RB, et al. General Thoracic
Surgery. 6th ed. Philadelphia, PA: Lippincott Williams & Wilkins,
2004.

P
ACHALASIA

PATHOPHYSIOLOGY
The term "achalasia" is derived from the Greek, meaning "failure to relax." It is defined as
absence of peristalsis in the smooth muscle of the distal esophagus and failure of the lower
esophageal sphincter (LES) to relax with swallowing. The exact cause of primary achalasia is not
well understood, but studies have shown abnormalities in the myenteric plexus (Auerbach).
Histologic evidence shows inflammation, fibrosis, and loss of ganglion cells. Possible causes are
infectious (neurotropic virus) or degenerative.

In South and Central America, the most common cause of secondary achalasia is Chagas disease
(American trypanosomiasis).

EPIDEMIOLOGY
Achalasia is the most common disorder of esophageal motility but is actually rare, with an
incidence of 0.5 per 100,000. Men and women are equally affected. Patients usually present
between 20 and 50 years of age.

HISTORY
Patients complain of dysphagia to solids and liquids. Because ingested material is unable to pass
into the stomach, a column of food or liquid rises in the esophagus. When there is a change in
position, liquid spills into the mouth or into the lungs, and patients complain of regurgitation or
have a history of recurrent aspiration pneumonia. Because the regurgitant does not include
gastric contents, it is usually not sour tasting.

P
DIAGNOSTIC EVALUATION
Chest x-ray may show an air-fluid level in the esophagus. A prominent gastric bubble may be
noted because of the highly competent LES. Changes from aspiration pneumonia may be present.
CT scan shows a dilated, thin-walled esophagus containing food material.

Contrast esophagogram shows a dilated, smooth-walled esophagus tapering to a "bird's beak" at


the LES. Dynamic video imaging or fluoroscopy reveals reduced or absent peristalsis of the distal
esophagus.

Esophagoscopy should also be performed to evaluate for strictures and rule out cancer. Malignant
tumors of the gastroesophageal junction can mimic the findings of achalasia, known as
pseudoachalasia.

Motility and pressure studies confirm the diagnosis. The two key findings on manometry that are
required to make the diagnosis are incomplete relaxation of the LES and aperistalsis of the body
of the esophagus.

TREATMENT
There is no cure for achalasia, given the still unelucidated underlying process that causes
esophageal neural pathology. Treatment is focused on relief of symptoms (dysphagia) caused by
achalasia.

Medical treatment consists mainly of pneumatic balloon dilatation (Fig. 19-7) of the lower
esophageal sphincter to reduce outflow obstruction and facilitate gravity drainage. This
technique is minimally invasive, has good initial results, and can be repeated. However, it carries
the risk of esophageal perforation and over the long term has inferior results when compared
with surgical therapy. Another medical treatment is LES relaxation by paralysis achieved by
endoscopic injections of botulinum toxin (Botox). This may be an option for frail, high-risk
patients, as all patients experience symptomatic relapse.
Figure 19-7 • Pneumatic balloon dilatation of lower esophageal
sphincter for achalasia. (A-C) The dilator is passed, guided by a
previously inserted guidewire. (D) When the balloon is in proper
position, it is distended by pressure sufficient to dilate the
narrowed area of the esophagus.

From Smeltzer SC, Bare BG. Brunner & Suddarth's Textbook of


Medical-Surgical Nursing. 9th ed. Philadelphia, PA: Lippincott
Williams & Wilkins, 2000.

Surgical therapy by esophageal myotomy (Heller myotomy) is the definitive treatment for
achalasia.
The goal of myotomy is to reduce LES pressure to facilitate gravity drainage while also P
performing a partial fundoplication to ensure a degree of gastro-esophageal reflux control. The
procedure is performed using the laparoscopic approach, with low morbidity. Longitudinal
separation of the esophageal musculature is carried from the distal esophagus onto the proximal
stomach, and partial fundoplication (Dor or Toupet wrap) completes the procedure. Myotomy can
also be performed through the chest by thoracoscopy or thoracotomy, but with greater
morbidity. Surgical therapy provides superior long-term results over medical interventions.

ESOPHAGEAL PERFORATION

ETIOLOGY
Esophageal perforation occurs most commonly after instrumentation (iatrogenic) but also from
ingested foreign bodies or penetrating trauma. Spontaneous esophageal rupture occurring after
violent emesis is known as Boerhaave syndrome. The latter is named after Hermann Boerhaave,
who first reported distal esophageal rupture in 1724 during the autopsy of Baron von Wassenaer,
the infamous overindulging Grand Admiral of the Dutch Fleet who showed very poor judgment by
engaging in self-induced, chemically assisted vomiting after feasting and drinking.

HISTORY
Recent instrumentation of the upper airway or esophagus should raise the possibility of
esophageal injury. Boerhaave syndrome should be suspected in cases involving recent emesis.
Epigastric abdominal pain and shoulder pain are frequent complaints.

PHYSICAL EXAMINATION
The degree of presenting symptoms is usually proportional to the time from when perforation
occurred. Subcutaneous emphysema in the cervical region is often found, as well as abdominal
tenderness or distention. If a major delay in diagnosis has occurred (i.e., because of lack of
history in an unconscious patient), then fever, tachycardia, and hypotension resulting from sepsis
is common. The presence of a hydropneumo-thorax can result in diminished breath sounds over
the involved hemithorax.

DIAGNOSTIC EVALUATION
Chest x-ray can demonstrate pleural effusion, hydro-pneumothorax, and mediastinal
emphysema. An esophageal contrast study with either water-soluble contrast medium or water-
suspended barium sulfate can confirm the location of perforation. If diagnosis is still uncertain,
then intraluminal examination by flexible endoscopy can also be used. Thoracentesis can reveal
empyema.

TREATMENT
Immediate exploratory thoracotomy and repair of the perforation is indicated in almost all cases.
Iatrogenic injuries can usually be closed primarily if surgical intervention is immediate.
Perforations with devitalized surrounding tissue or with significant soilage and infection should
be repaired with the use of a pedicled flap (i.e., intercostal muscle flap). Pleural space drainage
with chest tubes continues postoperatively. Although small cervical lacerations may occasionally
be managed with antibiotics and close observation alone, mortality from perforation of the
thoracic esophagus is >50% if not treated within 24 hours.

KEY POINTS
The esophagus is a muscular tube with a rich submucosal lymphatic
network that lacks an outer serosal layer. Therefore, esophageal cancer is
often advanced at the time of diagnosis.
Most esophageal tumors are malignant. Survival after surgery is 20% to 30%.
Overall survival remains low at 5%.
Risk factors for esophageal cancer include Barrett esophagus from
gastroesophageal reflux disease, esophageal burns, and nitrosamine ingestion.
Surgery is the only chance for attaining cure. Various surgical approaches are
used, including transthoracic, transhiatal, and minimally invasive.
Achalasia is the most common disorder of esophageal motility. Motility studies
confirm the diagnosis. Esophagoscopy usually differentiates achalasia from
cancer.
Medical and surgical therapies are used for treating achalasia. Medical treatment
usually entails balloon dilation, whereas surgical esophageal myotomy provides
definitive treatment and symptom relief.
Esophageal perforation is frequently fatal if not diagnosed and treated early.
Authors: Karp, Seth J.; Morris, James P.G.; Zaslau, Stanley
Title: Blueprints Surgery, 5th Edition

Copyright ©2008 Lippincott Williams & Wilkins

> Table of Contents > Part V - Special Topics > Chapter 20 - Neurosurgery

Chapter 20
Neurosurgery

BRAIN TUMORS
Because the brain is encased in a nonexpandable bony skull, both benign and malignant brain
tumors can cause death if not appropriately diagnosed and treated. Brain tumors cause elevated
intracranial pressure (ICP) by occupying space, producing cerebral edema, interfering with the
normal flow of cerebrospinal fluid, or impairing venous drainage (Fig. 20-1). Patients may
present with progressive neurologic deficits owing to increasing ICP, tumor invasion, or brain
compression. Alternatively, they can present with headache or seizures.
Figure 20-1 • Pathways for the circulation of cerebrospinal fluid.

PATHOLOGY
Intracranial tumors can be classified as either intra-cerebral or extracerebral (Table 20-1).
Intracerebral tumors include glial cell tumors (astrocytomas, oligodendrogliomas, ependymomas,
primitive neuroectodermal tumors), metastatic tumors (lung, breast, skin [melanoma], kidney,
colon), pineal gland tumors, and papillomas of the choroid plexus. Extracerebral tumors arise
from extracerebral structures and include meningiomas, acoustic neuromas, pituitary adenomas,
and craniopharyngiomas.

TABLE 20-1 Intracranial Tumors

Intracerebral

Glial cell tumors—astrocytomas,


anaplastic astrocy
tomas, glioblastoma multiforme,
oligodendroglioma,
ependymoma, primitive
neuroectodermal tumors

Metastatic tumors—lung, breast,


melanoma, kidney,
colon

Pineal gland tumors

Papillomas of the choroid plexus


Extracerebral

Meningiomas

Neuromas, especially acoustic neuromas

Pituitary tumors

Craniopharyngiomas

Glial cell tumors and metastatic tumors are the most common central nervous system (CNS)
tumors seen in adults. Children have a higher proportion of posterior fossa tumors.

GLIAL CELL TUMORS


Glial cells account for approximately 50% of CNS tumors in adults. Different glial cell types
(astrocytes, oligodendrocytes, ependymal cells, and neuroglial precursors) give rise to various
histologic types of tumors. Although the term "glioma" can be used to describe the above glial
tumor types, its common use refers only to astrocytic tumors.

Astrocytic tumors are graded I to IV based on histologic evidence of malignancy; grade I and II
tumors are slow-growing malignancies. In children, astrocytomas located in the posterior fossa
(cerebellum) usually have cystic morphologies (pilocystic astrocytoma). Grade III tumors are the
more aggressive anaplastic astrocytomas. The most common as well as the most malignant
astrocytoma is the grade IV glioblastoma multiforme. Glioblastoma multiforme tumors often
track through the white matter, crossing the midline via the corpus callosum, resulting in the so-
called butterfly glioma on computed tomography (CT). Median survival is 1 year.

Oligodendrogliomas are slow-growing calcified tumors, often seen in the frontal lobes. They are
most common in adults and are often associated with seizures.

Ependymomas arise from cells that line the ventricular walls and central canal. Clinical signs and
symptoms of elevated ICP are the main features of presentation. Ependymomas are mostly seen
in children and usually arise in the fourth ventricle.

Classified by location, infratentorial posterior fossa tumors make up most of the lesions seen in
childhood. These are most commonly cystic cerebellar astrocytomas, ependymomas, and
medulloblastomas. Highly malignant medulloblastomas typically occur in the vermis in children
and in the cerebellar hemispheres in young adults.
METASTATIC TUMORS
Approximately 30% of patients with systemic cancer have cerebral metastases, which usually
originate in the lung, breast, skin (melanoma), kidney, and colon. Most lesions are supratentorial
and located at the cortical
white matter junction. Resection depends on the nature of the underlying malignancy, P
symptoms, and prognosis. Single, approachable lesions should be considered for surgical
resection in the appropriate setting. Radiation usually follows surgery. Stereotactic radiosurgery
is an option in patients with multiple lesions.

MENINGIOMAS
Slow-growing, meningiomas arise from the meninges lining the brain and spinal cord. Complete
tumor removal is curative, and residual disease can be observed or treated with radiosurgery.

History
Patients usually present with neurologic signs and symptoms attributable to cerebral compression
from the expanding tumor mass. Seizures are a common presentation. Headache, nausea,
vomiting, and mental status changes are the most common generalized symptoms of elevated
ICP. Classically, patients complain of diffuse headache that is worse in the morning after a night
of recumbency.

Physical Examination
Bilateral papilledema may occur, especially in the later stages of disease. Personality changes
may be
noted early on and may progress to stupor and coma as ICP increases and brain herniation occurs P
(Fig. 20-2). Speech deficits and confusion are common with dominant hemisphere lesions. Eye
deviation can be a sign of frontal lobe involvement. Ataxia is common with cerebellar tumors.
Motor or sensory deficits indicate involvement around the central sulcus or deep structures,
especially if combined with mental status changes.
Figure 20-2 • Examples of brain herniation: (1) cingulate gyrus
herniation across the falx; (2) temporal uncus herniation across the
tentorium; (3) cerebellar tonsil herniation through the foramen
magnum; (4) herniation of brain tissue through craniotomy defect.

Differential Diagnosis
The differential diagnosis for a patient presenting with central neurologic deficits and symptoms
includes cerebrovascular accident, neurodegenerative diseases, abscess, vascular malformations,
meningitis, encephalitis, communicating hydrocephalus, and toxic state.
Diagnostic Evaluation
CT and magnetic resonance imaging (MRI) assist in making the diagnosis and in localization of the
tumor. MRI with gadolinium enhancement is useful for visualizing higher-grade gliomas,
meningiomas, schwannomas, and pituitary adenomas. T2-weighted MRI is useful for low-grade
gliomas.

Treatment
Correct management of primary brain tumors requires knowledge of the natural history of
specific tumor types and the risks associated with surgical removal. When feasible, total tumor
removal is the goal; however, subtotal resection may be necessary if vital brain function is
threatened by complete tumor extirpation. If subtotal resection is performed, postoperative
radiation therapy can prolong life and palliate symptoms. Chemotherapy is also used for specific
tumor types.

Metastatic brain tumors are generally treated with whole-brain irradiation. Occasionally, single
lesions amenable to surgery are removed first, followed by whole-brain irradiation.

Perioperative management of increased ICP caused by cerebral edema is accomplished by using


corticosteroids (dexamethasone [Decadron]). If hydrocephalus is present, shunting of
cerebrospinal fluid may be required.

INTRACRANIAL ANEURYSMS
Intracranial aneurysms are saccular, berry-shaped aneurysms, usually found at the arterial
branch points within the circle of Willis (Fig. 20-3). Although they
rarely rupture, significant morbidity and mortality may result secondary to hemorrhage. P
Subarachnoid hemorrhage (SAH) develops when intracranial aneurysms rupture and bleed.
Figure 20-3 • Cerebral arterial circle of Willis.

HISTORY
Sudden onset of a severe headache, typically described as the "worst headache of my life,"
usually signals the rupture of an intracranial aneurysm. ICP transiently increases with each
cardiac contraction, causing a pulsating headache. Progressive neurologic deficits may develop
as a result of blood clot mass effect, vasospasm with infarction, or hydrocephalus. Coma and
death may occur.

A system for categorizing the severity of hemorrhage has been developed using clinical
assessment based on neurologic condition. The five-point Hunt-Hess grading system ranges from
grade 1, indicating good neurologic condition, to grade 5, indicating significant neurologic
deficits (Table 20-2).

TABLE 20-2 Hunt-Hess Classification of Subarachnoid Hemorrhage

Grade Description

1 Mild headache and slight nuchal


rigidity

2 Cranial nerve palsy, severe


headache,
nuchal rigidity

3 Mild focal deficit, lethargy, or


confusion

4 Stupor, hemiparesis, early


decerebrate
rigidity

5 Deep coma, decerebrate rigidity,


moribund
appearance
DIAGNOSIS
CT is useful for demonstrating SAH. If CT is negative in a patient with a highly suspicious
presentation, a lumbar puncture should be performed. If SAH is present, four-vessel cerebral
angiography is performed to define the aneurysm neck and relationship with surrounding vessels
(Fig. 20-4).
Figure 20-4 • A vertebral artery angiogram is shown in A; the same
view, but in a different patient, is shown in B, using digital
subtraction methods. Relevant arteries are labeled.
From Haines DE. Neuroanatomy: An Atlas of Structures, Sections,
and Systems. 6th ed. Philadelphia, PA: Lippincott Williams &
Wilkins, 2004.

TREATMENT
Initial medical treatment involves control of hypertension with intravenous medications.
Phenytoin is administered for prophylactic treatment of seizures, mannitol can be given to
control edema, and nimodipine is used to reduce the risk of developing delayed neurologic
deficits from vasospasm.

Emergency external ventricular drainage may be indicated to decrease the ICP. In rare cases with
progressive neurologic deterioration, emergency crani-otomy and evacuation of a blood clot are
required to prevent herniation. The definitive treatment is obliteration by microsurgical clipping
or endovascular coiling of the aneurysm.

EPIDURAL HEMATOMA
Epidural hematomas are usually seen in patients with head trauma who have sustained a skull
fracture across the course of the middle meningeal artery, causing an arterial laceration and an
expanding hematoma (Fig. 20-5). The increasing pressure of the arterial-based hematoma strips
the dura mater from the inner table of the skull, producing a lens-shaped mass capable of
causing brain compression and herniation.
Figure 20-5 • Epidural hemorrhage: epidural hematoma. This 8-
year-old boy presented after a sledding accident. He had no loss of
consciousness but complained of headache and vomiting. A head
computed tomography scan shows the classic biconvex hyperdensity
of an epidural hematoma.
From Fleisher GR, Ludwig S, Baskin MN. Atlas of Pediatric
Emergency Medicine. Philadelphia, PA: Lippincott Williams &
Wilkins, 2004.

HISTORY
The presentation of an epidural hematoma is commonly described as a head injury followed by
an initial loss of consciousness, recovery (the honeymoon period), and then progressive
deterioration. In reality, this sequence of events is unusual. It is more common not to have an
initial period of unconsciousness or to become unconscious initially and not to regain it.

PHYSICAL EXAMINATION
Assessing the level of consciousness is the most important aspect in evaluating head injuries. The
standard clinical tool for assessment is the Glasgow Coma Scale (GCS), which evaluates eye
opening, verbal response, and motor response. Patients with a GCS of ≤7 have severe head
injuries, those with scores of 8 to 12 have moderate injuries, and those with scores of >12 have
mild injuries. Patients with severe injuries (GCS <8) require immediate endotracheal intubation
for airway protection and rapid neurosurgical evaluation (Table 20-3).

TABLE 20-3 Calculation of Glasgow Coma Score

Best Eye Response (4 points possible)

1. No eye opening

2. Eye opening to pain

3. Eye opening to verbal command

4. Eyes open spontaneously

Best Verbal Response (5 points


possible)

1. No verbal response

2. Incomprehensible sounds

3. Inappropriate words

4. Confused

5. Orientated

Best Motor Response (6 points


possible)

1. No motor response

2. Extension to pain

3. Flexion to pain

4. Withdrawal from pain

5. Localizing pain

6. Obeys commands

DIAGNOSTIC EVALUATION
CT is crucial to establish a diagnosis and treatment plan.

P.157 P
TREATMENT
For patients presenting with a depressed skull fracture and a neurologic examination indicating a
deteriorating level of consciousness, airway control and emergency cranial decompression must
be performed. Burr holes are made over the area of hematoma seen on CT, a flap is quickly
turned, and the clot is decompressed, with resultant decrease of the ICP. Middle meningeal
artery bleeding is controlled, and the dura is fixed to the bone to prevent reaccumulation.

SUBDURAL HEMATOMA
In contrast with epidural hematomas, subdural hematomas are usually low-pressure bleeds
secondary to venous hemorrhage. Both spontaneous and traumatic subdural bleeds occur. The
source of hemorrhage is from ruptured bridging veins that drain blood from the brain into the
superior sagittal sinus.

RISK FACTORS
Most commonly, subdural hematoma is a result of trauma. Older adult patients, those with brain
atrophy, and patients treated with anticoagulants are at particularly increased risk. Less common
causes include rupture of a cerebral aneurysm, arteriovenous malformation, and metastatic
tumors.

HISTORY
Headache, drowsiness, and unilateral neurologic symptoms are the usual presenting symptoms.
Seizure activity and papilledema are uncommon.

EVALUATION
Prompt imaging is the cornerstone of diagnosis. Head CT should be performed in patients with
the appropriate history. MRI is more sensitive and may be considered. In patients without history
of trauma, angiography may be useful to identify the root cause.

P
TREATMENT
Nonoperative management is limited to patients who are stable, have small lesions, and do not
have evidence of herniation. Patients with significant neurologic deficits secondary to mass
effect may need urgent burr-hole decompression or craniotomy. Other indications for surgical
management include lesion size >10 mm, >5 mm of midline shift, or declining GCS. Generally, it
is important to identify and ligate the bleeding vessel.

SPINAL TUMORS
Tumors are defined by anatomic location as being extradural, intradural, or intramedullary (Fig.
20-6). Extradural tumors are most commonly lesions of metastatic disease from primary cancers
of the lung, breast, or prostate. Other common tumors are multiple myeloma of the spine and
lymphoma. Back pain or neurologic deficit from cord compression is the usual presenting
complaint.
Figure 20-6 • Topographic distribution of the common neoplasms of
the spinal meninges, spinal nerve roots, and spinal cord.
The most common intradural tumors are meningiomas, schwannomas, neurofibromas, and
ependymomas. A nerve root tumor may transverse the intervertebral foramen, forming a bilobed
lesion called a dumbbell tumor. Patients usually present with numbness progressing to weakness.

Intramedullary tumors include astrocytomas, ependymomas, and cavernous malformations. It is


important to differentiate cystic tumors from syringomyelia by gadolinium-enhanced MRI,
because both may present with sensory loss.

DIFFERENTIAL DIAGNOSIS
The differential diagnosis for patients presenting with signs and symptoms of spinal cord
pathology are cervical spondylitic myelopathy, acute cervical disc protrusion, spinal angioma,
and acute transverse myelitis.

PHYSICAL EXAMINATION
Patients with tumors of the spine typically present with complaints indicative of progressive
spinal cord compression, with evidence of a sensory level.

DIAGNOSTIC EVALUATION
Plain radiographs may demonstrate bony erosion. MRI is the modality of choice, because it
provides detailed anatomic definition. A CT myelogram may be performed if MRI is unavailable.

TREATMENT
The goal of spinal tumor treatment is to relieve cord compression and to maintain spinal
stability. These are interrelated goals, because removing a compressing tumor usually requires
surgery on the vertebral column.

The spine consists of two columns: the anterior column (vertebral bodies, discs, and ligaments)
and the posterior column (facet joints, neural arch, and ligaments). Damage sustained to one of
the columns may result in permanent spinal instability.

For anterior tumors that involve the vertebral body, tumor removal via the anterolateral
approach is performed. The vertebral body is resected and the defect repaired with a bone graft
and metal plate stabilization.

Posterior tumors can be removed by laminectomy that usually does not cause spinal instability. P
Metastatic and unresectable disease can be palliated and pain controlled with radiation therapy.
Occasionally, anterior and posterior approaches are combined; therefore, appropriate spine
stabilization requirements must be anticipated.

SPONDYLOSIS AND DISC HERNIATION


Degenerative changes in the spine are responsible for a large proportion of spine disease.
Intervertebral discs consist of two parts: the central nucleus pulposus, which acts as a cushion
between vertebrae, and the surrounding dense annulus fibrosus (Fig. 20-7). At birth, the nucleus
contains 80% water, but over time the disk dehydrates, and the disc space narrows. In the
cervical and lumbar spines, this narrowing causes abnormal vertebral stresses, which in turn
cause osteogenesis, producing osteophytes and bony spurs. These degenerative bone growths can
traumatize nerve roots. This degenerative process secondary to abnormal motion in an aging
spine is called spondylosis.
Figure 20-7 • Intervertebral disc: annulus fibrosus and centrally
located nucleus pulposus.

Structural failure of the intervertebral disc occurs when the nucleus pulposus herniates into the
spinal canal or the neural foramina through a defect in the circumferential disc annulus. Lateral
disc herniation can cause nerve root compression and radicular symptoms; central disc
herniation can cause myelopathy.

These two interrelated degenerative processes are responsible for most spine disease,
manifested by nerve root and spinal cord compression. The most mobile segments of the spine
(cervical and lumbar) are commonly affected by both processes (Fig. 20-8).
Figure 20-8 • (A) Normal disc space with normal rotatory
movement of one vertebra on the adjacent one. (B) Disc protrusion.
(C) Osteophytes developing secondary to disc degeneration (or disc
protrusion). Note the different origin of the disc protrusion and
osteophytes.

P
CERVICAL DISC DISEASE

HISTORY
Patients with cervical spondylosis and disc disease are typically older than 50 years and can
present with pain, paresthesia, or weakness. In the case of cervical spondylotic myelopathy
secondary to repetitive spinal cord damage by osteophytes, patients experience progressive
numbness, weakness, and paresthesia of the hands and forearms in a glove-like distribution. In
contrast, patients with radiculopathy secondary to disc disease complain of pain radiating down
the arm in a nerve root distribution, worsening on neck extension.

PHYSICAL EXAMINATION
Limitation of neck motion and straightening of the normal cervical lordosis are common findings.
Sensory and motor deficits in a radicular pattern and careful testing for signs of diminished
biceps, brachioradialis, and triceps reflexes assist with localization. Hyperreflexia and the
presence of the Hoffmann (reflex contraction of thumb and index finger on tapping the nail of or
flicking the middle finger) or Babinski reflex (extension of the great toe on stroking the lateral
aspect of the sole of the foot) help determine the presence of myelopathy and are important
signs to elicit.

DIFFERENTIAL DIAGNOSIS
All causes of cervical spinal cord or cervical nerve root compression must be considered. More
common causes of cord compression are rheumatoid arthritis and ankylosing spondylitis. For
nerve root compression, brachial plexus compression from a first or cervical rib and scalenus
anticus syndromes (thoracic outlet syndrome) should be ruled out. Peripheral nerve entrapment
(carpal tunnel syndrome, ulnar nerve palsy) and Pancoast tumor of the pulmonary apex should be
considered in patients who have arm pain without neck pain.

Diagnostic Evaluation
Cervical spine x-rays show straightening of the normal cervical lordosis, disc space narrowing,
osteophyte formation, and spinal canal narrowing. If the axial diameter of the cervical spinal
canal is ≤10 mm, risk is high for cervical cord compression.
CT myelography and MRI are used to evaluate the spinal cord and nerve roots and define their
relationships to other vertebral structures. Areas of cord and root compression can be identified
and intervention planned. MRI is the study of choice for initial evaluation of a herniated cervical
disc, whereas CT is preferred when more bony detail is required.

TREATMENT
All patients should initially be managed with medical therapy, except for those with myelopathy
or severe radicular weakness. Cervical traction, analgesics, and muscle relaxants are used. For
acute cervical radiculopathy caused by cervical disc herniation, >95% of patients improve
without surgery. However, patients with spondylosis and disc prolapse who fail to improve or who
exhibit progressive worsening may require surgical treatment.

Because the pathogenesis of degenerative osteogenesis is abnormal stress and movement


between vertebrae, procedures aimed at stabilizing the spine have shown significant success in
obtaining symptomatic relief and promoting osteophyte reabsorption. Anterior cervical fusion
produces immobilization by removal of the intervertebral disc, with bone graft replacement and
internal fixation. Both cervical spondylosis and cervical disc prolapse can be treated with this
procedure.

Decompression laminectomy is usually performed only on patients who have a diffusely narrow
spinal canal and who are rapidly worsening as a result of spondylotic myelopathy. The posterior
approach for lateral disc herniations is also used to avoid segmental fusion.

LUMBAR DISC DISEASE


Lumbar disc prolapse is a common disorder. Patients often present with pain radiating down the
lower extremity.

PHYSICAL EXAMINATION
Symptoms of sciatica are caused by disc herniation compressing a nerve root, leading to severe
radicular pain. The L4-5 and L5-S1 discs most commonly prolapse, leading to L5 and S1 nerve
root symptoms. Paresthesia, numbness, and weakness may be present.
Straight leg raise testing can be positive for pain radiating down the affected extremity, with P
both ipsilateral and contralateral leg raising. Other important signs indicating disc herniation
include absence of an ankle or knee reflex, weakness of foot dorsiflexion or plantar flexion, or
weakness of knee extension.

DIAGNOSIS
Clinical diagnosis is confirmed by MRI that demonstrates disc protrusion at the suspected level
(Fig. 20-9).
Figure 20-9 • The relations of a lumbar disc prolapse. The
protruding disc causes nerve root compression.

TREATMENT
Most patients improve without surgery. Elective surgery should be considered for patients with
chronic, disabling, intractable pain. The standard procedure of choice is open laminectomy and
discectomy of the appropriate interspace. Urgent surgery is indicated in patients with
progressive neurologic deficits (e.g., foot drop) and in those with acute onset of cauda equina
syndrome, which is a neurosurgical emergency and occurs as a result of a massive midline disc
protrusion that compresses the cauda equina. Typical findings of cauda equina syndrome include
urinary retention or overflow incontinence, bilateral sciatica, and perineal numbness and
tingling (i.e., saddle anesthesia). Bilateral laminectomy decompression with disc removal is the
procedure of choice.

KEY POINTS
Brain tumors cause elevated intracranial pressure by occupying space,
producing cerebral edema, blocking cerebrospinal fluid flow, or impairing
cerebral venous drainage, resulting in neurologic deficits.
Glioblastoma multiforme tumors are the most common and most malignant
astrocytic tumors. They can track across the corpus callosum and are then
called butterfly gliomas.
Most childhood tumors are located in the posterior fossa and include cystic
astrocytomas, ependymomas, and medulloblastomas.
Intracranial aneurysms are usually found at arterial branch points within the
circle of Willis.
Patients with low-grade presentations should have early aneurysm obliteration
to prevent rerupture. Patients with high-grade presentations are stabilized with
external ventricular drainage, and the aneurysm is obliterated early or in a
delayed fashion, depending on brain swelling. Endovascular treatment is
frequently considered for patients with higher-grade presentations.
Epidural hematomas arise from middle meningeal artery hemorrhage after head
trauma and produce a lens-shaped mass capable of causing brain herniation.
A "honeymoon" period may precede rapid progressive deterioration.
Emergency cranial decompression is lifesaving.
Subdural hematomas are usually low-pressure venous bleeds arising from
ruptured bridging veins that drain blood from the brain into the superior sagittal
sinus.
Older adult patients receiving anticoagulants are at increased risk. P.1
Neurologic deficit warrants urgent neurosurgical intervention.
Spinal tumors are extradural, intradural, or intramedullary.
Most extradural tumors are metastatic lesions.
Symptoms of spinal tumors include pain, myelopathy, disc protrusion, spinal
angioma, and transverse myelitis.
Spondylosis and disc herniation can manifest with nerve root or spinal cord
compression.
Most patients with spondylosis and disc herniation improve without surgery.
Cauda equina syndrome presents as urinary retention or overflow incontinence,
bilateral sciatica, and perineal numbness secondary to lumbar disc herniation.
Urgent decompressive laminectomy is indicated.
Authors: Karp, Seth J.; Morris, James P.G.; Zaslau, Stanley
Title: Blueprints Surgery, 5th Edition

Copyright ©2008 Lippincott Williams & Wilkins

> Table of Contents > Part V - Special Topics > Chapter 21 - Kidneys and Bladder

Chapter 21
Kidneys and Bladder

ANATOMY
The kidneys are retroperitoneal structures. They are surrounded by Gerota's fascia and lie lateral
to the psoas muscles and inferior to the posterior diaphragm. Blood supply is by renal arteries;
usually there is a single renal artery, but there may be more than one. The renal veins drain into
the inferior vena cava. The ureters course retroperitoneally, dorsal to the cecum on the right
and the sigmoid colon on the left. They cross the iliac vessels at the bifurcation between internal
and external and enter the true pelvis to empty into the bladder. The bladder lies below the
peritoneum in the true pelvis and is covered by a fold of peritoneum. Blood supply is from the
iliac arteries through the superior, middle, and inferior vesical arteries (see Color Plate 14).
Sympathetic nerve supply is from L1 and L2 roots, whereas parasympathetic is from S2, S3, and
S4.

URINARY TRACT INFECTION


Urinary tract infections are classified as lower if they involve only the bladder (acute cystitis),
and upper if they involve the kidneys (pyelonephritis). They are considered uncomplicated if
they occur in young, nonpregnant women and complicated if they occur in men, pregnant
women, or older women.

ETIOLOGY
Escherichia coli is by far the most common pathogen, accounting for more than two thirds of all
cases. Other Gram-negative rods, including Proteus, Klebsiella, Staphylococcus saprophyticus,
and Enterococcus, are also common. In hospitalized patients, more resistant organisms, including
Pseudomonas, may be found. In women, most infections are thought to result from fecal
colonization of the vagina.

EPIDEMIOLOGY
Infections of the lower tract are more common than those of the upper tract. In young women in
the United States, the incidence is approximately three in 1,000. Risk factors include diabetes,
sexual intercourse, stress incontinence, and previous urinary tract infections.

HISTORY
Patients often present with frequency, dysuria, nausea, and vomiting. Flank pain is generally
more associated with pyelonephritis and suprapubic pain with cystitis, but this is not reliable.
Previous urinary infections and multiple or new sexual partners may help lead to the diagnosis.

PHYSICAL EXAMINATION
Patients may have fever, flank or suprapubic tenderness, or foul smelling urine. In severe cases,
patients may be hypotensive.

DIAGNOSTIC EVALUATION
Pyuria is nearly ubiquitous in patients with urinary tract infections. Hematuria is also common.
Culture growth of >100,000 colony-forming units/mL of urine is diagnostic, although some
patients will have only 1,000 or
10,000 colony-forming units/mL. For uncomplicated cases, imaging is not necessary. Complicated P
cases may require cystoscopy with or without upper tract imaging.

TREATMENT
Antibiotics are the mainstay of treatment. Oral antibiotics are adequate for uncomplicated
cases, but hospital admission and parenteral antibiotics should be considered for complicated
cases. Fluoroquinolones are a reasonable first choice. For complicated cases, urologic evaluation
should be considered.

STONE DISEASE

ETIOLOGY
The most common kidney stones are calcium phosphate and calcium oxalate (80%); struvite
(15%), uric acid (5%), and cystine (1%) are other causes. Calcium stones are usually idiopathic but
can be caused by hyperuricosuria and hyperparathyroidism. Struvite stones are caused by
infection with urease-producing organisms, usually Proteus. Uric acid stones are common in
patients with gout and can occur with Lesch-Nyhan syndrome or tumors. Cystine stones are
hereditary.

EPIDEMIOLOGY
Approximately 20% of males and 10% of females will be affected by nephrolithiasis over their
lifetime. Calcium stones and struvite stones are more common in women, uric acid stones are
twice as common in men, and cystine stones occur with equal frequency in men and women.
HISTORY
Stone formation is associated with a number of dietary factors, which should be investigated.
Low fluid intake is a general risk factor. Diets high in salt promote excretion and increased
urinary concentration of calcium. High intake of animal protein results in increased calcium, uric
acid, citrate, and acid excretion. Low-calcium diets can also be problematic, as they increase
oxalate excretion.

Patients with stone disease usually present with acute onset of pain beginning in the flank and
radiating down to the groin, although the pain can be anywhere along this track. The patient is
often unable to find a comfortable position, and vomiting is common. Dysuria, frequency, and
hematuria may be described.

DIAGNOSTIC EVALUATION
Workup includes evaluation of urinary sediment that shows hematuria, unless the affected ureter
is totally obstructed. Crystals are frequently observed. It is imperative to determine the type of
stone to guide therapy. Urinary sediment may be extremely useful for this purpose. Calcium
oxalate stones are either dumbbell-shaped or bipyramidal and may be birefringent. Uric acid
crystals are small and red-orange. Cystine stones are flat, hexagonal, and yellow. Struvite stones
are rectangular prisms. Uric acid crystals and calcium oxalate crystals can be found in normal
individuals and thus are less useful when found in the sediment.

Blood work should evaluate for elevated serum calcium and uric acid. Measurement of
parathyroid hormone levels should be performed in patients with hypercalcemia or high urinary
calcium.

Spiral computed tomography is commonly used to make the diagnosis. The stones appear bright
in the thin cuts that are obtained with this technique. An abdominal radiograph will often show
the stones, as calcium, struvite, and cystine stones are all radiopaque. Intravenous pyelography
involves intravenous administration of an iodinated dye that is excreted in the kidneys. This
allows diagnosis of stones by outlining defects in the ureter or demonstrating complete
obstruction caused by stone disease. Retrograde pyelography involves injecting dye through the
urethra and is useful for assessing the degree and level of obstruction. Ultrasonography of the
kidneys can demonstrate the stone (Fig. 21-1) and hydronephrosis (Fig. 21-2) indicative of
ureteral obstruction. The presence of fluid jets at the entrance of the ureter in the bladder
precludes the diagnosis of total obstruction.
Figure 21-1 • Posterior shadowing: a stone within the renal pelvis
(large arrow) casts a posterior shadow (small arrow).
From Harwood-Nuss A, Wolfson AB, Lyndon CH, et al. The Clinical
Practice of Emergency Medicine. 3rd ed. Philadelphia, PA:
Lippincott Williams & Wilkins; 2001:128.1.
Figure 21-2 • Hydronephrosis: longitudinal view of the right kidney.
There is moderate hydronephrosis, which is seen as separation of
the pelvic calyces by fluid (arrow).
From Harwood-Nuss A, Wolfson AB, Lyndon CH, et al. The Clinical
Practice of Emergency Medicine. 3rd ed. Philadelphia, PA:
Lippincott Williams & Wilkins, 2001.

TREATMENT
In the acute setting, pain and nausea should be controlled with narcotics and antiemetics. Most
stones pass spontaneously; deflazacort and nifedipine or tamsulosin may be used to facilitate
stone passage. Stone size predicts spontaneous passage: asymptomatic stones <5 mm usually do
not require intervention. Stones >5 mm should be considered for intervention. Less invasive
options include extracorporeal shock wave lithotripsy (ESWL); percutaneous nephrolithotomy
(PCNL); and endoscopic lithotripsy using ultrasonic, electrohydraulic,
or laser energy to remove stones. ESWL is the most common approach and involves using high- P
energy shock waves that originate extracorporeally. Focusing the energy on the stone causes
fragmentation, which facilitates passage. This technique is not ideal for struvite or staghorn
calculi. PCNL involves placement of a nephrostomy tube and is more efficacious than ESWL for
stones that are large, complex, or composed of cysteine. PCNL and ESWL can be combined (Fig.
21-3). Open pyelolithotomy is reserved for patients who fail multiple attempts at less invasive
approaches.
Figure 21-3 • Combined percutaneous nephrolithotomy and
extracorporeal shock wave lithotripsy for staghorn calculi. A 35-
year-old woman presented with a Proteus urinary tract infection
and was found to have a large complete left staghorn calculus.
From Harwood-Nuss A, Wolfson AB, Lyndon CH, et al. The Clinical
Practice of Emergency Medicine. 3rd ed. Philadelphia, PA:
Lippincott Williams & Wilkins; 2001:63–65.

To prevent recurrent stones, patients should be counseled to increase fluid intake. Dietary
modifica-tions should be recommended based on the stone type. Pharmacologic interventions
include a thiazide diuretic for patients with hypercalciuria and allopurinol or potassium citrate
for uric acid stones. Hypocitraturia may be treated with potassium citrate. Oxalate stones may
be treated with calcium if urinary calcium is low.

RENAL CANCER

EPIDEMIOLOGY
Two percent of cancer deaths are attributable to renal cancer. Men are affected twice as often
as women, and smoking may be a risk factor.

P
PATHOLOGY
Renal cell cancer is classified as clear cell (80%), papillary (15%), and chromophobe (5%; see
Color Plate 15). Rare types include collecting duct carcinoma and unclassified renal cell cancer.
Other tumors that may arise in the kidney include Wilms tumor and sarcoma.

HISTORY
Patients may experience hematuria and flank pain, which can be sudden in the event of
hemorrhage. Fever and extrarenal pain from metastatic disease may be present. Approximately
30% of patients will present with metastatic disease.

PHYSICAL EXAMINATION
Tumors may be palpable.

DIAGNOSTIC EVALUATION
Intravenous pyelography demonstrates a defect in the renal silhouette. Computed tomography
can differentiate between cystic and solid lesions.

TREATMENT
Initial treatment in most cases is radical nephrectomy with attempt to remove all tumor. For
patients with metastatic disease, results with chemotherapy are disappointing. Gemcitabine and
fluorouracil demonstrate limited activity against the tumor. Interleukin-2 is a drug approved by
the Food and Drug Administration for renal cell cancer. Response rates are in the 15% to 20%
range.

BLADDER CANCER

PATHOLOGY
Transitional cell tumors make up 90% of bladder malignancies. The remainder are squamous cell
and adenocarcinoma.

EPIDEMIOLOGY
Men are more frequently affected than women by a ratio of 3:1. Smoking, beta-naphthylamine,
schistosomiasis, and paraminophenol all predispose a person to the development of bladder
cancer.

HISTORY
Most patients present with hematuria. Urinary tract infections are relatively common, as is
bladder irritability evidenced by frequency and dysuria.

DIAGNOSTIC EVALUATION
Urinary cytology may reveal the presence of bladder cancer. Cystoscopy with biopsy confirms the
diagnosis. Excretory urography may demonstrate the lesion.

TREATMENT
For local disease, transurethral resection with chemother-apy including doxorubicin, mitomycin,
or thiotepa is effective. For locally advanced disease, radical cystectomy (including
prostatectomy in men) is combined with radiation and gemcitabine and cisplatin.

KEY POINTS
Kidney stones are usually composed of calcium salts.
Symptoms of kidney stones include severe flank pain, which may radiate
to the groin.
Stones >5 mm should be considered for intervention. Most patients will respond
to minimally invasive methods of stone removal.
Renal cancer is responsible for 2% of cancer deaths, and treatment in most
cases is radical nephrectomy.
Patients with bladder cancer usually have hematuria.
Treatment for bladder cancer may be transurethral resection for local disease;
radical cystectomy is used for advanced disease.
Authors: Karp, Seth J.; Morris, James P.G.; Zaslau, Stanley
Title: Blueprints Surgery, 5th Edition

Copyright ©2008 Lippincott Williams & Wilkins

> Table of Contents > Part V - Special Topics > Chapter 22 - Prostate and Male Reproductive Organs

Chapter 22
Prostate and Male Reproductive Organs

BENIGN PROSTATIC HYPERPLASIA


Benign prostatic hyperplasia (BPH) is a common condition of the prostate gland seen in older
men. BPH is clinically important because it is the most common cause of urinary outlet
obstruction in men older than 50 years. Untreated, stasis occurs, which increases the risk of
urinary tract infection and bladder stones. Over time, bladder decompensation can result in
chronic urinary retention with overflow or renal failure secondary to high-pressure urinary
retention.

PATHOGENESIS
Prostate gland growth is influenced by steroid hormones. However, the exact mechanism of
prostatic hyperplasia remains unclear. Interestingly, BPH does not occur in castrated men or
pseudohermaphrodites, both of whom lack dihydrotestosterone, the active metabolite of
testosterone. Estrogens have also been implicated in prostatic hyperplasia, because in aging
men, the levels of estrogens increase and those of androgens decrease.

The specific area of cellular hyperplasia is the transitional zone or periurethral area of the
prostate. The periurethral glandular elements undergo hyperplasia, causing an increase in
glandular mass that results in compression of the prostatic urethra and the onset of obstructive
symptoms (Figs. 22-1 and 22-2).
Figure 22-1 • Benign prostatic hyperplasia causing urethral
compression. The enlarged periurethral glands are enclosed by the
orange peel–like surgical capsule, which is composed of compressed
true prostatic tissue.
Figure 22-2 • Normal prostate and nodular hyperplasia. In pros-
tatic hyperplasia, the nodules distort and compress the urethra and
exert pressure on the surrounding normal prostatic tissue. Image
from Rubin E, Farber JL MD. Pathology. 3rd ed. Philadelphia, PA:
Lippincott Williams & Wilkins, 1999.

EPIDEMIOLOGY
The prevalence of BPH increases with age. Autopsy studies show that at least 50% of men older
than 50 years have significant enlargement of the prostate as a result of BPH. Rarely does a
patient present before age 50 years, because the doubling time of the hyperplastic gland is slow.
By age 90 years, roughly 90% of males have a significant degree of hyperplasia. All men with
intact functional testes are at risk for development of BPH.

CLINICAL MANIFESTATIONS

History
Any older man presenting with obstructive urinary symptoms must be suspected of having BPH.
Symp-toms include urinary hesitancy, intermittency, decreased force of urinary stream, and a
sensation of incomplete bladder emptying after voiding. Secondary symptoms are a consequence
of urinary stasis. High postvoid residual volumes promote bacterial growth, leading to urinary
tract infection. Stasis can also promote the formation of bladder calculi. Most seriously, high-
pressure chronic retention can cause bilateral hydroureteronep-hrosis and subsequent renal
failure.

Physical Examination
A careful rectal examination reveals an enlarged symmetric rubbery gland. The size of the gland
has no relationship to symptomatology. A small gland may produce a high degree of outflow
obstruction, whereas a large gland may produce no symptoms at all. The suprapubic region
should be palpated to rule out a grossly distended bladder.

P
Diagnostic Evaluation
Urine should be obtained for sediment analysis and microbiologic cultures. Serum blood urea
nitrogen and creatinine levels should be checked for evidence of renal insufficiency. If chronic
urinary retention is suspected, a postvoid residual can be checked by straight catheterization or
bladder ultrasonography. Urinary flow rate is assessed by measuring the volume of urine voided
during a 5-second period. A flow rate of <50 mL in 5 seconds is evidence of bladder outlet
obstruction. Ultrasonography, intravenous pyelography, or computed tomographic (CT) scan can
be used for imaging the urinary tract. Information regarding size of the prostate, presence of
bladder stones, the postvoid residual volume, and hydronephrosis can be obtained. Transrectal
ultrasonography is used to evaluate either an irregular prostate when found on examination or an
elevated prostate-specific antigen level.

Treatment
The goals of drug therapy for BPH are to relax smooth muscle in the prostate and bladder neck
or to induce regression of cellular hyperplasia, thereby enhancing urinary outflow from the
bladder to the urethra. Alpha blockade of adrenergic receptors produces smooth muscle
relaxation of both prostate and bladder neck. An infrequent side effect of alpha-antagonists
(e.g., terazosin) is postural hypotension (2% to 8%). Prostatic hyperplasia can also be treated
with 5-alpha reductase inhibitors (e.g., finasteride), which block the conversion of testosterone
to dihydrotestosterone without lowering serum levels of circulation testosterone. However, the
effectiveness of 5-alpha reductase inhibitors is less than half that seen with alpha-blockers.

Surgical relief of obstruction is necessary when medical therapy fails. The indications for surgery
are a postvoid residual volume >100 mL, acute urinary retention, chronic urinary retention with
overflow dribbling, gross hematuria on more than one occasion, and recurrent urinary tract
infections. Additional indications are patient request for restoration of normal voiding pattern
because of excessive nocturia or dribbling.

The procedure of choice is transurethral resection of the prostate (TURP). With the patient in P
the lithot-omy position, the resectoscope is introduced via the urethra into the bladder. The
occlusive prostate tissue is identified, and under direct vision, the tissue is shaved away using an
electrified wire loop (Fig. 22-3).
Figure 22-3 • Prostate surgery procedures. (A) Transurethral
resection of the prostate (TURP). Portions of the prostate are
removed at the bladder opening. (B) Transurethral incision of the
prostate. One or two incisions are made in the prostate to reduce
pressure on the urethra.

From Cohen BJ. Medical Terminology. 4th ed. Baltimore, MD:


Lippincott Williams & Wilkins, 2003.

Extravasated blood and tissue fragments are evacuated as the bladder is continuously irrigated
with a nonelectrolytic isotonic solution. An indwelling catheter is left in place for 1 to 7 days. To
minimize the blood loss and side effects of the standard TURP technique, alternative minimally
invasive outpatient procedures have been developed and are applicable to select patients.
Transurethral needle ablation and focused ultrasound—both of which are less invasive modalities
than TURP—ablate prostate tissue by locally heating the tissue. Although these procedures have
excellent short-term results in selected patients, long-term efficacy is not well established.

PROSTATE CANCER
Prostate cancer is the most common malignancy of the male genitourinary tract. Indolent tumor
growth and a long latency period account for the majority of cases (approximately 80%) being
clinically silent. Most prostate cancers are only discovered on postmortem examination.
Management and prognosis depend on stage of tumor.

PATHOGENESIS
The vast majority of prostate cancer (approximately 95%) is adenocarcinoma. Tumors arise from
the glandular epithelium in the peripheral zone of the prostate. Tumor growth is hormonally
influenced; testosterone exerts a stimulatory effect, whereas estrogens and antiandrogens are
inhibitory. Tumors are histologically graded using the Gleason grading system, with scores from 2
(well differentiated) to 10 (poorly differentiated). Tumor grade is correlated with prognosis.

EPIDEMIOLOGY
Prostate cancer is a malignancy of older men, usually occurring after age 60 years. The disease is
more common in Black men than in White men.

CLINICAL MANIFESTATIONS
History
Early prostate cancer is usually asymptomatic and is typically only detected on screening
examination.
Many patients present with evidence of obstructive symptoms indicating invasion or compression P
(poor stream, incomplete bladder emptying, nocturia). Such patients are commonly
misdiagnosed as having BPH. Metastatic disease is often manifested by bony pain or ureteric
obstruction.

Physical Examination
Digital rectal examination and prostate-specific antigen level measurement are the principal
methods of screening. Tumor staging is based on the degree of spread: T1 to T2 indicates
localized spread within the prostate, T3 to T4 indicates local spread to seminal vesicles or pelvic
wall, and M1 indicates metastatic disease. Pattern of spread is via lymphatics to iliac and
periaortic nodes and via the circulation to bone, lung, and liver.

Diagnosis
As for most cancers, prostate cancer requires tissue diagnosis. Typically, a hard nodule is
detected on digital rectal examination and a follow-up transrectal ultrasound is obtained for
needle biopsy of the prostate. The procedure is well tolerated and performed on an outpatient
basis.

Chest x-ray is performed to evaluate for lung metastases. Liver function tests may detect liver
metastases. If bone metastases are suspected on the basis of presenting symptoms, a bone scan
is indicated.

Treatment
Treatment for prostate cancer is based on the stage and grade of the tumor. Two common staging
systems are the Gleason score and the TNM stage (Tables 22-1 and 22-2). The treatment options
for localized disease (T1 to T2) include radical prostatectomy, external-beam radiotherapy, or
interstitial irradiation with implants.
For local spread (T3 to T4), the treatment is external-beam radiotherapy, with the addition of P
hormonal therapy for more advanced cases. Treatment for metastatic disease is hormonal
ablation, as most prostate cancers are androgen sensitive. Methods of androgen ablation include
surgical and pharmacologic options. Bilateral surgical orchiectomy is the gold standard for
ablating testosterone production. Chemical castration using luteinizing hormone-releasing
hormone agonists in conjunction with antiandrogens, such as flutamide and cyproterone,
produces castrate levels of testosterone. Randomized trials demonstrate that patients with
hormone-refractory prostate cancer benefit from docetaxel-based chemotherapy.
TABLE 22-1 Gleason Score

Histopathologic Grade (G)

G1 Well-differentiated
(slight anaplasia)

(Gleason Well-differentiated
2–4) tumors with
cells that are expected to
grow
slowly and not spread
readily

G2 Moderately differentiated
(moderate anaplasia)

(Gleason Moderately differentiated


5–6) tumor
cells

G3–G4 Poorly differentiated or


undiffer-
entiated (marked
anaplasia)

(Gleason Poorly differentiated


7–10) tumors with
cells that are likely to
grow
rapidly and spread to
other parts
of the body (metastasize)

TABLE 22-2 AJCC Classification for Prostate Cancer

TNM Definitions

Primary tumor (T)

T0         No evidence of primary


tumor

T1 Clinically inapparent tumor


not palpable or
visible by imaging

T2 Tumor confined within


prostate

T3 Tumor extends through the


prostate capsule

T4 Tumor is fixed or invades


adjacent structures
other than seminal vesicles:
bladder neck,
external sphincter, rectum,
levator muscles,
and/or pelvic wall

Regional Lymph Nodes (N)

N0 No regional lymph node


metastasis

N1 Metastasis in regional lymph


node(s)

Distant Metastasis (M)

M0 No distant metastasis

M1 Distant metastasis

AJCC Stage Groupings

Stage I

T1a, N0, M0, G1

Stage II

T1a, N0, M0, G2–G4

T1b, N0, M0, any G

T1c, N0, M0, any G


T1, N0, M0, any G

T2, N0, M0, any G

Stage III

T3, N0, M0, any G

Stage IV

T4, N0, M0, any G

Any T, N1, M0, any G

Any T, any N, M1, any G

TESTES
Disorders of the testes requiring surgical management include congenital abnormalities, tumors,
and, in the emergent setting, testicular torsion.

CONGENITAL ABNORMALITIES

Cryptorchidism
Cryptorchidism is the failure of normal testicular descent during embryologic development. The
cause of failed descent is unknown but may be due to a selective hormone deficiency,
gubernaculum abnormality, or intrinsic testicular defect. Such cryptorchid testes fail in
spermatogenic function, but they may retain the ability to secrete androgens. The major risk of
cryptorchid testes is the increased risk of testes cancer (35 to 48 times more common than in
descended testes). Inguinal hernia is also found in at least 25% of patients with cryptorchidism.

Physical Examination
The testicle remains within the abdomen and cannot be palpated on physical examination.
Treatment
Because spermatogenic failure is progressive, surgical exploration and scrotal placement of the
testis should be performed before 2 years of age. If placement of the testicle into the scrotum is
not possible, orchiectomy is indicated, because the incidence of cancer of abdominal testes is
very high.

Incomplete Descent of the Testis


Incomplete descent of the testis implies a testicle arrested at some point in the path of normal
descent but palpable on physical examination. Such testes are usually located within the inguinal
canal between the deep and superficial rings (Fig. 22-4).

Figure 22-4 • Four degrees of incomplete descent of the testis. 1.


In the abdominal cavity close to the deep inguinal ring. 2. In the
inguinal canal. 3. At the superficial inguinal ring. 4. In the upper
part of scrotum.

From Snell RS. Clinical Anatomy. 7th ed. Philadelphia, PA:


Lippincott Williams & Wilkins, 2003.

Incompletely descended testes are often associated with congenital indirect hernias as a result
of the incomplete obliteration of the processus vaginalis.

Treatment
Because testicular function is less compromised than in a cryptorchid testicle, the usual
treatment is repositioning and orchiopexy within the scrotum. If present, an indirect hernia is
repaired concurrently.

Testicular Tumors
Tumors of the testicle are the most common genitourinary malignancy among young men
between the ages of 20 and 35 years. Virtually all neoplasms of the testicle are malignant.
Tumors are divided into either germ cell or non–germ cell tumors, depending on their cellular
origin. Germ cell tumors predominate, accounting for 90% to 95% of all tumors.

Pathology
Non–germ cell tumors, which arise from Leydig and Sertoli cells, produce excess quantities of
androgenizing
hormones. Germ cell tumors arise from totipotential cells of the seminiferous tubules. Germ cell P
tumors are divided into two categories: seminomas and nonseminomatous germ cell tumors
(NSGCTs). Seminomas are relatively slow growing and exhibit late invasion. They are usually
discovered and surgically removed before metastasis can occur. NSGCTs exhibit greater
malignant behavior and metastasize earlier. NSGCTs consist of embryonal (20%), teratoma (5%),
choriocarcinoma (<1%), or mixed cell type (40%). Choriocarcinomas are fortunately rare subtypes
but are highly invasive, aggressive tumors that metastasize via lymphatic and venous systems
early in the disease.

Epidemiology
Seminomas are the most common malignant germ cell tumor. Embryonal carcinoma is usually
seen in younger males during childhood. Non–germ cell tumors are relatively rare.
History
Tumors usually manifest as firm, painless testicular masses (Fig. 22-5). Occasionally, the mass
may cause a dull ache. Hemorrhage into necrotic tumor or after minor trauma may cause the
acute onset of pain. Approximately 10% of patients with testicular tumors have a history of
cryptorchidism. Because of excess androgen production, non–germ cell tumors can cause
precocious puberty and virilism in young males and impotence and gynecomastia in adults.

Figure 22-5 • Early testicular tumor.

From Weber J, Kelley J. Health Assessment in Nursing. 2nd ed.


Philadelphia, PA: Lippincott Williams & Wilkins, 2003.

Diagnostic Evaluation
Immediate evaluation should include serum for tumor markers (alpha-fetoprotein [AFP], beta-
human chorionic gonadotropin [β-hCG]), serum lactate dehydrogenase, and ultrasound. Tumor
markers are not always helpful because seminomas, the most
common testicular neoplasm, are usually negative for both AFP and β-hCG. In neoplasms with P
tumor markers, the level of tumor burden directly relates to AFP/β-hCG levels that can be
monitored during the postoperative period to evaluate the efficacy of treatment and to detect
recurrence.

Treatment
Initial treatment is always radical orchiectomy. Subsequent therapy depends on tumor type,
grade, and staging. Retroperitoneal lymph node dissection is standard therapy for stage I and II
NSGCT.

Seminomas are usually highly radiosensitive. Adjuvant treatment with radiation and
chemotherapy yields high 5-year survival rates for both localized and metastatic disease.

Torsion of the Spermatic Cord


Torsion of the spermatic cord is a urologic emergency because complete strangulation of the
testicular blood supply renders the testicle surgically unsalvageable after approximately 6 hours.

Pathogenesis
Torsion results from an abnormally high attachment of the tunica vaginalis around the distal end
of the spermatic cord. This abnormality allows the testis to hang within the tunica compartment,
like a bell clapper within a bell—hence the name bell clapper deformity. As such, the testicle is
free to twist on its own blood supply, causing pain and ischemic strangulation (Fig. 22-6).
Figure 22-6 • Torsion. Left image shows normal testicular anatomy;
a bell-clapper deformity in the tunica vaginalis (middle image) can
permit torsion of spermatic vessels (right image).

LifeART image copyright © 2009. Lippincott Williams & Wilkins. All


rights reserved.

History
Torsion is usually seen in young male patients, who present complaining of the rapid onset of
severe testicle pain followed by testicle swelling.

Physical examination reveals a high-riding, swollen, tender testicle, oriented horizontally in the
scrotum. Pain is worse with elevation of the testes, and the cremasteric reflex is often absent.

Diagnostic Evaluation
A color-flow Doppler ultrasound should be obtained to evaluate for blood flow within the
testicle. Absence of flow confirms the diagnosis.

Differential Diagnosis
The differential diagnosis of an acutely swollen, tender testicle includes essentially two
diagnoses: torsion of the spermatic cord and advanced epididymitis. Because one can mimic the
other, it is vitally important to arrive at a timely diagnosis. Other less common diagnoses include
torsion of an appendix testis or appendix epididymis (Fig. 22-7).

Figure 22-7 • Torsion of appendix testis or appendix epididymis (not


shown) can also cause testicular pain and tenderness.

Treatment
Prompt surgical exploration and orchiopexy are required to save the testicle from undergoing
ischemic necrosis. Because the bell clapper deformity is usually bilateral, orchiopexy of the
contralateral testicle is performed concurrently. If the diagnosis is unclear, surgical exploration is
required, because an uncorrected torsion will cause necrosis of the testicle.

KEY POINTS
Benign prostatic hyperplasia causes bladder outlet obstruction and
obstructive urinary symptoms. Urinary stasis leads to urinary tract
infections, bladder calculi, hydroureteronephrosis, and renal failure.
Benign prostatic hyperplasia is medically treated by alpha blockade to relax
prostate and bladder neck smooth muscle and occasionally by 5-alpha reductase
inhibition to block dihydrotestosterone production and cause regression of
cellular hyperplasia. Surgically treatment is by transurethral resection of the
prostate and other techniques.
Prostate cancer is the most common male genitourinary tract malignancy,
occurring in older men. Management depends on tumor stage. Treatment
options include surgery, radiotherapy, and hormonal ablation. P.1
Cryptorchidism and incomplete testicular descent are congenital abnormalities.
Abdominal testes have a high incidence of cancer.
Testicular tumors are either germ cell tumors (seminomas [the most common]
or nonseminomatous germ cell tumors [embryonal carcinoma, teratoma,
choriocarcinoma, or mixed]) or non–germ cell tumors (Leydig and Sertoli cell
tumors). Tumors usually manifest as firm, painless testicular masses.
Tumor diagnosis is by ultrasound and alpha-fetoprotein and beta-human
chorionic gonadotropin measurement. Orchiectomy, with or without radiation, is
standard treatment for testicular tumors.
Testicular torsion is caused by abnormally high attachment of the tunica
vaginalis around the distal end of the spermatic cord. Doppler ultrasound is used
for diagnosis because epididymitis can mimic torsion. Prompt surgical
exploration and bilateral orchiopexy is required to reverse ischemia from
strangulation.
Authors: Karp, Seth J.; Morris, James P.G.; Zaslau, Stanley
Title: Blueprints Surgery, 5th Edition

Copyright ©2008 Lippincott Williams & Wilkins

> Table of Contents > Part V - Special Topics > Chapter 23 - Skin Cancer

Chapter 23
Skin Cancer

ANATOMY
Skin is often referred to as the largest organ in the body, which, by mass, it is. The importance of the skin as a functional barrier to loss
of fluids and protection from microorganisms is immediately apparent to anyone caring for patients with significant burns. The
performance of these functions requires a highly specialized multi-layer construction.

The outer layer, the epidermis, comprises four layers. Moving inward, they are the keratin layer, consisting of dead skin cells, the
granular layer, the squamous layer, and the basal layer. Epithelial cells originate in the basal layer and are pushed outward over time to
form the upper layers. These basal cells are strongly attached to the basement membrane. Moving inward, the dermis is composed of
collagen, elastin, and glycosaminoglycans. Cellular components are typically fibroblasts. Below the dermis is a layer of subcutaneous fat
that is home to sweat glands, hair follicles, and sebaceous glands.

OVERVIEW AND PREVENTION


Skin cancer is the most common malignancy in the United States, with the less aggressive forms, basal cell and squamous cell, being
much more prevalent than melanoma skin cancer. Approximately 1 million nonmelanoma skin cancers will be diagnosed in the United
States in 2007, but only 2,000 deaths will be attributable to the disease. The incidence of these types of cancers is increasing in the
United States.

It is generally accepted that these cancers are due to sun exposure. Professional and public education campaigns regarding the dangers
of excessive sun exposure are expected to decrease the incidence over time. Methods to reduce sun exposure include limiting daytime
activity during peak sun hours, between 11 AM and 3 PM; using protective clothing, including long sleeves and hats; and using high-
opacity sunscreens. Because sunburns seem to be associated with higher rates of skin cancer, these should be avoided.

Skin cancer seems to be more prevalent in people with light skin or numerous moles. Regular check-ups for both these groups are
critical to make an early diagnosis and initiate treatment.

BASAL CELL CARCINOMA


Basal cell carcinoma (BCC) is the most common form of skin cancer in Caucasians. It is rare in Asians and exceedingly rare in darkly
pigmented individuals. The predominant cause is long-term excess exposure to ultraviolet B (UVB) radiation. Accordingly, BCC is a
disease of adults, and tumors arise from sun-exposed skin, namely the head and neck. The cellular origin of BCC has traditionally been
thought to be the basal cell of the epidermis. More recently, an alternative theory posits that the originating cell type is a pluripotent
epithelial cell. BCC is categorized into three types: noduloulcerative, superficial, and sclerosing.

NODULOULCERATIVE BASAL CELL CARCINOMA


Lesions have a pearly, dome-shaped, nodular appearance, with associated telangiectasia and an ulcerated center (see Color Plate 16).
Telangiectasia is secondary to
tumor-induced angiogenesis, and ulceration results from outgrowth of the local blood supply. Noduloulcerative lesions are the most P
common type of BCC.

Tumors <1 cm in diameter are rarely invasive and can be treated with cautery and curettage or cryosurgery. Tumors >1 cm are treated
with surgical excision. High-risk sites of tumor growth are areas with underlying bone and cartilage (i.e., nose, ear), because the
growing tumor tends to track along these structures. Such tumors have a high recurrence rate. Therefore, high-risk tumors and
recurrent tumors should be treated with Moh micrographic surgery to ensure complete excision. In this procedure, the entire visible
tumor is excised and the margins are carefully marked. Frozen sections are then examined to determine whether the tumor has been
completely excised, and if not, to guide further resection. The wounds are not closed until the pathologist determines that all margins
are negative. In this way, a maximum amount of skin and soft tissue can be saved, providing a good cosmetic result.
SUPERFICIAL BASAL CELL CARCINOMA
The second most common BCC is the superficial type. Lesions usually appear on the trunk and proximal extremities and clinically
resemble thin, scaly, pink plaques with irregular margins (see Color Plate 17). These horizontally expanding tumors are often dismissed
as dermatitis; subsequently, tumors may reach diameters of several centimeters by the time of diagnosis. By this late stage, ulceration
and deep dermal invasion are present. Standard treatment has been wide-margin excision, with skin grafting if necessary. However, this
approach may be unacceptably morbid, leaving a large skin defect. Recently, topical chemotherapy with fluorouracil, cryosurgery, and
cautery/curettage has shown cure rates similar to those of traditional wide excision.

SCLEROSING BASAL CELL CARCINOMA


Sclerosing BCC is the least common type. The anatomic distribution is similar to that of the noduloulcerative type, but histologically,
the lesions appear as narrow cords of tumor cells encased in a proliferation of connective tissue. Macroscopically, lesions are smooth,
atrophic, and indurated and easily mimic scar tissue. This deceptive appearance is unfortunate, because sclerosing tumors are more
aggressive than other basal cell tumor types. The growth pattern follows tissue planes and neurovascular bundles, resulting in deep soft
tissue invasion. Moh micrographic surgery is the preferred management technique.

MELANOMA
Melanoma is the most frequent cause of death of all skin cancer types. It results from malignant transformation of the normal
melanocyte, usually located in the basal layer of the epidermis. Many melanomas are curable by surgical excision.

PATHOGENESIS
Ultraviolet (UV) light is suspected to play a role in the development of all types of skin cancer, including melanoma. Although the
precise etiologic role of UV light in the malignant transformation of skin cells remains unresolved, both ultraviolet A (UVA) and UVB are
thought to have carcinogenic potential. UVA penetrates deep into the dermis, damaging connective tissue and intrinsic skin elasticity.
Excessive UVB exposure results in sunburn.

EPIDEMIOLOGY
Melanoma accounts for 5% of all skin malignancies and 3% of all cancers. The diagnosis of melanoma carries a 50% mortality rate in the
United States, and the incidence has dramatically increased during the past 10 to 15 years. Many lesions arise from preexisting moles. A
mole that shows rapid growth or heterogeneous pigmentation should be evaluated and possibly biopsied to rule out melanoma. Fair-
skinned individuals have a higher incidence of melanoma than the general population. The five signs of melanoma can be remembered
as the ABCDE's: asymmetric shape, irregular border, mottled color, large diameter, and progressive enlargement.

RISK FACTORS
Risk factors include the following: a mole that shows persistent changes in shape, size, or color; persons having more than 100 nevi;
atypical nevi (5% of population); personal history or family history of melanoma; excess
sun exposure (especially in childhood); fair complexion; and tendency to freckle and sunburn. P

MELANOMA TYPES
Superficial spreading melanoma can occur anywhere, on both sun-exposed and nonexposed areas. The average age of diagnosis is 40 to
50 years. Lesions are commonly on the upper back and lower legs. Lesions show heterogeneous pigmentation with irregular margins.
The growth phase is radial with horizontal spread (Fig. 23-1).
Figure 23-1 • Superficial spreading melanoma. Note the central area (whitish gray) of regression.
From Goodheart HP. Goodheart's Photoguide of Common Skin Disorders. 2nd ed. Philadelphia, PA:
Lippincott Williams & Wilkins, 2003.

Lentigo maligna melanoma is usually seen in older individuals, with the average age of diagnosis being 70 years. Lesions appear on sun-
exposed surfaces, particularly the malar region of the cheek and temple. Lesions exhibit horizontal spread (Fig. 23-2).

Figure 23-2 • Lentigo maligna melanoma. Biopsy of this lesion demonstrated invasion into the
dermis.

From Goodheart HP. Goodheart's Photoguide of Common Skin Disorders. 2nd ed. Philadelphia, PA:
Lippincott Williams & Wilkins, 2003.
Acral lentiginous melanoma has an unusual distribution in that lesions appear on palms, soles, nail beds, or mucous membranes. The
most common mucous membrane site is the vulva. Other sites include the anus, nasopharynx, sinuses, and oral cavity. The average age
of diagnosis is 60 years. Spread is in a horizontal pattern.

Nodular melanoma can occur at any site and has a very early malignant potential secondary to a predominantly vertical growth phase.
In contrast to the three other melanoma types exhibit which radial growth phases with horizontal spread. Nodular lesions have well-
circumscribed borders and uniform black or brown coloring.

PROGNOSIS
As with other cancers, the extent of spread is an important prognostic factor. Stage I denotes local disease <1.5 mm. Stage II denotes
local disease >1.5 mm. Stage III denotes regional disease. Stage IV denotes metastatic disease. As indicated in Table 23-1, stage I
disease carries a relatively good prognosis as compared with the dismal prognosis of stage IV disease.

TABLE 23-1 Ten-Year Survival Rates in Patients with Melanoma by Tumor Thickness and Ulceration
4,568)a

Number of Patients with

10-Year Survival Rate


Thickness, No ulceration, Ulceration,
mm Percent Percent
No ulceration Ulceration P value

0.01–1.00 2017 (95.5) 96 (4.5) 92.0 69.1 <0.0001

1.01–2.00   944 (78.8) 255 (21.2) 77.7 62.9 <0.0001

2.01–4.00   500 (57.4) 372 (42.6) 59.5 53.2 0.006

>4.00   146 (38.1) 238 (61.9) 54.5 35.5 0.0006

aModified
from Buzaid AC, Ross MI, Balch CM, et al, Critical analysis of the current American Joint
Committee on Cancer staging system for cutaneous
melanoma and proposal of a new staging system. J Clin Oncol 1997;15:1039.

In melanoma, tumor thickness is inversely related to survival and is the single most important prognostic indicator. Historically, there
have been two systems for classifying melanomas: the Breslow thickness scale and Clark level of tumor invasion.

The Breslow scale defines primary melanomas that are <0.76 mm thick as local tumors. These tumors have >90% cure rates after simple
excision. Individuals with tumors deeper than 0.76 mm have a significant risk of metastatic disease and worse survival.

Clark levels of tumor invasion provide an anatomic description of tumor invasion. The level of tumor invasion can be used for discussing
prognosis and planning surgical management (Fig. 23-3).
Figure 23-3 • The Clark and Breslow classifications for melanoma.

TNM staging defines stage I tumors as T1 lesions (≤0.76 mm thick) or as T2 lesions (0.76 to 1.50 mm thick) with negative nodes and no
metastases. Stage II tumors are T3 lesions (1.51 to 4.00 mm thick) or T4 lesions (>4.00 mm thick) with negative nodes and no
metastases. Stage III tumors have fewer than three regional metastases (N1) and no distant metastases,
whereas stage IV tumors have metastases in skin or subcutaneous tissue, distant lymph node metastases, or visceral metastases. P

TREATMENT
Surgical excision is the treatment of choice for primary melanomas. The size of the surgical margin is based on the thickness of the
primary lesion (Table 23-2).

TABLE 23-2 Suggested Margins for Surgical Excision of Melanoma

Melanoma Thickness Margin

In situ 5 mm

<1 mm thick 1 cm

>1 mm thick 2 cm

Most tissue defects are primarily closed without skin grafting. If primary biopsy specimens are found to have tumor-negative margins,
no further surgical treatment is required. Primary mucosal melanomas have poor outcomes because disease is usually extensive. Nail
bed lesions require amputation at the distal interphalangeal joint for finger primaries and the interphalangeal joint for thumb
primaries.

Regarding regional disease, the performance of elective regional lymph node dissection for nonpalpable nodes is not routine. The
thickness of the primary melanoma is used to predict the chance of regional lymph node metastases. Thin lesions limited to the
epidermis have a low likelihood of lymph node metastasis, whereas thick lesions invading the subcutaneous fat have a higher likelihood
of spread. Regional lymphadenectomy is usually only performed for thick lesions with a high likelihood of nodal spread.

Because of the morbidity of formal regional lym-phadenectomy and to increase the ability of detect-ing microscopic nodal involvement
with minimal
P
invasiveness, the technique of sentinal lymph node biopsy was developed. Using lymphoscintigraphy, the sentinel node of the first-order
lymph node basin into which the tumor initially drains is identified. In lymphoscintigraphy, technetium and a sulfur colloid are
circumferentially injected around the primary lesion. The technetium drains via lymphatics to the sentinel node, which is identified
using a handheld Geiger Counter. The sentinel node is excised and evaluated microscopically for evidence of metastasis. Sentinel lymph
node biopsy is usually considered for lesions >1 mm in thickness.

In metastatic disease, individuals with a single identifiable metastatic lesion may benefit from surgical resection. However, resection is
generally not recommended for patients with multiple metastatic lesions. Metastatic sites include skin, lung, brain, bone, liver, and the
gastrointestinal tract. Treatment options include surgery, radiation, chemotherapy, and isolated limb perfusion.

SQUAMOUS CELL CARCINOMA


Squamous cell carcinoma (SCC) is the second most common form of skin cancer after BCC. Tumors arise from the skin and the oral and
anogenital mucosa. Multiple predisposing factors for development of SCC have been identified (Table 23-3).

TABLE 23-3 Predisposing Factors for Developing Squamous Cell Carcinoma

Sunlight exposure

Human papilloma virus infection

Immunosuppression (transplant
recipients)

Immunocompromise (HIV infection)

Chronic ulcers

Ionizing radiation (x-rays, gamma rays)

Tobacco use

Scars (burn injury)

PATHOGENESIS
The predominant cause of most SCC is chronic actinic damage that induces the malignant transformation of epidermal keratinocytes. A
similar effect is seen with exposure to ionizing radiation (x-rays and gamma rays). In darkly pigmented individuals, however, most
lesions arise from sites of chronic inflammation, such as osteomyelitis and chronic tropical ulcerations. Tumors also arise at
mucocutaneous interfaces, secondary to tobacco use or human papilloma virus (HPV) infection. Smokers typically present with
ulcerating lip and gum or tongue lesions, whereas invasive cancers of the vulva and penis are seen with HPV infection. Anogenital SCC is
linked to infection with HPV types 16, 18, 31, 33, and 35. Immunosuppressed or immunocompromised individuals—namely, transplant
recipients on immunosuppressive medication or those with HIV/AIDS— have an increased incidence of SCC and an elevated rate of
metastasis. Rarely, tumors arise from old scars (usually sustained secondary to burn injury), which form so-called Marjolin ulcers or burn
scar tumors. As a rule, actinically induced cancers infrequently metastasize, whereas tumors arising from other mechanisms have a
significantly higher rate of metastasis (Table 23-3).

HISTORY
SCC appears as an indurated nodule or plaque, often with ulceration, which has slowly evolved over time. Most lesions are on sun-
exposed areas, such as the face, ears, and upper extremities.

PHYSICAL EXAMINATION
Caucasians exhibit pinkish lesions, whereas darker-skinned individuals have hypo- or hyperpigmented lesions (Fig. 23-4). Regional
lymphadenopathy occurs in 35% of SCCs arising in the lip and mouth. Aberrant keratinization is often seen in SCC, occasionally causing
the growth of cutaneous horns. Therefore, the base of a cutaneous horn should always be examined for the presence of squamous cell
cancer.

Figure 23-4 • Squamous cell carcinoma. This patient has a nodular, ulcerative lesion on his lip.
From Goodheart HP. Goodheart's Photoguide of Common Skin Disorders. 2nd ed. Philadelphia, PA:
Lippincott Williams & Wilkins, 2003.

TREATMENT
The preferred treatment is tumor removal by surgical excision. The remaining defect is closed either primarily for smaller lesions or by
skin grafting or flap reconstruction for larger lesions. Cryosurgery or cautery/ curettage can also be used for small tumors.

PROGNOSIS
The overall cure rate for SCC is 90% after treatment. Tumors other than sun-induced SCC have a higher mortality rate because of the
greater likelihood of metastasis.

KEY POINTS
Basal cell carcinoma lesions appear mostly on sun-exposed areas (head and neck) and are treated by
surgical excision.
The three types of basal cell carcinoma are noduloulcerative, superficial, and sclerosing.
The five signs of melanoma are asymmetric shape, irregular border, mottled color, large diameter, and progressive
enlargement.
Prognosis for primary melanoma tumors is based on tumor thickness: Tumors <0.76 mm have >90% cure rates.
Primary melanoma tumors require excision margins based on tumor thickness, whereas nail bed tumors require
distal joint amputation.
When indicated, sentinal lymph node biopsy confirms regional spread of melanoma.
Sites of melanoma metastasis are lung, brain, bone, and the gastrointestinal tract.
Squamous cell carcinoma of the oral and anogenital mucosa is associated with tobacco use and human papilloma
virus infection.
Squamous cell carcinoma has an increased incidence in immunosuppressed and immunocompromised individuals
and exhibits an elevated risk of metastases.
Squamous cell carcinoma may arise in old burn scars as Marjolin ulcers or burn scar carcinoma, or in sites of
chronic inflammation, such as osteomyelitis and chronic ulcers.
Treatment of squamous cell carcinoma consists of surgical excision.
Overall cure rate for squamous cell carcinoma is 90%.
Authors: Karp, Seth J.; Morris, James P.G.; Zaslau, Stanley
Title: Blueprints Surgery, 5th Edition

Copyright ©2008 Lippincott Williams & Wilkins

> Table of Contents > Part V - Special Topics > Chapter 24 - Plastic Surgery

Chapter 24
Plastic Surgery

Plastic and reconstructive surgery encompasses repair, reconstruction, and aesthetic


improvement of bodily deformity resulting from congenital defects, posttraumatic tissue loss,
and postablative defects, as well as overall enhancement in normal appearance. Reconstruc-tive
surgeons seek to restore form and function while simultaneously achieving aesthetic normalcy.
This chapter focuses on the principles of wound healing and tissue transfer.

THE RECONSTRUCTIVE LADDER


The principles of successful wound care include healing by primary or secondary intention,
vacuum-assisted closure, skin grafts, local flaps, and distant flaps. A conceptual hierarchy has
been established based on a reconstructive approach. The first principle of this approach is that
wounds should be managed with the simplest technique to successfully heal a wound. (Fig. 24-1).
A plastic surgeon is often consulted to determine which rung (technique) of the "reconstructive
ladder" is appropriate from which reconstruction can proceed. Plastic surgeons perform
reconstructive surgery in almost all areas of the body. Therefore, plastic and reconstructive
surgery demands thorough understanding of anatomy. Individualized patient care is critical for
successful reconstructive surgery. Two nearly identical wounds can have solutions that vary
greatly in their technique yet adhere to plastic surgery principles.
Figure 24-1 • The reconstructive ladder.

PRIMARY CLOSURE
This is one of the simplest and most common methods of wound closure. Surgical wounds created
at the time of an operation are usually closed primarily in a linear fashion. At the termination of
a procedure, typically sutures, tissue glues, or metal staples are placed to align the wound
edges. Different types of suturing techniques are frequently used, depending on the dimensions
and specific characteristics of a wound (see Chapter 1, Surgical Techniques, Fig 1-6).

After ensuring complete hemostasis within the wound, the skin edges (if already sharply incised)
are coapted in a tension-free manner. Sutures are carefully placed to evert the skin edges as this
ensures proper healing. It is important to handle skin, dermis, and subcutaneous tissues gently
during closure to prevent further inflammation and scarring. Healthy patients who suffer minimal
tissue injury without significant skin or subcutaneous loss heal rapidly. Wounds that result from a
sharp laceration generally heal faster and with less tissue edema than avulsion and crush
injuries. It is important to distinguish how a wound has occurred. Reconstructive plans should be
mindful of the mechanism of injury and what tissues are missing (e.g., skin, dermis, soft tissue,
cartilage, bone, and so on). Thorough wound irrigation with sterile saline solution is essential
before primary closure.

DELAYED PRIMARY CLOSURE


This method of closure is often chosen when a wound is contaminated, infected, or colonized by
bacteria or when skin and soft tissue are missing such that the margins of the wound cannot be
closed. Infected wounds with necrotic tissue or tissues heavily colonized with bacteria require
sharp debridement followed by dressing changes. Wound care may
be performed for several days to weeks. Wound contraction is a normal component of the P
healing process. This decreases the size and depth of wounds, enabling delayed primary closure.
Wound contamination must be controlled before considering delayed primary closure. Closing a
wound that is not clean results in the development of an infection with further delay to
definitive wound healing.

SECONDARY INTENTION
Wounds with heavy bacterial contamination or tissue devitalization requiring debridement are
often left open and unsutured. The bacterial burden upon the wound is reduced by sharp
debridement so that wound edges are clean and optimized for healing. The defect created by
serial debridement of nonviable tissue gradually fills with beefy red fibrovascular granulation
tissue, which can contract and epithelialize over time to enable complete wound closure. A
major recent advance is the use of the wound vacuum to speed healing. This device consists of a
sponge attached to a suction machine. The sponge is placed in the wound bed attached to
suction tubing, and an airtight seal is created over the assembly. Alterations of the
microenvironment allow more rapid wound healing.

GRAFTS
When tissue loss prevents primary closure and waiting for a wound to heal by secondary
intention will take months or potentially lead to wound infection, a skin graft is considered for
wound closure. Skin grafting is a technique whereby skin is transferred from one area of the
body to another. Most grafts are autografts (patient is both donor and recipient), although other
types of grafts can be used in certain situations (isograft, allograft, or xenograft). Skin grafts
lack an intrinsic blood supply. Therefore, the skin must become revascularized at the site of
insertion, known as the recipient bed. Only vascularized beds are able to be covered with a skin
graft. Once applied to a recipient bed, there are three phases of skin graft adherence. During
the initial phase, the skin graft adheres as a result of fibrin deposition, known as plasmatic
imbibition. The imbibition phase allows the skin graft to remain viable before being
revascularized. This phase lasts approximately 48 hours. The second phase, called inosculation,
involves anastomotic connections between the recipient bed and graft vessels. Simultaneously,
capillaries from the recipient bed grow into the skin graft. Skin grafts are secured to the
recipient bed with a bolster, preventing fluid accumulation and shearing between the
undersurface of the graft and the wound bed. Other tissues can be used as grafts, such as
cartilage, fat, nerve, bone, or tendon.

Skin is the most common tissue graft. Open wounds require barrier reconstitution to prevent
bacterial invasion and fluid loss. Skin is collected as either split-thickness or full-thickness grafts.

Split-thickness skin grafts are thin sheets of skin collected from donor sites; they contain varying
amounts of dermis and the overlying epidermis (Fig. 24-2). A skin graft can be meshed by
creating many tiny incisions; this allows the graft to expand over a larger surface area and better
conform to undulating surfaces. Meshed skin grafts also allow drainage of underlying
wound exudates; however, as a disadvantage, they also leave large areas within the skin graft to P
heal by secondary intention. This leads to a cobblestone appearance and irregular contour. In
contrast, a sheet (skin) graft often has a nicer aesthetic appearance but does not allow serum or
blood to drain through it. Sheet graft survival relies solely on the recipient bed for successful
adherence. Additionally, a sheet graft needs a larger donor site surface area for tissue
collection. Because they are nonmeshed, sheet grafts contract considerably less than meshed
grafts. The anterior thigh is a common donor site for skin graft tissue collection. Donor sites re-
epithelialize from epithelial cells remaining within transected hair follicles and generally require
7 days to repopulate. Full-thickness skin grafts contain the entire dermis and epidermis. They
are used to close surgical defects when durability, color match, appearance, and lack of wound
contraction are required. The groin, flank, and postauricular area are common donor sites. Full-
thickness donor sites require primary closure, whereas split-thickness skin grafts epithelialize
(see Color Plate 18).
Figure 24-2 • Cross section of skin showing various thickness of
split-thickness grafts.

LOCAL FLAPS
Local flaps are blocks of tissue that maintain an intrinsic blood supply and are derived from
tissue immediately adjacent to the recipient bed. A local tissue flap either advances (V-Y
advancement flap, rectangular advancement flap; Fig. 24-3 and 24-5) or pivots (rotation flap or
transposition flap; Fig. 24-4). It is important to distinguish and define a graft from a flap. A graft
does not have its own blood supply, whereas a flap does (Fig 24-5A, B). When transferring a graft
to cover a wound, it must be revascularized to survive. A flap is transferred to cover a defect
with its blood supply intact.
Figure 24-3 • (A) Advancement flap. (B) V-Y advancement flap.
Figure 24-4 • (A) Rotation flap. (B, C, D, E) Transposition flap.
From Taylor J. Blueprints Plastic Surgery. Boston, MA: Blackwell
Science, 2004:Fig. 5.

Figure 24-5 • Volar V-Y advancement flap; volar V-Y flap. (A)
Preoperative outline of the skin incision used to create a volar skin
flap in preparation for distal advancement. The tip of the V incision
should be at the DIP joint flexion crease. (B) The flap has been
advanced distally to cover the tip defect. The resultant defect has
been closed, converting the V defect to a Y.
From Strickland JW, Graham TJ. Master Techniques in Orthopaedic
Surgery: The Hand. 2nd ed. Philadelphia, PA: Lippincott Williams &
Wilkins, 2005.

When dealing with scar contracture that affects a patient's function and aesthetic appearance,
plastic surgeons can use the Z-plasty technique to lengthen and alter the direction of the scar. A
Z-plasty uses two opposing transposition flaps that switch places and gain length along the
central axis of the original scar. Skin and tissue adjacent to the transposed flaps are recruited to
lengthen and blend the scar with the surrounding tissues. A Z-plasty changes the direction of the
central limb of scar by 90 degrees (Fig. 24-6). This technique improves the contour and aesthetic
appearance of scars by camouflage. Scars become softer and less pigmented once tension is
relieved via
a Z-plasty. The actual gain in scar length depends on the angle of the flaps, the length of the P
central limb, and the quality of the surrounding tissues.
Figure 24-6 • Z-plasty.
Taylor J. Blueprints Plastic Surgery. Boston, MA: Blackwell Science,
2004;Fig. 5.

REGIONAL FLAPS
When adjacent tissue is inadequate, a flap is raised from a distant location and inset for wound
closure. Flaps include adipocutaneous, fasciocutaneous, myocutaneous, osteocutaneous, or
muscular flaps (Figs. 24-7 and 24-8). Flaps that are interpolated over cutaneous skin require
pedicle transection once the flap becomes vascularized from the recipient tissue bed. The
classic operation of this type is the Tagliacotian operation—devised by the Italian surgeon
Gasparo Tagliacozzi (1546 to 1599)—in which a forearm flap is transferred to the nose (Italian
rhinoplasty). Other examples are groin flaps to the hand and cross leg flaps. Prolonged awkward
positioning is often required postoperatively until the pedicle is eventually transected.

Figure 24-7 • Cross section of soft tissues used for flaps.

P
Figure 24-8 • Lateral arm flap. (A) Debridement of ulcer and
surrounding induration creating exposure of a metacarpal and
extensor tendon. (B) Flap borders outlined slightly larger than the
size of the defect. (C) Flap in place 6 months after operation,
before final defatting procedure.
From Strickland JW, Graham TJ. Master Techniques in Orthopaedic
Surgery: The Hand. 2nd ed. Philadelphia, PA: Lippincott Williams &
Wilkins, 2005.

A common pedicled musculocutaneous flap used for breast reconstruction is the transverse
rectus abdominis myocutaneous flap. The contralateral rectus abdominis muscle is mobilized,
with the superior epigastric artery left intact to supply the large attached skin paddle. The bulky
paddle and muscle pedicle are tunneled superiorly and inserted into the breast defect, while the
abdominal donor site is closed (abdominoplasty; Fig. 24-9).
Figure 24-9 • Transverse rectus abdominis myocutaneous flap.
From Taylor J. Blueprints Plastic Surgery. Boston, MA: Blackwell
Science, 2004;Fig. 7.

P.187 P
TISSUE EXPANSION
Tissue expanders are placed beneath the dermis in a subcutaneous pocket. Their purpose is
generally to recruit skin. This is achieved via gradual expander inflation over weeks to months to
expand the skin superficial to the expander. The creation of additional skin surface area can be
used for breast reconstruction, scalp reconstruction, or any defect requiring local tissue transfer
(Fig. 24-10). Benefits of tissue expansion include good skin color match, as well as preservation
of hair and sensation. Tissue expansion can be complicated by pain, infection, and contour
deformity of surrounding structures.
Figure 24-10 • Tissue expansion with silicone balloons.

From Taylor J. Blueprints Plastic Surgery. Boston, MA: Blackwell


Science, 2004;Fig. 8.

FREE TISSUE TRANSFER


Free tissue transfer is used to reconstruct defects lacking local and regional pedicle flap options.
Advances in microsurgery have enabled distant tissue transfer to become a reliable and
predictable reconstructive option. Tissue (i.e., skin, subcutaneous fat, and muscle) can be
transferred to a distant site based on its feeding nutrient artery. The free flap is collected from a
distant donor site to cover a complex wound often with exposed vessels, bone, or other critical
structures. The arterial blood supply and vena comitantes of a free flap are coapted to a target
artery and vein within or adjacent to the defect using microvascular techniques (Figs 24-11A, B,
and C).
Figure 24-11 • Latissimus dorsi flap. (A) A 42-year-old woman with
extensive trauma to the right forearm and hand after a riding
lawnmower injury (B) Muscle elevated to tendon insertion and final
pedicle identification commencing. (C) Muscle flap applied to the
upper limb, vascular hookup performed, and muscle trimming and
insertion completed.

From Strickland JW, Graham TJ. Master Techniques in Orthopaedic


Surgery: The Hand. 2nd ed. Philadelphia, PA: Lippincott Williams &
Wilkins, 2005.

KEY POINTS
Goals of plastic surgery are to restore form and maximize function.
Complex wounds require thoughtful individualized solutions.
Methods of wound closure are based on the principles of the
reconstructive ladder.
Grafts do not have a blood supply; they become vascularized from a recipient
bed.
A flap has its own blood supply, generally rotated or advanced into complex
wounds.
Free tissue transfer involves collection of a distant flap with its arterial and
venous vessels.
Authors: Karp, Seth J.; Morris, James P.G.; Zaslau, Stanley
Title: Blueprints Surgery, 5th Edition

Copyright ©2008 Lippincott Williams & Wilkins

> Table of Contents > Part V - Special Topics > Chapter 25 - Orthopedic Surgery

Chapter 25
Orthopedic Surgery

OSTEOARTHRITIS

BACKGROUND
Osteoarthritis (OA) is the single most common disease of the joints; as such, it is one of the most impor-tant causes of morbidity in the
United States. Loss of articular cartilage in the synovial joint is the primary pathologic lesion. This loss can be due to repetitive load-
bearing stress or intrinsic properties of either the cartilage or bone or in the body's ability to repair these structures. Joints commonly
involved include the first carpometacarpal joint, hip, knee, and spine. Risk factors include age, obesity, and female sex. Secondary OA
can occur after major trauma to the joint; repetitive stress; prior inflammatory joint disease; metabolic disease, including ochronosis,
hemochromatosis, and Wilson disease; and endocrine disease, including acromegaly, diabetes, and hyperparathyroidism.

HISTORY AND PHYSICAL EXAMINATION


Patients are usually older than 55 years. Typical complaints include a deep ache in the affected joint that is exacerbated by activity.
Stiffness is common, especially after inactivity, but usually resolves with use. There are no systemic symptoms. Although OA is not
considered an inflammatory arthritis, inflammation does play a significant role in the development of asso-ciated pain and swelling.

Physical examination reveals limitation of motion secondary to pain. Localized tenderness and soft tissue swelling may be present.
Motion of the joint may produce bony crepitus. As the disease progresses, there may be gross deformities of the joint, with loss of
motion.

Hip OA may be experienced as groin pain. On examination, internal rotation is often the first motion to become painful and restricted.

Knee OA may result in stiff knees that are more painful with prolonged sitting, walking, and climbing stairs.

First carpometacarpal OA often results in stiffness and pain in the joint. On examination, the "grind test," in which the joint is
compressed, is positive.

DIAGNOSTIC EVALUATION
Radiographic examination reveals narrowing of joint spaces (Figs. 25-1 and 25-2). Subchondral cysts, bone sclerosis, and osteophytes
may be present. In primary OA, laboratory values are normal. Specific lab tests may be useful to diagnose causes of secondary OA.
Synovial fluid usually reveals a mononuclear leukocytosis; this test may be useful to exclude other diagnoses, including a septic joint.
Figure 25-1 • X-ray appearance of osteoarthritis of the knee.

Reproduced with permission from Duckworth T. Lecture Notes on Orthopaedics and Fractures. 3rd
ed. Cambridge, MA: Blackwell Science, 1995.
Figure 25-2 • X-ray appearance of osteoarthritic hip.
Reproduced with permission from Duckworth T. Lecture Notes on Orthopaedics and Fractures. 3rd
ed. Cambridge, MA: Blackwell Science, 1995.

It is important to note that radiographic findings of osteoarthritis are not always correlated with symptoms. A patient with significant
OA on x-ray may be symptom-free. The reciprocal is also true: a patient with minimal x-ray findings of OA may have significant
symptoms. The patient with OA must be treated, not the film.

TREATMENT
Therapy for osteoarthritis should be based on the severity of the symptoms. In general, exercises to stretch and strengthen the muscles
surrounding the
joint are critical. Joints face an enormous amount of daily stress. Descending stairs, for example, results in the knee bearing five to six P
times the body weight load. Weak or tight muscles result in excessive forces being placed through the joint itself, rather than being
relieved by the muscles.

No pharmacologic interventions can reverse the lesions of osteoarthritis, but nonsteroidal anti-inflammatory agents, as well as
analgesics (e.g., acetaminophen), can provide symptomatic relief for pain.

Nutritional supplements such as glucosamine and chondroitin sulfate have received much attention for the prevention and treatment of
osteoarthritis. Their usage remains controversial. However, because of their low side-effect profile (in contrast with nonsteroidal anti-
inflammatory drugs), their use has become more widespread. Superficial joints, such as those of the fingers and toes, may benefit from
topical analgesics.

Intra-articular corticosteroids can be extremely effective for pain. The number of corticosteroid injections that can be performed per
joint remains controversial. Some physicians advocate a total of three corticosteroid injections in the life of a joint, whereas others
argue that three injections per year are safe.

Hyaluronic acid injections may also be of benefit. These injections are approved by the U.S. Food and Drug Administration for knee OA,
and they are being used with increasing frequency, off-label, for hip OA. Some studies are also investigating their use for ankle,
shoulder, and elbow OA.

Surgical treatment should be reserved for patients with debilitating pain or compromised quality of life after nonsurgical options are
exhausted. In patients with mild disease, particularly younger patients who want to remain more active, osteotomy may produce
significant pain relief. In the knee, an osteotomy is not an option if the patient is unable to flex the knee past 90 degrees or if he or she
has more than 15 to 20 degrees of valgus deformity.

Total joint arthroplasty for hip and knee OA has excellent results in patients with more advanced disease. In the hip, a
polymethylmethacrylate cemented or cementless prosthesis may be used. A cemented prosthesis has the advantage of allowing the
patient full weight bearing and quicker healing immediately after the procedure. Cementless prostheses have the potential advantage
of ultimately achieving longer life and durability once they have matured. The disadvantage is that the patient may have greater
postoperative pain and must remain toe-touch weight bearing (putting less than 10% of body weight on the affected limb) for at least 6
to 8 weeks. In general, a cemented prosthesis is recommended for an older, less active patient, and a cementless prosthesis is
recommended for a younger, more active patient. A hemiarthroplasty in the hip is another option. In contrast to a total joint
arthroplasty, which involves replacing the femoral head and the acetabulum, a hemiarthroplasty involves only replacing the femoral
head.

The most common surgical approach for a total hip replacement is posterior. After a posterior approach,
hip precautions include (1) no hip flexion beyond 90 degrees, (2) no hip adduction past neutral, and (3) no hip internal rotation beyond P
neutral.

In the knee, cemented, cementless, and hybrid prostheses are available. However, cement fixation is by far the most common. After a
cemented total knee replacement, patients should expect to gradually return to their activities in 1 to 3 months.

When considering any major surgery, it is vital to consider anticoagulation after the procedure. This is particularly true with total knee
and hip replacements. After a total knee replacement, as many as 64% of patients may develop a clot if not anticoagulated. Only 6% to
24% may develop a clot if properly anticoagulated. In the hip, >50% of patients who do not receive adequate anticoagulation may
develop a clot. Anticoagulation options include warfarin (keeping the international normalized ratio between 2 and 3), aspirin 325 mg
orally twice per day, subcutaneous heparin 5,000 U twice per day, low molecular heparin, and possibly an inferior vena cava filter for
patients who are not candidates for anticoagulation.
JOINT PAIN
It is important to distinguish between acute and chronic causes of pain. Critical questions include history of trauma, swelling, decreased
range of motion, and problems with weight bearing. Positions or circumstances that alleviate or exacerbate the pain may yield clues to
the diagnosis.

On physical examination, a careful assessment for swelling, joint effusion, range of motion, deformity, tenderness, and instability
should be made.

KNEE PAIN: COMMON DIAGNOSES


Meniscal Injury
Meniscal injuries are more common in men but can occur in women. Injury to the medial meniscus is more common than injury to the
lateral meniscus. Pain is the most common symptom. Patients also may complain of knee locking or clicking. Effusions may be present,
and weight bearing may be difficult. Examination usually reveals joint line tenderness. A McMurray test is often positive. In this test
(Fig. 25-3), the patient is in the supine position. The examiner flexes the knee while palpating the joint line. Using the ankle as a
fulcrum, the leg is externally rotated and a valgus stress is applied as the knee is extended. A palpable or audible click, or recreation of
symptoms in the knee, is considered a positive test result. The grinding and distraction test of Apley is also useful (Fig. 25-4). In this
test, the patient lies prone with the knee flexed to 90 degrees. Using the ankle as a fulcrum, again, the examiner first internally and
externally rotates the leg while compressing the tibia into the table. The examiner then internally and externally rotates the leg while
distracting the tibia. If pain is produced while grinding but not while distracting, the meniscus is implicated as the cause of
pain. If pain is produced during both distraction (when the meniscus is unloaded) and grinding, the ligaments are more likely involved. P

Figure 25-3 • The McMurray test is performed with the leg externally rotated and applying a
valgus stress to test the medial meniscus.

Reproduced with permission from Gross JM, Fetto J, Rosen E. Musculoskeletal Examination. 2nd
ed. Malden, MA: Blackwell, 2002.
Figure 25-4 • The grinding/distraction test of Apley. The tibia is compressed first (A), then
rotated while compression force is maintained (B). Distraction with rotation tests the collateral
ligaments, whereas compression with rotation tests the menisci.

Reproduced with permission from Gross JM, Fetto J, Rosen E. Musculoskeletal Examination. 2nd
ed. Malden, MA: Blackwell, 2002.

Magnetic resonance imaging (MRI) is accurate and sensitive for meniscal injury. Not all meniscus lesions require surgery. Conservative
therapy is indicated, particularly if the tear is ≤10 mm or partial thickness or if a radial tear is ≤3 mm. However, even larger tears may
respond to conservative therapy. If locking is present and significant, surgical intervention should be considered sooner. Surgical options
include arthroscopic meniscectomy and meniscus repair. For tears in the vascular region (in the peripheral part of the meniscus),
arthroscopic meniscus repair with fixation using the outside-in, inside-out, or all-inside method is recommended.

Cruciate Ligament Injury


The cruciate ligaments stabilize the knee to translational motion. Acute pain in the setting of trauma and subsequent swelling should
alert the examiner to this possibility. Typically, a twist or hyperextension trauma causes these lesions. The pain may improve after the
initial injury, but chronic pain may develop. The anterior cruciate ligament (ACL) is most commonly torn. Between 30% and 50% of
people will report hearing a popping sound at the time of tearing their ACL. The most sensitive diagnostic maneuver for ACL is the
Lachman test (Fig. 25-5). In this test, the knee is flexed 20 to 30 degrees and the examiner applies an anteroposterior glide. A few
degrees of anteroposterior glide may be normal. It is important to always compare side-to-side findings. A loose end point or excessive
glide is a positive finding for an ACL tear.
Figure 25-5 • The position of the examiner and patient for the Lachman test. It is important that
the patient be relaxed for this test.
Reproduced with permission from Gross JM, Fetto J, Rosen E. Musculoskeletal Examination. 2nd
ed. Malden, MA: Blackwell, 2002.

The medial collateral ligament is evaluated by applying a valgus stress and comparing glide side-to-side (Fig. 25-6). The posterior
cruciate ligament and lateral collateral ligament are likewise evaluated by stressing those ligaments.
Figure 25-6 • Valgus strain (medial gapping).
Reproduced with permission from Gross JM, Fetto J, Rosen E. Musculoskeletal Examination. 2nd
ed. Malden, MA: Blackwell, 2002.

Treatment is typically conservative, focusing on strengthening the surrounding muscles and gradually returning to activity. Depending on
the severity of injury, the ACL may be repaired by arthroscopic allograft or autograft. The allograft, taken from a cadaver, has the
advantage of not requiring tissue collection of the graft from the patient. Allografts are irradiated to greatly reduce the chance of
infection. Autografts are performed by either taking a segment of the patient's hamstrings or patella tendon. The disadvantage of these
procedures is primarily the occurrence of increased postoperative wound pain. Both surgical approaches have similar long-term
outcomes.

Medial collateral ligament and posterior cruciate ligament injuries are generally treated conservatively unless the ligament is severely
torn.

P
Patellofemoral Syndrome
Patellofemoral syndrome is common in young adults. The pain is generally anterior. A positive "theatre sign" is characteristic. When
positive, the patient has trouble sitting for a long period of time (such as at a theatre) and must extend the knee (placing his or her
foot into the aisle) to relieve the discomfort. Pain is also exacerbated by running and ascending/descending stairs. Tenderness is usually
present under the patella. The Q-angle, which is the angle formed by a line drawn from the anterior superior iliac spine to the mid
patella and transected by a line drawn from the mid patella to the tibial tubercle with the knee in full extension, is usually increased
(>17 degrees in women and >14 degrees in men; Fig. 25-7).
Figure 25-7 • Measurement of the Q-angle.
Reproduced with permission from Gross JM, Fetto J, Rosen E. Musculoskeletal Examination. 2nd
ed. Malden, MA: Blackwell, 2002.

The problem is primarily caused by patellar mis-alignment and abnormal tracking. Classically, the patella tracks excessively laterally.
Treatment focuses on strengthening the quadriceps, particularly the vastus medialis oblique, and stretching the iliotibial band. Patellar
taping or bracing may also be somewhat helpful. Surgical correction is rarely indicated but may be used when conservative care is not
sufficient and an identifiable anatomic abnormality is present, such as isolated lateral patellar tilt that may be arthroscopically
released.

SHOULDER PAIN: COMMON DIAGNOSES


A variety of structures within the shoulder can be injured or inflamed, but most problems respond to conservative therapy.

Impingement Syndrome
Impingement syndrome includes rotator cuff tendonitis and subacromial bursitis. These two conditions often coexist and are treated in
much the same way. To diagnose impingement syndrome, Neer and Hawkins' tests are useful. In the Neer test, the examiner stabilizes
the patient's scapula with one hand and internally rotates and flexes the shoulder with the other. As the shoulder is elevated, the
greater tuberosity of the humerus abuts the coracoacromial arch, placing pressure on the rotator cuff tendon. Result for this test is
considered positive when symptoms are reproduced. The same maneuver but with the shoulder in external rotation often produces less
or no pain. In the Hawkins' test, the patient's shoulder is abducted to 90 degrees in the plane of the scapula and the elbow is flexed to
60 to 90 degrees. The shoulder is then placed firmly into internal rotation (Fig. 25-8). When this reproduces pain, the test result is
considered positive for impingement syndrome.
Figure 25-8 • The supraspinatus impingement test (Hawkins' test) being performed.
Reproduced with permission from Gross JM, Fetto J, Rosen E. Musculoskeletal Examination. 2nd
ed. Malden, MA: Blackwell, 2002.

The best test for impingement syndrome is to inject 3 to 5 mL of 1% lidocaine into the subacromial space and repeat the Neer and
Hawkins' tests. With the lidocaine injected, these maneuvers should be significantly less painful. If so, the patient likely has
impingement syndrome.

Impingement syndrome responds well to physical therapy that focuses on scapular stabilization and rotator cuff strengthening exercises.
A subacromial corticosteroid injection can also be very effective and help speed recovery.

P
Biceps Tendonitis
The biceps tendon crosses the shoulder joint and may become inflamed where it traverses the bicipital groove. This is best identified by
tenderness to palpation and a positive Speed test result. In the Speed test, the patient's shoulder is flexed to 90 degrees with the elbow
extended. The patient then resists the examiner who applies an inferior force to the patient's arm. Reproduction of pain is consistent
with bicipital tendonitis.

Glenohumeral Labral Tear


A labral tear may be identified on physical examination by the O'Brien test. In this test, the patient's shoulder is flexed, adducted, and
externally rotated. An inferior force is applied by the examiner. The arm is then placed in internal rotation and an inferior force is again
applied (and resisted by the patient). If symptoms are reproduced when the shoulder is internally rotated (thumb facing down) but not
in external rotation (thumb facing up), the test is considered positive for a labral tear. An MRI is the best way to evaluate for a labral
tear. Not all labral tears require surgery. Conservative care includes physical therapy and, in some cases, an intra-articular
corticosteroid injection.

Rotator Cuff Tear


The rotator cuff is a collection of the tendons of the supraspinatus, infraspinatus, subscapularis, and teres minor. The most common
tendon to tear is the supraspinatus. On examination, a supraspinatus tear will result in weak abduction, particularly from 0 to 30
degrees. MRI is sensitive and specific for rotator cuff injuries (Fig. 25-9). Surgery is an option if conservative measures fail.
Figure 25-9 • Magnetic resonance imaging of normal rotator cuff (*) and surrounding structures.
H, humerus, G, glenoid.

Reproduced with permission from Gross JM, Fetto J, Rosen E. Musculoskeletal Examination. 2nd
ed. Malden, MA: Blackwell, 2002.

BACK PAIN
Back pain is an epidemic in the United States, causing chronic disability in 1% of the population, with a cost of approximately $50 billion
annually. Back pain can be divided into acute pain from a muscle strain or spasm and subacute or chronic pain that is more likely a
result of intervertebral disc irritation, zygapophysial (facet) joint pain, or sacroiliac joint pain. A radiculitis can also result if a nerve
root is irritated. A radiculitis characteristically involves pain radiating into the lower extremity.

FACET JOINT PAIN


The facet joints in the spine are synovial joints (similar to the knee joint). These joints can become arthritic, just like the knee. These
joints are a common source of lower back pain. Pain is usually exacerbated by trunk extension and oblique extension. Treatment
options include physical therapy, facet joint injections, and radiofrequency neurotomy to essentially sever the medial branches of the
dorsal rami that innervate the involved joints.

DISCOGENIC PAIN
The disc is the most common cause of chronic lower back pain in the younger population and is also a common cause of pain in older
populations. A herniated disc is not synonymous with discogenic back pain. When discs cause pain, it is because the annulus of the disc
has torn internally and the nucleus pulposus (which is full of inflammatory proteins) is irritating the pain receptors in the outer third of
the annulus. A painful disc may appear normal on MRI. Reciprocally, a herniated disc may not be the source of back pain. A herniated
disc is more commonly associated with nerve root impingement or irritation, which would result in a radiculitis, causing electric,
radiating pain into the lower extremity.

Discogenic pain is typically worse with trunk flexion. Prolonged sitting may make the pain worse. Though somewhat controversial,
definitive diagnosis is accomplished using provocative discography in which dye is injected under x-ray guidance into the disc. When
this reproduces the patient's pain at low injection pressures, the disc is considered positive. Postdiscography computed tomography will P
often reveal the extent of the torn annulus (when present).

Treatment includes physical therapy that focuses on extension-based lumbar stabilization and stretching and strengthening the
surrounding muscles. Fusion is reserved for patients for whom more conservative measures have failed.
NEOPLASM
Metastatic cancer must always be included in the differential diagnosis of back pain. Cancer pain in the back tends to be worse at
night, and may wake the patient up from sleep. X-rays, bone scan, and MRI all may be used as part of the workup, depending on the
index of suspicion. Though much less common, primary bone tumors in the spine can also occur.

INFECTION
Osteomyelitis of the spine most commonly occurs in debilitated patients or intravenous drug abusers and is usually associated with some
type of systemic infection. Patients will usually have tenderness at the site and with movement. Pain is typically constant, and signs of
systemic infection are present. The sedimentation rate is often elevated. Plain films may be negative initially, MRI is usually diagnostic,
and a tagged white cell scan may light up the affected area.

HIP FRACTURE
Hip fracture is the most common fracture causing hospital admission. Approximately 300,000 hip fractures occur each year in the
United States. The generally advanced age of patients with this problem leads to a staggering 50% 1-year mortality rate after injury. Hip
fractures are twice as common in women. Three general types of fracture occur: fractures of the femoral neck, intertrochanteric
fractures, and subtrochanteric fractures (Fig. 25-10).The first two are by far the most common and usually occur in older adults as a
result of a relatively low-impact type of injury.

Figure 25-10 • Diagram of common points of femoral fracture.


Reproduced with permission from Uzelac A, Davis RW. Blueprints Radiology. 2nd ed. Philadelphia,
PA: Lippincott Williams & Wilkins, 2006.

The typical history involves an older adult patient who falls and has subsequent hip, groin, or knee pain. Weight bearing is often
difficult. On examination, patients with femoral neck fractures may have a shortened and externally rotated extremity. Hip motion may
be painful. Diagnosis is usually confirmed with plain radiograph films (Fig. 25-11). MRI should be obtained if x-rays appear normal but
clinical evidence is suggestive
of possible fracture (x-rays may miss up to 67% of acute hip fractures, according to one study). P
Figure 25-11 • Hip fracture. Intertrochanteric fracture in the right hip of an 83-year-old woman
who fell after getting out of bed.

Courtesy of Cedars-Sinai Medical Center, Los Angeles, CA.

Treatment is based on comorbidities and degree of fracture (Tables 25-1 and 25-2). If the acetabulum is involved in the fracture, total
hip replacement must be considered.

TABLE 25-1 Garden Classification and Treatment

Garden
Findings     Treatment
Type    

I Femoral head valgus Open reduction and internal


impaction fixation

II Complete, nondisplaced Open reduction and internal


fracture fixation

III Varus displacement of Open or closed fixation versus


femoral head total hip replacement

IV Complete loss of Open or closed fixation versus


fragment continuity total hip replacement

TABLE 25-2 Jensen Classification of Intertrochanteric Fractures

Type         Findings Stability

I Nondisplaced two-fragment fracture Stable

II Displaced two-fragment fracture Stable

III Three-fragment fracture without posterolateral Unstable


support

IV Three-fragment fracture without medial support Unstable

V Four-fragment fracture without posterolateral Unstable


and medial support

Both stable and unstable intertrochanteric fractures are typically treated with open reduction and internal fixation using the sliding
screw hip implant. Fracture healing time after surgery is approximately 12 weeks.

It is critical to properly anticoagulate patients after a hip fracture. Failure to do so may lead to a lower-extremity deep venous
thrombosis and/or fatal pulmonary embolism in 40% to 83% of patients. These numbers are greatly reduced (to 4% to 38%) in patients
who receive appropriate anticoagulation. High-risk patients, or patients who are not candidates for anticoagulation, may require an
inferior vena cava filter.

SEPTIC JOINT CONSIDERATIONS


A septic joint requires appropriate, prompt attention. Clinical signs and symptoms of a septic joint include fevers, chills, and a hot,
red, swollen joint. A low index of suspicion should be used for septic joint, because failure to diagnose and treat this condition quickly
may result in destruction of the joint. If suspected, the joint should be aspirated and the fluid sent for analysis. Blood tests should also
be performed. If clinical suspicion is high, treatment should be started before lab results have been obtained. Treatment includes
intravenous and/or oral antibiotic treatment.

Laboratory studies should include a complete blood cell count with differential and erythrocyte sedimentation rate. If positive,
complete blood cell count may show increased white blood cell count (>12,000/mL) with a left shift. Erythrocyte sedimentation rate is
typically elevated (>40 mm/h). Other laboratory tests may include a Lyme test and blood cultures.

If suspected, a urethral or cervical culture may be obtained to evaluate for Neisseria gonorrhoeae. The fluid aspirate should be sent for
Gram stain, culture, cell count, and crystal analysis. Table 25-3 presents results analysis.

TABLE 25-3 Synovial Fluid Classification


Quality Reference Noninflammatory Inflammatory Septic Crystal
Range

Volume <3.5 mL >3.5 mL >3.5 mL >3.5 mL <3.5 mL

Viscosity High High Low Variable Low

Color Clear Straw-yellow Yellow Variable Yellow-milky

Clarity Transparent Transparent Translucent Opaque Opaque

WBC <200/mL 200– 2,000– Often 500–


2,000/mL 75,000/mL >100,000/mL 200,000/mL

PMN <25% <25% >50% >75% <90

Culture Negative Negative Negative Often positivea Negative


result

Mucin Firm Firm to Friable Friable Friable


clot friable

Glucose ~Blood ~Blood Decreased Very decreased Decreased

Crystals Negative Negative Negative Negative Present

aSynovial
fluid culture results are positive in 85% to 95% of nongonococcal arthritis cases and
approximately 25% of gonococcal arthritis cases.

WBC, white blood cell count; PMN, polymorphonuclear neutrophils.

Modified from Schumacher HR. Pathologic findings in rheumatoid arthritis. Arthritis: An Illustrated
Guide to Pathology, Diagnosis and Management.
Philadelphia, PA: JB Lippincott, 1988. Available at:
http://www.emedicine.com/orthoped/topic437.htm .

Corticosteroid injection should NOT be administered if sepsis is suspected.

If infection is confirmed, drainage in combination with continued intravenous antibiotics is the treatment of choice. Drainage may be
attempted by percutaneous aspiration; however, surgical drainage is often required if percutaneous aspiration is not adequate.

P
KEY POINTS
Orthopedic injuries are a tremendous cause of economic loss, morbidity, and mortality in the United States.
Osteoarthritis is caused by loss of articular cartilage and is best managed conservatively, unless symptoms
are debilitating.
X-ray findings of osteoarthritis do not necessarily correlate with symptoms. The patient must be treated, not
the film.
Causes of knee pain include meniscal injury, cruciate ligament injury, and patellofemoral syndrome. Magnetic
resonance imaging is the study of choice to delineate the anatomy and to help guide treatment.
In the United States, back pain is an epidemic, with an economic cost of $50 billion per year. The most common
causes of back pain include discogenic and facet arthropathy.
Discogenic lower back pain is not necessarily caused by a herniated disc, but rather by internal derangement of the
disc, which may appear normal on magnetic resonance imaging.
Conservative therapy for back pain is usually best, unless there are neurologic symptoms or intractable pain.
Hip fracture is a common lesion in older adults and carries high mortality at 1 year.
It is important to keep a high index of suspicion for a septic joint. Failure to recognize and promptly treat this
pathology may lead to rapid destruction of the joint.
Authors: Karp, Seth J.; Morris, James P.G.; Zaslau, Stanley
Title: Blueprints Surgery, 5th Edition
Copyright ©2008 Lippincott Williams & Wilkins

> Table of Contents > Part V - Special Topics > Chapter 26 - Organ Transplantation

Chapter 26
Organ Transplantation

OVERVIEW
Organ transplantation is one of the great achievements of 20th-century medicine. Fueled by technological
advances in immunology and surgical technique, transplantation of the liver, kidney, small bowel, heart, and
lung is possible with good outcomes. Using various protocols, it is possible to perform transplantation when
patient and donor do not have compatible blood types, and even if the recipient is sensitized to the donor.
Researchers at various centers are also experiment-ing with hand, face, and uterus transplantation. The
decision to perform transplantation must take into account the risk of the surgery as well as long-term
immunosuppression.

BASIC SCIENCE
The key to successful organ transplantation, after the technical aspects have been performed successfully, is
the ability to prevent the recipient immunologic response from destroying the graft. In most cases, the donor
and recipient blood type will be matched, and control of the host's response to the donor's major
histocompatibility (MHC) antigens determines the success or failure of organ transplantation. MHC antigens
are coded by a single chromosomal complex. In humans, the MHC is named the HLA antigen (human leukocyte
antigen), which is located on the short arm of chromosome 6. HLA antigens are classified according to their
structure and function. Class I antigens are present on virtually all nucleated cells in the human body and act
as targets for cytotoxic T cells. Class II antigens are located on B cells, monocytes, macrophages, activated T
cells, and other antigen-presenting cells. The rejection reaction of a transplant recipient directed against
mismatched donor HLA antigens is a complex event that involves the actions of cytotoxic T cells, activated
helper T cells, B lymphocytes, activated macrophages, and antibodies. The reaction is primarily cellular in
nature and is T cell–dependent. Class I antigens stimulate cytotoxic T cells, directly causing donor tissue
destruction. Class II antigens activate helper T cells, which, along with activated cytotoxic T cells, elaborate
interleukin-1 (IL-1) and IL-2. IL-1 and IL-2, in turn, further activate macrophages and antibody-releasing B
cells.

Although most rejections are cell-mediated, humoral rejections are also possible. The exact pathogenesis of
humoral rejection is not well understood. When it occurs early after transplantation, it is generally due to
preformed antibodies against class I antigens in the recipient. These antibodies are commonly acquired via
blood transfusions, pregnancy, or prior transplantations. If this occurs in the period immediately after the
transplantation procedure, it is termed "hyperacute rejection." To avoid this, cross-matching of the recipient's
serum against the donor's lymphocytes is performed to confirm the absence of preexisting antibodies against
donor tissue antigens. Types of rejection are listed in Table 26-1.
TABLE 26-1 Classification Criteria for Allograft Rejection Responses

Type Time Course Target Response

Hyperacute Minutes to Vessels Humoral


hours

Acute Early after Parenchyma/vessels Cellular/humoral


transplantation

Chronic Late after Parenchyma/vessels Cellular/humoral


transplantation

Immunosuppressive regimens use a combination of agents among a number of classes. In the past, most
patients were treated with three drug regimens, but more and more transplantations are being performed
with corticosteroids only in the immediate postoperative period. Generally, patients will receive an antibody
as an induction agent for a few days at the time of the transplantation. In general, antibodies bind white
blood cells and trigger a destructive immune reaction. The most common agent is rabbit anti-thymocyte
globulin (Thymoglobulin), which profoundly depletes T cells. Also common are basiliximab and daclizumab, P
both of which are humanized antibodies to the IL-2 receptor on T cells.

Currently, the foundation of maintenance immunosuppression is a calcineurin inhibitor (either tacrolimus or


cyclosporine). This agent binds to immunophilins and inhibits calcineurin activity, which is necessary for the
transcription of genes that activate T cells, including IL-2, IL-3, IL-4, and interferon. Unfortunately,
nephrotoxicity of these medicines is associated with high rates of renal failure. The major alternative is
rapamycin, which does not seem to have nephrotoxicity but can impair wound healing, increase lipid levels,
and cause pulmonary toxicity and edema. Most regimens include an antimetabolite, such as mycophenolate
mofetil. Mycophenolate mofetil is rapidly converted to the morpholino ethyl ester of mycophenolic acid,
which inhibits inosine monophosphate dehydrogenases, blocking proliferation of T and B lymphocytes and
inhibiting antibody formation and the generation of cytotoxic T cells. Mycophenolic acid also down regulates
the expression of adhesion molecules on lymphocytes. Other drugs in this category include azathioprine, a
purine synthesis inhibitor.

Use of corticosteroids, once the mainstay of immunosuppression, is decreasing rapidly. Given the significant
long-term complications associated with corticosteroids (e.g., diabetes, heart disease, edema, avascular
necrosis of the hip, and easy bruisablity), alternatives have been actively sought. Current data suggest that if
an induction agent is included at transplantation, then maintenance corticosteroids are not required.
Corticosteroids alter the transcription and translation of several genes responsible for cytokine synthesis; they
inhibit T-cell activation by blocking IL-1, IL-2, IL-6, and interferon synthesis; and they have local anti-
inflammatory effects.

It is unusual to lose a graft from acute rejection. Antibody therapy that profoundly depletes T cells halts most
rejections. The most potent agents are antithymocyte globulin and OKT3, an antibody to CD3 which binds to
and depletes T cells.

DONORS
Organs can be used from living people or cadavers. Live donors for kidney transplants are routine, and the
majority of kidneys come from this source. The risk of death to the donor is approximately three in 10,000,
and complication rates are in the range of 2% to 3%. Live donors can be used for liver transplantation, but
donor risk is greater, with a mortality rate of approximately 0.2%, in addition to a biliary complication rate
that is much higher. Adult-to-child transplantation procedures usually use the left lateral segment of the liver,
which is safer than using the right lobe, as is common in adult-to-adult live-donor liver transplantation.

Other than the kidney and liver, organs are generally not obtained from living people. There are two types of
cadaver donors: heart beating or after cardiac death. Heart beating donors are pronounced brain-dead well
before the time of donation. In this case, the cadaver is taken to the operating room with full life support,
which is withdrawn after the aorta is cannulated. Cold perfusion is instituted immediately after the aorta is
clamped, so the organs are preserved immediately. In donation after cardiac death, the decision has been
made independent of the transplantation procedure that the patient's condition is terminal and life support
will be withdrawn. After this occurs and the heart has stopped by natural causes for 5 minutes, the patient is
pronounced dead and the organs are removed. This waiting period when the heart has stopped renders the
organs relatively ischemic, and outcomes using livers from these patients may not be as good as from heart-
beating donors.

LIVER TRANSPLANTATION

EPIDEMIOLOGY
There are approximately 17,000 patients on the waiting list for liver transplants in the United States, and
2,000 people on the waiting list die each year.

P
INDICATIONS
Liver transplantation is indicated for life-threatening or debilitating liver failure or early-stage hepatocellular
cancer that is not resectable because of tumor location or underlying liver disease. Disease states leading to
end-stage liver disease are listed in Table 26-2.

TABLE 26-2 Categories of Liver Disease Leading to Transplantation

Categories of Disease Examples

Noncholestatic Hepatitis B
cirrhosis

Hepatitis C
Alcoholic
cirrhosis

Autoimmune
hepatitis

Nonalcoholic
steatohepatitis

Cholestatic cirrhosis Primary biliary


cirrhosis

Primary
sclerosing
cholangitis

Caroli disease

Metabolic Wilson disease

Alpha-1
antitrypsin
deficiency

Tyrosinemia

Ornithine
carbamoyl-
transferase
deficiency

Cancer Hepatoblastoma
Hepatoma

Congenital Biliary atresia

Acute fulminant Tylenol


liver failure overdose

Halothane
hepatitis

Cryptogenic
cirrhosis

Viral hepatitis

Miscellaneous Budd-Chiari
syndrome

Polycystic liver
disease

LIVER ALLOCATION
Assignment of livers to patients depends on the Model for End-Stage Liver Disease (MELD), a formula that uses
the patient's creatinine, international normalized ratio (INR) of prothrombin time, and bilirubin.

Values <1.0 are set to 1.0; maximum creatinine level that may be used is 4.0 mg/dL, which is also the score
for dialysis patients. "Ln" is the natural logarithm.

This equation produces a number that directly correlates with mortality; livers are offered to the patients
with highest mortality. Waiting time is no longer a factor in liver allocation, unless patients have identical
MELD scores. This system has been prospectively analyzed and has resulted in fewer deaths on the waiting
list. Extra points are offered for certain early-stage tumors in recognition of excellent outcomes with liver
transplantation.
PROGNOSIS
Outcome after transplantation depends on a number of factors, including donor quality, recipient health, and
indication for operation. Overall, patient survival after liver transplantation is 93% at 3 months, 88% at 1 year,
80% at 3 years, and 74% at 5 years. Graft survival rates are 88% at 3 months, 81% at 1 year, and 66% at 5 years.
Fulminant hepatic failure carries poor prognosis after transplantation.

After successful liver transplantation, many patients return to a normal life, including work.

THE OPERATION: LIVER TRANSPLANTATION


Before beginning, it is imperative that adequate blood, platelets, and fresh frozen plasma are available. After
the organ is verified to be the correct ABO type, the entire abdomen is prepped, as is the groin in case bypass
will be necessary. Intravenous antibiotics are administered, and central monitoring is established. A wide
bilateral subcostal incision with midline extension is made. Goals are dissecting the suprahepatic cava,
isolating the portal vein in the porta hepatis, and freeing the liver from the diaphragm above the bare area of
the liver. The liver can be removed with or without the inferior vena cava. Most commonly, the liver is
removed with the retrohepatic cava (straight orthotopic). The new liver is sewn to the supra and infrahepatic
portions of the cava. In this case, because the cava is clamped, veno-venous bypass can be useful to preserve
venous return to the heart (Fig. 26-1). This also decreases bowel edema. It is also possible to leave the cava
intact in the recipient. In this case, the hepatic veins are
clamped, the liver is cut from the hepatic veins, and the new liver is sewn so it "hangs down" from the hepatic P
veins (piggyback technique). In both techniques, the portal vein is then reconstructed, followed by the
hepatic artery, and finally the bile duct (Fig. 26-2). Drains are placed, and the abdomen is closed.
Figure 26-1 • An example of veno-venous bypass during orthotopic liver
transplantation. The cannula drains the inferior vena cava, and the
centrifugal pump returns blood to the axillary vein. This circuit helps returns
blood to the heart during the anhepatic phase of the operation when the
inferior vena cava is occluded.

From Blackbourne LH. Advanced Surgical Recall. 2nd ed. Baltimore, MD:
Lippincott Williams & Wilkins, 2004.
Figure 26-2 • Diagram of orthotopic liver transplantation demonstrating the
hepatic artery, portal vein, bile duct, and infrahepatic caval anastomosis. Not
shown is the suprahepatic caval anastomosis.

From Blackbourne LH. Advanced Surgical Recall. 2nd ed. Baltimore, MD:
Lippincott Williams & Wilkins, 2004.

Complications
Technical complications after liver transplantation include caval stenosis, which can cause lower extremity
edema, or outflow problems for the liver, which causes swelling of the graft and dysfunction. A particularly
dangerous complication is hepatic arterial stenosis or thrombosis, which often leads to graft loss when acute
and intrahepatic abscesses when chronic. Portal vein stenosis or thrombosis can cause portal hypertension and
varices, as well as graft dysfunction or graft loss. Bile duct complications include stenosis and leak and may
be managed with operative repair, hepaticojejunostomy, or stents.
All patients with hepatitis C will experience recurrence in the graft. This can be a cause of graft loss both
early and late. Rejection typically presents with increasing liver function tests.

KIDNEY TRANSPLANTATION

EPIDEMIOLOGY
There are currently >60,000 people in the United States waiting for a kidney transplant, among a group of
more than 300,000 patients on dialysis.

INDICATIONS
Kidney transplantation is indicated for end-stage renal disease in patients otherwise healthy enough to
tolerate the surgery and subsequent immunosuppression. Common causes are given in Table 26-3.

TABLE 26-3 Disease States Leading to End-Stage Renal Disease

    Glomerulonephritis

    Chronic pyelonephritis

    Hereditary conditions—polycystic kidney


disease;
    nephritis, including Alport syndrome; tuberous
    sclerosis

    Metabolic conditions—diabetes mellitus,


hyperoxaluria,
    cystinosis, Fabry disease, amyloid, gout,
porphyria

    Obstructive uropathy

    Toxic insults

    Multisystem disease (lupus, vasculitis,


scleroderma)
    Hemolytic-uremic syndrome

    Tumors

    Congenital—hypoplasia, horseshoe

    Irreversible ATN

    Trauma

    Recurrences That Cause Graft Loss

    FSGS: 30%

    Mesangiocapillary type I glomerulonephritis: 20%

    Hemolytic-uremic syndrome: 50%

    Oxalosis: 90%

    Recurrences That Do Not Result in Graft


    Loss (generally)

    Membranous glomerulonephritis

    IgA nephropathy

    ATN, acute tubular necrosis; FSGS, focal


segmental glomerulosclerosis;
    IgA, immunoglobulin A.
KIDNEY ALLOCATION
Kidney allocation is based on a formula that incorporates the kidney's degree of match, the waiting time,
whether a person has been a kidney donor, the recipient's age, and the recipient's degree of sensitization
(Table 26-4).

TABLE 26-4 Kidney Allocation

  Criteria No. of Points

  Each year on the waiting list 1

  Very high preformed antibodies 4

  <10 years old 4

  11–17 years old 3

  Previous organ donor 4

  No mismatch 7

  One mismatch 5

  Two mismatches 2

PROGNOSIS
The half-life of a cadaver kidney, censoring for patient death, is >10 years, whereas for a live donor kidney,
half-life is close to 30 years. Transplantation is superior to dialysis for quality of life and survival, even in
older people.

THE OPERATION: KIDNEY TRANSPLANTATION


Before beginning, it is imperative to verify that the proper kidney has been received and is ABO compatible
and cross-match negative. Placement of the kidney on the right side is usually easier, as the right iliac vein is
more accessible than the left for anastomosis. Incising from the pubis to two fingerbreadths medial to the
anterior-superior iliac spine allows exposure of the anterior rectus sheath and the external oblique. Incision
through the muscular and fascial layers permits entry into the retroperitoneal space. Care is taken to
preserve the spermatic cord in males. Dissection of the retroperitoneal space allows the renal vessels to be
anastomosed to the external iliac artery and vein. Anastomosis of the ureter directly to the bladder re-
establishes drainage.

Complications
Technical complications after renal transplantation include venous and arterial thrombosis, which generally
result in graft loss. Ureteral complications include stricture and leak. Early problems should be fixed with re-
operation, either via direct repair or ureteroureterostomy with the recipient's native ureter. For chronic
problems, dilation and stenting can be attempted. Rejection typically presents with decreased urine output,
increasing creatinine, or edema.

PANCREAS TRANSPLANTATION
Pancreas transplantation is generally reserved for patients with severe type I diabetes who also need a kidney
transplant. In this setting, there is no additional risk of immunosuppression. In patients with hypoglycemic
unawareness, who can lose consciousness from low blood sugars, and other patients with severe
complications, pancreas transplantation alone is indicated. Transplantation will generally arrest the
development, but not reverse, diabetic complications. One- and 3-year graft survival of the pancreas in
kidney plus pancreas recipients is 85% and 71%, respectively. One- and 3-year graft survival of the pancreas
with pancreas transplantation alone is 73%, and 53%, respectively.

The pancreas is prepared with the donor duodenum intact to allow control of exocrine secretions. A P
bifurcated arterial graft is placed to connect the splenic artery and superior mesenteric artery of the
pancreas so that the entire gland receives blood. The gland is drained via its portal vein. The graft is placed
either in the right iliac fossa or in the middle of the abdomen. Right iliac placement requires inflow from the
right common iliac artery and drainage via the right common iliac vein, although the aorta and vena cava can
both be used. The pancreas can also be placed in the middle of the abdomen and have venous drainage
through the superior mesenteric vein (portal drainage). The duodenum may be anastomosed to the bowel or
bladder to control exocrine secretions.

Major complications include arterial or venous thrombosis, leak of the duodenal anastomosis, and
pancreatitis. Bladder drainage can lead to hematuria from bladder irritation and metabolic acidosis owing to
loss of bicarbonate secretions.

SMALL BOWEL TRANSPLANTATION


Small bowel transplantation is indicated for short bowel syndrome or intestinal failure, commonly resulting
from surgical resection. It is commonly performed in combination with liver transplantation. One- and 3-year
graft survival is 78% and 40%, respectively.

HEART TRANSPLANTATION
Heart transplantation is indicated for patients with end-stage heart failure whose symptoms cannot be
controlled with medications. The most common causes are coronary artery disease (45%) and congestive heart
failure (40%). Valvular disease, congenital, and other causes account for fewer than 10% of all transplant
recipients. One- and 3-year graft survival is 88% and 79%, respectively. Allograft vasculopathy is a major cause
of late graft loss.
LUNG TRANSPLANTATION
Lung transplantation is indicated for symptomatic end-stage lung disease. Chronic obstructive pulmonary
disease/emphysema accounts for >50% of transplants, and idiopathic pulmonary fibrosis another 26%. Less
common causes include alpha-1 antitrypsin deficiency, cystic fibrosis, sarcoidosis, and re-transplantation.
One- and 3-year graft survival is 83% and 64%, respec-tively. Because the lung is in direct contact with the
environment, infection is a major problem, and there is a constant struggle between overimmunosuppression
and rejection. Bronchiolitis obliterans syndrome is a poorly understood entity that is a major source of
morbidity and mortality in these patients.

Heart and lung transplantation are often performed together, and the operation is depicted in Figures 26-3,
26-4, 26-5, and 26-6.

Figure 26-3 • Heart/lung transplantation, step 1. Anterior view of recipient


heart and lungs before removal. Resection lines are shown dashed.

LifeART image copyright (c) 2009 Lippincott Williams & Wilkins. All rights
reserved.
Figure 26-4 • Heart/lung transplantation, step 2. Anterior view of recipient
mediastinum after heart and lungs have been removed. Shunts are in place.
Aorta is clamped.
LifeART image copyright (c) 2009 Lippincott Williams & Wilkins. All rights
reserved.

P
Figure 26-5 • Heart/lung transplantation, step 3. Anterior view of recipient
mediastinum after presentation of donor heart and lungs. Suturing has begun
along inferior vena cava and right atrium. Shunts are in place. Aorta is
clamped.

LifeART image copyright (c) 2009 Lippincott Williams & Wilkins. All rights
reserved.
Figure 26-6 • Heart/lung transplantation, step 4. Anterior view of recipient
mediastinum during last stages of transplantation. Recipient aorta is being
sutured to donor aorta.
LifeART image copyright (c) 2009 Lippincott Williams & Wilkins. All rights
reserved.

KEY POINTS
Control of the host response to donor major histocompatibility antigens is necessary
for successful organ transplantation.
Most current immunosuppressive regimens have moved away from long-term use of
corticosteroids.
Cadaver donors can be divided into heart-beating and non–heart-beating donors.
Liver transplantation is indicated for end-stage liver disease or unresectable early-stage
hepatocellular cancer and can be performed with removal or preservation of the recipient
vena cava.
Kidney transplantation is superior to dialysis for long-term patient survival and quality of
life.
Pancreas transplantation is indicated for patients with type I diabetes receiving a kidney, but
transplantation will generally not reverse diabetic complications.
Small bowel transplantation is indicated for patients with small bowel syndrome or
irreversible bowel failure.
Heart transplantation is a successful therapy for end-stage heart disease.
Allograft vasculopathy is a major source of morbidity in heart transplantation.
Bronchiolitis obliterans syndrome is an important cause of morbidity after lung transplant.
Authors: Karp, Seth J.; Morris, James P.G.; Zaslau, Stanley
Title: Blueprints Surgery, 5th Edition

Copyright ©2008 Lippincott Williams & Wilkins

> Table of Contents > Part V - Special Topics > Chapter 27 - Trauma

Chapter 27
Trauma

Trauma is the leading cause of death in the first four decades of life and the third leading cause
of death overall, trailing only cancer and coronary artery disease. Approximately 60 million
injuries occur annually in the United States. Although approximately 150,000 traumatic deaths
occur annually, the rate of disability from trauma is three times greater than mortality.
Therefore, issues relating to trauma care are of importance to all medical and surgical
specialists, from the trauma surgeon to the rehabilitation specialist.

Death due to trauma has been shown to occur in a trimodal distribution, during three
identifiable time periods. The first peak of death occurs within seconds to minutes of injury.
Lethal injury to the body's vital anatomic structures leads to rapid death, unless immediate
advanced intervention is performed. The second peak of death occurs within minutes to several
hours after the injury. Death during this second period is usually due to progressive neurologic,
cardiovascular, or pulmonary compromise. It is during this intermediate period that patients have
the greatest chance of salvage and toward which organized trauma care is focused. Rapid
resuscitation, coupled with the identification and treatment of potentially lethal injuries, is the
goal. The final third peak of death occurs several days to weeks after initial injury, usually
secondary to sepsis and multiorgan system failure.

This chapter discusses trauma management during the aforementioned second period.
Specifically, the steps of the initial assessment performed when the trauma patient arrives at the
hospital emergency room, the primary survey of the patient (ABCs), resuscitation, the secondary
survey (head to toe), and the institution of definitive care are examined.

PRIMARY SURVEY
The focus of the primary survey is to identify immediately life-threatening conditions and to
prevent death. Without a patent airway, adequate gas exchange, or sufficient intravascular
volume, any patient will die. Therefore, a simple mnemonic—ABCDE—is used to direct the
primary survey:

Airway with cervical spine control


Breathing and ventilation

Circulation and hemorrhage control

Disability and neurologic assessment

Exposure to enable examination

The principle here is that injuries are fixed as they are discovered, and this algorithm is to
remind the physician not to get distracted by obvious injuries. For example, a common exam
question goes something like this: "A patient is admitted to the emergency department after a
crush injury to the left upper extremity such that three fingers have been severed and one is
attached only by a thin piece of skin. What is your first intervention?" It is important to
demonstrate awareness that the airway should be assessed first. This should take approximately
15 to 30 seconds if there are no abnormalities.

(A) AIRWAY
The airway is immediately inspected to ensure patency and that any causes of airway obstruction
are identified (foreign body, facial fracture, tracheal/laryngeal disruption, cervical spine injury).
Cervical spine control must be maintained at all times—patients with multitrauma
must be assumed to have cervical spine injury until cleared radiographically. The chin thrust and P
jaw lift are methods of initially establishing airway patency while simultaneously protecting the
cervical spine. Conscious patients should be asked to open their mouths, which should be
inspected, and then to talk. This establishes at least a partially open conduit from the lungs to
the outside world.

Airway control in the conscious patient can be achieved with an easily inserted nasopharyngeal
trumpet; an oropharyngeal airway is used in the unconscious patient. Definitive control of the
airway and enhanced ability to ventilate and oxygenate the patient are achieved with
endotracheal intubation. Tube placement may be via the nasal or oral route. Nasotracheal
intubation is a useful technique for patients with cervical spine injuries; however, it is
contraindicated when midface or basilar skull fractures are suspected. When the trachea cannot
be intubated, as might occur with severe swelling or multiple fractures, a surgical airway is
indicated and should not be delayed. Jet insufflation of the airway after needle
cricothyroidotomy can adequately oxygenate patients for 30 to 45 minutes. Surgical
cricothyroidotomy with the insertion of a tracheostomy or endotracheal tube allows prolonged
ventilation and oxygenation. This is best achieved via a longitudinal incision that avoids the
anterior jugular veins, which can bleed significantly. The cricoid cartilage is palpated, and a
transverse incision is made, into which a tracheostomy tube can be placed.

(B) BREATHING
Once airway patency is established, the patient's ability to breathe must be assessed. Normal
function of the lungs, chest wall, and diaphragm is necessary for ventilation and gas exchange to
occur. Four entities should be of particular concern: open pneumothorax, tension pneumothorax,
hemothorax, and flail chest. Each can lead to rapid death resulting from respiratory
compromise. A mnemonic to remember this is to repeat "open, tension, hemo, flail" a number of
times. It has a regular cadence, and with enough repetition, it will come quickly back to you
when performing the breathing survey.

Open pneumothorax occurs when the integrity of the chest wall is compromised so that
diaphragmatic motion and decreased intrathoracic pressure cause air to enter the chest through
the hole in the chest instead of through the mouth. In this case, the lung will not expand.
Treatment requires closure of the chest wall defect and placement of a chest tube. Intubation
will also solve this problem because the positive pressure will expand the lungs.

Tension pneumothorax occurs when an injury to the lung parenchyma allows air to escape into
the pleural space. Often this will result in a simple pneumothorax, but if a valve effect occurs in
which the air cannot escape out of the plural space back into the lung, pressure in the pleural
space will continue to increase. This will render half of the lung useless for gas exchange, and
cause tremendous pressure (tension) on the mediastinum, pushing all structures away from the
side of the tension pneumothorax. This will impair venous return to the heart and will be fatal if
not recognized and treated (Fig. 27-1). The clinical picture of hypotension, tachycardia, tracheal
deviation, neck vein distention, and diminished unilateral breath sounds suggests the diagnosis of
tension pneumothorax. Immediate decompression by inserting a needle catheter into the second
intercostal space in the midclavicular line is indicated, followed by definitive treatment with
chest tube insertion into the fifth intercostal space at the anterior axillary line just lateral to the
nipple.
Figure 27-1 • Chest x-ray reveals a right-sided tension
pneumothorax. The right lung is collapsed and the increased
pressure in the right chest displaces the heart and other mediastinal
structures to the left.
From Fleisher GR, Ludwig S, Baskin MN. Atlas of Pediatric
Emergency Medicine. Philadelphia, PA: Lippincott Williams &
Wilkins, 2004.

Hemothorax occurs as a result of a ruptured blood vessel in the pleural space, which fills up with P
blood and compromises gas exchange (Fig. 27-2). This can cause life-threatening pulmonary
compromise or hypotension. Treatment involves tube thoracostomy and operation if bleeding
continues.

Figure 27-2 • Chest film reveals a left-sided hemothorax. The left


lung is compressed by the blood, and the mediastinum is shifted to
the right.
From Harwood-Nuss A, Wolfson AB, Lyndon CH, et al. The Clinical
Practice of Emergency Medicine. 3rd ed. Philadelphia, PA:
Lippincott Williams & Wilkins, 2001.
Flail chest occurs when multiple rib fractures prevent normal expansion of the lungs. In this
case, there may be paradoxical collapse of a portion of the chest on inspiration (Fig. 27-3). This
can be life-threatening if severe and is intensely painful. Often there is severe underlying lung
contusion, which impairs gas exchange. Treatment of pain with rib blocks may improve
ventilation. Intubation will allow lung expansion for the same reason for open pneumothorax.
Figure 27-3 • Flail chest. The paradoxical motion of the chest
cavity is shown. (top) As the patient inhales (diaphragm moves
down), the flail segment collapses, because the compromised chest
wall can no longer resist the decreased intrathoracic pressure. In
contrast, on expiration (bottom), exhalation causes a bowing out of
the flail segment. In these ways, flail chest results in dramatic
impairments in lung function.

Illustration by Neil O. Hardy, Westpoint, CT.

Examination of the chest during the primary survey includes observation, palpation, and
auscultation. Observation may reveal asymmetries in expansion, suggesting pneumothorax or
hemothorax, or an open, sucking wound. Palpation of the trachea may reveal deviation, and
palpation of the chest wall may reveal flail segments or asymmetrical excursion. Auscultation
may reveal diminished breath sounds as occurs in pneumo- and hemothoraces.

(C) CIRCULATION
Hypotension in the trauma patient is caused by blood loss until proven otherwise. There are only
five places to put enough blood to make someone hypotensive, so these areas require specific
attention: chest, abdomen, retroperitoneum, thighs, and outside world. External hemorrhage
can usually be identified and controlled by direct manual pressure. Tourniquets should be
avoided, because they cause distal ischemia. Internal hemor-rhage is more difficult to identify.
Therefore, hypotension
without signs of external hemorrhage must be assumed to be due to intra-abdominal or P
intrathoracic injury or from fractures of the pelvis or long bones. The hypovo-lemic hypotensive
patient usually exhibits a diminished level of consciousness as cerebral blood flow is reduced,
the pulse is rapid and thready, and the skin is pale and clammy.

(D) DISABILITY
Traumatic injuries may cause damage to the central and peripheral nervous systems. Spinal cord
injuries are most commonly seen in the cervical and lumbar regions. The thoracic spine is less
prone to injury because of the rigidity of the bony thorax. Complete spinal cord injury affects all
neurologic function below a specific level of the cord. Incomplete spinal cord injury exhibits
sacral sparing and may involve the central portion of the cord, as in central cord syndrome; a
single side of the cord, as in Brown-Séquard syndrome; or the anterior portion of the cord, as in
anterior cord syndrome. A rapid assessment of disability and neurologic function is vital so that
drug therapy and physical maneuvers can be initiated to prevent further neurologic injury.
(E) EXPOSURE
Exposure of the trauma patient is important for the entire body to be examined and injuries
diagnosed. Complete exposure entails removing all clothing from the patient so a thorough
examination can be performed, allowing for identification of entry and exit wounds, extremity
deformities, contusions, or lacerations; sometimes this is considered part of the secondary
survey. The patient should be rolled onto one side with the neck and back supported in a rigid
position to determine whether there are additional injuries. If there is suggestion of spinal injury
and no obvious hemorrhage is coming from the back, this maneuver can be postponed. After the
examination, attention should be paid to maintaining the patient's core body temperature.

EMERGENCY DEPARTMENT THORACOTOMY


In the severely injured, hypotensive patient, the question sometimes arises whether to perform
thoracotomy in the emergency department as a life-saving maneuver. In the blunt trauma
patient, this intervention is rarely successful and should be avoided, unless the vital signs are
lost just before or during the resuscitation and all other measures have failed to improve the
situation. In victims of penetrating trauma, this maneuver can be successful, especially if cardiac
tamponade is found. This can temporize until the patient can be taken to the operating room for
definitive repair.

VOLUME RESUSCITATION
Hemorrhage leading to hypovolemic shock is the most common cause of postinjury death in the
trauma patient. Rapid fluid resuscitation and hemorrhage control are the keys to restoring
adequate circulating blood volume. The fluid status of patients can be quickly evaluated by
assessing their hemodynamics (hypotension and tachycardia), their level of consciousness
(adequacy of cerebral perfusion), the color of their skin (pale skin indicates significant
exsanguination), and the presence and character of the pulse (absent central pulses indicate
profound hypovolemic shock). All sources of external hemorrhage must be identified and treated
by applying direct pressure. Indiscriminate hemostat usage should be avoided, because it may
crush and damage surrounding neurovascular structures. Sources of internal hemorrhage are
usually hidden and are suspected by unstable hemodynamics. Internal bleeding may occur in the
thorax as a result of cardiovascular or pulmonary injury, in the abdomen from splenic or liver
lacerations, or into the soft tissues surrounding femur or pelvic fractures.

Fluid resuscitation of the hypovolemic, hypotensive patient requires establishing adequate


intravenous access. Two large-bore catheters (14 gauge) should be placed in upper extremity
veins and rapid infusion of a balanced salt solution (Lactated Ringer's or normal saline) initiated.
If the pattern of injury allows, central access via the femoral vein approach using larger-
diameter catheters maximizes the rate of fluid administration. If percutaneous access is
unsuccessful, a cut-down of the greater saphenous vein at the anteromedial ankle is required.
After intravenous access is established, bolus infusion of crystalloid solution should be replaced
with O-negative or type-specific blood, once it becomes available.

TRAUMA RADIOGRAPHS
For patients with blunt trauma (automobile crashes, falls), three standard radiographic studies
are required to assess the neck, chest, and pelvis: cross-table lateral cervical spine,
anteroposterior chest, and anteroposterior
pelvis. Obtaining these three x-rays early in the resuscitation process allows potentially P
neurologically disabling cervical spine injuries, life-threatening chest wall and cardiopulmonary
injuries, and pelvic injuries to be identified and immediately treated. For patients with
penetrating trauma (gunshot, stabbing, impaling), an anteroposterior chest film and other films
pertaining to the site of injury should be obtained.

SECONDARY SURVEY
The secondary survey begins after the airway, breathing, and circulation have been assessed and
resuscitation has been initiated. This secondary survey is a head-to-toe evaluation of the body,
during which additional areas of injury are identified. A meticulous examination during this
phase of the trauma evaluation minimizes the chance of missing an important finding.

The final phase of acute trauma care is instituting definitive treatment. This may entail simple
wound care in the emergency room for minor injuries or, if the injuries warrant, transportation
to the operating room for surgical treatment.

THORACIC TRAUMA
Because of the great importance of the structures in the chest, thoracic trauma is second only to
head trauma as a cause of death in injured patients. After the interventions described in the
primary survey, many injures will remain that will require expeditious repair.

Open pneumothorax will generally require chest wall repair. The timing of this depends on
whether the defect can be controlled, how large it is, and the severity of the patient's other
injuries.

Tension pneumothorax, after emergent decompression, will require chest tube placement.

Hemothorax, after initial chest tube placement, requires careful monitoring to decide whether
the patient needs emergent thoracotomy. If the initial blood removed from the chest is >1,500
mL, or if ongoing blood loss is >200 mL/hr, the patient should be explored in the operating room
for hemorrhage control. Even if the patient does not require emergent thoracotomy, many
centers will aggressively clean the pleural space of blood with thoracoscopy at some point in the
future when the patient is stable.

Flail chest is often a sign of significant pulmonary contusion, which will cause a ventilation
perfusion mismatch. These patients can be very difficult to oxygenate. Treatment consists of
diuresis, pain control, and if necessary, prolonged intubation with positive end-expiratory
pressure. The flail chest itself can be stabilized operatively, but this is rarely necessary.

Cardiac tamponade is a life-threatening condition in which blood escapes into the pericardium.
Because the pericardium does not expand well, the blood compresses the heart and dramatically
decreases venous return and cardiac output. The diagnosis is made from the Beck's triad of
hypotension, distant heart sounds, and elevated central venous pressure. Pericardiocentesis can
remove blood from the space and temporarily restore normal hemodynamics. These patients
should generally be taken to the operating room to diagnose and repair cardiac injuries or other
sources of the bleeding.

Severe impact on the sternum can cause blunt cardiac injury or cardiac contusion. This lesion
can decrease cardiac output and predispose patients to arrhythmias. Although there is no
specific therapy, these patients should be monitored and supported if the cardiac output is
inadequate to perfuse the body.

Thoracic aortic injury from blunt trauma usually involves disruption of the aorta just distal to the
takeoff of the left subclavian artery where the aorta is tethered by the ligamentum arteriosum.
This lesion requires tremendous energy to produce and is most commonly due to falls or high-
speed frontal impacts. This lesion should be suspected in the appropriate clinical circumstances.
Chest radiograph may reveal a wide mediastinum, loss of aortic knob contour, nasogastric tube
deviation to the right, first or second rib fracture, or pleural cap, which is an accumulation of
blood above the pleura. Aortography, contrast computed tomography (CT), or transesophageal
echocardiogram can be used in diagnosis.

Diaphragmatic rupture may occur from penetrating or blunt trauma. Tremendous force is
required to cause this injury. Diagnosis is made with chest x-ray, demonstrating a nasogastric
tube or bowel in the chest, or with CT scan or contrast study. On the right side, the liver
prevents most cases, so that the left diaphragm is most commonly affected. Operative
exploration is indicated for repair and evaluation of other injuries.

Esophageal injury is diagnosed on contrast swallow or endoscopy. In general, operative repair


with wide drainage is necessary to prevent mediastinitis, which can be fatal.

Tracheal injury can present on physical examination with crepitus, difficulty speaking,
respiratory distress, or hemoptysis. Bronchoscopy is necessary to
define the lesion. Treatment is expectant or operative depending on the nature and severity of P
the injury.

ABDOMINAL TRAUMA
Complete examination of the abdomen is extremely difficult in the emergency department
because of the noise and number of people. Although classic teaching is to listen, observe,
palpate, and auscultate bowel sounds, findings can be unreliable because of altered states of
consciousness of the patient, ambient noise, and multiple procedures occurring at the same
time. The abdominal examination is best thought of in the context of all of the patient's injuries,
and it may be necessary to act definitively in spite of physical examination findings. For example
a hypotensive patient with no other injuries with a significant mechanism and no abdominal
tenderness may still have a hemoperitoneum. A soft abdomen does not preclude a hollow viscous
injury.

In addition to examination of the abdomen, rectal examination is performed to look for blood,
indicative of a rectal or colonic injury, and tone, lack of which may suggest a spinal cord injury.
In the male, a high-riding or nonpalpable prostate may suggest a urethral injury, and a
suprapubic tube should be placed instead of a Foley catheter. Blood at the urinary meatus or a
scrotal hematoma in the male also suggests a urethral injury, and again, a Foley should be
avoided. Pressure should be placed on the anterior-superior iliac spine to test for fractures.

Patients who are stable with a suspected abdominal injury should undergo abdominal CT scan.
Other options include focused assessment by sonography in trauma (FAST) ultrasound
examination or diagnostic peritoneal lavage. Diagnostic peritoneal lavage is performed through a
small infraumbilical incision. A supraumbilical incision is preferred if there is significant pelvic
trauma because of the possibility of a hematoma in the preperitoneal space below the umbilicus.
A catheter is placed in the abdomen, it is aspirated, and 1 L of Lactated Ringer's solution is
instilled. The test result is considered positive if the initial aspiration returns blood or bowel
contents, the aspirate after the liter contains >100,000 red blood cells or 500 white blood
cells/mL, if bacteria or bowel contents are returned, or if the fluid has a high amylase.

Injuries to the liver and spleen are generally treated conservatively unless there is ongoing
hemorrhage, making the patient unstable. Hollow viscous injuries are fixed immediately by
resection and either diversion or primary anastomosis, depending on the situation. Pancreatic
injuries can be treated by simple drainage or resection of a portion of the gland with or without
pancreaticojejunostomy. In general, "damage control" is used to describe the initial operation in
the multi-injured patient. Life-threatening injures are treated, and definitive treatment is
delayed in favor of resuscitation in the intensive care unit as necessary. This is to prevent
ongoing hypothermia and worsening coagulopathy, which can be fatal.

OPHTHALMIC TRAUMA
More than 1 million cases of ophthalmic trauma after penetrating or blunt injury are reported
annually in the United States. Prompt and appropriate care of many ophthalmic injuries may
prevent much visual disability.

CHEMICAL BURNS
Chemical burns to the eye represent an ophthalmologic emergency. If treatment is not begun
immediately, irreversible damage may occur. Alkaline substances (e.g., household cleaners,
fertilizers, and pesticides) cause the most severe damage, but acids may cause significant ocular
morbidity as well.
Treatment
A detailed history is not required before beginning copious irrigation with any available water
source for at least 15 to 20 minutes. After initial irrigation, visual acuity and pH should be
measured. If the pH has not returned to the normal value of 7.5, irrigation should be continued.
Prompt ophthalmologic referral should be obtained in all cases of acid or alkali burns and for
patients with decreased visual acuity, severe conjunctival swelling, or corneal clouding. All other
patients should see an ophthalmologist within 24 hours.

SUPERFICIAL FOREIGN BODIES


Foreign bodies that have an impact on the surface of the cornea or conjunctiva represent
approximately 25% of all ocular injuries.

History
An accurate history often provides the diagnosis and should be used to judge the risk of
intraocular foreign body (see below). Symptoms range from mild ocular irritation to severe pain.
If symptoms began gradually
rather than suddenly, other causes, such as infectious keratitis, should be considered. P

Diagnostic Evaluation
Careful inspection of the cornea and conjunctiva using bright light and magnification often
reveals the foreign body.

Treatment
One should always measure visual acuity before making any attempt at removing the foreign
body. Superficial foreign bodies can usually be removed using topical anesthesia and a cotton
swab. After the foreign body is removed, Wood's lamp examination with fluorescein should be
performed to ascertain the size of any residual corneal epithelial defect. Eversion of the upper
eyelid should be carried out to look for residual foreign material under the lids. Ophthalmologic
referral is indicated when a foreign body cannot be safely removed or for any patient with a
large corneal epithelial defect.

BLUNT OR PENETRATING INJURY


Blunt or penetrating trauma to the eye represents a leading cause of vision loss in young people.
Blunt trauma most often causes ocular contusion or damage to the surrounding orbit. Penetrating
trauma causes corneal or scleral laceration (a ruptured globe) and represents an ophthalmologic
emergency requiring early intervention and repair. The possibility of a retained intraocular
foreign body should always be considered (see below). A high degree of suspicion must be
maintained in all cases of head and facial injuries.
History
History should include the mechanism of injury, the force of impact, the likelihood of a retained
foreign body, and any associated ocular or visual complaints.

Diagnostic Evaluation
Eyelid integrity, ocular motility, and pupillary reaction should be tested. Use a penlight to detect
conjunctival swelling or hemorrhage, corneal or scleral laceration, or hyphema (blood behind the
cornea obscuring details of the underlying iris or pupil). Pain and decreased vision with a history
of trauma should always lead to suspicion of perforation of the globe. Severe subconjunctival
hemorrhage, a shallow anterior chamber or space between the cornea and iris, hyphema, and
limitation of extraocular motility are often, but not invariably, present. Radiologic studies,
including CT of the head and orbits, should be obtained in cases of suspected blowout fracture or
to rule out a retained intraocular foreign body (see below).

Treatment
If the eye is lacerated or the pupil or iris is not visible, a shield should be placed over the eye,
and the patient should be referred immediately to an ophthalmologist. Eyelid lacerations that
involve the lid margin or lacrimal apparatus require meticulous repair to avoid severe functional
and cosmetic morbidity. If the eyelid margin and inner one sixth of the eyelid are not damaged,
the wound can be closed with fine sutures. If the eyelid margin is lacerated, accurate
realignment of the lid margin must be ensured before wound closure. Disruption of the inner one
sixth of the eyelid requires intubation of the lacrimal drainage system, with stent placement
before surgical repair, and should be carried out by an ophthalmologist or other appropriately
trained physician. Ophthalmologic referral after trauma is determined by ocular symptoms and
findings, as set forth in Table 27-1.

TABLE 27-1 Management of Ophthalmic Trauma

  Treat on Site and Refer Immediately

  Acid or alkali burn

  Unremovable corneal or conjunctival foreign body

  Refer Immediately
  Severe pain

  Subnormal visual acuity

  Irregular pupil

  Deformed globe

  Corneal or scleral laceration

  Corneal clouding

  Severe lid swelling

  Severe conjunctival chemosis

  Proptosis

  Hyphema

  Absent red reflex

  Suspected intraocular foreign body (history of


being
  struck by high-speed missile)

  Eyelid laceration that is deep, large, avulsed,


exposes
  fat, or extends through lid margin or lacrimal
  drainage apparatus

  Refer Within 24 Hours

  Pain

  Photophobia

  Diplopia

  Foreign-body sensation but no visible foreign


body
  or corneal abrasion

  Large corneal abrasion

  Moderate eyelid or conjunctival chemosis but


normal
  visual acuity

  Suspected contusion of globe

  Suspected orbital wall fracture

  Refer Within 48 Hours

  Mild contusion injury to orbital soft tissues


INTRAOCULAR FOREIGN BODIES
A high-speed missile may penetrate the cornea or sclera while causing minimal symptoms or
physical findings. Foreign-body composition is important, because certain metals, such as iron,
steel, and copper, produce a severe inflammatory reaction if left in the eye, whereas other
materials, such as glass, lead, and stone, are relatively inert and may not require surgical
removal. Retained vegetable matter is especially dangerous and may cause a severe purulent
endophthalmitis. A retained foreign body should be suspected in all cases of perforating injuries
of the eye or whenever the history suggests high-risk activities, such as drilling, sawing, or
hammering.

History
One should inquire about high-risk activities, a sensation of sudden impact on the eyelids or eye,
and any complaint of pain or decreased vision.

Physical Examination
Visual acuity should always be recorded before any manipulation of the eye or eyelids.
Inspection may reveal an entry wound, although this may be quite subtle and easily overlooked.
Specifically, one should look for a hyphema, pupillary distortion, or any alteration of the red
reflex on funduscopic examination.

P
Diagnostic Evaluation
Accurate localization may require soft tissue radiographs, orbital ultrasound, or CT. Magnetic
resonance imaging is contraindicated in all cases of suspected intraocular foreign body.

Treatment
If the history strongly suggests the possibility of a retained foreign body, urgent ophthalmologic
referral is indicated, even in the absence of physical findings. Prompt surgical removal of
intraocular debris is usually indicated to avoid the toxic effect of metallic foreign bodies on
intraocular tissue and secondary intraocular infection from retained organic material.

BURNS
Burns are characterized by their thickness and the extent of the body surface area they cover.
First-degree burns are limited to the dermis, do not blister, and are red and painful. Because the
dermis is intact, they heal without scarring. Second-degree burns extend partially into the
dermis and form blisters which are red and painful. The extent of dermal involvement
determines the extent of scar formed. Third-degree burns occur when the entire dermis is
destroyed. These areas of burn are insensate, because the nerve endings have also been
destroyed. They generally require skin grafting for healing.
Patients presenting with burns must be evaluated for inhalation injury if the burn was caused by
a fire. Carbon monoxide inhalation, heat causing edema of the oropharynx and larynx, and
damage caused by inhalation of hot embers can also be fatal. Patients with significant injury of
this type should be intubated early to maximize respiratory support and prevent airway
compromise.

Severely burned patients will lose tremendous amounts of fluid, and resuscitation forms a major
portion of the care of these patients. Percentage of body burn can be estimated by the rule of
nines: the head has 9% of body-surface area, the dorsal and ventral torso each 18%, each arm 9%,
and each leg 18%. On this basis, the Parkland formula determines the amount of fluid given and
should be initiated for patients with >10% to 15% of total body-surface affected. This is
calculated as the body weight in kilograms × percent of total body surface with second- or third-
degree burns × 4 mL. Half of this total should be given as Lactated Ringer's solution in the first 8
hours after injury, and the rest over the next 16 hours. Urine output and blood pressure should
be carefully monitored and infusions adjusted accordingly.

As treatment begins, eschars in the torso may constrict breathing, or eschars in the extremities
may compromise circulation. In this case, escharotomy should be performed to relieve the
compression (Fig. 27-4). In
this procedure, full-thickness incisions are made in the skin to release the constriction. P
Figure 27-4 • Incisions for escharotomy. From Blackbourne LH.
Advanced Surgical Recall. 2nd ed. Baltimore, MD: Lippincott
Williams & Wilkins, 2004

Sepsis is an ongoing problem for burn patients. The burn itself sterilizes the skin, but bacterial
repopulation occurs quickly. Great improvements in burn care have come from early excision and
grafting, which decrease the risk of infection. In this procedure, thin slices of skin are removed
with a dermatome until viable tissue is encountered, as determined by bleeding. At this point,
skin grafting is performed. For large burns, this procedure is limited by hypothermia and blood
loss, and is often done in sequential fashion. Local care with topical antibiotics and frequent
dressing changes is critical.

Burn patients often have tremendous nutritional requirements, with calorie and protein
requirements approaching twice normal. Enteral feeding should be initiated as quickly as
possible.

KEY POINTS
Trauma is the leading cause of death in the first four decades of life.
Trauma is the third leading cause of death overall, after cancer and heart
disease.
Traumatic death occurs in a trimodal distribution.
Trauma care involves the primary survey, resuscitation, secondary survey, and
definitive care.
The primary survey identifies immediately life-threatening injuries involving the
airway, breathing, and circulation.
Resuscitation involves airway control and ventilation and fluid infusion after
intravenous access is obtained.
The secondary survey is a head-to-toe examination to identify additional areas
of injury.
Standard radiographs required for trauma include lateral cervical spine,
anteroposterior chest, and anteroposterior pelvis.
"Open, tension, hemo, flail" is a pneumonic for life-threatening thoracic injuries
diagnosed during the primary survey.
Total surface burn area determines fluid requirements for burn patients, which
are calculated using the Parkland formula.
Chemical burns to the eye are a true ophthalmologic emergency.
Authors: Karp, Seth J.; Morris, James P.G.; Zaslau, Stanley
Title: Blueprints Surgery, 5th Edition

Copyright ©2008 Lippincott Williams & Wilkins

> Back of Book > - COLOR PLATES

COLOR PLATES
Color Plate 1 Gastric ulcer. The stomach has been opened to
reveal a sharply demarcated, deep peptic ulcer on the lesser
curvature.
From Rubin E, Farber JL. Pathology. 3rd ed. Philadelphia, PA:
Lippincott Williams & Wilkins, 1999.

Color Plate 2 Crohn's disease. The mucosal surface of the colon


displays a "cobblestone" appearance owing to the presence of
linear ulcerations and edema and inflammation of the intervening
tissue.
From Rubin E, Farber JL. Pathology. 3rd ed. Philadelphia, PA:
Lippincott Williams & Wilkins, 1999.
Color Plate 3 Crohn's disease. Active phase of Crohn's disease
shows cobblestoning, caused by interconnecting ulcerations
(left). An area of cobblestoning after therapy is shown on the
right.

From Yamada T, Alpers DH, Kaplowitz N, et al. Atlas of


Gastroenterology. 3rd ed. Philadelphia, PA: Lippincott Williams &
Wilkins, 2003.
Color Plate 4 Carcinoid tumor of the small intestine. A bisected
annular carcinoid tumor (arrows) constricts the lumen of the
small intestine. Lymph node metastases are evident.

From Rubin E, Farber JL. Pathology. 3rd ed. Philadelphia, PA:


Lippincott Williams & Wilkins, 1999.
Color Plate 5 Hepatic segmentation. A. The segmental anatomy
of the liver is based on the blood supply. Segment I is the caudate
lobe. Segments II, III, and IV are part of the left lobe, and
segments V, VI, VII, and VIII are part of the right lobe.

From Moore KL, Dalley AF. Clinically Oriented Anatomy. 4th ed.
Baltimore, MD: Lippincott Williams & Wilkins, 1999.

Color Plate 6 Hepatic adenoma. A surgically resected portion of


liver shows a tan, lobulated mass beneath the liver capsule.
Hemorrhage into the tumor has broken through the capsule and
also into the surrounding liver parenchyma. The patient was a
woman who had taken birth control pills for a number of years
and presented with sudden intraperitoneal hemorrhage.
From Rubin E, Farber JL. Pathology. 3rd ed. Philadelphia, PA:
Lippincott Williams & Wilkins, 1999.

Color Plate 7 Hepatocellular carcinoma. Cross-section of a


cirrhotic liver shows a poorly circumscribed, nodular area of
yellow, partially hemorrhagic hepatocellular carcinoma.
From Rubin E, Farber JL. Pathology. 3rd ed. Philadelphia, PA:
Lippincott Williams & Wilkins, 1999.
Color Plate 8 Inguinal region of a male. The aponeurosis of the
external oblique is partly cut away and the spermatic cord has
been cut and removed from the inguinal canal. The reflected
(reflex) inguinal ligament is formed by aponeurotic fibers of the
external oblique. Observe the iliohypogastric and ilioinguinal
nerves (branches of the first lumbar nerve) passing between the
external and internal oblique muscles. The ilioinguinal nerve is
vulnerable during repair of an inguinal hernia.

From Moore KL, Dalley AF. Clinically Oriented Anatomy. 4th ed.
Baltimore, MD: Lippincott Williams & Wilkins 1999.
Color Plate 9 Thyroid-related ophthalmopathy with proptosis, lid
retraction, and limited motility.

From Tasman W, Jaeger E. The Wills Eye Hospital Atlas of Clinical


Ophthalmology. 2nd ed. Philadelphia, PA: Lippincott Williams &
Wilkins, 2001.
Color Plate 10 Diffuse enlargement of the thyroid gland.

From Weber J, Kelley J. Health Assessment in Nursing. 2nd ed.


Philadelphia, PA: Lippincott Williams & Wilkins, 2003.
Color Plate 11 Medullary thyroid carcinoma. Coronal section of a
total thyroid resection shows bilateral involvement by a firm,
pale tumor.

From Rubin E, Farber JL. Pathology. 3rd ed. Philadelphia, PA:


Lippincott Williams & Wilkins, 1999.
Color Plate 12 Pheochromocytoma. The cut surface of an adrenal
tumor from a patient with episodic hypertension is reddish brown
with a prominent area of fibrosis. Foci of hemorrhage and cystic
degeneration are evident.
From Rubin E, Farber JL. Pathology. 3rd ed. Philadelphia, PA:
Lippincott Williams & Wilkins, 1999.
Color Plate 13 Calcific aortic stenosis. Large deposits of calcium
salts are evident in the cusps and the free margins of the
thickened aortic valve, viewed from above.

From Rubin E, Farber JL. Pathology. 3rd ed. Philadelphia, PA:


Lippincott Williams & Wilkins, 1999.
Color Plate 14 Posterior abdominal wall showing great vessels,
kidneys, and suprarenal glands. Most of the fascia has been
removed. Observe that the ureter crosses the external iliac
artery just beyond the common iliac bifurcation and that the
testicular vessels cross anterior to the ureter and join the ductus
deferens (vas deferens) to enter the inguinal canal. Renal arteries
are not seen because they lie posterior to the renal veins.

From Moore KL, Dalley AF. Clinically Oriented Anatomy. 4th ed.
Baltimore, MD: Lippincott Williams & Wilkins, 1999.
Color Plate 15 Renal cell carcinoma. The kidney contains a large
irregular neoplasm with a variegated cut surface, including
yellow areas that correspond to lipid-containing cells.

From Rubin E, Farber JL. Pathology. 3rd ed. Philadelphia, PA:


Lippincott Williams & Wilkins, 1999.
Color Plate 16 Pigmented basal cell carcinoma. Note the pearly,
waxy surface.

From Goodheart HP. Goodheart's Photoguide of Common Skin


Disorders. 2nd ed. Philadelphia, PA: Lippincott Williams &
Wilkins, 2003.
Color Plate 17 Nodular basal cell carcinoma. Note the rolled
borders with telangiectasia.

From Goodheart HP. Goodheart's Photoguide of Common Skin


Disorders. 2nd ed. Philadelphia, PA: Lippincott Williams &
Wilkins, 2003.
Color Plate 18 Common donor skin graft sites. Blue skin areas are
appropriate for full-thickness grafts; green areas are used for
split-thickness grafts and rose sites are used for fat-dermal
grafts.

From Smeltzer SC, Bare BG. Textbook of Medical-Surgical


Nursing. 9th ed. Philadelphia, PA: Lippincott Williams & Wilkins,
2000.
Authors: Karp, Seth J.; Morris, James P.G.; Zaslau, Stanley
Title: Blueprints Surgery, 5th Edition

Copyright ©2008 Lippincott Williams & Wilkins

> Back of Book > Appendix: Sample Operative Reports

Appendix: Sample Operative


Reports
After each operation, a complete and concise description, outlining the indications for surgery,
the operative findings, and the conduct of the operation, should be composed. This report is
vital for communication among health care providers, and it documents the intervention for
future reference. When dictating a report, follow a defined format, introduce only relevant
information to the narrative, and maintain an orderly flow from incision to closure. The
following operative reports are examples of routine general surgery procedures: open hernia
repair and laparoscopic cholecystectomy.

INGUINAL HERNIA
PREOPERATIVE DIAGNOSIS: Right inguinal hernia

POSTOPERATIVE DIAGNOSIS: Right indirect inguinal hernia

PROCEDURE PERFORMED: Open-mesh repair of right indirect inguinal hernia

SURGEON: James Morris, MD

ANESTHESIA: Local and intravenous sedation

INDICATIONS FOR OPERATION: Mr. Robert Hall is a 75-year-old male who presented complaining of
a symptomatic right inguinal hernia of 6 months' duration. On physical examination, a nontender
reducible right inguinal hernia was noted. Operative and nonoperative management options, as
well as the risks and potential complications of each approach, were discussed with the patient.
After all questions were answered, he requested open-mesh hernia repair. A request-for-surgery
form was signed and witnessed preoperatively.

INTRAOPERATIVE FINDINGS: Moderate indirect inguinal hernia sac, ligated and resected.
Polypropylene mesh plug inserted into deep ring with mesh overlay.

DESCRIPTION OF PROCEDURE: The patient was positioned supine on the operating table, and
preoperative antibiotics and intravenous sedation were administered. The right inguinal region
was prepped and draped sterilely. Combined lidocaine 1% and bupivacaine 0.25% was injected
subdermally, and an oblique skin incision was made above the inguinal ligament. The superficial
epigastric vessels were divided between clamps and tied. Scarpa's fascia was divided and the
external oblique fascia identified. A subfascial local anesthetic infiltration was performed, and
the external oblique fascia was opened in the line of its fibers down through the external ring.
The iliohypogastric nerve was identified and preserved. The ilioinguinal nerve was dissected
away from the cord structures and retracted caudad. Both nerves were preserved and uninjured
during the entire procedure. The cord was encircled at the pubic tubercle with a Penrose drain
and the floor inspected. There was no evidence of direct herniation; however, the floor was
somewhat attenuated. The cord was interrogated, and a moderate-sized indirect hernia sac was
identified and dissected free from the surrounding structures. The vas deferens and testicular
vessels were preserved and uninjured. Once the proximal sac was fully mobilized into the deep
inguinal ring, the distal sac was suture ligated with a 3-0 absorbable stitch and the redundant sac
excised and passed off the table as a specimen. A large-sized polypropylene plug was placed into
the indirect defect and sutured superiorly for fixation. Given the previously noted attenuated
floor, a polypropylene patch was sutured for reinforcement to the conjoined tendon, pubic
tubercle, and shelving edge of the inguinal ligament with interrupted 3-0 absorbable suture. The
ilioinguinal nerve was returned to its anatomic position alongside the cord structures, and the
lateral legs of the patch were positioned around the cord as it exited the deep ring, the legs
being secured with a single stitch. The iliohypogastric nerve was then returned to its anatomic
position, and the external oblique fascia was closed with a running 3-0 absorbable suture, taking
care to avoid entrapment of the ilioinguinal nerve. The Scarpa fascia and then the subdermal
layer were reapproximated with interrupted 3-0 absorbable sutures. Skin was further closed with
a running 4-0 absorbable subcuticular stitch. Paper reinforcing strips were applied across the
incision, followed by a sterile dressing. The patient was then transferred to the recovery room
awake and in stable condition. Total intravenous fluids were 300 mL of crystalloid, and blood loss
was nil. The only specimen was the hernia sac.

LAPAROSCOPIC CHOLECYSTECTOMY
PREOPERATIVE DIAGNOSIS: Symptomatic cholelithiasis

POSTOPERATIVE DIAGNOSIS: Same

OPERATION PERFORMED: Laparoscopic cholecystectomy P

SURGEON: Seth Karp, MD

ASSISTANT: James Morris, MD

ANESTHESIA: Local, general endotracheal

INDICATIONS FOR OPERATION: Ms. Gloria Brillantes is a 45-year-old female with a 2-year history
of episodic postprandial right upper quadrant abdominal pain radiating to the right flank, with
associated nausea and occasional emesis. Ultrasonography reveals multiple gallstones with
normal gallbladder wall thickness and normal common bile duct caliber. Liver function tests were
normal. After surgical consultation and review of her clinical situation, I discussed with the
patient the operative and nonoperative management options, including the risks and potential
complications of each approach. After all questions were answered, she requested laparoscopic
cholecystectomy. A request-for-surgery form was signed and witnessed preoperatively.

INTRAOPERATIVE FINDINGS: Normal-appearing, thin-walled gallbladder without adhesions


containing multiple small 5-mm cholesterol gallstones. Liver, stomach, and small and large
intestines were grossly normal.

DESCRIPTION OF PROCEDURE: The patient was positioned supine on the operating room table,
and preoperative antibiotics and general endotracheal anesthesia were administered. The
abdomen was prepped and draped sterilely. The Veress needle was inserted uneventfully through
a tiny subumbilical incision, after skin infiltration with combined lidocaine 1% and bupivacaine
0.25%. The abdomen was insufflated with carbon dioxide to 15 mm Hg pressure. A 5-mm port
was then placed subum-bilically and the 5-mm 30-degree endoscope inserted. The abdomen was
inspected and appeared grossly normal. Under additional local anesthesia, a 10-mm port was
placed in the epigastric region, and two more 5-mm ports were placed further laterally on the
right. The table was placed in reverse Trendelenburg position, with mild rotation to the patient's
left side. The gallbladder fundus was retracted cephalad, and the infundibulum was retracted
toward the right lower quadrant. Using careful blunt dissection, normal biliary anatomy was
encountered. The cystic duct and the artery were easily identified and divided between surgical
clips. The gallbladder was then dissected out of the fossa using electrocautery, removed from
the abdomen via the epigastric port, and passed off the table as a specimen. The operative field
was then inspected and found to be hemostatic and without bile leak. Intact clips were again
seen securing the duct and artery stumps. All trocars were then removed under vision, and no
port site bleeding was noted. The 10-mm epigastric fascial defect was closed with a figure-of-
eight 0 absorbable suture, and all skin incisions were closed with buried simple 4-0 absorbable
sutures. Paper reinforcing strips were applied to the incisions, followed by sterile dressings. The
patient was uneventfully extubated and transferred to the recovery room in stable condition.
Administered intravenous fluids were 500 mL of crystalloid, and blood loss was negligible.
Specimens included the gallbladder and contained stones.

P
Authors: Karp, Seth J.; Morris, James P.G.; Zaslau, Stanley
Title: Blueprints Surgery, 5th Edition

Copyright ©2008 Lippincott Williams & Wilkins

> Back of Book > Questions

Questions
1. A 71-year-old man with sudden onset of severe abdominal and back pain is brought to the
emergency department for evaluation. He has a history of hypertension. He weighs 300 lb.
He has a 45-pack-a-year history of smoking. Physical examination reveals a pulsatile
abdominal mass. Both lower extremities reveal pallor with diminished pedal pulses. What is
the most likely cause of this patient's condition?

a. Atherosclerosis

b. Marfan syndrome

c. Meningococcal infection

d. Syphilis

e. Trauma

View Answer

2. A 78-year-old man is brought to the emergency department with a 12-hour history of


abdominal pain, diarrhea, and vomiting. He has a history of atrial fibrillation and was
previously treated for congestive heart failure with digoxin. Physical examination reveals a
distended abdomen with significant guarding. Rectal examination reveals guaiac positive
stool in the vault. White blood cell count is 24,000/mL. Abdominal x-ray reveals edema of
the bowel wall. What is the most appropriate treatment for this patient?

a. Angiographic embolization

b. Antibiotic therapy with ampicillin and gentamicin

c. Antibiotic therapy with gentamicin

d. Heparinization followed by oral warfarin

e. Surgical exploration

View Answer
3. A 20-year-old male tennis player crashes into a fence trying to chase a ball he thought he
could catch up to during an important match. His right knee sustains the brunt of injury.
Physical examination reveals edema and decreased range of motion of the knee in flexion
and extension. Magnetic resonance imaging (MRI) is performed and reveals dislocation of the
joint. No pulse is palpable behind the knee joint. What is the most likely explanation for this
finding?

a. Anterior tibial artery rupture

b. Peroneal artery hematoma

c. Popliteal artery spasm

d. Posterior tibial artery hematoma

e. Superficial femoral artery spasm

View Answer

4. A 25-year-old woman found a lump in her right breast on self-examination. She has no
family history of breast cancer. The lump is freely mobile and well circumscribed. What is
the best option to evaluate a breast mass in a young female?

a. Biopsy

b. Mammography

c. Testing for breast cancer (BRCA) gene

d. Ultrasound

e. Watchful waiting

View Answer

5. A 19-year-old woman began breast-feeding for the first time. At first, it was difficult for
her infant to feed. Now, her breasts are red, warm, and sore. She has continued to breast-
feed, despite the pain; however, she has recently begun to use a breast pump instead of
breast-feeding. She is begun on a course of oral antibiotics. What condition is this patient at
risk of developing?

a. Breast abscess

b. Fibrocystic disease

c. Inflammatory breast cancer

d. Prolactinoma

e. Tuberculosis

View Answer
6. A 31-year-old premenopausal woman with a left breast mass undergoes a left modified
radical mastectomy. Pathology reveals infiltrating ductal carcinoma measuring 3 cm in size
with negative lymph nodes. Estrogen receptor status is negative. What is the most
appropriate adjuvant therapy for this patient?

a. Chemotherapy (multiagent)

b. External-beam radiotherapy

c. High-energy focused ultrasound therapy P

d. Tamoxifen

e. Watchful waiting

View Answer

7. A 31-year-old woman complains of a 6-month history of bloody diarrhea, abdominal pain,


and intermittent fevers. She has a history of irritable bowel syndrome but has had a
worsening of her symptoms during the above time period. Her past medical history is
unremarkable. Physical examination reveals abdominal distension. Bowel sounds are present
in all quadrants. Rectal examination reveals multiple anal fissures. What is the most
appropriate diagnostic testing for this patient?

a. Anoscopy

b. Colonoscopy

c. Flexible sigmoidoscopy

d. Rigid sigmoidoscopy

e. -No further diagnostic testing is required for this patient.

View Answer

8. A 71-year-old woman presents to her primary care physician complaining of rectal


bleeding. She had some mild left-sided abdominal cramps that subsided within a few
minutes. She has never had a prior episode of rectal bleeding. Physical examination reveals
mild left lower quadrant abdominal pain without evidence of guarding or rebound
tenderness. Rectal examination reveals no fresh blood in the rectal vault. Colonoscopy
reveals several outpouchings of the sigmoid colon wall without evidence of bleeding or
perforation. The remainder of the colonoscopy is within normal limits. White blood cell
count is normal. What is the most appropriate treatment for this patient?

a. Antibiotic therapy with ampicillin and gentamicin

b. Left hemicolectomy

c. Right hemicolectomy
d. Subtotal colectomy

e. Watchful waiting

View Answer

9. An 85-year-old man is brought to the emergency department because of acute abdominal


pain and progressive abdominal distention. He is a resident of a local nursing home. He has
not been eating because of progressive nausea. Abdominal radiographs reveal a massively
sigmoid colon. What is the initial treatment for this patient?

a. Gastrografin enema

b. High-fiber diet

c. Lactulose

d. Rectal tube decompression

e. Surgical resection

View Answer

10. A 41-year-old woman complains of constant headaches for the past 6 months. She has
also complained of female infertility and has been unable to have children, despite having
unprotected sexual intercourse with her husband during the past 15 years. Physical
examination reveals deficits in the extraocular movements bilaterally. Breast examination
reveals bilateral female gynecomastia. Which of the following laboratory tests would be most
useful in diagnosing this patient?

a. Ferritin

b. Hemoglobin

c. Hematocrit

d. Iron

e. Prolactin

View Answer

11. A 41-year-old woman with Crohn's disease has undergone multiple surgical procedures.
She has recently undergone an ileostomy but still has evidence of some distal jejunal
disease. Her current medications include prednisone and aminosalicylic acid. Which of the
following effects of prolonged therapy with glucocorticoids are possible for this patient?

a. Antibody production

b. Collagen formation

c. Fibroblast dysfunction
d. Inflammatory cell migration

e. Impaired Wound healing

View Answer

12. A 49-year-old obese man presents to his primary care physician for a follow-up
examination. He has a history of uncontrolled diabetes mellitus and bipolar disorder. His
current medications include lithium and milk of magnesium. Physical examination of the
heart, lungs, and abdomen are within normal limits. Laboratory studies reveal serum calcium
of 14 mg/dL. What is the most likely explanation for these findings?

a. Dietary indiscretion

b. Medication overdose

c. Milk-alkali syndrome

d. Parathyroid adenoma

e. Parathyroid hyperplasia

View Answer

13. A 41-year-old man has chronic gastroesophageal reflux. He is currently managed with an
H2-blocker. Physical examination of the heart, lungs, and abdomen are within normal limits.
Which of the following factors would be least protective of the esophagus in terms of the
continued exposure induced by this condition?

a. Arcuate ligament

b. Gastric emptying ability

c. Gravitational effect

d. Salivary gland secretory products

e. Secondary peristaltic waves

View Answer

14. A 40-year-old woman complains of chest pain and dysphagia to solids. She presents to a
specialist for evaluation. Esophageal manometric studies are performed
and reveal high-amplitude contractions and eventual normal relaxation of the lower P
esophageal sphincter. Barium swallow is normal. What is the most likely diagnosis?

a. Cricopharyngeal muscle spasm

b. Diffuse esophageal spasm

c. Scleroderma
d. Tuberculosis

e. Psychogenic swallowing disorder

View Answer

15. A 5-year-old boy is brought to the emergency department after ingesting liquid drain
cleaner. The boy was left unattended while his baby-sitter was on the telephone. The boy is
hoarse and has obvious stridor. What is the most appropriate initial treatment for this
patient?

a. Antibiotics

b. Corticosteroids

c. Induction of vomiting with ipecac

d. Placement of nasogastric tube and lavage

e. Tracheostomy

View Answer

16. A 76-year-old man with a history of vague right upper quadrant pain, a 25-lb weight loss,
and anorexia pre-sents to his primary care physician for evaluation. Physical examination
reveals scleral icterus. Abdominal examination reveals a right upper quadrant mass. Kidney,
ureter, and bladder (KUB) reveals a circular calcification in the right upper quadrant.
Exploratory laparot-omy reveals a neoplastic process involving the gallbladder and liver.
What is the most likely pathology causing this condition?

a. Adenocarcinoma

b. Sarcoma

c. Squamous cell carcinoma

d. Transitional cell carcinoma

e. Tuberculosis granuloma

View Answer

17. A 38-year-old woman presents to her primary care physician for evaluation of
intermittent vague right upper quadrant pain. She has a history of hypothyroidism and
hypertension. Her current medications include synthetic thyroid hormone replacement and a
calcium channel blocker. Physical examination reveals mild right upper quadrant pain to
deep palpation. Ultrasound reveals a 3-cm gallstone. What is the most likely type of stone to
be present in this patient?

a. Black gallstone
b. Brown gallstone

c. Calcium oxalate gallstone

d. Type I cholesterol stone

e. Type II cholesterol stone

View Answer

18. A 46-year-old woman presents to the emergency department complaining of right upper
quadrant pain and a fever to 102°F. Physical examination reveals scleral icterus and
significant right upper quadrant pain to palpation. Peritoneal signs are absent. Bowel sounds
are present. Which of the following should be included in the initial treatment of this
patient?

a. Antibiotics

b. Choledochojejunostomy

c. Decompression with T-tube

d. Endoscopic sphincterotomy

e. Percutaneous transhepatic drainage

View Answer

19. A 17-year-old boy is brought to the emergency department after suffering from chest
pain and dyspnea during a pickup basketball game. Physical examination reveals a systolic
crescendo-decrescendo murmur, heard best at the second right intercostal space. The
murmur radiates to the right carotid artery. Chest x-ray reveals a normal heart size. Which of
the following findings would be expected to be seen on an electrocardiogram in this patient?

a. Inversion of T waves in leads V1–V4

b. Left ventricular hypertrophy

c. Right bundle branch block

d. Right ventricular hypertrophy

e. Right atrial hypertrophy

View Answer

20. A 72-year-old man collapses while walking in a shopping mall. He is pulseless and apneic.
There is no history of trauma. Cardiopulmonary resuscitation is started until a rescue squad
arrives. Advanced cardiac life support protocol is initiated. He is pronounced dead 40
minutes later. Autopsy reveals myocardial necrosis with rupture of the left ventricle. Which
of the following is the most likely risk factor that contributed to his death?
a. Family history of diabetes mellitus

b. Hypotension

c. Obesity

d. Sedentary lifestyle

e. Trauma

View Answer

21. A 57-year-old man is brought to the emergency department complaining of dyspnea and
chest pain. He also admits to a 20-lb weight loss. He complains of fevers, chills, and night
sweats. Physical examination reveals supraclavicular adenopathy. Chest examination reveals
distant heart sounds. Laboratory studies reveal a white blood cell count of 170,000/mL.
Chest x-ray and echocardiography reveal a pericardial effusion. What is the most likely
explanation of these findings?

a. Atrial myxoma

b. Atrial fibrillation

c. Lymphoma

d. Metastatic colorectal carcinoma

e. Pericarditis

View Answer

22. A newborn male has an opening of the abdominal wall at the umbilicus. He has no other P
prior medical or surgical history. Birth history was unremarkable. During the remainder of
the physical examination and diagnostic testing, which of the following findings are most
likely?

a. Cleft lip

b. Cleft palate

c. Diaphragmatic hernia

d. Pericardium

e. Urinary bladder in retroperitoneum

View Answer

23. A 44-year-old male construction worker undergoes a right inguinal hernia repair. The
surgical procedure is uneventful. He has no prior medical or surgical history. He returns for
follow-up on postoperative day 3 for a wound check. The wound is clean, dry, and intact.
What is the optimal convalescent period required before returning to work for this patient?
a. 1 week

b. 4 weeks

c. 6 to 8 weeks

d. 12 weeks

e. Unknown

View Answer

24. A 40-year-old woman undergoes repair of a right femoral hernia. During the procedure,
the femoral canal is dissected. The anatomic boundaries of the femoral canal include which
of the following?

a. Cooper ligament

b. Inguinal ligament

c. Ischial spine

d. Lacunar ligament

e. Nerve (femoral)

View Answer

25. A 53-year-old man undergoes a radical prostatectomy for presumed organ-confined


prostate cancer. The most important factor in maintaining continence after radical
prostatectomy is preservation of the:

a. Bladder neck

b. External urethral sphincter

c. Levator ani muscle complex

d. Nervi erigentes

e. Puboprostatic ligaments

View Answer

26. A 27-year-old man has bulky retroperitoneal adenopathy after radical orchiectomy for a
mixed germ cell tumor. His chest x-ray is normal. Serum beta-human chorionic gonadotropin
(β-hCG) and alpha-fetoprotein (AFP) are markedly elevated. Liver enzymes are slightly
elevated, and the patient relates a history of ethanol excess. He receives three cycles of
chemotherapy. Restaging reveals a 3-cm retroperitoneal mass, a normal chest x-ray, and
normal serum β-hCG. However, the serum AFP is 20 IU/mL (normal = 0 to 9 IU/mL). What is
the next step in the management of this patient?
a. Computed tomography (CT)—guided needle biopsy

b. External-beam radiotherapy

c. Retroperitoneal lymph node dissection

d. Salvage chemotherapy

e. Serial markers and CT scans

View Answer

27. A 63-year-old man is disease-free two years after bacillus Calmette-Guerin therapy for
carcinoma in situ and a grade 2, stage T1 bladder cancer. In addition to physical
examination, cystoscopy, and urinary cytology, evaluation at this time should include:

a. Intravenous pyelogram

b. Prostatic urethral biopsy

c. Random biopsies of the bladder

d. Selective upper tract cytology

e. Urinary voided cytology, repeated three times

View Answer

28. A 78-year-old man presents to the emergency department for evaluation of progressive
right upper quadrant pain, nausea, vomiting, and a 30-lb weight loss in the past 3 months.
He has a prior medical history of cholelithiasis, diabetes mellitus, hypertension, and
dementia. Physical examination reveals scleral icterus bilaterally. Abdominal examination
reveals right upper quadrant tenderness and a palpable mass. Peritoneal signs are absent. CT
scan reveals pancreatic, duodenal, and choledochal lymph nodes. There is an asymmetric
thickening of the gallbladder. What is the most likely pathologic finding at exploratory
laparotomy and biopsy?

a. Adenocarcinoma

b. Fibroma

c. Lipoma

d. Myxoma

e. Myoma

View Answer

29. An 8-year-old boy undergoes a right upper quadrant ultrasound for persistent right upper
quadrant discomfort. He has no prior medical or surgical history. He has no known allergies
and takes no medications. His mother has a history of gallstones. Ultrasound findings include
a fusiform dilation of the common bile duct. What is the most likely explanation for these
findings?

a. Type I choledochal cyst

b. Type II choledochal cyst

c. Type III choledochal cyst

d. Type IV choledochal cyst

e. Type V choledochal cyst

View Answer

30. An 18-year-old man is stabbed in his abdomen multiple times by an assailant during an
altercation involving sale of illicit drugs. He is brought to the emergency
department for evaluation. He has four stab wounds of the abdomen—three are in the right P
upper quadrant, and one is in the left lower quadrant. Physical examination of the abdomen
reveals guarding and rebound tenderness. The patient is brought to surgery for an
exploratory laparotomy. A penetrating injury to the gallbladder is found. Which of the
following associated viscera are likely to be injured?

a. Aorta

b. Colon

c. Kidney

d. Liver

e. Urinary bladder

View Answer

31. A 62-year-old woman presents to her primary care physician with a cough. She also
complains of hemoptysis. Social history reveals a 55-pack-a-year history of smoking. She is a
recovering alcoholic. Physical examination reveals bilateral wheezes. Cardiac, pulmonary,
and abdominal examinations are unremarkable. Laboratory values reveal serum calcium of
13 mg/dL. Serum protein electrophoresis shows no abnormal spikes. What is the most likely
diagnosis?

a. Goodpasture's syndrome

b. Myeloma

c. Renal adenoma

d. Small-cell carcinoma of the lung

e. Squamous cell carcinoma of the lung


View Answer

32. A 10-year-old boy is brought to his primary care physician for evaluation of persistent
hoarseness. He has just begun to participate with his school chorus and notes that his
hoarseness worsens with singing. Physical examination of the heart, lungs, and abdomen are
unremarkable. Fiber-optic flexible laryngeal examination reveals multiple lesions on his true
vocal cords. What is the most likely diagnosis?

a. Gastroesophageal reflux

b. Granulomatous inflammation of the pharynx

c. Laryngeal papilloma

d. Singer's nodule

e. Thyroid carcinoma

View Answer

33. A 75-year-old man presents to his primary care physician because of hoarseness. He has
a 60-pack-a-year history of smoking. He also complains of a 25-lb weight loss over the past 4
months. Direct laryngoscopy reveals a sessile mass on the high right vocal cord. He also has a
palpable lymph node along the right anterior cervical lymph node chain. If dysplasia is found
on biopsy of the laryngeal lesion, what is the most likely diagnosis?

a. Adenoma

b. Laryngeal polyp

c. Laryngitis

d. Mucoepidermoid cystic disease

e. Squamous cell carcinoma

View Answer

34. A 21-year-old male college student presents to the outpatient clinic for a routine
examination at the beginning of the fall semester. He has a history of irritable bowel
syndrome. Physical examination of the heart, lungs, and abdomen are unremarkable.
Genitourinary examination reveals that the testes are descended bilaterally. A left grade 1
varicocele is present. There are no testicular masses. The penis is uncircumcised, and the
foreskin is unable to be retracted behind the glans. What is the most likely diagnosis?

a. Balanitis

b. Hypospadias

c. Epispadias
d. Paraphimosis

e. Phimosis

View Answer

35. A 71-year-old white male presents to his primary care physician complaining of a 1-
month history of nocturia, polyuria, and difficulty starting and stopping his urinary stream.
His American Urological Association Symptom Score is 17/35. Physical examination of the
prostate reveals an enlarged gland without masses. His testes are descended bilaterally. He
has a small right hydrocele that transilluminates. His prostate-specific antigen (PSA) level is 6
ng/mL, and urinalysis is negative. The patient is begun on dutasteride 0.5 mg daily. What is a
likely result of taking this medication?

a. Ejaculatory dysfunction

b. Maximal change in urinary flow rate

c. Prostate size decreases by 25%

d. Serum PSA increases by 50%

e. Symptom score remains unchanged

View Answer

36. A 34-year-old white male has a painless enlargement of his right testis in the past 4
months. He is brought to his primary care physician by his girlfriend, who urges him to seek
evaluation. He has recently become depressed because of this problem. He had a
cryptorchid right testis as an infant, which was surgically corrected. A scrotal ultrasound
confirms the presence of a 3 cm hypoechoic right testicular mass. What is the most likely
diagnosis?

a. Choriocarcinoma

b. Embryonal (mixed germ cell) carcinoma

c. Endodermal (yolk sac) tumor

d. Seminoma

e. Teratoma

View Answer

37. A 51-year-old man is found to have an intracranial mass and will undergo resection. The
surgical procedure is performed via a transoccipital approach. In this
approach, the patient develops a cerebrospinal fluid (CSF) leak. Which of the following P
statements is true regarding CSF?

a. Arachnoid villi act as two-way valves.


b. Arachnoid villi open at a pressure of 5 mm Hg.

c. CSF is absorbed through the spinal roots.

d. CSF enters through the foramen of Magendie.

e. Total CSF volume is 150 L.

View Answer

38. A 19-year-old college student is driving under the influence of alcohol, despite
recommendations from friends not to drive. She is struck by another driver. The force of
impact causes her to strike the temporal area of her skull against the window. She develops a
mild headache but does not lose consciousness. Several hours later, she develops a severe
headache with nausea and vomiting. Which is the most likely diagnosis?

a. Bacterial infection

b. Berry aneurysm

c. Epidural hematoma

d. Subarachnoid hematoma

e. Subdural hemorrhage

View Answer

39. A 59-year-old man presents to his primary care physician complaining of progressive
right-sided hearing loss and gait unsteadiness. He states that when he uses the phone, he
must use his left ear to listen instead of his right ear. He has a past medical history of
hypertension. His current medications include a calcium channel blocker. Physical
examination reveals loss of the right corneal reflex and facial weakness. Cardiac, pulmonary,
and abdominal examinations are within normal limits. What is the most appropriate next
best step in the diagnosis of this patient?

a. Audiometric testing

b. Brainstem-evoked potential testing

c. CT scan of the head without contrast

d. MRI of the head

e. Nystagmography

View Answer

40. A 47-year-old man with a history of end-stage pulmonary disease of his right lung is
scheduled for lung transplantation. Preoperative cardiac function is good. He has no history
of congenital defects. Which of the following is the most appropriate surgical incision for this
patient to have?

a. Chevron abdominal

b. Lateral thoracotomy

c. Midline abdominal

d. Transverse anterior thoracotomy

e. Pfannenstiel

View Answer

41. A 47-year-old man with multiple medical problems and end-stage pulmonary
parenchymal disease undergoes lung transplantation. He has a prior medical history of
obstructive lung disease. He has an uncle with cystic fibrosis. His father has restrictive lung
disease, and his brother has pulmonary hypertension. Which of the following portends the
best survival after lung transplantation for this patient?

a. Bronchogenic carcinoma

b. Cystic fibrosis

c. Obstructive lung disease

d. Pulmonary hypertension

e. Restrictive lung disease

View Answer

42. A 4-year-old boy is on the waiting list for a liver transplant. He has end-stage hepatic
disease and is currently hospitalized for esophageal variceal hemorrhage. What is the most
likely cause of liver failure in this patient?

a. Biliary atresia

b. Hepatitis A

c. Primary biliary cirrhosis

d. Primary sclerosing cholangitis

e. Tuberculosis

View Answer

43. A 23-year-old woman who complains of greasy and odorous stools, generalized weakness,
and hair loss presents to her primary care physician for evaluation. Physical examination of
the heart, lungs, and abdomen are unremarkable. She has no guarding or rebound
tenderness. Bowel sounds are present in all quadrants. Female pelvic examination was
deferred at the patient's request. What is the most likely explanation of these findings?

a. Gastric ulcer with bleeding

b. Glucose malabsorption

c. Menstruation

d. Pancreatic insufficiency

e. Pituitary tumor

View Answer

44. A 27-year-old woman is 12 hours status after cadaveric pancreas transplantation and
currently in the surgical intensive care unit. She has a medical history of insulin-dependent
diabetes since age 5. Her vital signs are normal. Chest is clear to auscultation, and cardiac
examination reveals a regular rate with a regular rhythm. Wound dressing is clean, dry, and
intact. Which of the following is the best method of monitoring the transplanted pancreas?

a. Serum amylase level

b. Serum glucose level

c. Serum insulin level

d. Ultrasonography of the pancreatic vessels

e. Urinary amylase level

View Answer

45. A 44-year-old man with recurrent pancreatitis is brought to the emergency department
with another bout of pancreatitis. Which of the following is the most reassuring factors
regarding the severity of his condition?

a. Age P

b. Blood glucose level of 300 mg/dL

c. Lactate dehydrogenase level of 400 IU/L

d. Serum calcium of 6 mg/dL

e. Serum hematocrit level of 29%

View Answer

46. A 41-year-old man with a long history of renal stones and hypercalcemia is found to have
an adenoma of the right superior parathyroid gland. He is going to undergo surgical excision
of this lesion. What is the best surgical landmark for this lesion?
a. Bifurcation of the carotid arteries

b. Carotid sinus

c. -Junction of the inferior thyroid artery and recurrent laryngeal nerve

d. -Junction of the upper and middle third of the thyroid gland

e. Recurrent laryngeal nerve

View Answer

47. Which of the following techniques is best used to define an enlarged parathyroid gland?

a. CT scan of the neck

b. Dual tracer imaging

c. MRI of the neck

d. Thyrocervical angiography

e. Ultrasonography

View Answer

48. A 44-year-old man with end-stage renal disease successfully undergoes renal
transplantation. He has a prior medical history of hyperparathyroidism. Six months after
renal transplantation, his serum calcium is still 13 mg/dL. Which of the following laboratory
findings are possible in this patient?

a. Elevated serum phosphate

b. Elevated serum lactic acid dehydrogenase

c. Elevated urine calcium

d. Elevated urine creatinine

e. Elevated urine protein

View Answer

49. A 46-year-old man presents to his primary care physician for evaluation of a skin lesion.
He complains of hypopigmentation of the skin of his lower back. He has a prior medical
history of eczema and basal cell carcinoma. He is a farmer who spends a great deal of time
outdoors. What cells are responsible for this condition?

a. Adipocytes

b. Keratin-producing cells

c. Langerhans cells
d. Melanocytes

e. Merkel cells

View Answer

50. A 69-year-old male presents to his dermatologist with a lesion present on his nose. He is
a gardener who spends a great deal of his time outdoors. He has a prior medical history of
allergic rhinitis, hypertension, and diabetes mellitus. His current medications include a beta-
blocker and an oral hypoglycemic. Physical examination of his nose reveals a raised, shiny,
papular lesion with small blood vessels. What is the most likely diagnosis?

a. Basal cell carcinoma

b. Histiocytosis X

c. Melanoma

d. Seborrheic keratosis

e. Squamous cell carcinoma

View Answer

51. A 29-year-old Black woman presents to her primary care physician because of a growth
on her left ear, which occurred after she had her ear pierced for the first time a week ago.
She noticed that her ear seemed to develop a growth on it quite rapidly. She had never had
her ear pierced before. What is the most likely explanation for these findings?

a. Basal cell carcinoma

b. Blue nevus

c. Juvenile melanoma

d. Keloid

e. Molluscum contagiosum

View Answer

52. A 52-year-old Asian American female has melanotic pigmentation of the buccal mucosa,
lips, and digits. Colonoscopy reveals hamartomas throughout the gastrointestinal tract. The
polyps were removed because of her increased risk of cancer. What other cancer is
associated with this condition?

a. Cervical cancer

b. Kidney cancer

c. Liver cancer
d. Ovarian cancer

e. Pancreatic cancer

View Answer

53. A 45-year-old female complains of chronic diarrhea and sweating. Colonoscopy is


performed, and a biopsy of a lesion in her ileum is performed. The pathology report shows
that the tumor is composed of neuroendocrine cells. What is a medical treatment for this
condition?

a. Corticosteroids, intravenous

b. Corticosteroids, topical

c. Furosemide

d. Octreotide

e. Tetracycline

View Answer

54. An 18-year-old male is brought to the emergency department with sudden excruciating
abdominal pain localized to the right lower quadrant, nausea and vomiting, mild fever, and
slight tachycardia. He has a prior medical history of recurrent otitis media. Physical
examination reveals marked right lower rebound tenderness and guarding. Serum white
blood cell count is 18,000/mL.
KUB x-ray reveals bowel gas in the small and large bowel. What is the most likely diagnosis? P

a. Appendicitis

b. Crohn's disease

c. Diverticulitis

d. Pancreatitis

e. Ulcerative colitis

View Answer

55. A 39-year-old woman presents to the emergency department complaining of severe


abdominal pain. She has a history of peptic ulcer disease. Physical examination reveals
guarding and rebound tenderness. She is taken to the operating room for exploratory
laparot-omy. During the procedure, the surgeon who opens the gastrosplenic ligament to
reach the lesser sac accidentally cuts an artery. Which of the following vessels is the most
likely one injured?

a. Gastroduodenal artery
b. Left gastric artery

c. Left gastroepiploic artery

d. Right gastric artery

e. Splenic artery

View Answer

56. A 2-year-old female is brought to the emergency department because of several episodes
of rectal bleeding. A technetium-99m perfusion scan reveals a 3-cm ileal outpouching
located 50 cm from the ileocecal valve. Which of the following types of ectopic tissue does
this structure most likely contain?

a. Duodenal

b. Esophageal

c. Gastric

d. Hepatic

e. Jejunal

View Answer

57. A 39-year-old woman complains of epigastric pain with eating over the past 3 or 4
months. She admits to a history of chronic back problems. She notes weight gain of 20 lb in
the past 4 months. She denies the use of non-steroidal anti-inflammatory agents. She denies
nausea and vomiting. Physical examination of the heart, lungs, and abdomen are within
normal limits. What is the most likely pathogen associated with this condition?

a. Enterohemorrhagic Escherichia coli

b. Escherichia coli

c. Helicobacter pylori

d. Shigella sonnei

e. Streptococcus pyogenes

View Answer

58. A 59-year-old man was injured in a car accident. An abdominal CT scan reveals a
ruptured spleen. His blood pressure is 90/40 mm Hg, and his pulse is 140 beats per minute.
The patient is taken for laparotomy. Splenectomy is performed. Which of the following
laboratory abnormalities is likely after this procedure?

a. Anemia
b. Basophilia

c. Eosinophilia

d. Thrombocytopenia

e. Thrombocytosis

View Answer

59. A 19-year-old man was kicked in the abdomen during a fight in a bar. He went to his
primary care physician, who ordered a CT scan, which revealed a subcapsular splenic
hematoma. The man was told to restrict physical activity. Two weeks later, he presents to
the emergency department because of severe abdominal pain. He undergoes a splenectomy.
Postoperatively, a peripheral smear is ordered. Which type of cell can be found in this
patient?

a. Basophilic stippling

b. Blister cells

c. Howell-Jolly bodies

d. Nucleated red blood cells

e. Spherocytes

View Answer

60. A 15-year-old African American male underwent a splenectomy after sustaining a knife
injury during a fight. He presents to his primary care physician for a sports physical. His
mother read on the Web that he is at increased risk for infection. He should receive which of
the following vaccines to prevent serious infections?

a. Rubella

b. Measles

c. Tetanus

d. Vaccines against common encapsulated organisms

e. Varicella

View Answer

61. A 53-year-old woman presents to her primary care physician with a 12-month history of
neck pain. She complains of a 15-lb weight gain and generalized malaise. She has a past
medical history of hypertension and diabetes mellitus. Her current medications include an
oral hypoglycemic. Physical examination reveals tenderness along the course of the thyroid
gland without evidence of a discrete mass. What is the most likely diagnosis?
a. Acute thyroiditis

b. Hashimoto thyroiditis

c. Papillary thyroid carcinoma

d. Riedel thyroiditis

e. Subacute thyroiditis

View Answer

62. A 47-year-old woman with a history of a left thyroid mass undergoes left thyroid
lobectomy. Pathology reveals a 1.3-cm papillary carcinoma with no evidence of extracapsular
extension. What is the most appropriate next step in the treatment of this patient?

a. External-beam radiotherapy

b. Multiagent chemotherapy

c. Subtotal thyroidectomy P

d. Total thyroidectomy

e. Watchful waiting with periodic follow-up

View Answer

63. A 34-year-old man with a thyroid nodule is undergoing a neck exploration. During the
procedure, it is possible that he will undergo thyroidectomy. Which of the following
statements about the superior laryngeal nerve and the innervation of the thyroid gland is
correct?

a. -Injury to the nerve causes bowing of the vocal cords during phonation.

b. Nerve injury may be unnoticeable in singers.

c. -The nerve is rarely at risk during thyroid surgical procedures.

d. The superior laryngeal nerve is chiefly a motor nerve.

e. -The superior laryngeal nerve is chiefly a sensory nerve.

View Answer

64. A 19-year-old man leaps from the third floor of his dormitory in an apparent suicide
attempt. He is brought to the emergency department unconscious. He has visible head and
lower extremity injuries. He has a pulse of 110 beats per minute but is apneic. What is the
best airway management for this patient?

a. Nasotracheal intubation

b. Oral intubation
c. Oral intubation with head-chin lift

d. Tracheostomy

e. Intubation is not necessary for this patient.

View Answer

65. A 21-year-old woman is stabbed in the chest by her boyfriend. She is brought to the
emergency department for evaluation. Her blood pressure is 130/80 mmHg, and her pulse is
90 beats per minute. Physical examination reveals a single stab wound to the left fifth
intercostal space in the midclavicular line. Neck examination is normal. Trachea is midline,
and the jugular veins are not distended. She does have decreased breath sounds in the left
lung fields. Which of the following diagnoses can be ruled out on the basis of the above
information?

a. Large left hemothorax

b. Open pneumothorax

c. Pericardial tamponade

d. Rupture of the left main stem bronchus

e. Tension pneumothorax

View Answer

66. A 41-year-old man suffers a traumatic amputation of three of his fingers in a meat slicer.
He has no prior medical or surgical history. Which of the following modalities should be used
to transport the amputated fingers with the patient?

a. Place in clean plastic bag and pack with dry ice.

b. -Place in clean plastic bag filled with room temperature water.

c. -Place in clean plastic bag in a chest filled with crushed ice and water.

d. -Place in clean plastic bag filled with hot water.

e. Wrap the amputated fingers in sterile dry gauze.

View Answer

67. A 19-year-old woman presents to the emergency department after sustaining an injury to
her right eye while placing her contact lens. She has significant right eye pain. She has a
prior medical history of seasonal allergies. Physical examination reveals a simple abrasion.
Fluorescein testing is performed and reveals no evidence of a stained epithelial defect. This
rules out the possibility of which of the following?

a. Bacterial infection
b. Iritis

c. Trauma

d. Viral infection

e. Ulcer

View Answer

68. A 29-year-old man who works in a factory sustained a foreign body injury to his right eye
when a piece of metal shot off a conveyer belt. He is brought to the emergency department
for evaluation. Physical examination of the right eye reveals a metallic foreign body on the
eye with an epithelial rust ring. What is the most useful instrument to remove this foreign
body?

a. Cyanoacrylate glue

b. Eye burr

c. Eye spud

d. Fine needle tip

e. Sterile water and alcohol

View Answer

69. A 37-year-old chemistry teacher sustains a chemical splash of acid to his right eye while
attempting to perform a demonstration to his high school science class. He is in significant
pain. While in the classroom and waiting for an ambulance to transport him to the hospital,
which of the following interventions should be performed?

a. Eyedrop instillation with normal saline

b. Eye patch placement

c. Flush eye with 1 to 2 L of normal saline

d. Placement of eye under direct sunlight

e. Watchful waiting until ambulance arrives

View Answer

70. A 37-year-old construction worker sustained a crush injury to his right thigh after a crane
fell on his leg at the work site. He is brought to the emergency department for evaluation.
He has significant right leg pain and pain with passive stretch. The leg is tense to palpation.
What is the most likely intracompartmental pressure measurement of this patient's right leg?

a. 5 mm Hg
b. 10 mm Hg

c. 15 mm Hg

d. 25 mm Hg

e. 35 mm Hg

View Answer

71. A 41-year-old woman who cleans houses for a living pre-sents to her primary care P
physician complaining of tenderness in her right knee. The pain is constant and has been
present for 3 weeks. She is in a monogamous relationship. Physical examination reveals that
her knee is slightly swollen and tender. Cardiac, pulmonary, and abdominal examinations are
within normal limits. A synovial aspiration is performed. The evaluation reveals no evidence
of crystals or bacteria. What is the most likely diagnosis?

a. Bursitis

b. Infectious arthritis

c. Rheumatoid arthritis

d. Septic thrombophlebitis

e. Trauma-induced infectious arthritis

View Answer

72. A 12-year-old boy who is the star pitcher of his little league team complains of right
shoulder pain. This is his pitching arm. He has no prior medical or surgical history. Physical
examination reveals weakness of the rotator cuff tendon. What is the most appropriate
treatment for this patient?

a. Injection of corticosteroids

b. Intravenous corticosteroids

c. Rest, elevation, and anti-inflammatory agents

d. Sling placement

e. Surgical repair

View Answer

73. A 65-year-old man with a history of coronary artery disease is undergoing an


aortobifemoral bypass. Which of the following intraoperative management maneuvers will
decrease his risk of intraoperative myocardial infarction?

a. Beta blockade
b. Calcium channel blockade

c. -Administration of normal saline instead of Lactated Ringer's solution

d. Use of propofol

e. Use of morphine

View Answer

74. A 35-year-old healthy man is diagnosed with an inguinal hernia. He has no history of
abnormal bleeding. Which of the following tests is absolutely required prior to taking him to
the operating room?

a. Hematocrit

b. Platelet count

c. Potassium

d. White blood cell count

e. None of the above

View Answer

75. A 50-year-old man has diarrhea after an uncomplicated bowel resection. The fluid choice
that most closely resembles his output is

a. Normal saline

b. Half normal saline with 20 mEq of potassium

c. D5W with 3 amp bicarbonate

d. Lactated Ringer's solution

e. D5NS

View Answer

76. A 27-year-old man is brought to the emergency department after slashing his hand with a
knife while attempting to slice a bagel. He has a prior medical history of recurrent sinus
infections. His prior surgical history is notable for repair of a nasal fracture. Physical
examination reveals a4-cm clean laceration along the palmar aspect of his hand. The
principles most relevant for this case would be:

a. Gentle handling of tissue and closure without tension

b. Skin grafting when necessary to cover area

c. Debridement of devitalized tissue

d. Use of vacuum sponge to promote healing


e. Flap reconstruction with vascularized tissue

View Answer

77. A 68-year-old man is brought to the emergency department complaining of abdominal


and leg pain for 2 weeks. He has a history of hypertension and hyper-cholesterolemia; he
weighs 290 lb. Physical examination reveals a pulsatile midline abdominal mass. The lower
extremities have unequal pulses. Which of the following is the best next step in the
evaluation of this patient?

a. Aortogram

b. CT scan of the abdomen and pelvis

c. Ultrasound, kidneys and bladder

d. Ultrasound, liver and spleen

e. Venacavogram

View Answer

78. A 28-year-old woman presents to her physician for evaluation of a lump in her right
breast found on self-examination. She has a family history of breast cancer in that her
mother died in her early 40s from this condition. The mother had a modified radical
mastectomy followed by chemotherapy. Physical examination reveals a breast lump that is
freely mobile and well circumscribed. There is no dimpling, asymmetry, or retractions. The
lesion measures 2 cm. What is the next step in the management of this patient?

a. Biopsy of the lesion with sonographic guidance

b. Mammography followed by stereotactic CT scan

c. Testing for BRCA gene

d. -Ultrasound of the breast and consideration for breast biopsy

e. -Watchful waiting and follow-up examination by primary care physician in 1 year

View Answer

79. A 28-year-old man with a history of recurrent abdominal pain and bloody diarrhea
presents to his physician complaining of significant rectal pain with bowel movements. He
has lost 15 pounds in the last 3 months. Physical examination reveals right and left lower
quadrant pain to palpation. Laboratory values reveal a hematocrit of 28% and an elevated
erythrocyte
sedimentation rate. Colonoscopy performed in this patient would likely reveal: P

a. Colonic mass lesion


b. Diverticulosis

c. Internal hemorrhoids

d. Normal bowel

e. Thickened friable colonic and rectal mucosa

View Answer

80. A 55-year-old male presents to his physician complaining of polyuria, polydipsia,


polyphagia, and a red, scaly rash on his face within the past 2 weeks. He voids 18 times/day
with a good force of stream. Fasting blood glucose was 325 mg/dL. He has lost 20 pounds in
the past 2 months and has never had elevated blood glucose levels in the past. Physical
examination of the heart, lungs, and abdomen are normal. What is the most likely
explanation for these findings?

a. Diabetes mellitus type I

b. Diabetes mellitus type II

c. Glucagonoma

d. Insulinoma

e. Verner-Morrison Disease

View Answer

81. A 72-year-old man with an 80 pack-year history of smoking presents to his physician
complaining of weakness and malaise. He has recently developed dysphagia to solid foods.
He has lost 15 pounds in the last 3 months. Physical examination reveals right
supraclavicular lymphadenopathy. Cardiac and pulmonary examinations are unremarkable.
He has no guarding or rebound tenderness. Which of the following studies will provide the
most definitive diagnosis?

a. Barium esophagogram

b. CT scan of the abdomen and pelvis

c. Esophagogastroscopy with biopsy

d. MRI of the abdomen

e. Ultrasound of the right upper quadrant

View Answer

82. A 42-year-old African American female undergoes a laparoscopic cholecystectomy for


chronic right upper quadrant pain. CT scan demonstrated gallstones and pericholecystic
fluid. The surgical procedure was uncomplicated. Pathologic analysis of the gallstones
revealed calcium bilirubinate stones. What is the most likely explanation of these findings?

a. High serum cholesterol levels

b. High serum lipid levels

c. High-fat diet

d. Sickle cell anemia

e. Tumor

View Answer

83. A 36-year-old female complains of jaundice and peripheral swelling. An echocardiogram


is ordered and the patient is determined to have right-sided heart failure with hepatic
congestion and peripheral edema. No murmur is detected. What is the most appropriate
explanation for these findings?

a. Aortic stenosis

b. Atrial septal defect

c. Patent ductus arteriosus

d. Tetralogy of Fallot

e. Ventricular septal defect

View Answer

84. A newborn male born at term to a 27-year-old intravenous drug-abusing female is found
to have a small umbilical hernia. His vital signs are stable. His cardiac and pulmonary
examinations are noncontributory. Which of the following is the most likely explanation for
this finding?

a. Patent foramen ovale

b. Patent omphalomesenteric duct

c. Patent umbilical ring

d. Patent urachus

e. Patent vitelline duct

View Answer

85. A 28-year-old man presents to the emergency department complaining of left flank pain,
nausea, and vomiting. Physical examination of the heart and lungs are normal. There is left
costovertebral (CVA) tenderness. Urinalysis reveals microhematuria (5 red blood cells/high-
power field). CT scan reveals a left-sided 4-mm renal stone, whereas KUB reveals a normal
bowel gas pattern and no evidence of calcifications. What is the most likely explanation of
these findings?

a. Calcium oxalate stone

b. Calcium phosphate monohydrate stone

c. Calcium phosphate dehydrate stone

d. Small struvite stone

e. Uric acid stone

View Answer

86. A 47-year-old woman with gallstone pancreatitis is hospitalized. She has a history of
hypertension and hyper-cholesterolemia. Early cholecystectomy is indicated to prevent
which of the following complications?

a. Cholangitis

b. Recurrent pancreatitis

c. Gastric ulcer

d. Gallstone ileus

e. Gallbladder perforation

View Answer

87. A 56-year-old man complains of recurrent cough and hemoptysis. He has a history of
recurrent pneumonias. He is a nonsmoker and has no occupational risk for pulmonary
disease. Physical examination reveals decreased breath sounds in the right upper lobe. Chest
x-ray reveals a small right upper quadrant mass. Bronchoscopy reveals an angioma. What is
the most appropriate treatment for this patient?

a. Antibiotics P

b. Corticosteroids

c. Left lobectomy

d. Right upper lobe lobectomy

e. Watchful waiting

View Answer

88. A 55-year-old male presents to his primary care physician after noticing some blood-
tinged urine 1 week ago. He denies any current pain and denies any fevers. His past medical
history includes chronic obstructive pulmonary disease from many years of smoking
cigarettes. CT scan reveals bilateral renal simple cysts, prostate enlargement, and an
asymmetric thickening of the left side of the bladder. Left hydronephrosis is also present. His
urinalysis is positive for microscopic hematuria. What is the most likely diagnosis?

a. Nephrolithiasis

b. Prostate cancer

c. Renal cell carcinoma

d. Transitional cell carcinoma of the bladder

e. Urinary tract infection

View Answer

89. A 30-year-old man undergoes a CT scan of his abdomen after a motor vehicle accident.
He was an unrestrained driver and was thrown from the vehicle. No acute abdominal injuries
are found. The CT scan reveals bilateral enlarged kidneys with multiple cysts present in
varying sizes. The right kidney is 15 cm and the left kidney is 16 cm in length. Physical
examination of the heart, lungs, and abdomen are within normal limits, other than some
mild tenderness to palpation in the right and left upper quadrants. Which of the following
central nervous system pathologies are most strongly associated with this finding?

a. Circle of Willis aneurysm

b. Cysticercosis

c. Infarction

d. Glioma

e. Subdural hematoma

View Answer

90. A 46-year-old female with polycystic kidney disease is receiving a kidney transplant.
Within minutes of the anastomoses of the renal artery and vein to the respective external
iliac artery and vein, the kidney rapidly regains a pink coloration and normal tissue turgor
and begins excreting urine. The patient is discharged from the hospital and is seen at 1
month follow up. Serum creatinine is 4.2 mg/dL. Urine output is 20 mL/hour. Physical
examination of the heart, lungs, and abdomen are within normal limits. The transplanted
kidney is palpable in the right iliac fossa. Subsequent biopsy of the transplant shows
extensive inflammation and edema. What is the most likely explanation for these findings?

a. Acute transplant rejection

b. Chronic transplant rejection

c. Graft-versus-host disease
d. Hyperacute transplant rejection

e. Normal posttransplant process

View Answer

91. A 52-year-old man complains of chronic abdominal pain. He has been hospitalized seven
times in the last 2 years for recurrent attacks of pain from chronic pancreatitis. He has been
treated with analgesics and a partial distal pancreatectomy. His pain has still persisted. What
is the next step in the treatment of this patient?

a. Continued use of oral analgesics

b. Corticosteroids

c. Splanchnicectomy

d. Total pancreatectomy

e. Watchful waiting

View Answer

92. A 39-year-old man is evaluated by his physician for recurrent kidney stones. He has been
treated in the past with extracorporeal shock wave lithotripsy, ureteroscopy, and a
percutaneous nephrolithotripsy. Which of the following characteristics would suggest the
diagnosis of primary hyperparathyroidism?

a. 1-mm stone on KUB x-ray

b. 2-mm stone on KUB x-ray

c. 1-mm and 2-mm right-sided stone on KUB x-ray

d. -1-mm, 2-mm, and 3-mm right- and left-sided stones on KUB x-ray

e. 4-mm right lower pole stone on CT scan

View Answer

93. A physician wishes to deliver a local anesthetic subcutaneously to a patient with a


suspicious lesion on his forehead. The lesion measures 1 cm and the surgeon plans an
elliptical incision to remove it. Which of the following epidermal skin layers will the
physician penetrate first with the local anesthetic needle?

a. Stratum basale

b. Stratum corneum

c. Stratum granulosum

d. Stratum lucidum
e. Stratum spinosum

View Answer

94. A 35-year-old man with a history of Crohn's disease pre-sents to his physician for a
follow-up examination. He has diffuse ileocolonic disease on a recent CT scan. His current
medications include sulfasalazine. Physical examination reveals right lower quadrant pain to
deep palpation. Should antibiotic therapy be considered in this patient, which of the
following organisms should be targeted?

a. Mycobacterium species

b. Pseudomonas aeruginosa

c. Staphylococcus aureus

d. Streptococcus pneumoniae

e. Streptococcus pyogenes

View Answer

95. A 46-year-old woman presents to her physician with a history of progressive dysphagia. P
She has a history of a 15-lb weight loss in the last 6 months. Physical examination of the
neck, heart, lungs, and abdomen are noncontributory. Laboratory testing reveals a
hematocrit of 33% and a mean corpuscular volume of 70. Upper gastrointestinal endoscopy
reveals an esophageal web. What is the most likely diagnosis?

a. Carcinoid tumor

b. Leukemia

c. Lymphoma

d. Plummer-Vinson syndrome

e. Rheumatoid arthritis

View Answer

96. A 47-year-old man is involved in a motor vehicle accident. He suffers fractures of ribs 9,
10, and 11 on the left side. He is hemodynamically unstable and has a blood pressure of
90/50 mm Hg and pulse of 120 beats/minute despite transfusion of 3 U of packed red blood
cells. He is taken to the operating room for exploratory laparotomy. A ruptured spleen is
identified and removed. In searching for a potential accessory spleen, which is the most
likely location to encounter it?

a. Greater omentum

b. Right lower quadrant


c. Right upper quadrant

d. Splenic hilum

e. Splenic ligaments

View Answer

97. A 12-year-old boy is brought to his physician for evaluation of a neck mass. He has a
history of recurrent sinusitis and tonsillar infections. Physical examination reveals a midline
neck mass measuring 1.5 cm that moves with swallowing. There is no evidence of
lymphadenopathy. What is the most likely diagnosis?

a. Leukemia

b. Lymphoma

c. Thyroglossal duct cyst

d. Thyroglossal fistula

e. Thyroid carcinoma

View Answer

98. During your Emergency Medicine rotation you are called to the emergency department to
evaluate a patient with an ophthalmic injury. Which of the following ophthalmic trauma
injuries requires immediate on-site management and referral to the ophthalmologist on call?

a. Acid burn

b. Corneal clouding

c. Corneal laceration

d. Hyphema

e. Severe conjunctival chemosis

View Answer

99. A patient presents to the emergency department after being slashed with a knife in the
left leg. The wound appears clean and the edges are well opposed. Which of the following is
the simplest method of wound closure for this injury?

a. Delayed primary closure

b. Graft

c. Local flaps

d. Primary closure

e. Secondary intention
View Answer

100. A 23-year-old female presents to her physician complaining of tenderness and pain in
the right knee. The pain is constant throughout the day and has been present for
approximately 3 weeks. Social history reveals that she is in a monogamous relationship.
Physical examination reveals that her knee is slightly swollen and tender. A synovial
aspiration is performed. Neither crystals nor bacteria are found. What is the most likely
diagnosis?

a. Bursitis

b. Gonococcal arthritis

c. Rheumatoid arthritis

d. Septic bursitis

e. Trauma

View Answer
Authors: Karp, Seth J.; Morris, James P.G.; Zaslau, Stanley
Title: Blueprints Surgery, 5th Edition

Copyright ©2008 Lippincott Williams & Wilkins

> Back of Book > Answers

Answers
1. a (Chapter 2)

This patient likely has an abdominal aortic aneurysm. Ninety-five percent of aneurysms of the
abdominal aorta are associated with atherosclerosis. This condition is responsible for
approximately 15,000 deaths per year. Men are affected nearly 10 times more than women.
Marfan syndrome can be associated with increased protease activity on histologic evaluation of
the aneurysm wall. Syphilitic aneurysms occur in late-stage syphilis. Meningococcal infection is
rarely associated with aneurysm formation. Trauma is a rare cause of abdominal aortic
aneurysm.

2. e (Chapter 2)

This patient likely has acute mesenteric ischemia. Patients complain of sudden onset of
abdominal pain with severe nausea, diarrhea, and/or vomiting. Pain is out of proportion to
physical findings. Treatment involves surgical resection of infracted bowel as soon as possible.
Aggressive surgical intervention should not be delayed because of the high index of suspicion of
infracted bowel. Angiographic embolization may be diagnostic and therapeutic but is not
considered a first-line therapy. Antibiotic therapy is considered an adjunctive therapy.
Heparinization is not a first-line therapy for this condition.

3. c (Chapter 2)

A major portion of circulation for the lower extremity begins with the superficial femoral artery,
which forms the popliteal artery passing behind the knee joint. This vessel then branches into
the anterior tibial, posterior tibial, and peroneal arteries. There is no evidence to suggest
rupture, hematoma, or spasm in these vessels. With knee injuries, the popliteal artery may go
into spasm because of its location just posterior to the joint. It is important for practitioners to
always palpate for a pulse in this artery in all patients with knee injuries. This vessel is also
important, as it determines vascular supply to the distal leg.

4. d (Chapter 3)

Younger women have more fibrous tissue, which makes mammograms harder to interpret. Thus
ultrasound is a useful testing modality. As women age, breast tissue transforms from fibrous
tissue to adipose tissue. This change makes it easier for mammography to detect masses. Thus
this modality is more useful in patients over the age of 35 years. Watchful waiting may be
considered if the lesion is benign. Testing for the BRCA gene may be considered if the patient is
suspect to a family history of breast cancer.

5. a (Chapter 3)

Women with mastitis need close follow-up for inflammatory breast disease. If a breast abscess
developed, she would need antibiotics. If her breast abscesses were recurrent, the physician
should consider resection of the involved ducts. If a patient with fibrocystic disease has straw-
colored fluid on aspiration, she would need to be observed closely. If the patient had
spontaneous galactorrhea, the physician would need to rule out a prolactinoma.

6. a (Chapter 3)

This patient should be treated with multiagent chemotherapy. This is the treatment of choice for
a premenopausal patient with stage I or II breast cancer (size <1 cm), negative lymph nodes, and
estrogen receptor—negative status. Watchful waiting may be appropriate for patients with small
tumors and negative lymph nodes. External-beam radiotherapy is not indicated for this patient.
High-energy focused ultrasound therapy is not indicated for the treatment of breast cancer.
Tamoxifen is considered in patients who are estrogen receptor positive and have tumor size >1
cm.

7. b (Chapter 4)

This patient likely has ulcerative colitis. Colonoscopy may reveal thickened, friable mucosa.
Fissures and pseudopolyps
may also be present. This disease almost always involves the rectum and extends backward P
toward the cecum to varying degrees. Anoscopy is a limited procedure and will not allow
visualization of the entire colon. Flexible or rigid sigmoidoscopy will allow visualization of the
rectum and sigmoid colon but will miss higher levels of the colon. This patient requires further
testing to establish a definitive diagnosis.

8. e (Chapter 4)

This patient has diverticulosis as a result of the presence of outpouchings in the wall of the colon
that occur where the arterial supply penetrates the bowel wall. Patients who stop bleeding and
are asymptomatic require no further treatment. Elective colectomy is not recommended at the
first episode; thus right hemicolectomy, left hemicolectomy, or subtotal colectomy are not
required. Intravenous antibiotic therapy is not required in this patient, because there is no
evidence of infection.

9. d (Chapter 4)

This patient has sigmoid volvulus. This condition can be reduced with a rectal tube, which is the
treatment of choice. In addition, one can consider decompression with enema. Cecal calculus is
treated with surgical intervention. High-fiber diet has no role in the treatment of volvulus.
Lactulose is unlikely to be of benefit in the management of this patient.

10. e (Chapter 5)
This patient likely has a prolactinoma, the most common type of pituitary neoplasm. Women
may present with headaches, irregular menses, amenorrhea, or galactorrhea. A serum prolactin
level of >300 µg/L suggests the diagnosis of pituitary adenoma. This can be confirmed with MRI.
Ferritin levels would likely be normal in this patient. Hemoglobin and hema-tocrit levels should
be normal in this patient. Iron levels should be normal in this patient.

11. c (Chapter 5)

This patient would be expected to have fibroblast dysfunction. Patients with inflammatory bowel
disease may require treatment with exogenous corticosteroids. These agents suppress the
immune system and impair inflammatory cell migration. Antibody production is impaired. This is
appropriate in Crohn's disease. Other effects of corticosteroids include fibroblast dysfunction and
impaired wound healing.

12. e (Chapter 5)

Renal failure is the most common cause of secondary hyperparathyroidism. This patient, who has
had severe uncontrolled diabetes and lab values consistent in patients with diabetes, is most
likely to have renal failure as the cause of his hypercalcemia. Whenever the kidney loses its
ability to reabsorb calcium and hydroxylate vitamin D for calcium absorption from the gut,
hypocalcemia triggers the parathyroid glands to increase their production of parathyroid
hormone. Milk-alkali syndrome can cause hypercalcemia in patients who eat many antacids or
drink an excessive amount of milk. This condition is more commonly found in patients who have
gastric ulcers and frequently depend on milk and antacids for relief. Lithium can cause
hypercalcemia by causing hyperparathyroidism. Parathyroid adenomas can cause hypercalcemia
by increasing parathyroid hormone secretion.

13. a (Chapter 6)

The arcuate ligament does not inhibit gastroesophageal (GE) reflux and will not protect the
esophageal mucosa from erosion. The esophagus limits its exposure to acid by several
mechanisms, including salivation, gravity, gastric emptying, and the activity of peptic acid. Also
important is the maintenance of a critical esophagogastric angle and appropriate diaphragmatic
location of the GE junction.

14. b (Chapter 6)

This patient has diffuse esophageal spasm. Patients present with chest pain and dysphagia.
Manometric studies reveal high-amplitude contractions and normal relaxation of the lower
esophageal sphincter. Cricopharyngeal spasm occurs because of muscular dysfunction.
Scleroderma is a collagen vascular disease that can affect the esophagus and cause motility
dysfunction. Tuberculosis can be associated with esophageal diverticula of the traction type. This
patient has no evidence to suggest a psychogenic swallowing disorder.

15. e (Chapter 6)
This child ingested a caustic alkaline substance. The child has difficulty breathing and has
stridor. Airway edema is likely. Thus tracheostomy should be performed first. Antibiotics and
corticosteroids are secondary to the important primary survey of airway, circulation, and
breathing in this patient. Vomiting should not be induced for a patient with a caustic ingestion.
Likewise, placement of a nasogastric tube should be deferred.

16. a (Chapter 7)

This patient likely has cancer of the gallbladder. Eighty percent of cases are due to
adenocarcinoma. Approximately 10% are anaplastic carcinoma, whereas 5% are squamous cell
carcinoma. A right upper quadrant mass may be palpable. Signs of jaundice are also possible.
This lesion is unlikely to be a sarcoma. Transitional cell carcinoma occurs in the urinary tract.
Tuberculosis granuloma is found in the lung.

17. e (Chapter 7)

This patient has a type II cholesterol stone. This is produced as a result of homogeneous
nucleation and can produce large gallstones. This type of gallstone represents 5% to 20% of all
gallstones. Type I cholesterol stones are small in size and often multiple. Calcium oxalate stones P
are often found in the kidney. Black and brown gallstones are smaller in size and multiple in
number.

18. a (Chapter 7)

This patient has the Charcot triad of fever, jaundice, and upper quadrant pain. This triad is seen
with acute cholangitis. Initial treatment consists of fluid resuscitation and antibiotics. Patients
who do not respond to this therapy need to be decompressed with percutaneous transhepatic
drainage. T-tube decompression can also be considered if there is failure to respond to
antibiotics. Choledochojejunostomy is considered when the bile duct is dilated.

19. b (Chapter 8)

This patient has evidence of aortic stenosis. Progressive degeneration and calcification of the
valve leaflets occur. Patients can complain of angina, syncope, and dyspnea. A crescendo-
decrescendo murmur can be heard best in the second right intercostal space. Electrocardiogram
reveals left ventricular hypertrophy. Bundle branch block is uncommon, as is T-wave inversion.

20. c (Chapter 8)

This patient has evidence of coronary artery disease that ultimately led to death. This is
confirmed with the autopsy findings of myocardial necrosis and rupture of the left ventricle. Risk
factors for coronary artery disease include hypertension, smoking, hypercholesterolemia, family
history of heart disease, personal history of diabetes mellitus, and obesity. Atherosclerosis is the
predominant pathogenic mechanism underlying obstructive disease of the coronary arteries.

21. c (Chapter 8)
This patient likely has a metastatic tumor to the heart. In this case, lymphoma is likely because
of the following symptoms: fever, fatigue, weight loss, and an elevated white blood cell count
beyond what would be expected with infection. Atrial myxoma would manifest as a mass lesion
detectible with echocardiogram. Atrial fibrillation is unlikely given the findings presented. This
patient has no colorectal symptoms; thus metastatic colorectal carcinoma is unlikely. Pericarditis
is unlikely given the presenting findings in this patient.

22. c (Chapter 9)

This newborn has omphalocele, an opening in the abdominal wall at the umbilicus that is due to
incomplete closure of the somatic folds of the anterior abdominal wall in the fetus. The
omphalocele can be a part of the pentalogy of Cantrell, which is associated with a diaphragmatic
hernia, cleft sternum, absent pericardium, intracardiac defects, and exstrophy of the bladder.
Cleft lip and palate are not present in these patients.

23. e (Chapter 9)

The optimal time of convalescence after hernia repair is unknown. After traditional open
surgery, patients have been asked to convalesce for 6 to 8 weeks. However, after laparoscopic
mesh repair, patients may return to strenuous activity in 2 to 3 weeks. However, the true optimal
time of convalescence is not known.

24. a (Chapter 9)

Femoral hernias are located in the femoral canal. The entrance to the canal is bounded
superiorly and medially by the iliopubic tract, inferiorly by Cooper ligament, and laterally by the
femoral vein. The inguinal ligament is more superficial. The ischial spine is not part of the
femoral triangle. The lacunar ligament is not part of the femoral triangle. The femoral vein, not
the femoral nerve, forms the lateral boundary of the triangle.

25. b (Chapter 10)

Although preservation of the bladder neck, nervi erigentes, and the size of the bladder neck
have all been associated with continence, the only factor that is generally accepted as being
related to urinary control after radical prostatectomy is preservation of the external sphincter.

26. c (Chapter 10)

This patient presents with a residual bulky mass after three courses of platinum-based
chemotherapy. Although the chest x-ray and β-hCG are normal, the serum AFP remains slightly
elevated. AFP production is usually attributed to yolk sac elements in a mixed germ cell tumor. It
is also seen with a number of other conditions, such as hepatocellular carcinomas and benign
hepatic disease, including alcohol hepatitis, as is probable in this case. Patients with persistent
marker elevations after chemotherapy are usually considered very likely to harbor residual
carcinoma and probably best managed by further chemotherapy. However, the AFP elevation
seen in this case is more likely due to benign liver disease. Consequently, this patient would be
best managed by retroperitoneal lymph node dissection instead. The most likely finding at
retroperitoneal lymph node dissection would be either fibrosis or residual teratoma. CT scan–
directed percutaneous needle biopsy would have considerable sampling error, and external-beam
radiotherapy has no efficacy, particularly in the management of teratoma. Further observation is
usually not warranted in patients who have residual retroperitoneal masses in excess of 2 to 3
cm.

27. a (Chapter 10)

The frequency of development of metachronous upper tract tumors in patients with superficial
transitional cell carcinoma (TCC) of the bladder is not exactly known but has been estimated to
be very low (1% to 3%). The incidence is higher in patients with higher stage (T2) primary lesions
(2% to 8%).
Patients treated for high-risk superficial TCC with BCG demonstrate a higher rate (13% to 18%) of P
upper tract tumors over 3 years of follow-up. The best follow-up approach in patients treated
with BCG is, therefore, the addition of upper tract imaging in the form of an intravenous
pyelogram or CT urogram. Selective cytology as a routine practice is not recommended.

28. a (Chapter 11)

This patient likely has adenocarcinoma of the gallbladder, the most common pathology of
gallbladder carcinoma. Ninety percent of patients have cholelithiasis. Metastases can occur to
the lymph nodes and to the liver. Prognosis is poor and has a 5-year survival rate ranging from 0%
to 10%. There are several rare benign tumors of the gallbladder, including fibroma, lipoma,
myxoma, and myoma.

29. a (Chapter 11)

This patient has a congenital malformation of the pancreaticobiliary tree. Specifically, this is a
type U choledochal cyst, which is a fusiform dilation of the common bile duct. The type II cyst is
a diverticulum of the common bile duct. The type III cyst is a choledochocele involving the bile
duct within the liver. The type IV cyst is a cystic celation of the intrahepatic ducts. The type V
cyst does not exist.

30. d (Chapter 11)

This patient has sustained a penetrating injury. Gallbladder injuries are uncommon but are seen
after such trauma, as in this patient. When the gallbladder is injured, one must search for other
injuries. The most frequent associated injury, in 72% of cases, is to the liver. Aortic injuries are
less common than are liver injuries. Colon injuries are less common than are liver injuries.
Kidney injuries are commonly associated with penetrating trauma. Urinary bladder trauma is
often associated with pelvic fractures.

31. e (Chapter 12)

The combination of cough, hemoptysis, wheezing, and smoking history suggests the diagnosis of
lung cancer. Of the two lung cancers listed, squamous cell carcinoma is the one that may
produce parathyroid hormone (PTH)–related peptide protein. PTH receptor leads to
hypercalcemia. Small-cell carcinomas commonly produce antidiuretic hormone or
adrenocorticotropin hormone. In a patient with Goodpasture, hemoptysis may present before
hematuria, but because of the other symptoms, squamous cell carcinoma is the better choice.
Renal cell carcinoma, not renal adenoma, may produce ectopic PTH-related protein, and
smokers do have an increased risk, but these patients present with hematuria, a palpable mass,
flank pain, and a fever. The lack of an immunoglobulin G or A spike on serum protein
electrophoresis should rule out multiple myeloma.

32. b (Chapter 12)

Laryngeal papillomas are benign neoplasms usually located on the true vocal cords. In children,
they present as multiple lesions and are usually caused by human papilloma virus. In adults, they
occur as single lesions and sometimes undergo malignant change. A singer's nodule is a small
benign laryngeal polyp associated with chronic irritation from excessive use or heavy cigarette
smoking and is usually found on the true vocal cords. Thyroid carcinoma is unlikely in children.

33. e (Chapter 12)

Squamous cell carcinoma is the most common type of cancer of the larynx. Cigarette smoking is
the most important risk factor. Laryngeal polyps are small and benign. They are usually
associated with chronic irritation from excessive use or heavy cigarette smoking.
Mucoepidermoid and adenocarcinoma of the larynx are not as common as squamous cell
carcinoma and don't have dysplasia as a precursor. Laryngitis is acute inflammation of the larynx,
trachea, and epiglottis and is most often caused by a viral infection.

34. e (Chapter 13)

Phimosis is an acquired or congenital condition in which the foreskin cannot be pulled back
behind the glans penis. In acquired phimosis, there likely is a history of poor hygiene, chronic
balanoposthitis, or forceful retraction of a congenital phimosis. Balanitis is inflammation of the
glans of the penis. Hypospadias is an anomaly in which the urethral meatus opens on the ventral
surface of the penis. Epispadias is an anomaly in which the urethral meatus opens on the dorsal
surface of the penis. Paraphimosis is an emergency condition in which the foreskin, once pulled
back behind the glans penis, cannot be brought down to its original position.

35. c (Chapter 13)

Dutasteride is 5-alpha reductase inhibitor used in the symptomatic treatment of benign prostatic
hyperplasia that blocks the conversion of testosterone to dihydroxy testosterone (DHT) in target
tissues. Because DHT is the major intracellular androgen in the prostate, dutasteride is effective
in suppressing DHT and, subsequently, stimulation of prostatic growth and secretory function.
Prostate size will decrease by approximately 25%. Ejaculatory dysfunction occurs in 5% to 8% of
patients. Serum prostate-specific antigen will decrease by 50%. American Urological Association
symptom scores typically improve by 5 to 7 points.

36. d (Chapter 13)


More than 90% of testicular tumors derive from germ cell tumors; the remainder are gonadal
stromal tumors or metastatic from another site. The most common solid tumor
in men between the ages of 15 and 40 is a seminoma. These tumors are typically confined to the P
testicle and associated with a hypoechoic area on ultrasound. This is an important feature, as
most other testis tumors are associated with mixed echogenicity on ultrasound.
Nonseminomatous germ cell tumors include embryonal carcinoma, choriocarcinoma, endodermal
yolk sac tumor, and teratoma.

37. b (Chapter 14)

Arachnoid villi open at a pressure of 5 mm Hg. They act as one-way valves. Cerebrospinal fluid
can be absorbed around the spinal nerve roots. Cerebrospinal fluid flows through the ventricles
and exits by the foramen of Magendie. The total volume of cerebrospinal fluid is 150 mL.

38. c (Chapter 14)

Epidural hematoma results from hemorrhage into the potential space between the dura and the
skull. The hemorrhage most likely results from rupture to a meningeal artery, which travels
within this plane; the middle meningeal artery, which branches off the maxillary artery in the
temporal area, is most common. Normally, the patient experiences a lucid interval, defined as
an asymptomatic period of a few hours after the trauma. A Berry aneurysm results from a defect
in the media of arteries and is usually located at bifurcation sites. Berry aneurysms are most
commonly found in the circle of Willis. The source of bleeding due to subdural hematoma is from
bridging veins; these often occur in older adults as a result of minor trauma, and symptoms
usually occur slowly—days to weeks. Bacterial meningitis diagnosis is confirmed with lumbar
puncture and demonstrates increased neutrophils/protein and decreased glucose in the
cerebrospinal fluid.

39. d (Chapter 14)

This patient may have an acoustic neuroma. These lesions arise from the vestibular portion of
cranial nerve VIII. MRI has now become the method of choice for evaluation of posterior fossa
and cerebellopontine angle tumors, because they are better seen on MRI as compared with CT.
Audiometric testing is useful for lesions of cranial nerve VIII. Brainstem-evoked potential testing
is useful for lesions of cranial nerve VIII. Nystagmography is useful for evaluation of vestibular
disorders.

40. b (Chapter 15)

The most appropriate incision for a single lung transplant is via lateral thoracotomy. Double lung
transplants are usually performed through a transverse anterior thoracotomy incision. Chevron
incisions are useful for renal surgery. Midline abdominal incisions are appropriate for abdominal
surgeries, not thoracic surgeries. Pfannenstiel incisions are appropriate to approach the female
genitourinary tract.

41. c (Chapter 15)


One thing to consider after lung transplantation is survival. It is similar when comparing single
with double lung transplantation. It is also related to diagnosis. The best survival is with
obstructive lung disease. This is followed by cystic fibrosis. The worst survival is associated with
pulmonary hypertension. Bronchogenic carcinoma also has a poor survival rate. Patients with
lung cancer are not considered candidates for lung transplantation.

42. a (Chapter 15)

Most candidates for liver transplantation have end-stage liver disease and are likely to die in 1 to
2 years. Children who require liver transplantation often have biliary atresia (in 50% of cases). In
adults, postnecrotic cirrhosis accounts for 55% of cases, of which most are due to alcoholism or
chronic hepatitis B. Approximately 15% of cases involve primary biliary cirrhosis and primary
sclerosing cholangitis.

43. d (Chapter 16)

Pancreatic insufficiency, which is commonly seen in patients with cystic fibrosis, presents with
malabsorptive issues and severe steatorrhea. Proper advice is to limit fat intake, as well as to
increase ingestion of fat-soluble vitamins. Glucose malabsorption would not have such effects
upon stooling. Menstrual loss could incur an anemic condition but not odorous stools. Bleeding
ulcers can also cause anemia and black tarry stools, without the odor issues.

44. e (Chapter 16)

The exocrine pancreas is anastomosed to the bladder, and by measuring amylase, the exocrine
product of the pancreas, one can monitor the functioning of the graft. Unless the patient had a
pancreatectomy, the native pancreas may be making amylase. For the first several days
postoperative, the serum glucose may not stabilize, and insulin may be required. The native
pancreas may make a variably small amount of insulin, so direct measurement of the graft is not
possible. The graft may have good blood flow but not be functioning well as a result of
microvascular damage.

45. a (Chapter 16)

Ranson developed 11 criteria to determine the severity of pancreatitis. These factors are divided
into admission criteria and initial 48 hours criteria. This patient's age of <55 years is reassuring.
His blood glucose level is worrisome. His serum lactate dehydrogenase (LDH) level is worrisome.
His serum calcium level is also worrisome. His serum calcium level is low and is also worrisome.

46. d (Chapter 17)

The superior parathyroid glands are located at the junction of the upper and middle third of the
thyroid gland on the
posteromedial aspect. The inferior parathyroids are located near the junction of the inferior P
thyroid and the recurrent laryngeal nerve. The carotid sinus is not near the location of the
superior parathyroid glands.

47. e (Chapter 17)


Ultrasonography will define an enlarged parathyroid gland in 70% to 80% of cases. Dual tracer
imaging can localize adenoma or hyperplasia in 70% of cases. Thyrocervical angiography is
reserved for patients with recurrent hyperparathyroidism after surgical neck exploration. CT and
MRI are helpful to locate enlarged parathyroid glands that are in the mediastinum.

48. c (Chapter 17)

This patient has tertiary hyperparathyroidism, which occurs in patients with chronic renal
disease despite successful renal transplantation. Patients will have hypercalcuria (elevated urine
calcium). There is no change in LDH levels. Serum phosphate levels are decreased. Serum and
urine calcium levels are increased.

49. d (Chapter 18)

Melanocytes produce melanin and are chiefly responsible for pigmentation of the skin. They are
of neural crest origin. One of the diseases associated with melanocytes is vitiligo, which is
characterized by flat, well-demarcated zones of pigment loss. Keratinocytes produce keratin,
which forms a waterproof layer. Langerhans cells are antigen-presenting cells. Merkel cells are
epidermal cells that function in cutaneous sensation. Adipocytes are fat storage cells.

50. a (Chapter 18)

Basal cell carcinomas are the most common skin tumors. They tend to involve skin-exposed
areas, most often in the head and neck. Grossly, they are characterized by a pearly papule with
overlying telangiectatic vessels. The lower lip is actually the most common site for a tobacco
user to develop squamous cell carcinoma. Malignant melanomas are the most likely primary skin
tumors to metastasize systemically. Histiocytosis X (Langerhans cell histiocytosis) is caused by a
proliferation of Langerhans cells, which are normally found in the epidermis. Seborrheic
keratosis is a benign squamoproliferative neoplasm, associated with sunlight exposure. Fair-
skinned persons are at increased risk. Depth of tumor correlates with risk of metastases.

51. d (Chapter 18)

A keloid is an abnormal proliferation of connective tissue with an abnormal arrangement of


collagen. This abnormal proliferation looks very similar to a tumorlike scar. Keloids are much
more common in African American individuals and usually follow some sort of trauma—in this
case, the ear piercing. The Spitz nevus can be confused with malignant melanoma. However, the
lack of color change or change in size would make melanoma a little less likely. Also, this patient
is much younger than the average age of patients who present with melanomatous lesions. Spitz
nevus is also known as juvenile melanoma; because of its benign nature, this name is falling out
of use. It is important to always think of melanoma when this type of lesion is seen and to order
appropriate tests to rule it out. Molluscum contagiosum is a viral disease caused by the DNA
poxvirus. It is contracted via direct contact, and its lesions are characteristically pink,
umbilicated, and dome-shaped.

52. d (Chapter 19)


Ovarian cancer is an associated risk for women with Peutz-Jegher's syndrome. Granulosa cell
tumor is the most common. This condition is not associated with cervical carcinoma. Hereditary
renal cell carcinoma can be associated with Von Hippel-Lindau disease. Hamartomas are not
associated with liver carcinoma or pancreatic carcinoma.

53. d (Chapter 19)

This patient has carcinoid syndrome, which is caused by the release of substances from a
carcinoid tumor. The medical treatment for carcinoid syndrome is octreotide, a somatostatin
analog. Topical or intravenous corticosteroids are not beneficial for this patient. Furosemide is a
loop diuretic used to treat fluid overload states. Tetracycline is an antibiotic and is not indicated
in the treatment of carcinoid syndrome.

54. a (Chapter 19)

Appendicitis is predominantly seen in young adults. It causes right lower quadrant pain, nausea,
vomiting, mild fever, and leukocytosis. The inflamed appendix may become gangrenous and
perforate in 24 to 48 hours. Therefore, immediate appendectomy is standard treatment.
Pancreatitis typically presents with epigastric pain radiating into the back, nausea, vomiting, and
fever. Crohn's disease and ulcerative colitis are inflammatory bowel diseases that typically
present with long-standing diarrhea. They do not typically present in an acute fashion, as in this
patient. Diverticulitis is predominantly found in older adults and typically presents with left
lower quadrant pain.

55. c (Chapter 20)

The left gastroepiploic artery runs through the gastrosplenic ligament to reach the greater
omentum. The gastroduodenal artery and the right gastric artery branch off of the common
hepatic artery. The gastroduodenal artery descends behind the first part of the duodenum. The
right gastric artery runs to the pylorus and then along the lesser curvature of the stomach. The
left gastric artery and the splenic artery arise from the celiac trunk. The left gastric artery runs
upward and to the left toward the cardia, giving rise to esophageal and hepatic branches, and
then turns right and runs along the lesser curvature within the lesser omentum to anastomose
with the right gastric artery.

56. c (Chapter 20) P

This child has a Meckel's diverticulum, which is a congenital anomaly resulting from an
unobliterated yolk stalk. More specifically, it is a vestigial remnant of the omphalomesenteric
duct. It presents as an ileal outpouching typically located close to the ileocecal valve. The
presence of inflammation, ulceration, and gastrointestinal bleeding due to the presence of
ectopic acid–secreting gastric epithelium is seen in approximately half of these patients.
Remember the rule of 2s with Meckel's diverticulum: It occurs in approximately 2% of children,
occurs within approximately 2 ft of the ileocecal valve, contains 2 types of ectopic mucosa
(gastric and pancreatic), and its symptoms usually occur by age 2.

57. c (Chapter 20)


Because of the symptoms of decreased burning with food intake and weight gain, a preliminary
differential diagnosis of Helicobacter pylori would be appropriate. Treatment with the triple
therapy of bismuth salicylate, metronidazole, and an antibiotic such as amoxicillin would be in
order. The presentation of Escherichia coli tends to be a more acute infection. E. coli is
associated with bloody diarrhea, and Shigella with abdominal cramping and diarrhea.

58. e (Chapter 21)

Thrombocytopenia is not a complication of a splenectomy. Thrombocytosis is a possible


complication postsplenectomy. Anemia is not a direct result of splenectomy. Patients are unlikely
to have basophilia or eosinophilia. Subphrenic abscess, atelectasis, pancreatitis, gastric dilation,
and sepsis are the possible complications of splenectomy.

59. c (Chapter 21)

The peripheral blood smear in a postsplenectomy patient will show Pappenheimer bodies,
Howell-Jolly bodies, and Heinz bodies. Nucleated red blood cells are found in the blood of sickle
cell patients. Basophilic stippling is found in the blood of patients with lead poisoning.
Spherocytes are found in patients with hemolytic anemia. Blister cells are found in the blood of
patients with glucose-6-phosphate deficiency.

60. d (Chapter 21)

Postsplenectomy patients should receive vaccinations against encapsulated organisms. The


common encapsulated organisms are Streptococcus pneumoniae, Neisseria meningitides, and
Haemophilus influenzae. The best time to vaccinate these patients is preoperatively.

61. b (Chapter 22)

This patient likely has Hashimoto thyroiditis. Patients have mild thyroid tenderness and fatigue.
Laboratory features include the presence of thyroid autoantibodies. Frequently, no treatment is
necessary for this condition. Acute thyroiditis is associated with fever, chills, and dysphagia.
Papillary carcinoma is associated with a palpable thyroid nodule. Riedel thyroiditis is associated
with thyroid fibrosis. Symptoms of tracheal and esophageal compression are possible.

62. e (Chapter 22)

This patient has evidence of papillary carcinoma of the thyroid. Only 5% of patients with
papillary carcinoma of the thyroid present with distant metastases. For tumors that are <1.5 cm
and that are disease-confined to one lobe and no extracapsular extension, treatment with
thyroid lobectomy is appropriate. External-beam radiotherapy is not required for this patient.
Multiagent chemotherapy is not required for this patient. Subtotal and total thyroidectomy are
not required for this patient.

63. a (Chapter 22)

Injury to the nerve causes bowing of the vocal cords during phonation. This can be a problem in
singers who have difficulty reaching high-pitched notes. The nerve can be at risk during thyroid
surgical procedures because of its proximity to the superior thyroid artery. The nerve is both
sensory and motor to the larynx.

64. d (Chapter 23)

This patient is apneic. An airway must be established for this patient. However, he may also have
fractures of the cervical spine. Thus the best treatment for this patient in terms of airway
management is a tracheostomy. Nasotracheal intubation is inappropriate for a patient who is
totally apneic. Oral intubation and oral intubation with head-chin lift is inappropriate because it
requires some hyperextension of the neck. Intubation is necessary for a patient who is apneic.

65. b (Chapter 23)

It is unlikely that this patient has an open pneumothorax. Patients with pneumothorax are in
obvious respiratory distress. They often have an obvious "sucking" chest wound. This patient has
neither of the above findings. Left pneumothorax is possible in this patient and needs evaluation
with a chest x-ray. Cardiac tamponade is possible in this patient, as is rupture of the main stem
bronchus. Tension pneumothorax is also a consideration for this patient.

66. c (Chapter 23)

An amputated upper extremity body part can be replanted if properly recovered and transported
with the patient. Cooling the body part in a chest filled with crushed ice and water may preserve
the body part for up to 18 hours. The body part should not be placed in dry gauze or packed with
dry ice. In addition, the body part should not be placed in warm water.

67. e (Chapter 23)

An ulcer will appear as a fluorescein-stained epithelial defect with a local corneal infiltrate. This
patient appears to have no
evidence of an ulcer. However, this form of testing does not rule out the presence of bacterial P
infection, iritis, trauma, or viral infection. This patient will benefit from treatment with
quinolone eyedrops and avoidance of eye patching. Close follow-up with a physician is also
recommended.

68. b (Chapter 23)

This patient has suffered a metallic foreign body to the eye with an epithelial rust ring. These
should be removed immediately with an eye burr. Foreign bodies of the cornea can also be
approached with a fine needle tip or an eye spud. Sterile water may be used, but alcohol
exposure to the eye should be avoided. Cyanoacrylate glue adheres to the eyes and should also
be avoided.

69. c (Chapter 23)

This patient has sustained a chemical burn to the eye. The eye should be immediately flushed at
the scene with 1 to 2 L of normal saline, which should be continued in the emergency
department. A topical anesthetic and a Morgan lens will facilitate flushing. Eye patch placement
is not indicated. Placement of the eye under direct sunlight may damage the eye. Watchful
waiting is not recommended; this patient needs eye lavage as soon as possible.

70. e (Chapter 25)

This patient likely has a compartment syndrome, which is caused by an increase in interstitial
fluid pressure within an osteofascial compartment, leading to compromise of the
microcirculation and myoneural necrosis. Diagnosis is confirmed by an intracompartmental
pressure of 30 mm Hg or higher. Treatment of this condition is surgical fascial release.

71. a (Chapter 25)

Bursae are fluid-filled sacs that cushion areas of friction between tendon and bone or skin.
Bursae are lined with special cells called synovial cells, which secrete a fluid rich in collagen and
proteins. This synovial fluid acts as a lubricant when parts of the body move. When this fluid
becomes irritated because of too much movement, the painful condition known as bursitis
results. Rheumatoid arthritis is a multisystem disorder that results in symmetrical joint
inflammation, articular erosions, and extra-articular complications. Infectious arthritis is unlikely
in the absence of joint fluid aspiration that reveals an organism. Septic thrombophlebitis is
unlikely given the history of this patient. Trauma is also unlikely given the history of this patient.

72. c (Chapter 25)

This patient may have suffered a tear of the rotator cuff. Most tears are small and may be
treated symptomatically with rest, elevation, and anti-inflammatory agents. If the shoulder still
demonstrates pain after a trial of conservative therapy, surgical repair should be considered.
Injection of corticosteroids or intravenous corticosteroids is not considered to be first-line
therapy for this patient.

73. a (Chapter 26)

Multiple randomized controlled studies have demonstrated decreased mortality and morbidity
with intraoperative beta blockade in high-risk patients. This is one of the few interventions that
has been clearly shown to improve outcomes.

74. e (Chapter 26)

In the setting of a normal history and physical, it is not necessary to obtain preoperative labs for
minor surgery, though many surgeons and institutions will do this.

75. d (Chapter 26)

Lactated Ringer's solution is a bicarbonate-rich solution with an electrolyte composition similar


to stool output.

76. a (Chapter 1)

Although all of the options contain important principles of wound closure, the most relevant
here is gentle handling of the tissue and closure without tension to prevent further damage that
can result in poor healing, infection, or hypertrophic scar formation. Although debridement of
devitalized tissue is important, in this type of injury, the surrounding tissue is generally healthy
and well vascularized.

77. b (Chapter 16)

This patient has signs and symptoms of an abdominal aortic aneurysm. This is suggested by the
mentioned risk factors and physical examination findings. CT scan will provide precise
anatomical detail and location of the aneurysm. This is particularly important for consideration
of stent graft repair. Aortogram may be useful as a secondary test to prepare for surgical
resection. Ultrasound could be performed to determine the presence of clot in the aortic lumen.
The ultrasound choices given in this question would be unlikely to determine clot in the lumen
because they are focusing on the upper quadrants and not the midline.

78. d (Chapter 14)

Younger women have more fibrous tissue, which makes mammograms harder to interpret. Thus
ultrasound is a useful testing modality. This patient has a family history of breast cancer.
Therefore, ultrasound and consideration for breast biopsy would be most prudent. As women
age, breast tissue transforms from fibrous tissue to adipose tissue. This change makes it easier
for mammography to detect masses. Thus this modality is more useful in patients older than 35
years. Watchful waiting may be considered if the lesion is benign. Testing for
the BRCA gene may be considered if the patient is suspect to a family history of breast cancer. P

79. e (Chapter 5)

This patient likely has ulcerative colitis. Patients complain of bloody diarrhea, fever, and weight
loss and have frequent attacks of symptoms. Colonoscopy often reveals thickened, friable
mucosa. Fissures and pseudopolyps are also common. Diverticulosis is rare in a patient younger
than 30 years. Although internal hemorrhoids are a possible cause of bleeding in this patient,
they do not present with the associated findings of abdominal pain and weight loss.

80. c (Chapter 15)

This patient has a glucagonoma, a tumor produced by pancreatic alpha cells that increases
glycogenolysis and gluconeogenesis and leads to increased blood glucose levels. Glucagonomas
are associated with necrolytic migratory erythema, a characteristically red, scaly rash usually
located on the face but also occurring in other locations. Typical patients are thin and have an
insidious onset of symptoms. Insulinoma would produce dizziness, diaphoresis, anxiety, and
tremor because of the increased production of insulin and resulting hypoglycemia. Patients with
type II diabetes mellitus are often obese and have a gradual onset of symptoms. Patients with
Verner Morrison disease often complain of watery diarrhea, hypokalemia, and achlorhydria.

81. c (Chapter 19)

This patient likely has esophageal cancer by history. Any older patient with dysphagia should be
assumed to have esophageal cancer until proven otherwise. This patient is a smoker with
progressive dysphagia and lymphadenopathy, placing him at high risk. Definitive diagnosis
requires biopsy at the time of esophagogastroscopy. CT and MRI may evaluate the presence of
regional lymphatic spread. Barium study will detect lesions in more than 90% of patients but will
not provide tissue for pathologic examination.

82. d (Chapter 7)

This patient likely has sickle cell anemia. Calcium bilirubinate stones are found in association
with the following conditions: chronic biliary infection, cirrhosis, and hemolytic processes such
as sickle cell anemia. A high-cholesterol diet has a role in the pathogenesis of cholesterol stones.

83. b (Chapter 17)

Atrial septal defects are often mild until later in life. As a result of the low pressures of both
atria, only a small left-to-right shunt will occur. Over time, this increase in blood return to the
right heart leads to pulmonary hypertension and right ventricular hypertrophy. Ventricular septal
defect is associated with late cyanosis because of the left to right shunt. Patent ductus arteriosis
is also associated with a left-to-right shunt and late cyanosis. The posterior descending artery
can close with indomethacin treatment. Tetralogy of Fallot is the most common cause of early
cyanosis. It is the result of a right-to-left shunt and results in early cyanosis.

84. c (Chapter 11)

Umbilical hernias are the result of a patent umbilical ring at birth. Some of the other choices in
this question, such as patent omphalomesenteric duct, are seen in association with umbilical
hernias, but this is likely coincidence as opposed to causality. Most of these hernias will resolve
without the need for operation.

85. e (Chapter 21)

This patient has a uric acid stone. These stones are visible on CT scan but not on KUB x-rays.
Calcium and struvite stones, on the other hand, are visible on both CT scan and on KUB x-rays.
Knowing the visibility of stones on imaging modalities assists with treatment decisions.

86. b (Chapter 6)

Recurrent pancreatitis occurs within 6 weeks in nearly 50% of patients. When this occurs,
complications of pancreatitis can be severe. Although the other choices are possible, they are
not nearly as common.

87. e (Chapter 18)

Angioma is a benign tumor that frequently regresses, and simple observation (watchful waiting)
with serial examinations and imagines studies is recommended. There is no need for other
treatments such as corticosteroids, antibiotics, or lobectomy.

88. d (Chapter 22)


This patient likely has transitional cell carcinoma of the bladder. This is a common cause of
painless gross hematuria in patients older than 50 years. This consideration is strengthened by
the CT scan findings in this question (hydronephrosis and bladder mass). Both renal cell
carcinoma and advanced prostate cancer can present with hematuria, but it is less likely. Urinary
tract infections are also a cause of hematuria, but are not common in men. Nephrolithiasis can
cause hematuria, but are more likely to cause flank pain.

89. a (Chapter 20)

This man most likely has adult polycystic kidney disease, an autosomal dominant disease. It is
the most common inherited disorder of the kidney. Adult polycystic kidney disease is often
associated with berry aneurysm in the circle of Willis. These aneurysms
are prone to rupture, leading to subarachnoid hemorrhage. Cerebellar hemangioma is associated P
with von Hippel-Lindau disease, an autosomal dominant disease of chromosome 3.

90. a (Chapter 26)

Acute rejection typically occurs days to months after transplantation and is characterized by
significant inflammation. Acute rejection is primarily T-cell mediated. These patients typically
present with symptoms of acute renal failure. Hyperacute rejection occurs within minutes or
hours of the transplantation and is due to preformed antibodies against the donor. In a
hyperacute kidney rejection, the graft would rapidly become cyanotic, mottled, and flaccid and
does not produce urine. Chronic rejection is probably more correctly termed chronic allograft
nephropathy and presents years after transplantation with chronic changes on biopsy. Patients
typically present clinically with a progressive increase in serum creatinine levels over a 4-to 6-
month period.

91. c (Chapter 9)

This patient has suffered from chronic pain which is presumed to be of pancreatic origin. This
can initially be treated with oral analgesics. For patients for whom this treatment fails,
operative drainage is considered next. All of the above steps have failed, and therefore, this
patient would best benefit from interruption of sympathetic nerve fibers (splanchnicectomy).

92. d (Chapter 13)

Patients with primary hyperparathyroidism can have renal stones. Typically, patients have
multiple and bilateral stones. Thus a patient with 1-mm, 2-mm, and 3-mm stones bilaterally and
visible on KUB suggests calcium stones.

93. b (Chapter 23)

Histologically, the epidermis has five layers, which are demarcated on the basis of microscopic
morphology. The most superficial layer, the stratum corneum, is characterized by anucleate cells
filled with keratin filaments. Beneath the stratum corneum is the stratum lucidum, which is only
seen well in thick skin and is considered to actually be a subdivision of the stratum corneum. The
stratum granulosum is beneath the stratum lucidum and contains keratohyalin granule-containing
cells. The stratum spinosum, beneath the stratum granulosum, is composed of spiny-looking
cells. Finally, the stratum basale, the deepest layer of the epidermis, is composed of a single
layer of stem cells from which keratinocytes arise.

94. a (Chapter 4)

Because of the presence of granulomas in the colon, mycobacterial infection is thought to be the
causative agent. Thus if one were to consider antibiotic therapy in such a patient, it should be
directed toward this agent. However, no specific immunologic disturbance has been described
for this disease.

95. d (Chapter 3)

This patient has postcricoid carcinoma, which is associated with Plummer-Vinson syndrome.
Dysphagia is nearly always a symptom of organic disease rather than a functional complaint.
Lymphoma and leukemia are rarely associated with iron deficiency anemia. Rheumatoid arthritis
is associated with arthralgias of joints of the hands. Iron deficiency anemia is uncommon with
this condition.

96. d (Chapter 8)

Accessory spleens are present in approximately 25% of patients. They are most often found in
the splenic hilum. The second most common location is the splenic ligaments and greater
omentum. Accessory spleens are almost never found in the right upper or lower quadrants.

97. c (Chapter 12)

This patient has a thyroglossal duct cyst. These are most commonly seen in children and appear
as a single painless lump in the midline that moves with swallowing. Thyroid carcinoma is rare in
children. Leukemia and lymphoma would be associated with palpable lymphadenopathy in the
neck.

98. a (Chapter 27)

There are two ophthalmic injuries that require immediate treatment: acid/alkali burns and an
unremovable corneal or conjunctival foreign body. The other choices should be referred
immediately and do not require immediate on site treatment.

99. d (Chapter 24)

Primary closure is the simplest and most common method of wound closure. Surgical wounds
created during a procedure are typically closed in this fashion. Delayed primary closure is
considered if the wound is contaminated or requires debridement.

100. a (Chapter 25)

Bursae are fluid-filled sacs that cushion areas of friction between tendon and bone or skin.
Bursae are lined with special cells called synovial cells, which secrete a fluid rich in collagen and
proteins. This synovial fluid acts as a lubricant when parts of the body move. When this fluid
becomes irritated because of too much movement, the painful condition known as bursitis
results. Rheumatoid arthritis is a multisystem disorder that
results in symmetrical joint inflammation, articular erosions, and extra-articular complications. P
Gonococcal arthritis is unlikely in the absence of joint fluid aspiration that reveals Gram-
negative diplococci. Rheumatoid arthritis is an autoimmune disease that affects the synovial
joints, with pannus formation in the joints (metacarpophalangeal, proximal interphalangeal),
subcutaneous rheumatoid nodules, ulnar deviation, and subluxation. Septic bursitis is unlikely in
the absence of a joint fluid aspiration that reveals organisms on Gram stain. Trauma is unlikely
given the history in this patient.

You might also like